Mastering Mrcp Vol2

  • Uploaded by: mostachek
  • 0
  • 0
  • January 2021
  • PDF

This document was uploaded by user and they confirmed that they have the permission to share it. If you are author or own the copyright of this book, please report to us by using this DMCA report form. Report DMCA


Overview

Download & View Mastering Mrcp Vol2 as PDF for free.

More details

  • Words: 110,854
  • Pages: 348
Loading documents preview...
Way to success

Mastering MRCP Volume 2 Dr. Ahmed Hakim 664 BOF + 31 EMQ

2010

2

Mastering MRCP Mastering MRCP Volume 2 By Dr. Ahmed Hakim MBChB , MRCP (UK)

First published in Iraq in 2010 All rights reserved. Apart from any use permitted under copyright law, this publication may only be reproduced, stored or transmitted, in any form, or by any means with prior permission in writing of the publishers or in the case of reprographic production in accordance with the terms of licenses issued by the Copyright Licensing Agency.

Whilst the advice and information in this book are believed to be true and accurate at the date of going to press, neither the authors nor the publisher can accept any legal responsibility or liability for any errors or omissions that may be made. In particular (but without limiting the generality of the preceding disclaimer) every effort has been made to check drug dosages; however it is still possible that errors have been missed. Furthermore, dosage schedules are constantly being revised and new side-effects recognized. For these reasons the reader is strongly urged to consult the drug companies’ printed instructions before administering any of the drugs recommended in this book.

Dedicated to All Iraqi Doctors When you read this book please ask God to forgive me ‫ﺍﺳﺌﻠﻜﻢ ﺍﻟﺪﻋﺎء ﻻ ﻏﻴﺮ‬

3

Mastering MRCP BOF: 1 A 30-year-old female presents with fatigue, weight loss, red, painful tender nodules over her shins and breathlessness. The chest X-ray shows bilateral hilar lymphadenopathy. Investigations show that she has hypercalcaemia. The cause of the hypercalcaemia is: a) Increased parathyroid hormone production in the lungs b) A high serum ACE causing increased hydroxylation of 25 hydroxycholecalciferol to 1,25 dihydroxycholecalciferol in the kidney c) Alpha hydroxylation of 25 hydroxycholecalciferol to 1, 25 dihydroxycholecalciferol by macrophages in the lung d) Lytic lesions in the bones e) Hyperventilation Answer: c) The patient has sarcoidosis. In this condition alpha hydroxylation of 25 hydroxycholecalciferol to 1,25 dihydroxycholecalciferol takes place in sarcoid macrophages in the lungs. This increases levels of calcitriol in the blood and results in hypercalcaemia.

BOF 2 The surgical registrar on duty calls you up to ask for advice. A 24-year-old female patient was admitted under their care the previous night and underwent a diagnostic laparoscopy for investigation of abdominal pain. The examination was normal, the pain has subsided and the plan is to discharge the patient. The patient is a tourist and wants to travel back to her home country. He would like to know how soon after laparoscopy a patient might undertake an airline flight. What would your reply be? a) Immediately b) After one week c) After five days d) 48 hours e) Ten days Answer: d) Modern aircraft normally cruise at between 35,000 to 43,000 feet. As this environment would be nonphysiological, the aircraft cabin is pressurized to a maximum cabin altitude of 8,000 feet. This reduced atmospheric pressure would cause gas in body cavities to expand by 30-40%. Hence, air travel should be delayed for 48 hours after laparoscopy to allow all gas to be absorbed.

BOF: 3 A 55-year-old male is admitted as an emergency with severe abdominal pain. He smokes 30 cigarettes a day and takes approximately 30 units of alcohol per week but admits to exceeding this amount sometimes. He also complains of sudden deterioration in vision. Ophthalmoscopy shows multiple micro infarcts (cotton wool spots). What investigation would best confirm your diagnosis and guide treatment? a) Upper GI endoscopy b) E.R.C.P.

4

Mastering MRCP c) Blood glucose d) Mesenteric angiogram e) CT scan abdomen Answer: e) Ischaemic retinopathy, which causes retinal oedema and micro infarcts, causes acute visual loss. This is a complication of acute pancreatitis. CT scanning will be useful in diagnosis and evaluation of pancreatitis.

BOF: 4 A patient has been transferred to your ward from the coronary care unit. He has suffered an uncomplicated myocardial infarct 3 days ago and at the moment he has no problems. He is a tourist and wishes to travel back home as soon as possible. How soon after an uncomplicated myocardial infarct may a patient undertake air travel safely? a) 10 days b) 48 hours c) 72 hours d) 14 days e) 21 days Answer: a) A patient should be fit to fly 10 days after an uncomplicated myocardial infarct.

BOF: 5 A 32-year-old female patient has had multiple resections of the bowel on account of recurrent Crohn's disease. This has resulted in intestinal failure and she is dependent on home parenteral nutrition. She has a Broviac catheter inserted for central venous access. She presents with fever accompanied by chills and rigors. No physical signs are demonstrable. Cultures taken both centrally and peripherally demonstrate the presence of methicillin-sensitive Staphylococcus aureus. Your next course of action would be a) Treatment with intravenous vancomycin for 14 days followed by repeat cultures b) Treatment with intravenous teicoplanin for 14 days followed by repeat cultures c) Determine the antibiotic sensitivity prior to commencing antibiotic treatment d) Do not use the intravenous line until the infection has been successfully eradicated. e) Remove the intravenous line Answer e) One of the main complications of parenteral nutrition is infection of the central line. Educating the patient and carriers may reduce this. However, line infections do occur and infection with Staphylococcus aureus and Candida are indications for line removal.

5

Mastering MRCP BOF: 6 A 76-year-old male has been admitted to your ward following a stroke that has resulted in a left hemiplegia. He is a mild diabetic and is hypertensive but both these conditions are well controlled. Seven days after admission the patient develops fever, tachycardia and tachypnoea. On auscultation of his chest crepitations are heard over both lung bases. Chest X-ray demonstrates bilateral basal pulmonary infiltrates. Your decision regarding empirical antibiotic treatment for this condition will be based on the assumption that the most likely causative organisms would be a) Methicillin-resistant Staphylococcus aureus b) Methicillin-sensitive Staphylococcus aureus c) Pneumococcus d) Gram-negative organisms e) Legionella Answer d) The diagnosis in this patient is most likely to be hospital-acquired pneumonia. After taking appropriate cultures, the patient should be started on antibiotics. Gram-negative organisms are the most likely cause of hospital-acquired pneumonia. Pneumococci are seldom isolated from culture and the most common gram-positive organism that causes hospital-acquired pneumonia is Staphylococcus aureus, particularly MRSA.

BOF: 7 A soldier returns from training in Belize. He has developed an ulcer on his right leg that has failed to heal despite treatment with antibiotics and daily dressings. Skin biopsy from the edge of the lesion has been examined histological and amastigotes bodies have been detected. Your next course of action would be. a) Leave the lesion alone and wait spontaneous healing. b) Freeze the lesion with liquid nitrogen c) Treat the patient with a twenty-day course of sodium stibogluconate d) Surgical excision e) Intralesional injection of sodium stibogluconate Answer c) Amastigotes (LD) bodies seen on biopsy suggest the diagnosis of Leshmaniasis. Cutaneous leishmaniasis may be divided into that of the Old World (Africa, Mediterranean, Afghanistan) and cutaneous leishmaniasis of the New World (Central and South America). Cutaneous leishmaniasis of the Old World heals in 4-18 months leaving a scar. No serious sequelae occur. It may be left alone. It could be frozen with liquid nitrogen. A pentavalent antimonial such as sodium stibogluconate may be administered intralesionally. Cutaneous leishmaniasis of the New World runs a more protracted course. Without treatment the ulcers may persist for years.

6

Mastering MRCP Some of the patients may develop the mucocutaneous form where the infection spreads from the skin the nasopharyngeal mucosa and this leads to progressive destruction of the nose and pharynx. New World cutaneous leishmaniasis should be treated and the drug used is sodium stibogluconate.

BOF: 8 One of your patients suffers from a cardiac arrhythmia that gives rise to syncope. He continues to drive despite you having made all reasonable efforts to explain to him that this is unsafe. Your response should be a) It is the legal obligation of the patient to inform the authorities of his disability; hence you are not obliged to take any further action b) As the patient has a right to confidentiality you may not take the matter any further c) Inform the police d) Inform the next of kin or the driving authority e) Inform the patient's solicitor Answer: d) In such a situation your duty to society overrides any right of an individual to confidentiality. The ultimate responsibility is yours and you have to inform the next of kin or the driver licensing authority.

BOF: 9 A 26-year-old male has returned from a backpacking trip to India. It is a week since arriving back in Britain. He has had fever for two weeks and has been treated with an antibiotic for a few days whilst in India. He does not know the name of the drug used. On examination he is febrile, has a few rose spots on his abdomen and splenomegaly. What source is the most likely to provide a positive culture in view of the fact that he has been exposed to antibiotics? a) Blood b) Rose spots c) Bile d) Urine e) Bone marrow Answer: e) In typhoid fever, positive cultures may be obtained from blood, bone marrow, urine, faeces, rose spots and bile. Bile has a high concentration of bacteria and the method of culturing it is by using a duodenal string capsule to obtain intestinal fluid. Bone marrow gives a good yield on culture and may provide culture even after antibiotic treatment has been commenced.

7

Mastering MRCP BOF: 10 A fifty-year-old patient attending the hypertension clinic has refractory hypertension. Random aldosterone: plasma renin activity has shown a ratio of greater than 750. In order to differentiate the underlying cause of primary aldosteronism, demonstrated in this patient, you arrange for the patient to have aldosterone: plasma renin activity measured in the morning (at 8 a.m.) with the patient in the supine position and again at noon with the patient in the erect position. This test helps to differentiate the causes of primary hyperaldosteronism because a) ACTH suppresses aldosterone secretion in adrenal adenoma b) ACTH has no effect on glucocorticoid suppressible hyperaldosteronism c) Erect posture increases plasma aldosterone in adrenal hyperplasia d) ACTH levels are higher at noon e) The supine position increases aldosterone secretion in glucocorticoid suppressible hyperaldosteronism. Answer: c) In primary aldosteronism, the aldosterone: PRA (plasma renin activity) ratio is greater than 750. The causes of primary aldosteronism are · An aldosterone producing adenoma (Conn's syndrome) · Bilateral adrenal hyperplasia · Glucocorticoid suppressible hyperaldosteronism · Adrenal carcinoma (rare) · Unilateral hyperplasia (rare) By measuring aldosterone: PRA ratio at 8a.m. in the supine position and at noon in the erect position one may differentiate between these conditions. Posture has an effect on aldosterone levels in adrenal hyperplasia. Erect posture increases plasma aldosterone. ACTH produces a marked effect in patients with glucocorticoid suppressible hyperaldosteronism. It increases aldosterone levels. ACTH has no effect in adrenal hyperplasia ACTH has only a small effect on adrenal adenoma At 8a.m. ACTH levels are higher than at noon.

BOF: 11 A 32-year-old male has returned from a holiday in Gambia six weeks ago. He presents with fever, chills and rigors and on clinical examination is found to have an enlarged spleen. Initial thin film examination for malarial parasites is negative. You are aware that local experience and expertise in diagnosing malaria is not adequate As the clinical suspicion of malaria is very high in this patient, what would your next course of action be? a) Repeat the thin film examination in 48 hours b) Do a thick film examination c) Do a dipstick assay of Plasmodium Lactate dehydrogenase (pLDH) d) Give empirical treatment for malaria e) Repeat thick and thin films in 72 hours Answer: c)

8

Mastering MRCP In patients with malaria, examination of a thin film is useful for making the diagnosis of the particular type of malaria and assessing the degree of parasitaemia. However, this depends on local expertise. If an initial film is negative it is best to repeat the test in 12 hours and then again in 24 hours if still negative. Thin films are more sensitive than thick films but this depends on the expertise of the person performing the examination. Dipstick assay is available for detection of all four types of malaria. The assay can differentiate between P.falciparum and P.vivax The test is an antigen detection system for Plasmodium Lactate Dehydrogenase (pLDH) p LDH is an abundant intracellular enzyme produced by malarial parasites. The dipstick is coated with monoclonal antibodies against this intracellular metabolic enzyme pLDH. The test is effective, sensitive and may be performed rapidly. It has almost the same sensitivity as examination of a thick film by an expert. It should be used as a supplement to thin film examination where expertise is lacking.

BOF: 12 A 60-year-old male is admitted with a history of acute on chronic breathlessness. He has been a longstanding smoker. On examination, he is breathless at rest with pursed lip breathing, use of accessory muscles of respiration and a barrel shaped chest. He is not oedematous. Which of the following physical signs will allow you to infer that on chest X-ray hyper inflated lung fields will be seen? a) Determining respiratory movements b) Vocal fremitus c) Percussion d) Auscultation of breath sounds e) Vocal resonance Answer: c) Absence or reduction of dullness over the praecordium (cardiac dullness) or reduction of dullness over the right lower chest (liver dullness) will allow you to infer that on chest X-ray the lung fields will be hyper inflated.

BOF: 13 A 56-year-old male presents with pain in the lower back. The pain has a girdle like distribution beginning in the lower back and radiating to the lower abdomen. He has not been on any drugs. The patient is hypertensive but there are no other physical signs of note. Investigations reveal a normocytic normochromic anaemia, raised erythrocyte sedimentation rate and C-reactive protein. Renal function is impaired. Ultrasound scanning reveals bilateral hydronephrosis. Which of the following investigations is most likely to give you the diagnosis? a) Intravenous urogram b) Retrograde urogram c) Isotope renogram d) Computerised tomogram of abdomen

9

Mastering MRCP e) Renal biopsy Answer: d) The patient has idiopathic retroperitoneal fibrosis (peri-aortitis). This is a condition in which the ureters become embedded in dense fibrous tissue, usually at the junction of the middle and lower thirds of the ureters. This results in unilateral or bilateral ureteric obstruction. CT scanning will show a peri-aortic mass. Histological confirmation is obtained by CT guided biopsy or laparotomy

BOF: 14 A 54-year-old male has a longstanding cough productive of sputum and breathlessness on exertion. The breathlessness increases following upper respiratory tract infections. He has been smoking 20 cigarettes a day over a period of 30 years. On examination, there is no clubbing the patient is breathless with use of accessory muscles and resting activation of the abdominal muscles. The chest is barrel shaped and cardiac dullness is reduced. Breath sounds are reduced on auscultation and a few rhonchi are audible. In your assessment of the patient which one of the following is known to be a predictor of mortality? a) The degree of breathlessness b) The presence of pursed lip breathing c) The body mass index (BMI) d) Spirometry e) Blood gas analysis Answer: c) The patient has chronic obstructive pulmonary disease. A low body mass index is a known predictor of mortality in patients with chronic obstructive pulmonary disease. A change in BMI with treatment suggests a better prognosis. Ankle swelling is also an important prognostic feature. This is because ankle swelling suggests the development of right ventricular failure

BOF: 15 A 54-year-old male with known chronic obstructive pulmonary disease is breathless on minimal exertion and is almost incapacitated by this. Chest X-ray has demonstrated the presence of large emphysematous bullae. In this patient you would arrange a CT scan of his chest for the following reason. a) To exclude a co-existent lung cancer b) To exclude co-existent heart failure c) To document the presence of bronchiectasis d) To determine the distribution of the emphysema e) To detect co-existent pulmonary emboli Answer:

10

Mastering MRCP d) This patient has incapacitating chronic obstructive pulmonary disease and has emphysematous bullae. This patient will be considered for lung volume reduction surgery. The distribution of the emphysema is an important variable when lung volume reduction surgery is being considered

BOF: 16 A 63-year-old male presents with a history of itching after he has a hot shower. There are no other clinical features of note. He has no significant past illnesses and is not taking any medication. Which of the following investigations is most likely to lead you to a diagnosis? a) Alkaline phosphatase levels b) Blood urea and electrolytes c) Thyroid stimulating hormone levels d) The fasting blood glucose levels e) A Full Blood Count Answer: e) Aquagenic pruritus is caused by the slow release of histamine when the skin is exposed to water. The slow rate of release of histamine gives a sufficient concentration to cause itch but is insufficient to cause visible vascular changes. Aquagenic pruritus is a feature of Polycythaemia Rubra Vera but may also occur in conditions such as Hodgkin's disease and myeloid metaplasia. Out of the choices given, the full blood count will pick up this haematological condition, which is the most likely cause of this type of pruritus in a male in this age group. Remember that aquagenic pruritus may precede the development of Polycythaemia Rubra Vera by several years.

BOF: 17 A 25 year old male who is known to have insulin dependent diabetes mellitus presents with nausea, vomiting and abdominal pain. He has a tachycardia, a postural drop in blood pressure. He is hyperglycaemic and has ketonuria. The clinical feature that will help differentiate abdominal pain due to diabetic ketoacidosis from a surgical emergency is: a) Postural hypotension b) The presence of diarrhoea c) Abdominal pain preceding the onset of vomiting d) Vomiting preceding the onset of abdominal pain e) Colicky pain in the right iliac fossa Answer: d) Abdominal pain may be a feature of diabetic ketoacidosis. It is usually a dull, persistent discomfort centered on the umbilicus. If vomiting precedes the onset of abdominal pain it is more likely to be due to ketoacidosis than to be a surgical emergency.

11

Mastering MRCP BOF: 18 Inferior extension of a pituitary tumour may result in a) Ethmoid sinusitis b) Sphenoid sinusitis c) Damage to the maxillary division of the trigeminal nerve d) Sixth nerve palsy e) Quadrantic hemianopia Answer: b) The inferior relationship of the pituitary is the sphenoid sinus and rarely inferior extension of a pituitary tumour may result in sphenoid sinusitis.

BOF: 19 An obese 45-year-old female is referred to the clinic by her general practitioner as she has been found to have a raised alanine aminotransferase level. She is not on any medication and does not take alcohol. On examination she is obese. In this patient you would expect: a) Type 2 diabetes mellitus b) Insulin resistance c) Hyperlipidaemia d) Acanthosis nigricans e) Hepatosplenomegaly Answer: b) This patient has non-alcoholic fatty liver disease. The metabolic abnormalities leading to accumulation of lipid within hepatocytes are poorly understood but one of the most reproducible features in the development of this process is insulin resistance. Type 2 diabetes mellitus and hyperlipidaemia frequently co-exist with this condition but not necessarily. Acanthosis nigricans is a feature of non-alcoholic fatty liver in children. Hepatomegaly is a common finding but not hepato-splenomegaly.

BOF: 20 A 45-year-old female being treated for rheumatoid arthritis is admitted complaining of breathlessness on exertion. She is found to have a Haemoglobin level of 8.5 g/dl with a MCV of 102. The white cell count and platelets are normal. The anaemia is most likely to be due to: a) Treatment with Diclofenac b) Treatment with methotrexate c) Anaemia of chronic disease d) Treatment with penicillamine e) Felty’s syndrome Answer: d)

12

Mastering MRCP Treatment with non-steroidal anti-inflammatory drugs would result in anaemia as a consequence of gastrointestinal haemorrhage. This would cause a microcytic hypochromic anaemia if chronic or normocytic normochromic anaemia if acute. Methotrexate treatment would result in marrow aplasia and a pancytopaenia Anaemia of chronic disease would result in normocytic normochromic anaemia. In Felty’s syndrome the patient would be neutropaenic. Treatment with penicillamine could result in haemolytic anaemia with macrocytosis

BOF: 21 A 33-year-old female who has had multiple resections of the small bowel has been left with 90 cms of jejunum anastamosed to the colon. She is maintained on a diet high in polysaccharides and manages well on this diet. She is admitted to the ward with ataxia, blurred vision, ophthalmoplegia and nystagmus. The likely cause of this complication is a) Thiamine deficiency b) Vitamin B 12 deficiency c) Magnesium deficiency d) L (+) lactic acidosis e) D (-) Lactic acidosis Answer: e) In patients with a short small bowel and an intact colon, energy is absorbed from the colon by bacterial fermentation of polysaccharides to short chain fatty acid, which can be absorbed by the colonocytes. In rare instances mono and oligosaccharides may be metabolised to D (-) lactic acid by abnormal bacteria. The normal lactic acid produced by man is L (+) lactic acid. Absorption of D (-) lactic acid results in ataxia, blurred vision, ophthalmoplegia and nystagmus. Treatment is with broad-spectrum antibiotics such as neomycin or vancomycin, thiamine and a change in diet to one high in polysaccharides and low in mono and oligosaccharides.

BOF: 22 A 28-year-old male presents with a painful swollen knee. He feels generally unwell and has fever. He has a psoriasiform rash on his glans penis and he also complains of low backache. Six weeks previously he has had a self-limiting episode of diarrhoea. In this patient a) Prompt treatment will reduce the chance of recurrence b) High dose steroids should be used without delay c) Prolonged antibiotic treatment will prevent the disease becoming chronic d) If the disease becomes chronic sulphasalazine and methotrexate are useful second line agents e) He has a greater than 50 % chance of developing erosive disease or spondylitis Answer: d) This patient has developed reactive arthritis. If possible athrocenetesis should be preformed to exclude septic arthritis. Prednisolone does help to control symptoms in active disease but is not the drug of first choice. Non-steroidal anti-inflammatory drugs should be used. Antibiotics should be used if active infection

13

Mastering MRCP is demonstrated but prolonged therapy is of no benefit. More than 50 % of patients will experience further episodes. Sulphasalazine and methotrexate are useful second line agents if the disease becomes chronic. About 15 % of patients go on to develop erosive disease or spondylitis.

BOF: 23 A 35-year-old female presents with a mutilating arthritis of the hands with associated pitting of the nails. In treating this patient a) Sulphasalazine is unlikely to be effective b) Methotrexate is unlikely to be effective c) Antimalarials are best avoided d) Tumour necrosis factor alpha antagonists are not effective e) Oral corticosteroids should be the drugs of first choice as they help the nail condition as well Answer: c) The patient has psoriatic arthropathy. Treatment is usually with non-steroidal anti-inflammatory drugs although there is a risk of worsening the psoriasis. Sulphasalazine and methotrexate are useful in this condition. Tumour necrosis factor alpha antagonists are effective but are expensive to use. Steroids are seldom needed and may provoke worsening of the psoriases on withdrawal. Antimalarials are best avoided as they can cause an acute psoriatic skin reaction.

BOF: 24 A 45-year-old male presents with a sudden onset of pain and swelling of the metatarsophalangeal joint of the right big toe. In this patient a) A normal serum uric acid concentration excludes the diagnosis of gout b) The x-ray changes would be characteristic c) Synovial fluid analysis should be delayed to allow crystals to aggregate and become easier to visualise d) Fever, leucocytosis and elevated ESR would suggest septic arthritis e) The first attack is seldom associated with residual disability Answer: e) In an acute attack of gout serum uric acid is raised in only about 60 % of patients. Similar x-ray changes may occur in inflammatory and degenerative arthritis. Synovial fluid analysis should be undertaken immediately following aspiration of joint fluid. The characteristic changes being the demonstration of needle shaped negatively birefringent crystals of mono-sodium urate in synovial fluid neutrophils by polarizing light microscopy. A raised ESR fever and leucocytosis can accompany very acute attacks of gout and do not necessarily indicate sepsis. The first attack of gout is seldom associated with residual disability.

BOF: 25 A sixty five year old male who is on treatment for chronic heart failure with diuretics, angiotensin converting enzyme inhibitors, beta-blockers and spironolactone presents with sudden onset of pain and

14

Mastering MRCP swelling of the metatarsophalangeal joint of his right big toe. Aspiration of the joint demonstrates crystals of monosodium urate. In this patient a) Moderate doses of aspirin would be beneficial b) Non-steroidal anti-inflammatory digs would be the drugs of first choice c) Highly selective cyclooxygenase 2 inhibitor should be used d) Colchicine would be the best choice e) Parenteral colchicine may be safely used to counter nausea and diarrhoea Answer: d) This patient has gout. Aspirin unless used in high doses causes uric acid retention. Non-steroidal antiinflammatory drugs would be contraindicated in view of the heart failure. Highly selective COX 2 inhibitors may not be used with co-existing heart failure. Intravenous colchicine is potentially hazardous.

BOF: 26 A 50-year-old female presents with a sudden onset occipital headache followed by a decreased level of consciousness. On examination she has neck stiffness and a positive Kernig’s sign. CT scanning shows blood in the sub-arachnoid and intraventricular space. The patient improves initially but 10 days following admission her level of consciousness begins to deteriorate. The next step in management would be: a) Decompression by lumbar puncture b) Lumbar puncture followed by high dose broad spectrum antibiotics until cultures are available c) High dose dexamethasone d) CT scan followed by a ventricular jugular shunt e) Cisternal puncture for decompression Answer: d) The patient has had a sub-arachnoid haemorrhage. Deterioration coming on after an initial improvement is most likely due to the development of secondary hydrocephalus due to blockage of CSF flow by blood. The management would be CT scan to confirm the diagnosis followed by a procedure to drain CSF.

BOF: 27 A 60-year-old female presents with a severe left-sided temporal headache. The temporal artery is tender, pulsation is lost and the overlying skin is erythematous. The E.S.R. is 80 mm in the first hour. In this patient a) A short course of high dose steroid should be prescribed b) The E.S.R. is not a reliable guide to use when reducing the dosage of steroids c) The headache subsides within hours of commencing the patient on high dose steroid d) Lifelong steroid treatment will be required e) As steroids may be harmful in elderly patients, treatment should be delayed until the results of temporal artery biopsy are available Answer:

15

Mastering MRCP c) The patient has temporal arteritis. The diagnosis is confirmed by biopsy but in view of the serious complications that may occur treatment with high-dose steroids should be started immediately. Reduction of steroid dosage is guided by the fall in the E.S.R. but the duration of treatment would be several months to years

BOF: 28 A 30-year-old female presents with a history of weakness and fatigability of the ocular, bulbar and limb muscles. On examination she has bilateral ptosis and extra-ocular muscle weakness. Reflexes are preserved, there is no muscle wasting. The Edrophonium test is positive. In this patient a) Thymectomy has no long term benefit b) If a thymoma is present the muscle weakness would improve c) In non-thymoma patients improvement will be seen in 60 % of patients d) The prognosis is worse as the patient is under 40 years of age e) Thymectomy should not be performed if the patient has positive receptor antibodies Answer: c) In myasthenia gravis thymectomy offers long-term benefits. It improves the prognosis in patients below 40 years, in those with positive receptor antibodies and in those who have had the disease for less than 10 years. Following thymectomy 60 % of non-thymoma patients will improve. In thymoma although surgery is necessary as the tumour is potentially malignant, the myasthenia is unlikely to improve.

BOF: 29 A 70-year-old male is referred by his general practitioner as he has had a stroke. On examination the patient has left sided complete third nerve palsy with a contralateral hemiplegia. The lesion is likely to be in the a) The pons b) The medulla c) The mid-brain at the level of the inferior colliculus d) The mid-brain at the level of the superior colliculus e) The thalamus Answer: d) The lesion involves the mid-brain at the level of the superior colliculus damaging the third nerve nucleus and the cerebral peduncles.

BOF: 30 A 30-year-old male presents with a chronic cough productive of copious amounts of thick yellow sputum and occasional haemoptysis. He also complains of bad breath and recurrent episodes of fever. On examination he has clubbing and on auscultation over the lung bases coarse crepitations are heard. The test that would identify the cause of his condition would be:

16

Mastering MRCP a) Sweat electrolytes b) Sinus x-ray c) Bronchoscopy d) High resolution CT scanning e) Bronchography Answer: d) The patient has bronchiectasis. High resolution CT scanning would be the investigation of choice. It would show bronchial dilatation and wall thickening.

BOF 31 A 68-year-old female patient is seen on the ward. She complains of severe pain in her right eye. There is blurring of vision and she feels nauseated and has vomited several times. Earlier in the day she has undergone colonoscopy for evaluation of her long-standing Crohn's disease. The endoscopist has made a comment that the examination was difficult. What is the likely cause of her painful red eye? a) Anterior uveitis b) Acute conjunctivitis c) Episcleritis d) Sub-conjunctival haemorrhage e) Acute angle closure glaucoma Answer e) Anticholinergic agents are sometimes used during endoscopy to cause smooth muscle relaxation to aid examination when difficulty is encountered. These agents cause pupillary dilatation thus precipitating acute angle closure glaucoma in susceptible patients. In patients with a history of glaucoma, glucagon is used instead of anticholinergics. BOF: 32 A 65-year-old male presents with a chronic cough. He is a heavy smoker of over 40 cigarettes a day. CXR shows a peripheral right-sided lesion, which on CT guided lung biopsy, is shown to be squamous carcinoma. No regional lymph nodes are involved. Lung function tests show a FEV1 of less than 1.5 liters. The treatment most likely to benefit this patient would be: a) Surgery b) Chemotherapy c) High dose radiotherapy d) Combination chemotherapy and radiotherapy e) Combination chemotherapy and surgery Answer: c) A FEV1 of less than 1.5 liters is not compatible with an active life following surgery. High dose radiotherapy can produce good results and is the treatment of choice in patients with poor lung function.

17

Mastering MRCP BOF: 33 A 45-year-old male homeless alcoholic has been referred to the medical ward after being brought in to casualty. He has a chronic cough productive of sputum, loss of weight, and night sweats. On examination he is unkempt and emaciated. His trachea is deviated to the left and there are crepitations over the apex of the left lung. CXR shows fibrosis and cavitation in the left apex. The investigation most likely to confirm the diagnosis would be a) Sputum examination for acid and alcohol fast bacilli b) High resolution CT scan c) Fibreoptic bronchoscopy d) Mantoux test e) Gastric washings Answer: a) The patient has a productive cough. The chances are that AAFB will be identified in these specimens. If sputum were not produced bronchoscopy would be preferred to gastric washings.

BOF: 34 A fifty –five year old male presents with a history of anorexia, nausea and vomiting and abdominal pain. His skin is pigmented with pigmentation of palmar creases and sun exposed areas. He has a few patches of vitiligo. His blood pressure is low and there is a postural drop. In this patient the blood urea and electrolytes are likely to show the following a) Decreased Na, Decreased K, Normal Urea b) Decreased Na, Increased K, Increased Urea C) Decreased Na, Increased K Normal Urea d) Increased Na, Decreased K, Increased Urea e) Decreased Na, Decreased K, Increased Urea Answer: b) The patient has Addison’s disease. The Na will be low with an increase in K and increase in blood urea

BOF: 35 A sixty-year-old female presents with a history of palpitations and swelling in the neck. On examination of the pulse there is atrial fibrillation and in the neck there is a large multinodular goiter. In this patient which one of the following are likely a) Eye signs are common b) Eye signs are rare c) Spontaneous remission is likely d) Long term antithyroid drugs are effective in controlling symptoms e) Thyroxine will help to reduce the size of the goiter Answer: b)

18

Mastering MRCP This patient has toxic multinodular goiter. In this condition eye signs are rare unlike Grave’s disease. Spontaneous remission is rare. Antithyroid drugs will increase the size of the goiter and are only used as a temporary measure prior to definitive treatment. Thyroxine will not reduce the size of the goiter

BOF: 36 A sixty-year-old female presents with a history of nausea, lethargy and depression. Her skin is pigmented and there is vitiligo. Her blood pressure is low and there is a postural drop. In this patient which one of the following are true a) Eosinophilia is a feature b) The ESR is decreased c) Hyperglycemia is a feature d) The heart size is small e) Hypokalaemia would occur Answer: d) The patient has Addison’s disease. In this condition the heart size is small. The eosinophil count may be elevated, the ESR may be high, hypoglycemia may be a feature, and hyperkalaemia would be a feature

BOF: 37 A sixty-year-old man presents with a history of increased sweating. He also complains of headaches. On examination the patient has large hands and the facial features are exaggerated with large nose, prominent jaw and thick lips. In this patient which of the following may be used as a screening test a) Growth hormone level b) Glucose Tolerance Test c) Prolactin level d) Plasma Insulin-like Growth Factor levels e) Serum calcium Answer: d) The patient has acromegaly. Plasma Insulin-like Growth Factor may be used as a screening test

BOF: 38 A fifty-five year old man is admitted with a history of fatigue, weight loss and jaundice. His alcohol intake is sixty units a week. On examination he has clubbing, Dupuytren’s contracture, palmar erythema, flapping tremor, parotid enlargement, spider naevi, gynaecomastia, hepatosplenomegaly. Which of the following signs is indicative of a poor prognosis? a) Clubbing b) Parotid Enlargement c) Gynaecomastia d) Flapping Tremor e) Splenomegaly

19

Mastering MRCP Answer: d) The patient has alcoholic liver disease with clinical evidence of cirrhosis. The features of a poor prognosis are hepatic encephalopathy, low serum albumin concentration, and low serum sodium and prolonged prothrombin time.

BOF: 39 A sixty-year-old man who is known to have ischaemic heart disease is admitted with a history of sudden onset abdominal pain, followed by watery diarrhoea and subsequent profuse rectal bleeding. The likely diagnosis is a) Small bowel infarction b) Large bowel infarction c) Volvulus of the sigmoid colon d) Colon cancer with intussusceptions e) Ulcerative colitis Answer: b) The history of pain followed by diarrhoea and bleeding per rectum in a patient with known macro vascular disease is typical of large bowel infarction, which occurs in the region of the splenic flexure.

BOF: 40 A thirty five year old female has had a right hemicolectomy and resection of 30 cms of terminal ileum for ileocaecal Crohn’s disease. She has persistent diarrhoea, which is not explosive. She does not have abdominal pain, bloating, or loss of weight. Investigations have failed to demonstrate evidence of recurrent Crohn’s disease. Treatment that would relieve symptoms and give a clue to the underlying diagnosis would be: a) Loperamide b) Steroids c) Cyclical antibiotics d) Cholestyramine e) Tricyclic antidepressants Answer: d) The patient has had resection of the terminal ileum and the cause of the diarrhoea is likely to be Bile Acid Malabsorption (BAM). Treatment with a Bile Acid Sequestrants such as Cholestyramine would relieve symptoms and point to the diagnosis

BOF: 41 A thirty four year old female presents with a deep vein thrombosis. She has a history of weight loss, recurrent mouth ulcers and chronic diarrhoea with the passage of a bulky stool, which is difficult to flush away. She has a macrocytic anaemia. The underlying biochemical abnormality that would explain the DVT would be:

20

Mastering MRCP a) Vitamin B 12 deficiency b) Folic acid deficiency c) Iron deficiency d) Protein S deficiency e) Protein C deficiency Answer: b) The patient has coeliac disease. In coeliac disease folic acid deficiency with resultant hyperhomocystinaemia increases the tendency to thromboembolic phenomena

BOF: 42 A twenty seven year old female presents with a rash. The rash consists of erythematous plaques, excoriations, and vesicles some of which have ruptured leaving a crust. She also complains of diarrhoea with the passage of a bulky stool, which is difficult to flush away. In this patient: a) The rash responds rapidly to a gluten free diet b) The rash responds slowly to treatment with Dapsone c) The rash responds rapidly to treatment with Dapsone d) Doxycycline is the drug of choice e) Oral Steroids should be commenced immediately Answer: c) The patient has dermatitis herpetiformis, which is associated with coeliac disease. The rash responds within a few hours to treatment with dapsone.

BOF: 43 A twenty four year old female who is known to have ulcerative colitis presents with an ulcer above the medial malleolus. This is large has a necrotic base and the edges are undermined. In this patient: a) Immediate treatment should be with broad spectrum antibiotics b) High dose steroids should be used in the first instance c) Surgery is the first line treatment of the ulcer d) Treatment of the colitis with high dose mesalazine preparation will cause the ulcer to heal e) Colectomy is indicated Answer: b) The patient has pyoderma gangrenosum complicating ulcerative colitis. The initial treatment should be with high dose steroids.

21

Mastering MRCP BOF: 44 A 50-year-old male presents with oral ulceration and flaccid blisters on the skin especially the trunk. The blisters are sore but not itchy and they rapidly denude leaving weeping, erythematous erosions. Gentle sliding pressure on the blisters makes them extend. In this condition: a) Low dose steroids are effective in controlling formation of new blisters b) Local steroids are effective in controlling the disease c) Long term tetracycline is effective treatment d) Gold may be effective therapy in steroid resistant patients e) Dapsone is effective on controlling the disease Answer: d) The patient has pemphigus vulgaris. The physical sign described is Nikolsky's sign( extension of the blister with pressure). In pemphigus vulgaris high dose steroids may be needed to prevent formation of new blisters. Azathioprine and methotrexate may be needed to reduce the dose of steroids. Gold may be effective in steroid resistant patients.

BOF: 45 In patients with cystic fibrosis a FEV1 of less than 30 % would predict a) 80% of patients would be infected with Pseudomonas aeroginosa b) 60% would develop a digestive tract malignancy c) 80 % would develop a variceal haemorrhage d) 50 % would be dead within two years e) 60 % would respond to treatment with high dose ibuprofen Answer: d) In patients with cystic fibrosis a FEV1 of less than 30 % would predict 50 % mortality within two years

BOF: 46 In lung cancer which of the following features would suggest response to cytotoxic chemotherapy a) Horner’s syndrome b) Wasting of small muscles of hand c) Ectopic ACTH syndrome d) Bone metastases e) Pleural effusion Answer: c) In small cell lung cancer the ectopic ACTH syndrome may occur. Small cell lung cancer responds to cytotoxic chemotherapy

BOF: 47 Which one of the following auscultatory signs is confirmatory for bronchial breathing? a) Aegophony

22

Mastering MRCP b) Whispering pectoriloquy C) Increased vocal resonance d) Course crepitations e) Fine crepitations Answer: b) Whispering pectoriloquy is the confirmatory sign for bronchial breathing

BOF: 48 A 24-year-old male presents with sudden onset left sided pleuritic chest pain and difficulty in breathing. On examination he is distressed, tachypnoeic and has tracheal displacement to the right together with a hyperresonant percussion note on the left side with absent breath sounds on the left. In this patient: a) The lung will spontaneously expand within one week b) Immediate insertion of an intercostal drainage tube is required c) Aspiration of the pneumothorax should be undertaken in the first instance d) Surgical pleurodesis will be required e) Chemical pleurodesis will be required Answer: c) Treatment of a complete pneumothorax would be initial aspiration. This is less painful, leads to a shorter duration of admission, reduces the need for pleurectomy, there is no increase in recurrence rate at one year

BOF: 49 In a man the commonest presentation of prolactinoma is with: a) Galactorrhoea b) Gynaecomastia c) Impotence d) Adiposity e) Apathy Answer: c) In men with prolactinoma the commonest of the features mentioned above is impotence. Approximately 8% of men presenting with sexual dysfunction have hyperprolactinaemia

BOF: 50 An elderly female is admitted with loss of consciousness. She is hypothermic, has a bradycardia, evidence of cardiac failure, hypoventilation, hypoglycemia and hyponatraemia. In this condition one of the treatment strategies would be: a) Thyroxine 1000 micrograms by slow intravenous infusion every 8 hours b) Prednisolone 60 mgs orally c) Oral thyroxine 125 micrograms daily

23

Mastering MRCP d) Oral thyroxine 25 micrograms daily e) Hydrocortisone 100 mgs IV 8 hourly Answer: e) The patient has myxoedema coma. Hydrocortisone 100 mgs IV 8 hourly should be used to protect against the possibility of associated adrenocortical deficiency.

BOF: 51 The commonest clinical manifestation of primary hyperparathyroidism is: a) Renal stone disease b) Bone disease c) Peptic ulceration d) Constipation e) Polyuria Answer: a) Renal stone disease occurs in 50 % of patients

BOF: 52 A 50-year-old female has been referred for investigation of abnormal liver function tests. She consumes 52 units of alcohol per week. On examination she is obese with mainly truncal obesity, with a moon face and a buffalo hump shaped deposit of fat across her shoulders. Her face is plethoric and there are numerous telangiectasias. The abdomen is protuberant and there are striae. In this patient: a) The biochemical abnormalities do not return to normal with abstinence b) There is little relationship between the degree of liver damage and the plasma cortisol levels c) Urinary 17 – hydroxycorticosteroid levels are suppressed d) Plasma cortisol levels are suppressed e) Plasma ACTH levels are elevated Answer: b) The patient has alcoholic pseudocushings. There is little relationship between the degree of liver damage and the plasma cortisol levels

BOF: 53 A 50-year-old man presents with malaise, weight loss, diarrhoea and pain in the joints. He is pigmented, has clubbing and lymphadenopathy. He has ascites and ophthalmoplegia. Investigations show him to be anaemic. ECG shows a right bundle branch block and paracentesis abdominis reveals chylous ascites. The investigation most likely to give a diagnosis would be: a) Lumbar puncture b) CT head c) Ascitic fluid cytology d) Small bowel biopsy

24

Mastering MRCP e) Transoesophageal echocardiography Answer: d) The patient has Whipple’s disease, which is diagnosed by the demonstration of multiple macrophages in the lamina propria and the presence of rod shaped bacteria both within and without the abnormal macrophages

BOF: 54 A 32-year-old female who is known to have Crohn’s disease presents with increased frequency of micturition. She is demonstrated to have sterile pyuria. The lesion is likely to be: A) Left colonic b) Colo-vesical fistula c) Terminal ileal d) Recto-vesical fistula e) Vesico-vaginal fistula Answer: c) Crohn’s disease of the terminal ileum or left colon may involve the right ureter and result in sterile pyuria

BOF: 55 A 30-year-old male presents with several weeks history of blood and mucous diarrhoea. He has a frequency of bowel movement of 8 times during the day and has to wake up 3 times at night to have a bowel movement. Unprepared flexible sigmoidoscopy shows an inflamed colonic mucosa. A feature that will show that this patient has disease affecting the right colon would be: a) Serum albumin level of less than 30 g /l b) CRP of greater than 100 c) Thumb printing on plain abdominal x-ray d) Distended loops of small bowel e) A tachycardia of greater than 100 per minute Answer: d) The patient has ulcerative colitis. The presence of distended small bowel loops would suggest incompetence of the ileo-caecal valve indicating right colonic involvement.

BOF: 56 A 20-year-old male presents with bleeding oesophageal varices. On examination he has Kayser- Fleischer rings in the cornea. The inheritance of this disorder is: a) Autosomal dominant b) Autosomal recessive c) X-linked recessive d) X-linked dominant

25

Mastering MRCP e) Polygenic inheritance Answer: b) This patient has Wilson’s disease, which is inherited in an autosomal recessive fashion due to a mutation on the long arm of chromosome 13

BOF: 57 A 35-year-old female presents with acute severe abdominal pain associated with tachycardia, hypotension and tachypnoea. The finding of a raised serum amylase in this patient: A) is diagnostic of acute pancreatitis b) Makes ectopic pregnancy less likely c) Makes diabetic ketoacidosis less likely d) Could indicate a perforated duodenal ulcer e) Makes mesenteric ischaemia less likely Answer: d) A raised serum amylase may occur in all these conditions

BOF: 58 In primary sclerosing cholangitis: a) There is clear female predominance b) It is associated with Crohn’s colitis but not with ulcerative colitis c) The liver biopsy features are diagnostic d) Colectomy prevents the progression of the disease e) Ursodexoycholic acid improves clinical symptoms and reduces the level of cholestatic enzyme markers Answer: e) Ursodexoycholic acid improves clinical symptoms and reduces biochemical evidence of cholestasis but no effect on morbidity and mortality has been demonstrated

BOF: 59 In psoriatic arthropathy: a) There is a strong relationship between the skin changes and the severity and extent of arthritis b) There is a close relationship between the onset of nail dystrophy and arthritis c) Nail dystrophy is more common in patients with the spinal form of arthritis d) Longitudinal ridging of the nails is diagnostic of psoriatic arthropathy e) Acute anterior uveitis is common in patients with the mutilating form of psoriatic arthropathy Answer: b) There is poor correlation between the skin changes and the arthritis although there is closer relationship between nail changes and arthritis. Nail dystrophy is common in those with distal interphalangeal joint

26

Mastering MRCP involvement. Longitudinal ridging is not a specific associated feature of psoriatic arthropathy. Anterior uveitis is associated with sacroiliitis

BOF: 60 In polymyalgia rheumatica: a) The disease is common in those over 80 years of age b) A dramatic onset is unusual c) The onset is more common in the pelvic girdle d) Morning stiffness is not a common feature e) Anorexia and weight loss are features Answer: e) Polymyalgia rheumatica is common between 60 and 70 years. It is seldom seen before 45 or after 80. The onset is often dramatic. Immobility is most severe on waking and may persist for hours. Anorexia and weight loss may be striking.

BOF: 61 An elderly lady has features of osteoporosis. She appears to have lost height due to osteoporotic collapse of the vertebrae but she does not know what her former height was. Which measurement will give an indication of her former height? a) Length of manubrium sterni b) Distance between the costal margin and the iliac crest c) Diameter of the rib cage d) Waist measurement e) Arm span Answer: e) In an adult the arm span and height are approximately equal. In osteoporotic collapse of the vertebrae, the arm span becomes greater than the height

BOF: 62 In osteoporotic collapse of the vertebra: a) The absence of precipitating stress makes it less likely b) The absence of severe pain will make the diagnosis less likely c) The pain is very well localised and the absence of this symptom will exclude the diagnosis d) The vertebrae are tender to percussion and the absence of this sign will exclude the diagnosis e) Frequently occurs without symptoms Answer: e) Osteoporotic collapse of the vertebrae frequently occurs without symptoms

27

Mastering MRCP BOF: 63 The three main sites at which osteoporotic fractures occur are: a) Vertebrae, pelvis, femoral neck b) Vertebrae, femoral neck, forearm c) Vertebrae, forearm, pelvis d) Vertebrae, femoral neck, humerus e) Vertebrae, forearm, ribs Answer: b) The commonest sites of osteoporotic fracture are vertebrae, femoral neck, and forearm

BOF: 64 In contrast to other forms of osteoporosis, steroid induced osteoporosis affects the: a) Skull b) Forearm c) Vertebrae d) Pelvis e) Femoral neck Answer: a) Osteoporosis is thought not to affect the skull except in steroid induced osteoporosis

BOF: 65 A 45-year-old female is admitted with a subarachnoid haemorrhage. She initially, makes satisfactory progress but 5 days later her level of consciousness begins to deteriorate. The most likely cause of the deterioration is: a) Bacterial meningitis complicating lumbar puncture b) Cerebral abscess c) Coning of the medulla d) Acute hydrocephalus e) Dural sinus thrombosis Answer: d) Organized blood in the subarachnoid space may cause obstruction to the flow of cerebrospinal fluid and result in hydrocephalus.

BOF: 66 A 67-year-old man presents with a history of falls. He has difficulty in reading and coming down stairs. He has dysarthria, and akinesia and rigidity can be demonstrated. Power of the muscles is normal, reflexes are brisk. What physical sign will help to make the diagnosis? a) Jaw jerk b) Romberg’s sign

28

Mastering MRCP c) Tandem walking d) Eye movements e) Plantar response Answer: d) The patient has progressive supranuclear palsy and demonstration of impairment of voluntary gaze will allow one to make a diagnosis

BOF: 67 A 25-year-old female has recently had a forceps delivery. She complains of pain in the groin and on examination she has weakness of adduction and internal rotation of the hip. There is sensory impairment over the medial aspect of the thigh. The affected nerve is the: a) Femoral nerve b) Sciatic nerve c) Lateral cutaneous nerve of the thigh d) Tibial nerve e) Obturator nerve Answer: e) The obturator nerve supplies gracilis, adductor longus and brevis, adductor magnus, obturator externus and the skin over the medial aspect of the thigh

BOF: 68 A 20-year-old male presents with wasting and weakness of the muscles of the pelvic girdle. There is evidence of generalised muscular hypertrophy. His maternal grandfather had a similar disorder. The mode of inheritance is a) Autosomal dominant b) Autosomal recessive c) X-linked recessive d) Polygenic inheritance e) X-liked dominant Answer: c) The patient has benign x-liked muscular dystrophy (Becker type)

BOF: 69 Myotonia aggravated by cold is a feature of: a) Myotonia dystrophica b) Myotonia congenita c) Chondrodystrophic myotonia d) Paramyotonia e) Hypothyroidism

29

Mastering MRCP Answer: d) Myotonia appearing on exposure to cold occurs in Paramyotonia. Chondrodystrophic myotonia causes choking on cold drinks.

BOF: 70 In myasthenia gravis the tendon reflexes are: a) Brisk b) Absent c) Depressed d) Delayed e) Pendular Answer: a) In myasthenia gravis the tendon reflexes are characteristically brisk. If the reflexes are depressed or absent one should think of Eaton-Lambert syndrome.

BOF: 71 A 25-year-old male who is known to have ankylosing spondylitis presents with a painful, aching, photophobic red eye. Examination shows cells floating in the anterior chamber and precipitated on the back of the cornea. In this patient treatment should be commenced with: a) High dose oral steroids b) Broad spectrum antibiotics c) Local steroids d) Local steroids and dilator e) Local steroids and a constrictor Answer: d) The patient has anterior uveitis. Treatment should be with local steroids and a dilator to break adhesions to the lens and allow the iris to remain mobile

BOF: 72 In diabetes mellitus, eye complications that do not result in visual impairment are: a) Cataract b) Lipaemia retinalis c) Rubeosis iridis d) Retinal vein occlusion e) Diabetic retinopathy Answer: b)

30

Mastering MRCP Lipaemia retinalis is seen in association with hypertriglyceridaemia and reverses with metabolic control. All other complications impair vision. Rubeosis iridis can result in glaucoma by neovascularisation of the drainage channels of the aqueous in the anterior chamber.

BOF: 73 A 27 year old female presents with abdominal pain, weight loss, diarrhoea and mouth ulcers. On clinical examination apart from mouth ulcers no abnormality is detected. Full blood count reveals normocytic normochromic anaemia, inflammatory markers are raised, and biochemical investigation reveals a raised alkaline phosphatase. Barium follow through shows terminal ileal inflammatory disease. In this patient long term remission may be maintained by the use of: a) Low dose prednisolone b) Mesalazine c) Azathioprine d) Infliximab e) Elemental diet Answer: c) The patient has Crohn’s disease. Induction of remission of Crohn’s disease may be achieved by the use of steroids, elemental diet, surgery or Infliximab. Mesalazine may maintain remission if induction has been achieved by surgery and if an 8-week course of metronidazole is administered. In other cases the most useful drug to maintain remission is azathioprine

BOF: 74 An 80-year-old man has had a stroke and has a poor swallow. He keeps pulling his nasogastric tube out and is not adequately nourished. He develops diarrhoea. Stools were examined and show the presence of Clostridium difficile toxin. He has not been treated with antibiotics. In this patient the next course of action should be: a) Arrange a colonoscopy b) Arrange a barium enema c) Treat with metronidazole d) Treat with Loperamide e) Treat with Cholestyramine Answer: c) Infection with Clostridium difficile is usually established upon exposure to antibiotics. However, it may also occur in debilitated patients who have not been exposed to antibiotics.

BOF: 75 A 50-year-old male presents with haematemesis and melaena. He gives a history of alcohol abuse of several years duration. On examination he has bilateral parotid enlargement, spider naevi, Dupuytren’s contracture, jaundice and ascites. Whilst awaiting endoscopy the initial management of this patient should be:

31

Mastering MRCP a) Administration of intravenous pantoprazole b) Administer terlipressin c) Insert a Sengstaken tube d) Nasogastric tube and aspiration to decompress the stomach e) Administer sucralfate Answer: b) The patient has clinical evidence of cirrhosis of the liver (features of portal hypertension and features of hepatocellular failure). The likely cause of the haematemesis and melaena is bleeding oesophageal varices. Until endoscopy and definitive treatment one should attempt to reduce portal venous pressure with terlipressin

BOF: 76 The cardinal manifestation of acute hepatic failure is: a) Jaundice b) Prolonged prothrombin time c) Hepatic encephalopathy d) Ascites e) Elevated ALT Answer: c) Cerebral disturbance (hepatic encephalopathy) is the cardinal manifestation of acute hepatic failure

BOF: 77 A 45-year-old female who is known to have autoimmune hepatitis, which has progressed to cirrhosis of the liver documented on liver biopsy, complains of breathlessness. The breathlessness is better when she lies down and gets worse when she is upright. Her arterial oxygen saturation is reduced on standing. The condition that this patient suffers from is an: a) Indication for treatment with carvedilol b) Indication for treatment digoxin c) Indication for liver transplantation d) Indication for home oxygen treatment e) Indication for oral beta agonists Answer: c) Breathlessness on standing relieved by lying down (platypnoea) together with decreased arterial oxygen saturation on standing (orhtodeoxia) is features of the hepato-pulmonary syndrome which is an indication for liver transplantation

BOF: 78 A 26-year-old female who has been on the contraceptive pill presents with abdominal pain and distension of 5 days duration. On examination she has no stigmata of chronic liver disease. She has distended veins

32

Mastering MRCP over the anterior abdominal wall with flow of blood in a caudal to cephalic direction .She also has ascites, an enlarged tender liver with absent hepato-jugular reflux. Her lower limbs are oedematous. In this patient the oedema of the lower limbs is due to: a) Hypoalbuminaemia b) Hepato-renal syndrome a) Inferior vena cava thrombosis b) Portal venous thrombosis c) Lymphatic obstruction Answer: c) Budd-Chiari syndrome is thrombosis of the hepatic veins. Lower limb oedema could occur due to associated thrombosis of the inferior vena cava

BOF: 79 A 40-year-old male presents with a history of intermittent, but slowly progressive dysphagia for both solids and liquids. He experiences pain on swallowing and has regurgitation of food swallowed several hours earlier. He has no heartburn but has anorexia and weight loss. Ba swallow demonstrates proximal dilatation of the oesophagus and failure of relaxation of the lower oesophageal sphincter. In this patient good medium to long term relief of symptoms may be obtained by: a) Oesophageal myotomy b) Amyl nitrite c) Amlodipine d) Intrasphincteric botulinum toxin e) Oesophageal bouginage Answer: a) The patient has achalasia of the cardia, which is a functional obstruction at the lower oesophageal sphincter caused by a failure of relaxation. Oesophageal myotomy (Heller’s operation) and balloon dilatation give good medium to long-term results. Bouginage is not used. Drug therapy does not achieve medium to longterm relief

BOF: 80 A 22-year-old female of Irish descent has been admitted under the care of the surgeons with a history of abdominal pain. They have concluded that the patient has non-specific abdominal pain, which they define as abdominal pain that cannot be treated by an operation. No abnormality has been detected clinically or by routine investigation. They wish to discharge the patient but would welcome your opinion. You agree to see the patient in clinic following discharge but request that they perform an investigation prior to discharge. This investigation would be: a) B12 and folate levels b) Urine for porphyrins c) ANCA screen d) Mesenteric angiogram

33

Mastering MRCP e) Anti gliaden and anti endomysial antibodies Answer: e) Coeliac disease presents with abdominal pain and a coeliac screen would be a good screening test for this condition.

BOF: 81 A 55-year-old Caucasian male presents with a 2-year history of arthritis, fever, recurrent cough and chest pain. He has been feeling generally unwell. Recently he has developed diarrhoea (steatorrhoea), abdominal pain and weight loss. On examination he is pigmented, there is finger clubbing and lymphadenopathy. On auscultation of the heart a pan-systolic murmur is heard. Which of the following investigations would confirm your clinical diagnosis? a) ECHO b) Blood cultures c) Lymph node biopsy d) Small bowel biopsy e) Mesenteric angiogram Answer: d) The patient has Whipple’s disease, which may be confirmed by small bowel biopsy. Small bowel biopsy will show large, foamy PAS positive macrophages in the lamina propria

BOF: 82 A 35-year-old male intravenous drug abuser presents to his GP with a complaint of severe fatigue and a blistering eruption on the sun exposed areas of his skin. He also abuses alcohol consuming between 30-40 units a week. The GP carries out blood test and refers him for investigation of abnormal liver function tests. In this patient: a) Combination therapy will induce a response in 90% of patients b) Development of cirrhosis does not preclude a good response to treatment c) If the patient develops cirrhosis he has an 80-90% chance of developing hepatocellular carcinoma. d) Without treatment end stage liver failure will develop in 2-3 years e) In patients without cirrhosis, combination therapy induces a response in 45% of patients Answer: e) The patient has hepatitis C with associated porphyria cutanea tarda. Significant liver disease develops in 2030 % of patients who have necro-inflammatory disease. The process takes 20-30 years. Development of cirrhosis results in a poor response to combination therapy (Interferon and ribavarin) although the newer PEGylated interferons yield a better virological response. Once cirrhosis is established the incidence of hepatocellular cancer is 1.5-2 % per year

34

Mastering MRCP BOF: 83 A 50-year-old male has been seen by his GP who has made a diagnosis of diabetes mellitus. On routine investigation he has been noted to have abnormal liver function tests. The GP refers him to the clinic querying non-alcoholic fatty liver disease. On examination he is pigmented, there is loss of body hair, gynaecomastia, testicular atrophy, and an arthropathy of his knee joints. In this patient treatment of the underlying condition will: a) Increase the severity of the diabetes mellitus b) Increase the size of the liver c) Reverse any changes of cirrhosis d) Only results in decrease in malaise and decreases liver size e) Improves 5- year survival rate Answer: e) The patient has haemochromatosis. Venesection is the best method of depleting body iron stores. It decreases the severity of diabetes mellitus and results in a general improvement in malaise and decreases liver size. The changes of cirrhosis are not reversible. Five-year survival has improved from 4.4 years in 1935 to 89% in 1969.

BOF: 84 A 40-year-old female presents with hepatitis. She also has arthralgia and autoimmune thyroid disease. Investigations reveal high serum transaminases, increase in total globulin and smooth muscle antibodies are positive. Where treatment of this condition is concerned: a) Steroids are contraindicated as they increase the risk of osteoporosis b) Steroids reduce the enzyme levels and decrease jaundice but are of no long term benefit c) Steroids improve survival rate, but do not prevent development of cirrhosis d) Steroids should be used continuously for 4 years e) Azathioprine is as effective as steroids and can be used instead of steroids in older females Answer: c) The patient has autoimmune hepatitis. In this condition response to steroids is excellent and improves fiveyear survival but does not prevent development of cirrhosis. Azathioprine is a useful adjunct to steroids and allows a lower dose of steroids to be used and thus reduces the risk of osteoporosis. If the disease is inactive for 2 years steroids may be cautiously withdrawn. Relapse is common (60-80 %) and necessitates reintroduction of steroids.

BOF: 85 A 50-year-old female presents with weakness of her left upper limb, which developed overnight. She says she fell asleep on a chair. On examination there is weakness of extension of her left elbow, a wrist drop and absent sensation over the first interosseus space of her left hand on the dorsal aspect. The lesion is in the: a) Ulnar nerve at the elbow b) Musculocutaneous nerve

35

Mastering MRCP c) Radial nerve in the spiral groove d) Radial nerve in the axilla e) Upper brachial plexus Answer: d) The weakness of extension of the elbow indicates weakness of the triceps hence the lesion should be in the axilla. Lesions of the radial nerve in the spiral groove spare the triceps

BOF: 86 A 32-year-old female who has had surgery on her cervical spine following a car accident presents with sudden onset of left-sided weakness. The weakness developed when she looked up whilst bending to pick up an object. On examination she has a hemiplegia affecting the left face arm and leg. She also has unilateral internuclear ophthalmoplegia with failure of adduction to the left and nystagmus to the left. She also has early papilloedema. In this patient MRI shows: a) Right mid brain infarct b) Left mid brain infarct c) Right pontine infarct d) Left pontine infarct e) Lateral medullary syndrome Answer: a) The patient has unilateral internuclear ophthalmoplegia. The side of the lesion is the side of the failure of adduction not the side of the nystagmus. This localises the lesion to the right medial longitudinal fasiculus. To cause weakness of face arm and leg the lesion must be above the pons

BOF: 87 A 32-year-old female who has had surgery on her cervical spine following a car accident presents with sudden onset of left-sided weakness. The weakness developed when she looked up whilst bending to pick up an object. On examination she has a hemiplegia affecting the left face arm and leg. She also has unilateral internuclear ophthalmoplegia with failure of adduction to the left and nystagmus to the left. She also has early papilloedema. In this patient the papilloedema is due to: a) Obstruction of the foramen of Magendie b) Obstruction of the foramen of Luschka c) Obstruction of the aqueduct of Sylvius d) Obstruction of the foramen of Monro e) Obstruction of the foramen of Morgagni Answer: c) The aqueduct of the midbrain (the aqueduct of Sylvius) runs in the tegmentum of the midbrain and joins the third and fourth ventricles. Oedema around the midbrain infarct would have compressed the aqueduct and resulted in obstructive hydrocephalus and papilloedema

36

Mastering MRCP BOF 88 A 57-year-old female who is known to have rheumatoid arthritis has been on penicillamine for treatment of her condition. She presents with oedema, proteinuria, hypoalbuminaemia and her serum cholesterol levels are elevated. In this patient renal biopsy: a) Is indicated as she may have developed a vasculitis b) Is indicated to decide if she needs steroids and immunosuppression c) Should be performed in order to demonstrate minimal change nephropathy as this will respond to steroids d) Is indicated to look for amyloidosis e) Is not indicated Answer: e) The patient has nephrotic syndrome. In patients on drugs such as penicillamine it is best to stop the drugs and assess response first, rather than proceed to renal biopsy. In nephrotic syndrome, renal biopsy is not indicated in: Young children, especially males, with a highly selective protein leak, no hypertension, no red cells or red cell casts in the urine Long standing insulin dependent diabetes mellitus with associated retinopathy or neuropathy as here the most likely diagnosis is diabetic nephropathy Patients on drugs, which should be stopped first

BOF: 89 A 60-year-old male with uncomplicated stable alcoholic cirrhosis presents with severe breathlessness on exertion. On examination he was found to be anaemic and jaundiced. He has splenomegaly. His Hb is 8 gm/dl with normal MCV. WBC and platelet count are normal. There is no evidence of blood loss. Iron studies and red cell folate assay are normal. What is the most likely explanation for his anaemia? a) Burr cell anaemia b) Paroxysmal cold haemoglobinuria c) Autoimmune haemolytic anaemia d) Spur cell anaemia e) Hypersplenism Answer: d) Spur cell anaemia should be suspected when the anaemia is more severe than is observed in otherwise uncomplicated cirrhosis. Splenomegaly is always present. The RBC is irregularly shaped with multiple spicules. The surface membrane of a spur cell contains 50 – 70% excess cholesterol. Spur cells are distinguished from regularly spaced cremated RBC (Burr cells), which are present in some patients with uremia. It is not hypersplenism because his white cells and platelets are normal

37

Mastering MRCP BOF: 90 Which of the following conditions is most likely to be associated with Paroxysmal Cold Haemoglobinuria? a) Mycoplasma infection b) Lymphoma c) Systemic lupus erythematosus d) Tertiary syphilis e) Chronic lymphocytic leukemia Answer: d) This is a rare disorder now. It was more frequent when tertiary syphilis was prevalent. Now most cases are either secondary to a viral infection (measles and mumps in children) or are autoimmune. Paroxysmal Cold Haemoglobinuria results from the formation of the Donath-Landsteiner antibody, an IgG antibody that is directed against the P antigen. This can induce complement-mediated lysis. Attacks are precipitated by exposure to cold and are associated with haemoglobinemia and haemoglobinuria

BOF: 91 A 40-year-old male was admitted with sudden onset headache and generalised tonic-clonic convulsion. MRI scan and subsequent MRI venography revealed sagittal sinus thrombosis. He recently recovered from an episode of aplastic anaemia. Investigation reveals anaemia with reticulocytosis. What is the most likely diagnosis? a) Homocystinuria b) Thrombotic thrombocytopenic purpura c) Paroxysmal Cold Haemoglobinuria d) Paroxysmal Nocturnal Haemoglobinuria e) Thalassaemia Answer: d) Paroxysmal Nocturnal Haemoglobinuria (PNH) is an intracorpuscular defect acquired at the stem cell level. Three common manifestations are haemolytic anaemia, venous thrombosis and deficient haematopoiesis. Granulocytopaenia and thrombocytopaenia are common and reflect deficient haematopoiesis. Clinical haemoglobinuria is intermittent in most patients and never occurs in some, but haemosiderinuria is usually present. Venous thrombosis is a common complication of patients of European origin. Thrombosis can occur in cerebral venous sinuses and is a common cause of death in a patient with PNH.

BOF: 92 Thrombocytosis does not occur in: a) Essential Thrombocytosis b) Systemic Lupus Erythematosus c) Inflammatory Bowel Disease d) Acute Myeloid Leukemia e) Hyposplenism Answer: d)

38

Mastering MRCP Acute Myeloid Leukemia (AML) causes thrombocytopaenia. Chronic Myeloid Leukemia can cause thrombocytosis. Idiopathic sideroblastic anaemia and Myelodysplasia can cause thrombocytosis. Essential thrombocytosis is a clonal disorder of unknown aetiology and manifests clinically by the overproduction of platelets. It is often identified incidentally. Patients with Essential Thrombocytosis do have haemorrhagic and thrombotic tendencies

BOF: 93 Which of the following statements is not true about Waldenstrom’s Macroglobulinaemia? a) Hypercalcaemia is common b) Renal disease is not common c) Peripheral neuropathy is not uncommon d) Hepatosplenomegaly can occur e) Lymphadenopathy is not uncommon Answer: a) Waldenstrom’s Macroglobulinaemia is a malignancy of lymphoplasmacytoid cells. The disease is associated with lymphadenopathy, hepatosplenomegaly and the hyperviscosity syndrome. The disease involves the bone marrow, but doesn’t cause lytic bone lesions or hypercalcaemia. IgM paraprotein has got very little excretion through urine because of its size. Therefore renal disease is not common

BOF: 94 Alopecia is not a feature of: a) Secondary Syphilis b) Hypothyroidism c) Hypopituitarism d) Hyperthyroidism e) Hypoparathyroidism Answer: e) Other systemic diseases that cause protein/iron/zinc/biotin, HIV infection

BOF: 95 Hyperpigmentation is not a manifestation of: a) Addison’s disease b) Vitamin B12 deficiency c) Pellagra d) Whipple’s disease e) Systemic Lupus Erythematosus Answer: e)

alopecia

are

lupus

erythematosus,

deficiencies

of

39

Mastering MRCP Scleroderma can cause hyperpigmentation. Other conditions causing pigmentation are Nelson syndrome, Porphyria Cutanea Tarda, Haemochromatosis, Folate deficiency, Malabsorption, Biliary Cirrhosis, Eosinophilia-myalgia syndrome and POEMS syndrome

BOF: 96 The following statements about Dermatitis Herpetiformis is false: a) Papulovesicular lesions over the extensor surfaces are common b) The rash is non-pruritic c) Almost all patients have associated sub-clinical gluten-sensitive enteropathy d) IgA is deposited in the skin e) Increased incidence of thyroid abnormalities are found Answer: b) Dermatitis Herpetiformis is intensely pruritic. Most patients with dermatitis herpetiformis do not report overt gastrointestinal symptoms or laboratory evidence of malabsorption. They also have increased incidence of thyroid abnormalities, achlorhydria, atrophic gastritis and antigastric parietal cell antibody. Dapsone is the initial treatment with institution of a gluten free diet.

BOF: 97 A 55-year-old male presents with acute bilateral visual loss. Both the optic discs are swollen and a central scotoma is detected. In further investigation of the cause of this condition the following blood levels are not required: a) Ethylene glycol b) Thiamine c) Vit.B12 d) Folate e) Vitamin A Answer: e) The diagnosis is Toxic Optic Neuropathy. This can happen from exposure to ethambutol, methyl alcohol, ethylene glycol or carbon monoxide. Deficiency states induced either by starvation, malabsorption or alcoholism can lead to insidious visual loss. Thiamine, vitamin B12 and folate level should be checked in any patient with unexplained bilateral central scotoma and optic pallor

BOF: 98 Which of the following is not a clinical sequel of rhabdomyolysis: a) Hypovolaemia b) Metabolic alkalosis c) Hyperkalaemia d) Acute renal failure E) Disseminated intravascular coagulation Answer:

40

Mastering MRCP b) Metabolic acidosis occurs in rhabdomyolysis due to release of cellular phosphate and sulphate.

BOF 99 Which one of the following tumours, in advanced stages, is not poorly responsive to chemotherapy? a) Pancreatic carcinoma b) Hypernephroma c) Gall Bladder carcinoma d) Ovarian carcinoma e) Non-small cell lung carcinoma Answer d) Other cancers that are poorly responsive to chemotherapy are thyroid carcinoma, carcinoma of vulva, colorectal carcinoma, prostate carcinoma, melanoma and hepatocellular carcinoma

BOF: 100 A 22-year-old male presents with gradual onset loss of central visual field in the right eye. He also had the same process in the left eye 3 months back. Ophthalmoscopy reveals bilateral optic atrophy. What is the possible diagnosis? a) Leber’s Optic Neuropathy b) Central Retinal Artery Occlusion c) Glaucoma d) Central Retinal Vein Occlusion e) Sarcoidosis Answer: a) This is a disease of young men. Visual loss is painless and gradual onset. There is no treatment. Curious male predilection is a mystery

BOF: 101 A 75-year-old male presents with lethargy and dizziness on standing. He gives a history of having had treatment for tuberculosis when he was young. On examination he looks underweight, his skin is pigmented and there is pigmentation of his buccal mucosa and pigmentation of the palmar creases. His nipples and areola are pigmented. There is no vitiligo. His blood pressure is 120/70 lying down and 90/60 on standing. His blood urea 10.3 mmol/L, Na 120 mmol/L, K 5.9 mmol/L Which of the following investigations is most likely to identify the aetiology of his condition? a) 0900 ACTH level b) Short synacthen test c) Abdominal X-ray d) Long ACTH stimulation test e) Serum aldosterone /plasma renin activity

41

Mastering MRCP Answer: c) The patient has Addison’s disease. With the history of tuberculosis the likely aetiology is tuberculosis of the adrenal gland and this may result in calcified adrenals

BOF: 102 A 60-year-old female is admitted with a history of syncopal episodes and breathlessness of one-week duration. No history of immobilisation or recent travel. On examination she looks distressed her conjunctiva and tongue are pale. Her pulse rate is 120 beats per minute, blood pressure 90/60, JVP elevated 5 cms. Respiratory rate 26 per minute, apex 5 LICS MCL heart sounds triple rhythm, soft systolic murmur at the left sternal edge. Lungs clear. HB 13.5 g/dL Blood gases pH 7.46 Pa CO2 3.2 kPa, Pa O2 7.7 kPa The next step in management of this patient is: a) Immediate thrombolysis followed by an ECHO to confirm the diagnosis b) Immediate thrombolysis followed by computerized tomographic pulmonary angiography (CTPA) to confirm the diagnosis c) Immediate thrombolysis followed by ventilation perfusion scan to confirm the diagnosis d) Urgent CT pulmonary angiography followed by thrombolysis e) Full dose fragmin followed by ventilation perfusion scan Answer: d) The clinical features in this patient suggest massive central pulmonary embolus. It is best to obtain an urgent CT PA or ECHO prior to thrombolysis. In a cardiac arrest or peri-arrest situation it may be justifiable to thrombolyse the patient and confirm the diagnosis by CTPA or ECHO if the patient survives

BOF: 103 A 52-year-old female presents with numbness and weakness of her upper and lower limbs. She developed asthma at the age of 50 and her general practitioner treated her with steroids and antibiotics approximately one month prior to the onset of these symptoms. On examination she looks unwell. She has palpable purpura over her face and over her elbows and knees. On neurological examination she has a left sided wrist drop and there is weakness of dorsiflexion of her right hallux and weakness of dorsiflexion of her right foot Sensation is impaired over the dorsum of her right foot. Lungs are clear. CXR is normal Hb 10.9 g/dL MCV 88 fl WBC 23,000 10 9 /l Eosinophils 12% ANCA negative In this patient abdominal pain could be due to: a) Pancreatic infarction b) Gall bladder infarction

42

Mastering MRCP c) Vasculitis of the gastrointestinal tract d) Renal infarction e) Splenic infarction Answer: c) The patient has Churg-Strauss syndrome. The diagnosis may be made on the basis of late onset asthma, vasculitic rash and mononeuritis multiplex. ANCA is positive in approximately 60 % of the patients hence a negative ANCA does not rule out the diagnosis. In this condition vasculitis of the gastrointestinal tract may result in mass lesions that cause intestinal obstruction

BOF: 104 A 45-year-old female presents with an 8-week history of diarrhoea with no blood or mucous, colicky abdominal pain, vomiting and loss of weight of 10 kgs. She has a history of appendicectomy and has fibromyalgia for which she has been on Diclofenac SR 75 mgs b.d. On examination she was afebrile and looked pale. The abdomen was diffusely tender but soft. Bowel sounds were normal and rectal examination was normal. Investigations showed: Hb 7.5 g/dL, MCV 70 fl CRP 235, coeliac screen negative LFTs normal, urea and electrolytes normal Colonoscopy revealed several webs with superficial ulceration in the ascending colon. Biopsies from these areas showed non-specific inflammation. In this patient, the most important aspect of treatment is: a) Discontinue Diclofenac b) Treat with Steroids c) Treat with Mesalazine d) Treat with Azathioprine e) Treat with Ciprofloxacin Answer: a) The patient has non-steroidal anti-inflammatory drug colopathy. This can occur after short or long term treatment with NSAIDS. These patients should be warned against any further use if NSAIDS

BOF: 105 A 30-year-old female is under investigation for excessive weight gain. She is 1.62 m tall and weighs 80 kgs. She has central obesity with abdominal striae. She is hirsute. A dexamethasone suppression test has been performed and the results are as follows: Cortisol levels nmol/L Dexamethasone Day 0 0900 990 8mgs Day 1 0900 320 8mgs Day 2 0900 130

43

Mastering MRCP

The patient has: a) Cushing’s disease b) Adrenal tumour c) Bronchial carcinoma d) Simple obesity e) Polycystic ovarian syndrome Answer: a) In the dexamethasone suppression test normal individuals suppress cortisol levels to < 50 nmol/L 90 % of patients with pituitary dependent disease suppress production of cortisol to < 50 % of that on day 0 on day +2 Failure of suppression suggests ectopic ACTH production or adrenal tumour

BOF: 106 A 14-year-old male has had a cardiac catheterization to document the diagnosis of an abnormality of his heart. The results are as follows: Chamber Superior vena cave Inferior vena cave Right atrium Right ventricle Pulmonary artery Left atrium Left ventricle Femoral Artery

Pressure in mm Hg

10 25/0 25/10 10 110/0 110/50

% Oxygen saturation 68 64 82 81 81 96 95 95

In this patient, on auscultation of the heart one may hear: a) Loud single second heart sound b) Soft single second heart sound C) Fixed splitting of the second heart sound d) Reversed splitting of the second heart sound e) Varying split of the second heart sound Answer: c) The patient has an atrial septal defect. In this condition fixed splitting of the second heart sound occurs.

BOF: 107 A 45-year-old female presents with backache. She has had a thyroidectomy for thyrotoxicosis 18 months previously. She is a vegan. Her blood results are as follows:

44

Mastering MRCP Hb 10.0 g/dL Ca 1.8 mmol/L PO4 0.72 mmol/L Alkaline phosphatase 200 U/L Albumin 36 g/L Urea 6.5 mmol/L Sodium 140 mmol/L Potassium 3.8 mmol/L The test that will define this patient’s condition is: a) PTH level b) Serum 25-hydroxy vitamin D3 level c) Urinary calcium excretion d) Urinary phosphate excretion e) Iliac crest biopsy Answer: b) The patient has osteomalacia with secondary hyperparathyroidism causing low phosphate levels

BOF: 108 A 14-year-old male, a recent immigrant to the country is been investigated for a murmur. On examination he is in sinus rhythm, he has a loud pan systolic murmur best heard at the left sternal edge and a mid-diastolic murmur best heard at the apex. The first heart sound is not loud; there is no opening snap and no pre-systolic accentuation of the diastolic murmur. Cardiac catheterisation results are as follows Chamber Superior vena cave Inferior vena cave Right atrium Right ventricle Pulmonary artery Left ventricle

Pressure in mm Hg

3.5 35/0 35/10 100/0

% Oxygen saturation 66 68 67 80 81 96

In this patient the mid-diastolic murmur is due to: a) Increased flow across the mitral valve b) Increased flow across the tricuspid valve c) Mitral stenosis d) Tricuspid stenosis e) Lutembacher’s syndrome Answer: a) The patient has a ventricular septal defect. This can be deduced from the step up in oxygen saturation at the level of the right ventricle due to the left to right shunt at this level. This causes increased flow into the left atrium and this causes a flow murmur in mid-diastole.

45

Mastering MRCP Stenosis of the mitral valve would result in a loud first heart sound, opening snap and presystolic accentuation of the murmur. Tricuspid stenosis would have caused a high pressure in the right atrium. Lutembacher’s syndrome is a combination of atrial septal defect and mitral stenosis.

BOF: 109 A 60-year-old male is under investigation for weakness of his legs, pigmentation of the skin, hypertension and glycosuria. Results of serum cortisol estimations are; 0900 1170 nmol/l 2400 1100 nmol/l After 48 hours of dexamethasone 8 mgs daily: 0900 1030 nmol/l In further investigation of this patient the test most likely to identify the cause of his condition is: a) MRI head b) CT scan head c) Perimetry to define visual fields d) Chest x- ray e) ACTH levels Answer: d) The patient has Cushing’s syndrome with failure to suppress cortisol levels with dexamethasone. This will indicate either an adrenal tumour or an ectopic source of ACTH. Out of the list provided the most likely investigation that will determine the cause of the lesion is CXR

BOF: 110 A 31-year-old female presents with a severe headache, which woke her up from sleep at approximately 3 a.m. She has never had a headache like this before. She had a mastoidectomy several years ago. She has no other symptoms and was otherwise well; she is not pregnant and was on no drugs in particular she was not on hormonal contraception. She had not undertaken any severe exercise. On examination she was afebrile, conscious and alert. On examination of the optic fundus the cup of the optic disc was filled and the medial margins of the disc were blurred. No other CNS abnormality in particular no neck stiffness. Examination of the other systems did not reveal any abnormality. A CT scan was reported as normal by the duty radiology registrar, Full Blood Count was normal. In this patient the investigation that is likely to lead to a diagnosis is: a) MR venogram b) Lumbar puncture c) Vitamin A level d) Cisternal puncture e) E.S.R. Answer: a) The patient has sagittal sinus thrombosis. This can be secondary to Thrombophilia, which may be primary or secondary to diseases such as nephrotic syndrome, paroxysmal nocturnal haemoglobinuria. It can also

46

Mastering MRCP occur in pregnancy and in patients taking hormonal contraception. Dehydration is another cause. It may be secondary to sinusitis and mastoiditis or it may be idiopathic. In this patient the treatment of choice would be: a) Heparin b) Steroids c) Vitamin A d) Acetazolamide e) Repeated lumbar puncture Answer: a) Sagittal sinus thrombosis is best treated with heparin. There are reports in the literature of successful thrombolysis by local instillation of thrombolysis

BOF: 111 A young female who is short, plump and has a rounded facies has the following haematology and biochemical profile: Hb 12.8 g/dl Serum Calcium 1.62 mmol/l Serum Phosphate 2.9 mmol/l Creatinine 44 micromoles/l Albumin 38 g/l Which radiological abnormality is likely in this patient? a) Defective mineralisation of the pelvis B) Looser’s zones c) Nephrocalcinosis d) Subperiosteal erosions in the middle or terminal phalanges of the hands e) Short fourth and fifth metacarpals Answer: e) The patient has pseudohypoparathyroidism, which is associated with short metacarpal bones Looser’s zone are linear areas of low density surrounded by sclerotic borders. This and defective mineralisation of the pelvis are x-ray features of osteomalacia. Nephrocalcinosis and subperiosteal erosions occur in hyperparathyroidism

BOF: 112 A 60 year old male presents with backache and discomfort in his pelvis. He is a widower and lives alone in a small flat. Investigations are as follows: Serum calcium 1.9 mmol/l Serum phosphate 0.6 mmol/l Alkaline phosphatase 140 U/l Urea 5.5 mmol/l Albumin 36 g/l

47

Mastering MRCP In this patient the following x-ray changes are likely: a) Calcification of the basal ganglia b) Pseudofractures of the pubic rami c) Osteoporosis circumscripta d) Rounded lytic lesions in the skull e) Nephrocalcinosis Answer: b) The patient has osteomalacia. The x-ray feature that may occur in this condition is pseudofracture of the pubic rami. Calcification of the basal ganglia is a feature of hypoparathyroidism. Osteoporosis circumscripta refers to a front of resorbing bone in the skull. It occurs in Paget’s disease of bone. Rounded lytic lesions in the skull occur in myeloma. Nephrocalcinosis is a feature of hyperparathyroidism

BOF: 113 A 74-year-old male who is known to have ischaemic heart disease and has had a myocardial infarction presents with breathlessness. His extremities are cold and clammy; his pulse rate is 120 beats per minute, blood pressure 80/40. JVP elevated, heart sounds triple rhythm, crepitations are heard in both bases Investigations show: pH 7.24 Pa O2 8.5 kPa Pa CO2 4.4 kPa Blood lactate 5.9 mmol/l Urine pH 5.2 Urine osmolarity 320mmol/l Blood glucose 8.6 mmol/l In this patient the abnormal acid-base balance is due to: a) Aspirin overdose b) D-lactic acidosis c) Type A L-lactic acidosis d) Type B L-lactic acidosis e) Diabetic ketoacidosis Answer: c) Type A L-lactic acidosis occurs when excess lactic acid is formed by anaerobic metabolism. Type B L-lactic acidosis is due to decreased hepatic lactate metabolism caused by insulin deficiency, drugs such as biguanides, haematological malignancies, enzyme deficiencies. D-lactic acidosis is caused by bacterial fermentation in the gut in the short bowel syndrome

48

Mastering MRCP BOF: 114 A 56-year-old female presents with a history of increasing tiredness. On examination there is pigmentation of her skin creases and buccal mucosa. Her blood pressure is 90/50. Investigations are as follows: Blood urea 8.4 mmol/l Na 130 mmol/l Potassium 6.1 mmol/l Chloride 96 mmol/l Bicarbonate 23 mmol/l Which of the following tests will confirm the diagnosis? a) Random cortisol measurement b) Short ACTH stimulation test c) Long ACTH stimulation test d) 0900 Plasma ACTH level and cortisol measurement e) Abdominal x-ray Answer: d) This patient has Addison’s disease. A high 0900 h plasma ACTH level with low or normal cortisol will confirm the diagnosis of primary hypoadrenalism The short ACTH (synacthen) test would show that the adrenal gland is not responding to ACTH. This may be due to primary hypoadrenalism or secondary to adrenal suppression by steroids or ACTH deficiency. Hence it will not confirm the diagnosis. She does not have adrenal suppression by steroids or ACTH deficiency as her skin creases and buccal mucosa are pigmented. Hence, a long ACTH stimulation test is not the answer

BOF: 115 A young female has the following somatic features: Short stature, round face, short neck, short fourth and fifth metacarpals and metatarsals Her investigations are as follows: Serum calcium 2.2 mmol/l Serum phosphate 1.2 mmol/l Alkaline phosphatase 100 U/L Urea 4.0 mmol/L Creatinine 90 mmol/L The diagnosis is: a) Pseudohypoparathyroidism b) Hypoparathyroidism c) Rickets D) Vitamin D resistant rickets e) Pseudopseudohypoparathyroidism Answer: e) This patient has the somatic features of pseudohypoparathyroidism but biochemistry is normal indicating pseudopseudohypoparathyroidism

49

Mastering MRCP BOF: 116 A 50-year-old male who is known to have chronic bronchitis presented to the accident and emergency department. After initial treatment he was sent up to the ward. His blood gases on admission to the ward were as follows: pH 7.22 PaO2 14.4 kPa PaCO2 18.8 kPa The physical sign that alerted the examining clinician to this condition was: a) Tachycardia b) Pulsus paradoxus c) Flapping tremor d) Elevated JVP e) Fourth heart sound Answer: c) A flapping tremor is indicative of hypercapnoea

BOF: 117 A 35-year-old female has been on treatment for depression. She complains of constipation. Her blood results are as follows: Serum calcium 2.77 mmol/l TSH 20 mU/l Which drug is responsible for these features? a) Amitryptiline b) Fluoxetine c) Venlafaxine d) Mirtazepine e) Lithium Answer: e) Lithium can cause hypercalcaemia and hypothyroidism. The other effects are fine tremor, metallic taste in the mouth, weight gain, oedema, goiter, acne rash, diabetes insipidus, and cardiac arrhythmias

BOF: 118 An 18-year-old female presents to casualty with abdominal pain. On examination she is in distress, she has a pulse rate o f 130 beats per minute her blood pressure is 80/50 her respiratory rate is 38 per minute. Abdominal examination revealed generalized tenderness. Her blood results are as follows: Sodium 110 mmol/l Potassium 5.5 mmol/l Urea 11 mmol/l Which of the following tests will determine the cause of this patient’s condition?

50

Mastering MRCP a) Blood gases b) Triglycerides c) Blood sugar d) Salicylate levels e) Amylase Answer: c) The patient has presented with diabetic ketoacidosis. The blood sugar will determine the cause of the condition

BOF: 119 Your junior asks for advice regarding blood results on a 45-year-old female who is a known hypertensive and is complaining of polyuria, polydypsia and muscle weakness. Blood pressure 170/110. The results are: Potassium 2.5 mmol/l Arterial pH 7.49 The next question you would ask is: a) Is there a family history of hypertension? b) Does the patient have a bruit over the renal artery? c) Is the patient on a diuretic? d) What is the ratio of aldosterone to plasma rennin activity? e) Does the patient have papilloedema? Answer: c) Hypertension, hypokalaemia and alkalosis should make one suspect Conn’s syndrome if the patient is not on diuretics. Hence this is the most important question to answer. Renal artery stenosis also causes hypertension with hypokalaemia and is a more common cause of refractory hypertension with low potassium. Secondary hyperaldosteronism may be due to Renal artery stenosis Accelerated hypertension Diuretics Heart failure Liver failure

BOF: 120 A 56-year-old male presents with a history of impotence, decreased facial hair and a discharge from his nipples. A basal plasma prolactin, taken at non-stressful venepuncture, is raised. The patient also has gastro-oesophageal reflux disease and hypertension and is on a number of drugs. Which of the following drugs could cause a raised prolactin level? a) Omeprazole b) Metoclopramide c) Lisinopril

51

Mastering MRCP d) Bendrofluazide e) Atenolol Answer: b) The drugs that may increase prolactin levels are: Phenothiazines, haloperidol, metoclopramide, methyldopa, oestrogens, TRH Which of the following diseases does not cause an increase in prolactin levels? a) Non-secretory pituitary adenoma b) Chronic renal failure c) Hypothyroidism d) Sarcoidosis e) Chronic obstructive pulmonary disease Answer: e) The diseases that may cause an increase in prolactin levels are: Prolactinoma, pituitary adenoma, pituitary stalk section, hypothalamic disease, hypothyroidism, chronic renal failure, sarcoidosis Pituitary stalk section causes loss of inhibition of prolactin production by dopamine. Similarly large pituitary adenomas can cause loss of inhibition of prolactin production

BOF: 121 A patient presents with tiredness, dizziness, headache, visual disturbance, impotence and decreased facial hair. On examination he has a blood pressure of 110/70 lying 80/ 60 standing. On examination of his visual fields he is found to have a bitemporal hemianopia. In this patient the prolactin level is 4000 mIU/L. An insulin tolerance test is performed and the blood glucose levels falls to 1.2 mmol/L and the patient becomes sweaty and tachycardic. Blood taken at this time would show: a) Absent cortisol response b) Exaggerated cortisol response c) Increased thyroxine release d) Attenuated thyroxine release e) Exaggerated growth hormone release Answer: a) The combination of headache, bitemporal hemianopia and the prolactin level of > 3,600 mIU/L suggests the patient has a macroadenoma. Chromophobe adenoma and acidophil adenoma can produce prolactin. This patient is likely to have an impaired response to the insulin tolerance test as prolactinoma may cause hyposecrestion of other hormones. It is likely that this patient has corticotrophin deficiency as he has features that are suggestive of this (tiredness, dizziness, low blood pressure with postural drop)

BOF: 122 A 57-year-old male presents with an acute confusional state. This was preceded by a few days of headache, nausea and vomiting and visual disturbance. His GP had been treating him with an antihistamine as he had

52

Mastering MRCP complained of itching over the last few months. The GP had also treated him a few weeks earlier for a swollen, hot, painful joint. He had prescribed NSAIDS and antibiotics. On examination he was overweight; he looked plethoric, and cyanosed. There was no clubbing. No murmurs, lungs clear. Liver was enlarged 3 fingerbreadths and the spleen was enlarged about 5 fingerbreadths. No focal neurological signs. On examination of the optic fundus engorged veins were seen. Investigations: Hb 22.4 g/dl MCV 73 fl WBC 20.3 Na 138 mmol/l K 3.8 mmol/l Urea 5.6 mmol/l Creatinine 90 micromoles/l Chloride 97 mmol/l Bicarbonate 30 mmol/l Calcium 2.32 Albumin 37 g/l Phosphate 1.09 mmol/l ESR 1 mm/1st hour In this patient your course of action should be: a) Blood gas analysis followed by oxygen therapy b) Bone marrow examination c) Venesection d) Blood cultures followed by broad spectrum antibiotics until cultures are available e) CT scan and lumbar puncture Answer: c) This patient has polycythaemia rubra vera. He has pruritus, gout and he has presented with a hyperviscosity syndrome. The immediate management should be venesection and appropriate fluid replacement

BOF: 123 A 67-year-old female presents with anorexia, nausea vomiting and loss of weight. No other symptoms of note. On examination she was mildly dehydrated and looked unwell. Investigations: Hb 16.6 g/dl WBC 4.7 x 109/l ESR 13mm in the 1st hour Sodium 136 mmol/l Potassium 3.9 mmol/l Chloride 111 mmol/l Bicarbonate 27 mmol/l Urea 10.2 mmol/l Creatinine 126 micromoles/l Albumin 44 g/l

53

Mastering MRCP Calcium 3.87mmol/l Phosphate 0.62 mmol/l Alkaline phosphatase 120 U/l Which of the following is an unlikely cause for her hypercalcaemia? a) Thyrotoxicosis b) Malignant disease c) Hyperparathyroidism d) Vitamin D therapy e) Renal failure Answer: a) Severe hypercalcaemia (Serum calcium > 3 mmol/l) may be associated with malignant disease, hyperparathyroidism, renal failure and vitamin D therapy Thyrotoxicosis and Addison’s disease may cause hypercalcaemia but it is usually mild. What is the best emergency treatment for this patient? a) Intravenous Bisphosphonates b) Steroids c) Calcitonin d) Intravenous phosphate e) Oral phosphates Answer: a) Intravenous bisphosphonates are now considered the treatment of choice for severe hypercalcaemia. Adequate rehydration is an essential prerequisite. Calcitonin has a short-lived action and is not used. Prednisolone may be effective in myeloma, sarcoidosis, and Vitamin D excess but is ineffective in most case. Oral phosphate causes diarrhoea intravenous phosphate is dangerous and is not used for hypercalcaemia

BOF: 124 A 30-year-old Chinese patient presents with a history of weakness of both lower and upper limbs. The weakness started overnight in his lower limbs and spread to involve the upper limbs as well. The patent had been well till then and had not had any diarrhoea or vomiting. He had not lost any weight. There were no significant past illnesses. On examination he looked well, fully orientated in place, time and person. No evidence of weight loss, temperature 37°C. Pulse rate 65 beats per minute regular. BP 120/60. No rash, skin warm and dry. No lymphadenopathy. On neurological examination of his limbs tone was normal there was weakness of all four limbs, reflexes were diminished, plantar downgoing, no sensory loss. Investigations FBC normal ESR not elevated LFTs normal Glucose 7.5 mmol/l Na 140 mmol/l

54

Mastering MRCP K 1.6 mmol/l Chloride 104 mmol/l pH 7.39 In this patient, whilst awaiting the results of his blood tests, what bedside investigation would give you a clue to the cause of the patient’s weakness? a) ECG b) Tidal volume c) Urine for ketone bodies d) Urine analysis e) Specific gravity of urine Answer: a) The patient has hypokalaemic periodic paralysis. The ECG would show U-waves. What other investigation would you arrange for this patient to determine the aetiology of his condition? a) Thyroid function tests b) Overnight dexamethasone suppression test c) Aldosterone plasma rennin activity ratio d) Chest x-ray e) Adrenal CT scan Answer: a) The patient has thyrotoxic periodic paralysis. Thyrotoxic periodic paralysis is more common in Asian patients. Usually the patient has clinical features of thyrotoxicosis but clinical features of thyrotoxicosis may not be apparent

BOF: 125 A 22-year-old female presents with severe colicky abdominal pain, vomiting and constipation of 3 days duration. She had a previous history of admission to hospital with similar features. She was admitted under the care of a surgical team who did not find any specific abnormality on examination. Her abdominal x-ray and ultrasound scan were normal. She was treated with antibiotics, analgesics and antiemetics. She developed urinary retention and was catheterized. It was noticed that her urine was discolored but this was thought to be due to trauma. Whilst in ward she developed two tonicclonic seizures. You are asked to see the patient and after taking these features into account you make a diagnosis. The patient’s father wishes to know how this condition is inherited and your answer is: a) Autosomal recessive b) Autosomal dominant c) X-linked recessive d) Polygenic inheritance e) Not an inherited disease Answer: b) The patient has Acute Intermittent Porphyria. The inheritance of this condition is autosomal dominant

55

Mastering MRCP BOF: 126 A 20-year-old female presents with weakness of all four limbs of one days duration. She had a history of similar episodes in the past. No other significant illnesses in the past. She is not on any drugs. No significant family history. On examination her general condition was satisfactory, no signs of thyrotoxicosis. Conscious alert rational, cranial nerves normal. On examination of her limbs there was no wasting, tone was decreased, weakness of all four limbs unable to move them against gravity tendon reflexes were normal no sensory loss. No other abnormality of note. Investigations: Blood sugar 5.7 mmol/l Na 138 mmol/l K 2.0 mmol/l Chloride 112 mmol/l Bicarbonate14 mmol/l Blood urea 5.0 mmol/l pH 7.28 Pa CO 2 3.8 kPa Pa O2 13.0 kPa Urine pH 7 In this patient treatment should be: a) Give iv Potassium before bicarbonate b) Give iv bicarbonate before potassium c) Give iv bicarbonate alone d) Give iv potassium alone e) Give iv calcium Answer: a) The patient has hypokalaemic periodic paralysis. The underlying diagnosis is distal renal tubular acidosis. Potassium should be given before bicarbonate as bicarbonate can precipitate further hypokalaemia. In this patient the underlying diagnosis is: a) Proximal renal tubular acidosis b) Distal renal tubular acidosis c) Type A L- lactic acidosis d) Type B L-lactic acidosis e) D-lactic acidosis Answer: b) The patient has a normal anion gap acidosis with failure to acidify urine, normal K hence the likely diagnosis is distal renal tubular acidosis

BOF: 127 A 50-year-old male presents with malaise. He finds it difficult to cope with his work, sleeps poorly and has an itch especially at night. He has difficulty climbing stairs and also finds it difficult to stand up from a

56

Mastering MRCP seated position. He had no significant past illnesses but had been told that he had high blood pressure years earlier but had taken no action regarding this. On examination he was underweight and pale. His skin was dry and covered in scratch marks. Blood pressure was 190/110. No oedema. Heart sounds were normal, lungs were clear. Fundus showed arteriovenous nipping and thickened arteries. On neurological examination it was noticed that he had difficulty standing from the seated position. Investigations: Hb 8.0 MCV 90 fl WBC 4.8 x 10 9 /l Na 138 mmol/l K 6.1 mmol/l Chloride 90 mmol/l Urea 53 mmol/l Creatinine 1850 micromoles/l Calcium 1.40 mmol/l Phosphate 2.31 mmol/l Albumin 30 g/l Fasting Blood Glucose 5.2 mmol/l In this patient: a) Excellent control of blood pressure is not a priority as he is not diabetic b) Control of blood pressure is essential but care should be taken not to lower the blood pressure to less than 130/80 as this will compromise renal function c) ACE inhibitors should be avoided as they will cause further deterioration of renal function d) Treatment of hypertension with ACE inhibitors would be of particular value e) Low dose diuretics should be used so as to avoid dehydration which will further compromise renal function Answer: d) Non-diabetics with chronic renal failure benefit from excellent control of blood pressure. Treatment should aim to control blood pressure to 130/80 or lower. Treatment of hypertension with ACE inhibitors, which reduce intraglomerular capillary pressure and proteinuria, is of particular benefit in chronic renal failure. Large doses of diuretics are usually required to control sodium and water retention. In this patient: a) Treatment of hypercalcaemia should aim to keep the PTH level below three times the upper limit of normal b) Calcitriol should be given immediately c) Dietary restriction of phosphate will control the high phosphate levels d) Calcium carbonate will reduce bioavailability of dietary phosphate e) H2 antagonists should be used to increase the effectiveness of phosphate binders Answer: d) Reduction of PTH levels to less than 3 times the upper limit of normal carries the risk of development of adynamic bone disease. Increasing calcium before lowering phosphate increases the risk of tissue calcification.

57

Mastering MRCP Dietary phosphate restriction is seldom effective as many foods contain phosphate. H2 antagonists decrease the effectiveness of phosphate binders

BOF: 128 A 30-year-old female who is known to be hypothyroid and is on thyroxine presents complaining of loss of hair. On examination there are localised patches of hair loss with short stubby hair similar to an exclamation mark at the edges of the lesions. The cause of hair loss in this patient is: a) The patient has been non-compliant with her treatment and failed to take thyroxine b) Telogen effluvium c) The patient has hypothyroidism secondary to pan-hypopituitarism which has been uncovered by treatment with thyroxine d) Alopecia areata e) Secondary to iron deficiency caused by menorrhagia on account of hypothyroidism Answer: d) All the conditions mentioned above may be associated with hair loss. However apart from alopecia areata all the other conditions result in diffuse loss of hair. Localised hair loss with exclamation mark hairs at the edges of the lesions is characteristic of alopecia areata

BOF: 129 A 27-year-old female has been admitted to your ward with fever following a trip to India. A diagnosis of Plasmodium vivax malaria was made and she has been successfully treated. In this patient treatment with primaquine to eradicate liver forms: a) Will not result in clinical problems as Glucose-6-phosphatase dehydrogenase is inherited in an X-linked recessive manner b) May result in clinical problems as Glucose-6-phosphatase dehydrogenase is inherited in an X-linked dominant manner c) May result in clinical problems as Glucose-6-phosphatase dehydrogenase is inherited in an autosomal dominant manner d) May result in clinical problems as Glucose-6-phosphatase dehydrogenase is inherited in an autosomal recessive manner e) May result in clinical problems due to lysonisation Answer: e) Glucose-6-phosphatase dehydrogenase is an enzyme inherited in an X-linked recessive manner. Hence, most clinical problems occur in males. However, in females, lysonisation or random inactivation of an Xchromosome may result in two populations of red cells; one normal and one G-6-P D deficient and this may result in clinical problem

58

Mastering MRCP BOF: 130 A 25-year-old male has been admitted to your ward with fever. He had been backpacking in India and this raised concerns that he may have contracted malaria. Blood film confirms that the patient has infection with Plasmodium vivax and treatment is commenced with Chloroquine and Primaquine. The patient becomes unwell and starts passing dark urine and develops jaundice. A blood film is performed and it is reported as showing anaemia with many irregularly contracted cells, bite cells, blister cells and reticulocytes. This film suggests: a) Infection with Plasmodium falciparum as well b) Hypersplenism c) Glucose-6-phosphate dehydrogenase deficiency d) Backwater fever e) Infection with Plasmodium ovale as well Answer: c) In G-6-PD deficiency the changes seen during an attack are irregularly contracted cells, bite cells (cells with an indentation of the membrane) blister cells (cells in which the haemoglobin appears detached from the cell membrane) Heinz bodies and reticulocytosis

BOF: 131 A 45-year-old male presents with a history of slowly progressive, abnormal movements of his body. His wife reports a change in personality. No family history is available, as the patient had been adopted in infancy. On examination you observe jerky, semi-purposive movements involving the entire body. On asking the patient to protrude his tongue you notice that his tongue keeps darting in and out of his mouth. On testing grip strength you notice that he keeps alternately squeezing and releasing pressure on your fingers. The above clinical features are not caused by : a) Thyrotoxicosis b) Polycythaemia rubra vera c) Systemic lupus erythematosus d) Neuroleptic use e) Hypothyroidism Answer: e) The patient has chorea. The most likely cause is Huntington’s chorea. In an adult the other conditions that may result in chorea are: Thyrotoxicosis Polycythaemia rubra vera Drug use neuroleptics and oral contraceptives Systemic lupus erythematosus

BOF: 132 A 35-year-old female presents with a history of change in personality. There is a family history of Huntington’s chorea and the family is worried that she may be developing this condition.

59

Mastering MRCP The inheritance of Huntington’s chorea is: a) Autosomal recessive b) Autosomal dominant c) X-linked recessive d) Polygenic inheritance e) X-linked dominant Answer b) Huntington’s chorea is inherited as an autosomal dominant trait with full penetrance

BOF: 133 A post-mortem examination is carried out on a patient who had Huntington’s chorea. The main histological abnormality will be seen in the: a) Corpus callosum b) Hippocampus c) Superior colliculus d) Inferior colliculus e) Caudate lobe Answer: e) The main histological abnormality in Huntington’s chorea is seen in the caudate lobe and putamen where there is extensive neuronal loss

BOF: 134 A 22-year-old male presents with painless enlargement of the cervical lymph nodes. He also complains of fever and pruritus. Lymph node biopsy demonstrates Reed-Sternberg cells. In this patient the feature that would indicate the worst prognosis is a) Obstruction of the superior vena cava b) Inguinal lymphadenopathy c) Bone marrow involvement d) Pruritus e) Fever Answer: c) The patient has Hodgkin’s disease. The presence of Reed-Sternberg cells is pathognomonic. Involvement of the bone marrow would classify the patient as stage IV (modified Ann Arbor classification) indicating poor prognosis

BOF: 135 An absolute contraindication to peritoneal dialysis is: a) Previous peritonitis causing peritoneal adhesions b) Abdominal hernias

60

Mastering MRCP c) Visual impairment d) Active intra-abdominal sepsis e) Severe arthritis Answer: d) There are few absolute contraindications to peritoneal dialysis. Unwillingness or inability to learn the technique makes it impossible. Active intra-abdominal sepsis too is an absolute contraindication. Previous peritonitis causing adhesions may make placement of a catheter difficult but surgical placement may be possible. A stoma (colostomy, ileostomy, ileal conduit) makes placement of a dialysis catheter unlikely. Abdominal hernias expand during dialysis and ideally should be repaired prior to dialysis. Visual impairment makes peritoneal dialysis difficult but even blind people may be taught the technique. Severe arthritis too makes it difficult but mechanical aids are available

BOF: 136 An elderly patient presents with fluctuating episodes of confusion, attention problems and visual hallucinations. There is also a history of falls. In this patient which type of drug should be avoided, as it would result in an increased mortality? a) Antidepressants b) Neuroleptics c) Benzodiazepines d) Anticonvulsants e) Antihypertensives Answer: b) The patient has Lewy body dementia. The clinical presentation of this condition is with fluctuating episodes of confusion, problems with attention and visual hallucinations. They commonly fall down. In these patients, neuroleptics cause an extrapyramidal syndrome and increase mortality

BOF: 13 A 45-year-old female presents with a history of Raynaud’s phenomenon. She also complains of lethargy, anorexia and weight loss. She has heartburn. On examination the skin over her hands is tight and there are multiple telangiectasia over the malar region. Her blood pressure is 140/100 and this increase is sustained. Urine proteins ++ Her Investigations are as follows: Blood urea is 8.7 mmol/l Creatinine 120micrmoles/l Na 138 K 4.2 HB 11.0 g/dL MCV 90 ESR 60

61

Mastering MRCP In this patient the drug of first choice for control of her blood pressure would be a: a) Thiazide b) Beta-blocker c) Calcium antagonist d) ACE inhibitor e) Vasodilator Answer: d) The patient has systemic sclerosis. Renal involvement in systemic sclerosis requires intensive control of hypertension and ACE inhibitors are the drugs of first choice

BOF: 138 A 45-year-old female presents with a rash. She has recently been treated by her GP for an urinary tract infection. On examination she has a symmetrical rash mainly affecting the limbs, especially her hands and feet. The lesions consist of three concentric rings of colour change; a central blister surrounded by a pale oedematous zone, which in turn is surrounded by an area of erythema. The most likely drug that has been prescribed by the GP is: a) Trimethoprim b) Co-trimoxazole c) Augmentin d) Ampicillin e) Ciprofloxacin Answer: b) The patient has erythema multiforme. The most likely cause is the sulphonamide component of cotrimoxazole. (Sulfamethoxazole and trimethoprim)

BOF: 139 A patient who is known to have hereditary spherocytosis and has mild jaundice and gallstones is awaiting splenectomy. In this patient you would like to make sure that Pneumococcal immunisation is administered: a) 24 hours prior to splenectomy b) 2-3 weeks prior to splenectomy c) 2-3 months prior to splenectomy d) 72 hours prior to splenectomy e) 6 weeks prior to splenectomy Answer: b) Following splenectomy patients are at increased risk of overwhelming infections especially pneumococcal infections. Pneumococcal immunisation should be administered to the patient 2-3 weeks before splenectomy

62

Mastering MRCP BOF: 140 A 78-year-old male presents with a history of tremor and difficulty in getting out of chairs and getting up from bed. He has also noticed that his handwriting has been becoming smaller. On examination he has a tremor at rest mainly affecting his hands. When examining tone in his arms cogwheel rigidity is detected and when he walks you notice that he walks with a stoop and there is no arm swing. In this patient which class of drug should not be used in combination with laevodopa? a) Dopamine receptor agonist b) Aromatic amino acid decarboxylase inhibitor c) Catechol-O-methyl transferase inhibitor d) Monoamine oxidase inhibitor Type A e) Monoamine oxidase inhibitor Type B Answer: d) The patient has Parkinson’s disease. Monoamine oxidase inhibitors type A are absolutely contraindicated in combination with laevodopa

BOF: 141 An 18-year-old male presents with a history of syncope related to exercise. There is a family history of sudden cardiac death. General examination does not reveal any abnormality, his pulse rate is 70 beats per minutia regular, the carotid pulse is jerky, and there is a double apical pulsation. On auscultation there is a fourth heart sound, an ejection systolic murmur heard at the base of the heart and a pan systolic murmur heard at the apex. The inheritance of this condition is: a) Autosomal dominant b) Autosomal recessive c) X-linked recessive d) X-linked dominant e) Polygenic inheritance Answer: a) The patient has the features of Hypertrophic Obstructive Cardiomyopathy. The inheritance of this condition is autosomal dominant

BOF: 142 A patient is on ciclosporin following a renal transplant. Which of the following does not increase the plasma concentration of ciclosporin? a) Carbamazepine b) Ketoconazole c) Erythromycin d) Grapefruit juice e) Calcium channel antagonists Answer:

63

Mastering MRCP a) The plasma concentration of ciclosporin and risk of toxicity is increased by: Ketoconazole, erythromycin, chloroquine, cimetidine, oral contraceptives, anabolic steroids, calcium channel antagonists, grapefruit juice The plasma concentration of ciclosporin is reduced and loss of effect may be caused by: Enzyme inducing antiepileptics (phenytoin, carbamazepine, phenobarbitone) and rifampicin

BOF: 143 A 28-year-old female of Scandinavian origin presents with intense pruritus in her third trimester of pregnancy. Clinically there are no abnormalities apart from scratch marks. Investigations reveal: ALT 120 U/L Alkaline phosphatase 350 U/L Serum bilirubin 60 micromoles per liter Hb 11.0g/dL WBC 4.3 x109 /l Platelets 250,000 x 109/l In this patient the next step should be: a) Immediate delivery of the baby b) Ursodeoxycholic acid c) Cholestyramine d) Liver biopsy e) Steroids and azathioprine Answer: b) The patient has intrahepatic cholestasis of pregnancy (obstetric cholestasis). Treatment is symptomatic with ursodeoxycholic acid

BOF: 144 A 50-year-old male presents with severe pain in his left loin radiating to the left groin. Bedside testing of urine demonstrates haematuria. Abdominal ultrasound does not demonstrate any abnormality and X-ray KUB does not show stones. The reason the X-ray KUB does not show a stone may be because the type of calculus the patient has is a: a) Uric acid stone b) Calcium oxalate stone c) Calcium phosphate stone d) Triple phosphate stone e) Cystine stone Answer: a)Radiopaque stones are: Calcium oxalate, calcium phosphate, triple phosphate, cystine stones Radiolucent stones are: Urate, xanthine stones

64

Mastering MRCP BOF: 145 A 70-year-old male has been admitted under your care with atrial fibrillation and fast ventricular rate. Your junior has added amiodarone to the patient’s prescription of digoxin, warfarin and amlodipine. The interactions that may occur are: a) Digoxin effect is enhanced, warfarin effect is enhanced and the depressant effect of amiodarone on the SA and AV node is decreased b) Digoxin effect is enhanced, warfarin effect is enhanced and the depressant effect of amiodarone on the SA and AV node is increased c) Digoxin effect is enhanced, warfarin effect is reduced, the depressant effect of amiodarone on the SA and AV node is decreased d) Digoxin effect is reduced, warfarin effect is reduced and the depressant effect of amiodarone on the SA and AV node is decreased e) Digoxin effect is reduced, warfarin effect is enhanced and the depressant effect of amiodarone on the SA and AV node is enhanced Answer: b) Amiodarone increases the effect of digoxin (by displacing it from tissue binding sites and interference with its metabolism) Amiodarone increases the effect of warfarin (by inhibiting its metabolism) Beta-blockers and calcium antagonists augment the depressant effect of amiodarone on the SA and AV nod

BOF: 146 A 35-year-old male who suffers from schizophrenia has been on chlorpromazine. He presents with gynaecomastia and galactorrhoea. In this patient you would recommend changing from chlorpromazine to: a) Haloperidol b) Thioridazine c) Risperidone d) Amisulpiride e) Olanzapine Answer: e) Classical antipsychotics increase plasma prolactin concentrations by their blocking action on dopamine receptors in the tuberofundibular pathway. They can cause gynaecomastia and galactorrhoea and menstrual disturbances. A change to an atypical agent such as quetiapine or olanzapine should minimize this effect. Risperidone or amisulpiride do not minimize the effect. If continuation of the existing drug is mandatory then bromocriptine or amantadine may be used

BOF: 147 A patient, who is known to have COPD and has severe arterial hypoxaemia, is shown to have profound nocturnal hypoxemia. In this patient which of the following statements is false? a) Alveolar hypoventilation occurs due to an increase in upper airway resistance due to a reduction in muscle tone during sleep

65

Mastering MRCP b) Shallow breathing in REM sleep reduces alveolar ventilation c) Inhibition of intercostal and accessory muscles in REM sleep reduces alveolar ventilation d) Increased pulmonary artery pressure is caused by increased venous return to the right heart during deep sleep e) Severe secondary polycythaemia occurs partly as a result of nocturnal hypoxemia Answer: d) In chronic obstructive pulmonary disease, patients with severe arterial hypoxaemia may have severe nocturnal hypoxemia, which may result in the PaO2 falling as low as 2.5 kPa This is due to: Alveolar hypoventilation caused by inhibition of intercostal and accessory muscles in REM sleep Shallow breathing in REM sleep. This reduces ventilation. An increase in upper airways resistance because of reduction in muscle tone during sleep This results in: An increase in pulmonary artery pressure due to vasoconstriction Severe secondary polycythaemia

BOF: 148 A 27-year-old male presents with painless enlargement of his cervical lymph nodes. He also complains of fever, night sweats and loss of weight. A lymph node biopsy is performed and this reveals a background of lymphocytes, plasma cells, histiocytes, eosinophils, neutrophils and fibroblasts. Scattered within this background infiltrate are a number of large cells with two large nuclei with prominent nucleoli. The diagnosis in this patient is: a) Tuberculosis b) Sarcoidosis c) Hodgkin’s lymphoma d) Brucellosis e) Lyme disease Answer: c) The clinical features are suggestive in Hodgkin’s disease and histology demonstrates Reed-Sternberg cells, which are pathognomonic. A background reactive infiltrate of cells as described is seen with a few ReedSternberg cells

BOF: 149 A 47-year-old female presents with a long history of Raynaud’s phenomenon. She also complains of heartburn and reflux. On examination she has a small mouth and a beak like nose. The skin over her fingers is tight and shiny. She has multiple telangiectasias over her face and nail-fold capillary loops are seen. Her investigations show she has Anticentromere antibodies. In this illness a later complication that is significant is: a) Severe erosive arthropathy

66

Mastering MRCP b) Oesophageal Adenocarcinoma c) Renal failure d) Pulmonary hypertension e) Heart failure Answer: d) The patient has the clinical features of limited cutaneous scleroderma (LcSSc- formerly CREST syndrome). Anticentromere antibodies occur in 70-80% of these patients. A significant later complicating that causes death is pulmonary hypertension

BOF: 150 A young male presents with a history of dyspnoea. On examination he has a jerky carotid pulse a double apical impulse on palpation of the apex beat, an ejection systolic murmur is audible at the base of the heart and a pan systolic murmur is heard at the apex. Echocardiography demonstrates asymmetric left ventricular hypertrophy and systolic anterior motion of the mitral valve. In this patient, dyspnoea may be treated with: a) Diuretics b) ACE inhibitors c) Long acting nitrates d) Digoxin e) Beta-blockers Answer: e) The patient has the clinical and echocardiographic features of hypertrophic obstructive cardiomyopathy. In these patients dyspnoea is best treated with beta-blockers and verapamil either alone or in combination. If they are ineffective disopyramide may be used. Other types of treatment include dual chamber pacing, alcohol ablation of the septum and resection of the myocardium

BOF: 151 A 27-year-old Caucasian female presents with a long history of abdominal pain and loose motions. She also gives a history of recurrent mouth ulcers. Recently the pain has become more intense, she has begun to lose weight and has developed fever with night sweats and she also complains of a rash over her legs. On examination she appears to have lost weight, she has clubbing and erythema nodosum. On examination of the abdomen a firm tender mass can be palpated in the right iliac fossa. In this patient, which of the following investigations would help guide your subsequent management? a) Small bowel follow through b) Labelled white cell scan c) Colonoscopy d) Barium enema e) CT scan of the abdomen Answer:

67

Mastering MRCP e) The clinical features in this patient point to a diagnosis of Crohn’s disease. However, the features of increased pain, fever sweats and a mass suggest the patient may have developed a collection of pus in the right iliac fossa. Hence, imaging this area with CT scan to detect and treat the collection is the investigation required in this patient

BOF: 152 A patient has had a splenectomy because of hereditary spherocytosis. In this patient penicillin prophylaxis should be: a) Lifelong b) Prior to surgical procedures or invasive investigation c) For 5 years d) If the patient has an upper respiratory tract infection e) For 10 years Answer: a) Following splenectomy patients should receive lifelong penicillin prophylaxis

BOF: 153 Antinuclear anybodies develop in the plasma of patients taking hydralazine if they are: a) Fast acetylators b) Slow acetylators c) Slow oxidizers d) Extensive oxidizers e) Thiopurine methyl transferase deficient Answer: b) Drugs that possess an amide group (-NH2) are metabolized by acetylation. Slow acetylators are likely to develop adverse effects whereas rapid acetylators need higher doses of drugs. Hydralazine and procainamide cause antinuclear antibodies to appear in the plasma of slow acetylators and some may progress to develop systemic lupus erythematosus

BOF: 154 A 50-year-old male who is known to have chronic obstructive pulmonary disease is admitted with a history of increasing breathlessness. This has been precipitated by the development of a cough with production of purulent sputum. The organisms that are likely to cause infective exacerbations of COPD are: a) Streptococcus pneumoniae, Haemophilus influenzae, Moxarella catarrhalis b) Streptococcus pneumoniae, Haemophilus influenzae, Klebsiella pneumoniae c) Streptococcus pneumoniae, Haemophilus influenzae, Staphylococcus aureus d) Streptococcus pneumoniae, Haemophilus influenzae, Mycoplasma pneumoniae e) Streptococcus pneumoniae, Haemophilus influenzae, Legionella pneumophilia Answer:

68

Mastering MRCP a) In infective exacerbations of COPD the usual organisms are Streptococcus pneumoniae and Haemophilus influenzae. Occasionally Moxarella catarrhalis may be the causative organism

BOF: 155 A 47-year-old male presents with sudden onset of a right-sided foot drop with numbness over the dorsum of his right foot. He has weakness of dorsiflexion and eversion of his right foot. There is an area of diminished sensation over the dorsum of the right foot. He also has a left wrist drop with loss of sensation over the dorsal aspect of the first interosseus space. For the last few weeks he has had fever, he has been feeling unwell, he has lost weight and he has had aching of his muscles. Investigations: Hb 11.0 g/dL MCV 90 WBC 12 x109/l Neutrophils 8.5 x109/l Lymphocytes 3.0 x109/l Monocytes 0.3 x109/l Eosinophils 0.1 x109/l Basophils 0.1 x109/l ESR 47 mm/1st hour Urine: Proteins ++ Blood ++ ANCA negative The most likely diagnosis is: a) Sarcoidosis b) Systemic lupus erythematosus c) Rheumatoid arthritis d) Polyarteritis nodosa e) Churg-Strauss syndrome Answer: d) The patient has a systemic illness with mononeuritis multiplex and renal involvement. All the conditions listed above can cause mononeuritis multiplex. PAN is one of the few connective tissue disorders that usually occur in middle-aged men unlike RA and SLE, which are commoner in females. In addition no mention has been made of skin or joint involvement. There is no history of asthma and the eosinophil count is normal and ANCA negative hence, Churg-Strauss syndrome becomes less likely. Sarcoidosis does not usually cause renal involvement

BOF: 156 A female patient on long-term warfarin consults you as she wishes to conceive a baby. What would your advice be regarding anticoagulation? a) It is safe to carry on with warfarin without any change

69

Mastering MRCP b) Stop warfarin prior to conception, change over to heparin and continue heparin to term c) Stop warfarin prior to conception, change over to heparin for the first trimester and then restart warfarin d) Stop warfarin prior to conception and substitute with heparin, continue heparin through the first trimester, change back to warfarin and re-start heparin near term. e) Stop warfarin prior to conception, use heparin for the first two trimesters and then restart warfarin Answer: d) Warfarin use in early pregnancy may cause skeletal disorders (bossed forehead, sunken nose, calcification in the epiphyses) and absence of the spleen. Hence warfarin should be stopped prior to conception and heparin substituted. After the first trimester heparin should be discontinued and warfarin restarted as long term heparin can cause osteoporosis. Warfarin should be discontinued and heparin restarted prior to term as warfarin can exacerbate neonatal hypoprothrombinaemia and its control is not precise enough during labour

BOF: 157 An obese middle-aged male presents with a history of numbness and tingling of the upper outer part of the left thigh. On examination there is sensory impairment over the anterolateral aspect of the thigh. Tinel’s sign is positive on tapping just below the junction of the inguinal ligament and the anterior superior iliac spine. The lesion is in the: a) Ilioinguinal nerve b) Femoral nerve c) Lateral cutaneous nerve of thigh d) Genitofemoral nerve e) Intermediate femoral cutaneous nerve Answer: c) The patient has meralgia paraesthetica, an entrapment neuropathy of the lateral cutaneous nerve of the thigh as it passes under the inguinal ligament. Features may be unilateral or bilateral. Weight reduction should be advised but quite often the condition subsides spontaneously. Decompression is rarely required

BOF: 158 A 28-year-old Caucasian female has been started on azathioprine as she has Crohn’s colitis. She has a full blood count performed for routine monitoring and the results are as follows: Hb 8.0 g/dL MCV 85 fl WBC 2.3 x109 /l Neutrophils 1.1 x109 /l Lymphocytes 0.9 x109 /l Monocytes 0.2 x109 /l Eosinophils 0.1 x109 /l Platelets 75,000 x109 /l

70

Mastering MRCP This type of reaction is more likely if the patient: a) Has Glucose-6- phosphate dehydrogenase deficiency b) Is a slow acetylator c) Is a fast acetylator d) Has thiopurine methyl transferase deficiency e) Is a slow oxidizer Answer: d) The patient has developed a pancytopaenia due to azathioprine toxicity. Approximately 1 in 300 Caucasian people have thiopurine methyl transferase (TPMT) deficiency. This is a result of genetic polymorphism. TPMT is the enzyme that metabolises 6-mercaptopurine. TPMT deficiency results in high risk of toxicity to azathioprine

BOF: 159 A 28-year-old female has been started on treatment for pulmonary tuberculosis. She has asked you for advice on contraception. You would not recommend oral contraception, as failure of oral contraception is a recognised interaction of: a) Isoniazid b) Ethambutol c) Rifampicin d) Pyrazinamide e) Thiacetazone Answer: c) Rifampicin is a powerful enzyme inducer and speeds up the metabolism of steroid contraceptives. Alternative methods of contraception should be advised. Rifampicin also speeds up the metabolism of drugs such as warfarin, narcotic analgesics, oral antidiabetic agents, phenytoin and dapsone

BOF: 160 Weakness of biceps, coracobrachialis and brachialis and sensory loss over the lateral aspect of the forearm may be caused by a lesion of the: a) Axillary nerve b) Radial nerve c) Musculocutaneous nerve d) Ulnar nerve e) Median nerve Answer: c) Damage to the musculocutaneous nerve causes weakness of the biceps, coracobrachialis and brachialis. It also results in sensory loss over the lateral aspect of the forearm. Lesions of the musculocutaneous nerve art rare alone but it may be damaged along with the brachial plexus

71

Mastering MRCP BOF: 161 The registrar on duty at the accident and emergency department rings you up for advice regarding a 24year-old male who has presented to them with sudden onset of dyspnoea. He says the only physical sign he can elicit is a clicking sound in the heart in systole. He wishes to discharge the patient but you advise him that he should arrange: a) Echocardiogram b) Electrocardiogram c) Blood gases d) Chest x-ray e) Lung function tests Answer: d) Left sided pneumothorax can result in a clicking sound synchronous with the heartbeat (Hamman’s sign). In this patient who has presented with breathlessness and a systolic click chest x-ray is mandatory

BOF: 162 A 30-year-old female has been started on azathioprine as a steroid-sparing agent because she has ileocaecal Crohn’s disease. She complains of malaise, abdominal discomfort and fever. Which of the following steps would you take in this patient? a) Stop azathioprine immediately b) Increase the patient’s steroid dosage c) Decrease azathioprine dosage d) Increase azathioprine dosage e) Change from azathioprine to mercaptopurine Answer: e) Intolerance of azathioprine is manifested by malaise, abdominal discomfort and fever. Pancreatitis occurs in up to 5 % of patients. These effects are caused by the imidazole side chain of the azathioprine molecule and mercaptopurine, which is azathioprine without the imidazole side chain, may be better tolerated

BOF: 163 A 50-year-old male presents with painless jaundice. He has pale stools and dark urine. On examination he is deeply jaundiced and has multiple scratch marks all over his body. On examination of the abdomen a distended gall bladder is palpable. In this patient, jaundice is due to obstruction at the level of the: a) Intrahepatic ducts b) Left hepatic duct c) Right hepatic duct d) Cystic duct e) Common bile duct Answer: e)

72

Mastering MRCP The physical signs in this patient demonstrate Courvoisier’s sign (palpable gall bladder) and Courvoisier’s law (obstruction below the level of the cystic duct causes the gall bladder to become distended and it may be palpable). The cause of the obstruction is most likely to be carcinoma of the pancreas

BOF: 164 A cardiac catheterisation study performed on an asymptomatic 21-year-old male demonstrated the following

SVC

Pressure mm Hg 7

Oxygen Saturation % 66

RA (mean) RV PA LA (mean) LV

7 26/2 25/7 7 114/0

79 78 80 98 97

Aorta

110/60

97

In this patient, which of the following statements about the E.C.G. is true? a) The ECG is likely to show right bundle branch block, right axis deviation, sharp right atrial P waves, and prolongation of PR interval, absent QR in V5-V6 b) The ECG is likely to show rSR in V1 with left axis deviation, P mitrale and absent QR in V5-V6 c) The ECG is likely to show P mitrale d) The ECG is likely to show a short PR interval, right bundle branch block, left axis deviation and deep Q waves in V5-V6 e) The ECG is likely to show right bundle branch block, right axis deviation, sharp left atrial P waves and deep Q waves in leads V5-V6 Answer: a) The cardiac catheterisation data with a step up of saturation in the RA suggests that the patient has an atrial septal defect. As the patient is asymptomatic and has normal pulmonary artery pressure at the age of 21, it is likely to be an ostium secundum defect rather than an ostium primum defect. The characteristic ECG changes of ostium secundum defects are RBBB with right axis deviation. Sharp right atrial P waves occur and prolongation of the PR interval (slight) is seen. QR is absent in leads V5-V6 Ostium primum defects extend into the anterior leaflets of the mitral valve and result in mitral regurgitation. This increases the left to right shunt with earlier onset of symptoms and results in a worse prognosis. The ECG changes in ostium primum defects are rSR in V1 with left axis deviation. P mitrale is common and prolongation of the PR interval is likely

73

Mastering MRCP BOF: 165 Furosemide (furosemide) produces a diuresis by acting on the: a) Proximal convoluted tubule b) Descending loop of Henle c) Ascending loop of Henle d) Distal convoluted tubule e) Collecting ducts Answer: c) Furosemide acts on the thick portion of the ascending loop of Henle. It can cause up to 25% of filtered sodium to be excreted

BOF: 166 A 28-year-old male presents with haematemesis. He has been out drinking with his friends and drank about 10 pints of beer and several shots of vodka after which he had felt sick and vomited several times. The last time he vomited he brought up several cupfuls of blood. The most likely cause of his haematemesis is: a) Oesophageal varices b) Peptic ulcer c) Dieulafoy lesion d) Severe gastritis e) Mallory-Weiss tear Answer: e) A Mallory-Weiss tear is a linear mucosal tear that occurs at the gastro-oesophageal junction when there is a sudden increase in intra-abdominal pressure after a bout of coughing or retching. It commonly occurs after an alcohol binge. Dieulafoy lesion is an ectatic (Dilated) blood vessel in the stomach

BOF: 167 Complete inability to raise the arm at the shoulder with sensory loss over the deltoid is a result of damage to the: a) Radial nerve in the axilla b) Radial nerve in the spiral groove c) Suprascapular nerve d) Dorsal scapular nerve e) Axillary nerve Answer: e) The axillary nerve supplies the deltoid and teres minor and the skin over the deltoid

74

Mastering MRCP BOF: 168 A 25-year-old male has been diagnosed as having autosomal dominant polycystic kidney disease. His mother has had a subarachnoid haemorrhage. In this patient: a) Screening for intracranial aneurysm is of no benefit b) Screening should be performed by MRI or spiral CT c) Screening should only be carried out if there is uncontrolled hypertension d) Screening should be carried out only if there is renal dysfunction e) Screening should be carried out if only there is associated mitral valve prolapse Answer: b) Approximately 8% of patients with have asymptomatic intracranial aneurysms. Then prevalence is twice as high in those with a family history of intracranial aneurysms or subarachnoid haemorrhage. Screening for intracranial aneurysm is recommended for patients with ADPKD aged 18-40 years who have a positive family history

BOF: 169 A 55-year-old male is on azathioprine to maintain remission of Crohn’s disease. He has attended the cardiovascular risk clinic and has been started on allopurinol as he has been noted to have a high uric acid level. He asks you whether he should take allopurinol and your reply is: a) There is no interaction between allopurinol and azathioprine b) Allopurinol potentiates toxicity of azathioprine c) Azathioprine potentiates toxicity of allopurinol d) Allopurinol reduces bioavailability of azathioprine e) Azathioprine reduces bioavailability of allopurinol Answer: b) Allopurinol specifically inhibits xanthine oxidase and prevents metabolism of azathioprine to mercaptopurine. It can thus cause dangerous toxicity

BOF: 170 A 65-year-old male has been admitted to your ward with left ventricular failure. The house physician has administered Furosemide 100 mgs intravenously and the patient is less dyspnoeic. The blood results are now available and he notes that there is evidence of renal failure. He rings you up to ask whether the furosemide he administered might result in an adverse effect in view of the renal impairment. Your reply is that in a patient with renal failure rapid i.v. injection of furosemide may result in: a) Ototoxicity b) Hepatotoxicity c) Nephrotoxicity d) Cardiotocxicity e) Gout Answer:

75

Mastering MRCP a) Rapid i.v. injection of furosemide in patients who are in renal failure may rarely cause deafness which is usually transient

BOF: 171 A 55-year-old male is in coronary care unit having suffered an anterior myocardial infarction. Three days after admission, he complains of a sharp, substernal pain, which is relieved by leaning forward and aggravated by lying down. The pain is also aggravated by movement and respiration. On examination he is febrile and has a tachycardia and on auscultation of his chest a pericardial friction rub is audible. Your house physician has started the patient on NSAIDS in high dosage and asks for your advice regarding the use of steroids on this patient. You inform him that in this patient the use of steroids and NSAIDS: a) Increases the risk of re-infarction b) Increases the risk of pulmonary embolism c) Increases the risk of myocardial rupture d) Increases the risk of brady-arrhythmias e) Increases the risk of tachy-arrhythmias Answer: c) The patient has post-myocardial infarction pericarditis that occurs in about 20% of patients within a few days of the infarction. The likelihood of a patient developing this complication is greater in those who have had an anterior infarction, Q wave infarction and in those who have high serum cardiac enzymes. In these patients the use of NSAIDS (other than aspirin) and corticosteroids increases the risk of myocardial rupture

BOF: 172 A 53-year-old male presents with sudden onset of severe crushing central chest pain associated with a feeling of nausea and profuse sweating. On clinical examination the only abnormalities found are profuse sweating and bradycardia. E.C.G shows sinus bradycardia with ST segment elevation in leads II, III, aVF. In this patient the lesion is likely to be in the: a) Obtuse marginal artery b) Circumflex artery c) Anterior interventricular artery d) Right coronary artery e) Diagonal artery Answer: d) The patient has an inferior infarction. This area of the myocardium is supplied by the right coronary artery . The other arteries are branches of the left coronary artery. The SA nodal artery also arises from the right coronary and this explains the bradycardia

76

Mastering MRCP BOF: 173 A 65-year-old male who is known to have ischaemic heart disease is admitted with left ventricular failure. He has been on NSAIDS for some time as the GP has treated him with these drugs for osteoarthritis. In this patient because he has been on NSAIDS one would expect: a) No difference in furosemide induced diuresis b) Increased furosemide induced diuresis c) Reduced furosemide induced diuresis d) Increased tendency to hypokalaemia e) Increased tendency to hyperkalaemia Answer c) NSAIDS especially indomethacin reduce furosemide induced diuresis probably by inhibiting the formation of vasodilator prostaglandins in the kidney

BOF: 174 A 64-year-old male presents with oliguria, peripheral oedema and shortness of breath. He gives a history of intermittent haemoptysis and dyspnoea over several months. There is no history of significant past illnesses. He smokes about 20 cigarettes a day On examination he is oedematous, cyanosed, tachypnoeic and there are widespread inspiratory crackles throughout the lung fields. Urine: Proteins ++ Blood ++ Immunology: Anti glomerular basement membrane antibody + In this patient renal biopsy is likely to show: a) Membranous glomerulonephritis b) Membranoproliferative glomerulonephritis c) Focal glomerulosclerosis d) Mesangial proliferative glomerulonephritis e) Proliferative glomerulonephritis (crescentic glomerulonephritis) Answer: e) The patient has Goodpasture’s syndrome. On renal biopsy the features that occur are: Initially there is a focal proliferative glomerulonephritis. In the later stages diffuse glomerular involvement occurs. The proliferative changes are frequently associated with epithelial crescent formation and necrosis

BOF: 175 A patient presents with wasting of the quadriceps, weakness of knee extension, loss of the knee jerk and sensory impairment over the front of the thigh and over the subcutaneous surface of the tibia. This is due to a lesion of the: a) Femoral nerve b) Obturator nerve

77

Mastering MRCP c) Sciatic nerve d) Lumbosacral plexus e) Tibial nerve Answer: a) The femoral nerve supplies iliacus, pectineus, sartorius, quadriceps femoris. It supplies cutaneous branches to the front of the thigh and the continuation of the femoral nerve, the saphenous nerve, supplies the skin over the medial aspect of the lower leg as far as the medial malleolus

BOF: 176 A 66-year-old male who is known to have chronic bronchitis has presented to his GP with a history of gradually increasing breathlessness and wheeze. He takes salbutamol and beclomethasone inhalers and has recently been started on timolol eye drops for glaucoma and omeprazole for heartburn. In this patient what steps would you take to improve this patient’s symptoms? a) Increase the dosage of inhaled salbutamol and beclomethasone b) Add in oral steroids c) Treat with a week course of antibiotics d) Stop omeprazole e) Stop timolol eye drops Answer: e) Timolol (beta-blocker) eye drops may drain down the lacrimal duct and aggravate bronchoconstriction

BOF: 177 A 50-year-old male presents with malaise, weight loss and diarrhoea. On examination his skin is pigmented, he has clubbing, anaemia and lymphadenopathy. Upper gastrointestinal endoscopy is performed and distal duodenal biopsies demonstrate stunted villi. The lamina propria is distended with multiple periodic acidSchiff (PAS) positive macrophages. The aetiological agent in this condition is: a) Mycobacterium avium intracellulare b) Giardia lamblia c) Trophyrema whippelleii d) Epstein Barr virus e) HIV virus Answer: c) The clinical features with demonstration of PAS positive macrophages in the lamina propria of the small intestine is diagnostic of Whipple’s disease which is caused by Trophyrema whippelleii

78

Mastering MRCP BOF: 178 A 25-year-old male presents with a history of recurrent episodes of haematuria. This is related to upper respiratory infections and comes on within approximately 12-24 hours of development of pharyngitis. He describes passage of frank blood. Clinically he looks well, there is no oedema, blood pressure is normal and renal function is not impaired. In this patient the light microscopic features one would expect on renal biopsy are: a) Proliferative glomerulonephritis without crescent formation b) Proliferative glomerulonephritis with crescent formation c) Mesangial proliferation d) Membranous thickening e) Minimum change Answer: c) The description of recurrent pharyngitic haematuria in a young male with no physical signs and no deterioration in renal functions should make one suspect IgA nephropathy (Berger’s disease). The light microscopic feature of this is mesangial proliferation. Immunofluorescence would demonstrate IgA deposition in the mesangium as confluent masses or discrete granules

BOF: 179 A 22-year-old male is admitted with severe crushing central chest pain with associated nausea and profuse sweating. He admits to snorting cocaine. In this patient what is the mechanism of myocardial ischaemia? a) Increased platelet adhesiveness b) Coronary vasospasm c) Severe tachycardia d) Hypotension e) Cardiac arrhythmia Answer: b) Cocaine causes coronary vasospasm sufficient to present as severe ischaemia or infarction

BOF: 180 A patient presents with foot drop. On examination there is weakness of dorsiflexion and eversion at the ankle and weakness of toe extension. Loss of sensation is demonstrated over the lateral aspect of the lower leg and ankle and over the dorsum of the foot. In this patient the lesion is in the: a) Common peroneal nerve b) Sural nerve c) Tibial nerve d) Sciatic nerve e) Femoral nerve Answer: a) The common peroneal nerve divides into two branches:

79

Mastering MRCP Superficial peroneal: Supplies peroneus longus and brevis and the skin over the lateral aspect of the lower leg. Deep peroneal: Supplies tibialis anterior, extensor hallucis longus, extensor digitorum longus, peroneus tertius, extensor digitorum brevis, and skin of the adjacent sides of the first two toes. Damage to the common peroneal is common because of its superficial position.

BOF: 181 A middle-aged male has slowly progressive weakness of his upper limbs. On examination of the patient the physical signs are kyphoscoliosis, wasting and weakness of the small muscles of the hand, flattening of the muscles of the ulnar border of the forearm. No reflexes can be elicited in the upper limb. Pain and temperature sensation are lost over the upper limbs and upper chest whereas light touch and proprioception remain intact. Lower limb reflexes are exaggerated and plantars are up going. In this patient the associated feature could be: a) Arnold-Chiari malformation b) Rheumatoid arthritis c) Cervical spondylosis d) Vitamin B12 deficiency e) Dumbbell meningioma Answer: a) The patient has the features of an intramedullary lesion of the spinal cord. A middle-aged male with slowly progressive neurological features suggests that the patient has syringomyelia. Arnold-Chiari malformation is associated with syringomyelia. Arnold-Chiari malformation refers to aberrant cerebellar tissue, which extends through the foramen magnum (herniation of the cerebellar tonsil)

BOF: 182 In the management of cocaine overdose which one of the following statements is untrue: a) Haloperidol may be used for mental disturbance b) Diazepam may be used for convulsions c) A calcium channel blocker may be used for hypertension d) Glyceryl trinitrate may be used for myocardial ischaemia e) A beta blocker may be used for myocardial ischaemia Answer: e) Beta-blockers aggravate cocaine-induced coronary vasospasm

BOF: 183 A 56-year-old female presents with a history of progressive difficulty in climbing stairs and rising from chairs. She also complains of difficulty in swallowing. She complains that her hands become cold, painful and turn blue when exposed to cold.

80

Mastering MRCP On examination she has purple discoloration of her eyelids and periorbital oedema. There is weakness of the muscles of her limb girdles. In this patient the dysphagia is most likely to be due to: a) Carcinoma of the oesophagus b) Bronchial carcinoma causing extrinsic compression of the oesophagus c) Benign stricture of the oesophagus d) Crico-pharyngeal web e) Pharyngeal muscle weakness Answer: e) The clinical features suggest the patient has dermatomyositis. In this condition involvement of the pharyngeal musculature results in dysphagia BOF: 184 A patient has multiple skin lesions that consist of sessile and pedunculated papules and nodules distributed over the entire surface of his body. In addition there are multiple pigmented macules on his trunk and freckling in the axilla. There are small pigmented hamartoma in the iris. The inheritance of this condition is: a) Autosomal dominant b) Autosomal recessive c) X-linked recessive d) X-linked dominant e) Polygenic inheritance Answer: a) The clinical features point to a diagnosis of neurofibromatosis. The inheritance of this condition is autosomal dominant

BOF: 185 A 24-year-old male presents with multiple painful ulcers on the prepuce and frenulum of his penis. He also has unilateral inguinal lymphadenopathy. He gives a history of unprotected sexual intercourse 5 days prior to the onset of the lesions. The diagnosis in this patient is: a) Primary syphilis b) Chancroid c) Lymphogranuloma venereum d) Gonorrhoea e) Chlamydia urethritis Answer: b) Chancroid is a sexually transmitted infection caused by Haemophilus ducreyi It causes multiple painful ulcers and the incubation period is 3-10 days. The chancre in primary syphilis is a single ulcer, which is painless. The incubation period is 10-90 days. Lymphogranuloma venereum causes a painless ulcer; the incubation period is 7-21 days.

81

Mastering MRCP Gonorrhoea and Chlamydia cause urethritis but do not cause ulcers

BOF: 186 A 65-year-old male presents with sudden onset dysphagia and dysarthria, vomiting hiccup and vertigo. On examination he has a right sided Horner’s syndrome, right-sided cerebellar ataxia, loss of pain and temperature sensation on the right hand side of the face and loss of pain and temperature sensation in the left upper and lower limbs. In this patient the lesion is in the: a) Pons b) Upper midbrain c) Lower midbrain d) Lateral medulla e) Medial medulla Answer: d) The patient has the lateral medullary syndrome (Wallenberg’s syndrome). It is due to a lesion involving the: 9th and 10th nerve causing dysphagia and dysarthria Nucleus ambiguous causing vomiting and hiccup Vestibular nuclei causing vertigo Inferior cerebellar re peduncle causing ipsilateral cerebellar ataxia Descending autonomic fibers causing Horner’s syndrome Fifth nerve nucleus causing loss of pain and temperature sensation over the face (ipsilateral) Lateral leminiscus causing loss of pain and temperature sensation in the contralateral limbs

BOF: 187 A 27-year-old female presents with restlessness, tremor, shivering and myoclonus. She has been on sertraline for depression and has taken medication for an attack of migraine. Which agent is she likely to have taken for migraine? a) Paracetamol b) Aspirin c) Diclofenac d) Sumatriptan e) Ibuprofen Answer: d) The patient has developed the serotonin syndrome, which is a rare but dangerous complication of SSRIs The features of this syndrome are: Restlessness, tremor, shivering and myoclonus. It may progress to convulsions, coma and death. Risk of development of this complication is increased by co-administration of drugs that enhance serotonin transmission especially MAOIs, sumatriptan and St. John’s Wort

82

Mastering MRCP BOF: 188 A 45-year-old female is admitted with a history of epistaxis and iron deficiency anaemia. On examination of the patient multiple telangiectasia e are noted around her lips and in her mouth. In this patient family history is important, as you know the mode of inheritance of this condition is: a) Autosomal dominant b) Autosomal recessive c) X-linked recessive d) X-linked dominant e) Polygenic inheritance Answer: a) The patient has the features of hereditary haemorrhagic telangiectasia (Osler-Rendu-Weber syndrome). The inheritance of this condition is autosomal dominant BOF: 189 A young man presents with a history of chest pain. On examination he has a jerky carotid pulse, double apical pulsation and an ejection systolic murmur. Echocardiography demonstrates asymmetric left ventricular hypertrophy and systolic anterior motion of the mitral valve. In this patient, the chest pain should respond to treatment with: a) Isosorbide dinitrate b) Isosorbide mononitrate c) Amlodipine d) Beta blockers e) ACE inhibitors Answer: d) The patient has features of hypertrophic obstructive cardiomyopathy. In these patients chest pain responds to treatment with beta-blockers and verapamil either alone or in combination. Other treatments that may be used are: Disopyramide Dual chamber pacing Alcohol ablation of the septum Resection of septal myocardium

BOF: 190 In anti glomerular basement membrane disease (Goodpasture’s syndrome), the chances of pulmonary haemorrhage are increased: a) In current smokers b) Non-smokers c) In patients with severe hypertension d) In patients with severe renal impairment e) In patients with high levels of circulating IgM antibody Answer: a)

83

Mastering MRCP In Goodpasture’s syndrome, almost all current smokers have lung haemorrhage whereas it is rare in nonsmokers

BOF: 191 A 28-year-old Japanese male presents with anterior uveitis. He gives a history of recurrent oral ulceration (more than three times a year) .On examination of his genitals scarring is noted (he has had ulcers on his genitals) You think he may have Behcet’s disease but you are aware that to make a diagnosis of this condition the criteria that should be fulfilled are: a) Recurrent oral ulceration plus two of the following: recurrent genital ulcers, eye lesions, skin lesions, positive pathergy test b) Recurrent oral ulceration, a positive pathergy test, plus one of the following; recurrent genital ulcers, eye lesions, skin lesions c) Recurrent oral ulcers plus recurrent genital ulcers plus one of the following; skin lesions, eye lesions, positive pathergy test d) Recurrent oral ulcers plus two of the following; recurrent genital ulcers, eye lesions, skin lesions, arthritis, positive pathergy test e) Recurrent oral ulcers plus two of the following; recurrent genital ulcers, eye lesions, skin lesions, neurological signs, arthritis, positive pathergy test Answer: a) The international study group criteria for the diagnosis of Behcet’s disease are: Recurrent oral ulcers (minor aphthous, major aphthous, or herpetiform ulcers which have recurred at least three times a year) Plus any two of the following: Recurrent genital ulcers (ulcers or scarring) Eye lesions (anterior uveitis, posterior uveitis, cells in the vitreous on slit lamp examination, retinal vasculitis) Skin lesions (erythema nodosum, pseudofolliculitis, papulopustular lesions, acneiform lesions in post adolescents not on corticosteroids) Positive pathergy test (skin injury by needle prick leads to a papule or pustule formation in 24-48 hours)

BOF: 192 A 45-year-old female who is on disease modifying agents for rheumatoid arthritis presents with a blistering rash, which is sore but not itchy. On the skin there are multiple superficial bullae, which rupture easily, leaving erosions, which are painful. The base of the erosion is red and bleeds easily. The epidermis at the edge of the blister is easily dislodged by sliding pressure. In the mouth the blisters rupture easily leaving superficial erosions. The disease-modifying agent that the patient has been treated with is likely to be: a) Methotrexate b) Azathioprine c) Penicillamine d) Gold

84

Mastering MRCP e) Sulphasalazine Answer: c) The description of the lesions point to a diagnosis of pemphigus 9% of patients treated with penicillamine for rheumatoid arthritis develop pemphigus

BOF: 193 A 35-year-old male who is homeless is admitted with a history of breathlessness and swelling of his ankles. On examination there is peripheral oedema, he is in atrial fibrillation, has an elevated JVP, displaced apex beat, a third heart sound, reversed splitting of the second heart sound and a soft systolic murmur at the apex. You take a detailed dietary history in this patient, as you are aware that this condition may be caused by deficiency of: a) Thiamine b) Riboflavin c) Pyridoxine d) Niacin e) Biotin Answer: a) The patient has the clinical features of dilated cardiomyopathy. This may be caused by deficiency of thiamine, selenium

BOF: 194 A friend of yours is off on a trekking holiday to the Himalayas. He asks you whether it is correct that acetazolamide can be used to prevent high-altitude (mountain) sickness. You reply that it is effective because it stimulates respiration by: a) Causing a metabolic acidosis b) Causing a metabolic alkalosis c) Causing respiratory acidosis d) Causing respiratory alkalosis e) Direct effect on the respiratory centre Answer: a) Acetazolamide is a carbonic anhydrase inhibitor. Carbonic anhydrase facilitates the reaction between CO2 and water to form carbonic acid, which then breaks down into H+ and HCO3-. When acetazolamide acts on the kidney it reduces HCO3- reabsorption from the tubule and results in metabolic acidosis. This metabolic acidosis increases respiratory drive.

BOF: 195 A 70-year-old male presents with sudden onset of right-sided paralysis of his tongue with left sided weakness of the limbs and loss of postural sense on the left hand side. In this patient the lesion is in the:

85

Mastering MRCP a) Pons b) Upper midbrain c) Lower midbrain d) Lateral medulla e) Medial medulla Answer: e) The patient has developed the medial medullary syndrome, which is caused by an ipsilateral 12th nerve lesion and contralateral weakness and loss of postural sense due to lesions in the pyramidal tract and the medial leminiscus

BOF: 196 A 54-year-old male wakes up with excruciating pain in his right big toe. On examination he has swelling of the metacarpophalangeal joint of his right hallux with overlying erythema. It is tender to touch and the patient is febrile. Initial investigations reveal a raised white cell, count and an elevated ESR. In this patient: a) A high serum uric acid is the most specific and diagnostic test b) Joint fluid aspirate for culture is the most specific and diagnostic test c) Joint fluid aspirate for microscopy is the most specific and diagnostic test d) Demonstration of periarticular deposits on X-ray, giving a halo of radio-opacity is the most specific and diagnostic test e) Demonstration of clearly defined (“punched out”) bone cysts on X-ray is the most specific and diagnostic test Answer: c) The clinical features in this patient suggest a diagnosis of gout. Fever, elevated ESR and leucocytosis can occur in severe attacks and do not signify infection. In gout, the diagnosis is made by joint fluid aspirate and microscopy by compensated polarised light microscopy, which shows strongly birefringent (negative sign) needle-shaped crystals. The X- ray changes described occur in chronic tophaceous gout. Uric acid levels may fall immediately after an acute attack and in practice they should be repeated several weeks after the attack

BOF: 197 A 45-year-old male presents with cough and breathlessness. H e has had a long-standing cough productive of copious amounts of sputum. Sputum production is increased in the morning and on changing posture. A few days prior to presentation the sputum has changed colour becoming brown in colour. It has also become thicker and foul smelling. On examination the patient is febrile, breathless, has a tinge of central cyanosis, clubbing, tachycardia and coarse, bilateral basal crepitations (crackles) The major pathogens that cause infective exacerbations in the condition this patient has are: a) Staphylococcus aureus, Pseudomonas aeruginosa, Haemophilus influenzae, anaerobes b) Streptococcus pneumoniae, Klebsiella pneumoniae, Aspergillus fumigatus anaerobes c) Streptococcus pneumoniae, Staphylococcus aureus, Haemophilus influenzae anaerobes

86

Mastering MRCP d) Staphylococcus aureus Klebsiella pneumoniae Aspergillus fumigatus anaerobes e) Pseudomonas aeruginosa, Klebsiella pneumoniae, Aspergillus fumigatus anaerobes Answer: a) The clinical features suggest the patient has bronchiectasis. In bronchiectasis the organisms that are likely to cause infective exacerbations are: Staphylococcus aureus, Pseudomonas aeruginosa, Haemophilus influenzae, anaerobes

BOF: 198 A 27-year-old female has been diagnosed as having thyrotoxicosis and has been started on carbimazole. When counseling this patient you would tell her to stop the drug immediately and report to a doctor if she develops: a) Anorexia b) Abnormalities of taste c) Abnormalities of smell d) Arthralgia e) Mouth ulcers Answer: e) Carbimazole is a member of the thionamide group of drugs. The adverse effects that occur when treating patients with this group of drugs are: Minor effects: Rash, urticaria, arthralgia, fever, anorexia, abnormalities of taste and smell Major effects: Agranulocytosis, thrombocytopenia, acute hepatic necrosis, cholestatic hepatitis, lupus-like syndrome, vasculitis Patients should be advised to stop the drug and report to a doctor for a blood count if they develop: Sore throat, fever, bruising or mouth ulcers

BOF: 199 A 30-year-old male, who admits to having unprotected sexual intercourse 3 days before presentation, presents with multiple, painful ulcers on his penis. There is unilateral inguinal lymphadenopathy. In this patient the causative organism is likely to be: a) Treponema pallidum b) Haemophilus ducreyi c) Chlamydia trachomatis d) Neisseria gonorrhoea e) Klebsiella granulomatis Answer: b) The clinical features suggest the patient has chancroid, which is caused by Haemophilus ducreyi. Treponema pallidum causes syphilis, which presents as a solitary painless chancre. Chlamydia trachomatis types LGV 1, 2, 3-cause lymphogranuloma venereum, which presents as a painless ulcerating papule and regional lymphadenopathy.

87

Mastering MRCP Neisseria gonorrhoea causes gonorrhoea Klebsiella granulomatis causes Donovanosis, which presents as a heaped up ulcerating lesion with prolific red granulation tissue on the external genitalia, perianal skin, or inguinal region within 1-4 weeks of exposure

BOF: 200 A 64-year-old patient complains of headache. On examination of his visual fields you detect a right upper quadrantopia. In this patient the lesion is in the: a) Right temporal lobe b) Left temporal lobe c) Right parietal lobe d) Left parietal lobe e) Optic chiasma Answer: b) Quadrantopia refers to loss of one quadrant of the visual field. Upper quadrantopia is due to temporal lobe lesions, lower quadrantopia to parietal lobe lesions. Hence, right upper quadrantopia would be due to a left temporal lobe lesion. Lesions in the optic chiasma typically cause bi-temporal hemianopia

BOF: 201 A 42-year-old female presents with stiffness, pain and swelling of her hands and wrists of several months duration. She feels generally unwell and tired. On examination she has firm subcutaneous nodules over her elbows, swelling of her wrists, ulnar deviation at the metacarpophalangeal joints and swelling of her proximal interphalangeal joints. Investigations reveal a normocytic, normochromic anaemia, elevated ESR and CRP. She is sero-positive. The commonly employed test to detect sero-positivity tests: a) IgG antibody b) IgA antibody c) IgM M antibody d) IgE antibody e) IgD antibody Answer: c) The patient has features of rheumatoid arthritis. The rheumatoid factor is a circulating antibody directed against the Fc fragment of immunoglobulin. The antibody may IgM, IgG, or IgA. The commonly employed tests detect IgM rheumatoid factor

BOF: 202 A 30-year-old male presents with diarrhoea with passage of blood and mucous. The diarrhoea has been a feature for approximately 8 weeks but has increased over the last week. At present he is going to the toilet > 10 times a day, passing a lot of blood and mucous. On examination he looks unwell, has decreased skin

88

Mastering MRCP elasticity, a pulse rate of 110 beats per minute. Abdomen mildly distended, diffuse tenderness, bowel sounds normal. Stool culture performed by his GP prior to admission has not revealed any pathogens. Unprepared sigmoidoscopy shows an erythematous, granular mucosa with multiple small ulcers and contact bleeding. In this patient the factors determining severity of the attack are: a) Stool frequency > 6 /day with blood, fever > 37.5 ° C, Tachycardia >90 /minute, ESR 30 mm in 1st hour, Anaemia <10g/dL, Albumin < 30 g/L b) Stool frequency > 6 /day with blood, fever > 37.5 ° C, Tachycardia >90 /minute, AXR showing thumb printing, Anaemia <10g/dL, Albumin < 30 g/L c) Stool frequency > 6 /day with blood, blood urea > 10 mmol/L, Anaemia <10g/dL, Albumin < 30 g/L, fever > 37.5 ° C, Tachycardia >90 /minute d) Stool frequency > 6 /day, with blood fever > 37.5 ° C, Tachycardia >90 /minute AXR showing dilatation of the colon, Anaemia <10g/dL, Albumin < 30 g/L e) Stool frequency > 6 /day with blood, ESR .30 mm in 1st hour, Anaemia <10g/dL, Albumin < 30 g/L, blood urea > 10 mmol/L, AXR showing thumb printing Answer: a) The presenting features are suggestive of ulcerative colitis. In UC the factors that determine severity of the attack are: Stool frequency > 6 /day with blood Fever > 37.5 ° C Tachycardia >90 /minute ESR .30 mm in 1st hour Anaemia <10g/dL Albumin < 30 g/L

BOF: 203 A 64-year-old male has been started on sildenafil by his GP. The patient has angina pectoris. Which of the following drugs should be not be used in this patient? a) Beta blockers b) Nitrates c) Aspirin d) Clopidrogel e) ACE inhibitors Answer: b) When using sildenafil, nitrates and nicorandil should not be used concomitantly as they cause an enhanced hypotensive effect

BOF: 204 A 65-year-old male presents with haemoptysis. He does not smoke but gives a history of having worked in the shipbuilding industry. On clinical examination no abnormalities are seen. CXR show pleural thickening affecting the parietal pleura and calcification if the parietal and diaphragmatic pleura.

89

Mastering MRCP In this patient which of the following statements is true regarding the risk of developing bronchial cancer: a) The risk is not increased above the normal, as the patient does not smoke. b) The risk of bronchial cancer is increased only if there is evidence of parenchymal lung disease c) He is at high risk of developing small cell lung cancer d) If he was a smoker, the risk of developing lung cancer would have been additive to the risk involved in his condition e) Pleural thickening alone confers an increased risk of developing lung cancer, even in non-smokers Answer: e) The radiological features suggest the patient has asbestos related pleural plaques. The history of working in the shipbuilding industry is significant in this regard. Asbestos related lung disease confers an increased risk of development of lung cancer. It is usually bronchial adenocarcinoma. The risk is increased in those who have parenchymal disease but it is also increased in those who have pleural plaques without parenchymal involvement. In smokers there is a synergistic effect between smoking and asbestos related disease. This causes a fivefold multiplication in risk rather than an additive effect

BOF: 205 A 68-year-old male who has worked in the building industry presents with a history of chest pain and dyspnoea. On examination he has evidence of a right-sided pleural effusion, which is confirmed by chest xray. Pleural aspiration is performed and a pleural biopsy taken. Histology from the biopsy shows mesothelioma. In this patient: a) Chemotherapy should be given to prevent seeding of tumour cells along the biopsy track b) Surgery should be performed to prevent seeding of tumour cells along the needle track c) Radiotherapy should be given to prevent seeding of tumour cells along the needle track d) Curative chemotherapy should be given e) Curative radiotherapy should be given Answer: c) In mesothelioma no treatment has been shown to influence the universally fatal outcome. After obtaining a positive biopsy. Radiotherapy should be given in an attempt to prevent seeding of tumour cells along the needle track

BOF: 206 A 26-year-old male is admitted with severe pain radiating from his left loin to the left groin. He also has haematuria. He gives a history of previous urinary tract infections. Clinical examination is unremarkable. Investigations are as follows: Serum Sodium 140mmol/L Serum Potassium 2.8 mmol/L Urea 5.7 mmol/L Creatinine 107 micromoles/L

90

Mastering MRCP Chloride 115 mmol/l Bicarbonate 16 mol/L In this patient which of the following would you use to prevent recurrence of his problems? a) Allopurinol b) Probenecid c) Sulphinpyrazone d) Long term antibiotics e) Sodium bicarbonate Answer: e) The patient presented with a history of renal colic and previous urinary tract infections Investigations reveal a normal anion gap acidosis. These features suggest a diagnosis of distal renal tubular acidosis. Sodium bicarbonate is effective in decreasing the acidosis and also halts progression of nephrolithiasis

BOF: 207 These results were obtained at cardiac catheterisation in a 28-year-old male who complains of fatigue and dyspnoea on exertion: Pressure mm Hg SVC

Oxygen Saturation % 72

IVC

76

RA (mean) RV PA LV

8 52/0 52/17 98/10

75 84 85 98

Aorta

100/70

97

The diagnosis is: a) Ventricular septal defect b) Ebstein’s anomaly c) Primary pulmonary hypertension d) Patent ductus arteriosus e) Fallot’s tetralogy Answer: a) There is a step up in oxygen saturation in the right ventricle suggestive of a left to right shunt at the level of the ventricular septum. The pulmonary artery pressure is high suggesting that pulmonary hypertension has developed but the shunt has not reversed

91

Mastering MRCP BOF: 208 A 24-year-old male who is known to have sickle cell anaemia is admitted with a history of headache, fever and photophobia. On examination he is febrile, no rash, he has neck stiffness. A lumbar puncture is performed and reveals turbid CSF under pressure. Which organism is most likely to be responsible for this condition? a) Haemophilus influenza b) Neisseria menigitidis c) Streptococcus pneumoniae d) Cryptococcus neoformans e) Listeria moncytogenes Answer: c) In sickle cell anaemia susceptibility to infections increases. This is particularly so for infections with Streptococcus pneumoniae which can cause fatal meningitis or pneumonia

BOF: 209 A 24-year-old female is admitted with a history of vague headache, lassitude, anorexia and vomiting of a few weeks duration. There is a history of seizures prior to admission. Over the last few months she has been treated for oral thrush on several occasions by her GP. She has been steadily losing weight. On examination she is febrile, drowsy and she has neck stiffness. CT scan does not show a mass lesion. Blood: Urea: 8.2 mmol/L Sodium 140 mmol/L Potassium 3.8 mmol/L Glucose 6.6 mmol/L CSF: Red cells 12/ml White cells 110 /ml (66% lymphocyte, 30 % polymorphs) Protein1.8g/L Glucose 2.6 mmol/L Which organism is the likely cause of this patient’s condition? a) Neisseria meningitidis b) Streptococcus pneumoniae c) Staphylococcus aureus d) Echo virus e) Cryptococcus neoformans Answer: e) The patient has chronic meningitis on a background of loss of weight and probable immunosuppression as suggested by the oral candidiasis. The likely cause is cryptococcal meningitis

92

Mastering MRCP BOF: 210 A 32-year-old male presents with a history of dysuria, discharge per urethra and discomfort in the penis. He gives a history of unprotected sexual intercourse with a casual partner 4 days prior to the development of symptoms. An urethral swab is examined and on gram stain this shows numerous polymorphs with many intracellular gram-negative diplococci. The causative organism is: a) Neisseria gonorrhoeae b) Trichomonas vaginalis c) Chlamydia trachomatis d) Ureoplasma urealyticum e) Mycoplasma genitilium Answer: a) The patient has urethritis. Urethritis may be gonococcal or non-gonococcal. Non-gonococcal urethritis (NGU) may be due to Chlamydia trachomatis, Trichomonas vaginalis, Ureoplasma urealyticum, Mycoplasma genitilium and Bactroides spp. HSV may also cause NGU. Chancres and warts within the urethra can also cause NGU. The presence of gram-negative intracellular diplococci suggests the diagnosis is gonorrhoea

BOF: 211 A 32-year-old male is admitted for investigation of fever. The fever is intermittent and comes on every 72 hours and is accompanied by chills and rigors. His spleen is just palpable and firm. A Giemsa stained thin blood film shows malarial parasites. Which species is it likely to be? a) Plasmodium falciparum b) Plasmodium vivax c) Plasmodium ovale d) Plasmodium malariae e) Plasmodium simium Answer: d) Plasmodium malariae causes quartan fever (every 72 hours) whereas the other species cause tertian fever (every 48 hours). Plasmodium simium is a simian (monkey) plasmodium that may rarely infect humans

BOF: 212 A 57-year-old male presents with a history of feeling tired and lethargic. There is a history of ureteric colic. Investigations reveal: Corrected calcium 3.1 mmol/L Serum phosphate 0.5 mmol/L Plasma parathyroid hormone 6.2 pmol/L (normal 0.9-5.4 pmol/L) In this patient, which of the following features would suggest the hypercalcaemia is long-standing?

93

Mastering MRCP a) Polyuria b) Abdominal pain c) Depression d) Corneal calcification e) Dehydration Answer: d) The patient has primary hyperparathyroidism. Hypercalcaemia may result in all the above features. Corneal calcification causes no symptoms but reflects long-standing hypercalcaemia

BOF: 213 A 67-year-old diabetic male who has peripheral vascular disease is being investigated for hypertension. On examination of his abdomen a bruit is audible in the epigastrium. Urine: Proteins ++ Abdominal ultrasound reveals a difference in length between the two kidneys of 2.0 cms In this patient the investigation that is most likely to be of benefit is: a) Isotope renography b) MRI c) CT scan d) IVU e) Renal arteriogram Answer: e) The patient is an arteriopath (generalized and unspecified atherosclerosis )who has features of renal artery stenosis. A difference in length of over 1.5 cms between the two kidneys is significant. All the tests listed above will point to this diagnosis. Renal arteriography is considered to be the gold standard and in addition it has therapeutic potential in that angioplasty may be performed

BOF: 214 A 28-year-old male who is known to have hereditary spherocytosis (heterozygous) is married to an unaffected female. What is the chance of their first-born child being affected? a) 0% b) 25% c) 50% d) 75% e) 100% Answer: c) Hereditary spherocytosis is inherited in an autosomal dominant manner. Thus the chance of the first child being affected is 50%

94

Mastering MRCP BOF: 215 An 18-year-old male has a chronic cough productive of copious sputum. He has a history of recurrent bronchopulmonary infections. On examination he is clubbed and there are coarse mid and late inspiratory crepitations (crackles) at both lung bases. His sweat sodium concentration is 80 mmol/L (normal 60 mmol/L) In this patient the mode of inheritance of the condition is: a) Autosomal recessive b) Autosomal dominant c) X-linked recessive d) X-linked dominant e) Polygenic inheritance Answer: a) The clinical features suggest the patient has bronchiectasis and together with a high sweat sodium concentration this should point to the diagnosis of cystic fibrosis. The condition is inherited in an autosomal recessive manner and is caused by a gene mutation on the long arm of chromosome 7

BOF: 216 A 38-year-old female is being investigated for hypertension. She complains of polyuria, and muscle weakness. There is no family history of early hypertension. Investigations reveal: Serum sodium 145 mmol/L Serum potassium 2.2 mmol/L Chloride 103 mmol/L Bicarbonate 32 mmol/L Urea 7.4 mmol/L Which of the following conditions is most likely to account for her condition? a) Adrenal adenoma b) Bilateral adrenal hyperplasia c) Adrenal carcinoma d) Glucocorticoid remediable aldosteronism e) Unilateral adrenal hyperplasia Answer: a) The features are suggestive of primary hyperaldosteronism. The most common cause of primary hyperaldosteronism is unilateral adenoma (Conn’s syndrome). This is more common in young females and hence in this patient this is the most likely cause. Bilateral adrenal hyperplasia is less likely below the age of 40 and is more common in males. The absence of a family history of early hypertension makes GRA less likely. Adrenal carcinoma and unilateral adrenal hyperplasia are exceedingly rare

BOF: 217 A 45-year-old female presents complaining of bone pain and muscular weakness. On examination she has a waddling gait. X-ray shows pseudofractures of her pubic rami.

95

Mastering MRCP Investigations show: Serum corrected calcium 2.09 mmol/L Serum phosphate 0.48 mmol/L Serum alkaline phosphatase 220 U/L Which of the following is an unlikely cause of her condition? a) Malabsorption syndrome b) Renal failure c) Chronic liver disease d) Mesenchymal tumour e) Calcium deficiency Answer: e) The clinical radiological and biochemical features in this patient suggest she has osteomalacia. Osteomalacia may be caused by deficiency of vitamin D or phosphate deficiency. Malabsorption syndrome, renal failure and liver disease result in vitamin D deficiency. Mesenchymal tumours may result in phosphate deficiency. Calcium deficiency per se very rarely causes osteomalacia

BOF: 218 A 70-year-old male is admitted to your ward with pneumonia. He has impaired hearing in his left ear, his skull vault is enlarged, the right tibia is bowed and warm to touch and he has a kyphoscoliosis. In this patient one of the complications of immobilisation specific to his condition is: a) Hypercalcaemia b) Heart failure c) Gout d) Skin ulceration e) Pathological fracture Answer: a) The patient has the clinical features of Paget’s disease of bone. In this condition immobilisation is likely to cause hypercalcaemia

BOF: 219 A 24-year-old male presents with fatigue, poor exercise tolerance and loss of weight. On examination he looks underweight, pulse rate of 120 beats per minute regular. The pulse volume decreases during inspiration. His JVP is elevated 5 cms and the level increases during inspiration. On auscultation of the heart sounds an early diastolic sound is audible at the left sternal edge. On examination of the abdomen there is 4 fingerbreadths of hepatomegaly and ascites. In this patient the early diastolic added sound is due to: a) Opening of a thickened atrioventricular valve b) Tumour plop c) Rapid ventricular filling d) Arrest of ventricular filling e) Pericardial crunch

96

Mastering MRCP Answer: d) The patient has the clinical features of constrictive pericarditis. In constrictive pericarditis an early diastolic sound is heard (pericardial knock). This sound is due to turbulence caused by arrest of rapid ventricular filling by the non-distensible pericardial sac

BOF: 220 A 45-year-old male is being investigated in the hypertension clinic. He complains of panic attacks, fever, headache, tremor and palpitations. On examination he is hypertensive, has a tachycardia and has glycosuria. 24 hour urinary vanillyl mandelic acid is measured at 70 micromoles/ 24h (normal 5-35 micromoles/24 h) In this patient treatment of hypertension should be with: a) Phenoxybenzamine before propranolol b) Propranolol before phenoxybenzamine c) Clonidine d) Labetolol e) Carvedilol Answer: a) The clinical features and investigations suggest the patient has a phaeochromocytoma. The treatment of hypertension in phaeochromocytoma is with alpha blockade prior to beta blockade. Alpha blockade reverses the peripheral vasoconstriction whereas beta blockade prevents tachycardia. The preferred alphablocker is not a selective alpha one blocker but an irreversible alpha-blocker whose effects cannot be overcome by a surge of catecholamines. Hence, phenoxybenzamine is the drug of choice

BOF: 221 An 18-year-old male presents with a history of dyspnoea on exertion and orthopnoea, fatigue and anorexia. On examination he has peripheral oedema, a rapid low volume pulse, no paradox, elevated JVP with a rapid y descent, no inspiratory increase in JVP, a quiet praecordium and characteristic auscultatory features of his condition. He also has hepatomegaly and ascites. His chest X-ray demonstrates a small heart with no pericardial calcification. ECG shows a widened QRS complex with diffuse non-specific repolarisation changes. In this patient the characteristic auscultatory feature mentioned above is: a) Third heart sound b) Diastolic knock c) Fourth heart sound d) Pericardial rub e) Pericardial crunch Answer: c) The clinical radiological and ECG features suggest the patient has a restrictive cardiomyopathy. The characteristic auscultatory feature of this condition is a fourth heart sound reflecting increased atrial contraction in an effort to overcome the reduced compliance of the ventricle.

97

Mastering MRCP A third heart sound may rarely be heard in the late stage of this disease. A diastolic knock is a feature of constrictive pericarditis, which has been ruled out by the absence of Kussmaul’s sign (inspiratory increase in JVP) and absence of pericardial calcification. A pericardial crunch is heard in left sided pneumothorax or pneumomediastinum

BOF: 222 40 year old female returns from a holiday and presents to you with fever, headache, anorexia and constipation. She admits to having partaken of food and water from sources, which were of questionable hygienic standards. She also admits to heavy alcohol consumption of over 35 units per week over several years. On examination she is febrile, temperature of 38.7°C. She has faint salmon pink blanching, macules on her chest and abdomen. Her pulse rate is 68 beats per minute. The spleen is just palpable and is soft. In this patient you arrange for blood cultures and a Widal test. In the interpretation of the Widal test which of the following is true? a) Paired specimens taken one week apart should show a twofold rise b) It is highly sensitive c) It is highly specific d) False positive tests occur in cirrhosis of the liver e) Paired specimens taken two weeks apart show a twofold rise Answer: d) The patient has a febrile illness following exposure to unhygienic sources of food and water. She has constitutional symptoms and on examination she has a relative bradycardia, rose spots and splenomegaly. The clinical diagnosis is enteric (typhoid) fever. In the Widal test a fourfold rise in titer in paired specimens taken two weeks apart suggests Salmonella typhi infection. The test is not sensitive (30%) in culture proven patients. It is non-specific. Non-typhoid salmonella can cross-react and cirrhosis is associated with non-specific antibody production

BOF: 223 An 18-year-old male is admitted with severe colicky abdominal pain. On examination he has multiple, small, pigmented macules around his lips and in his mouth. His pulse rate is 110 beats per minute, his abdomen is distended and percussion note over it is tympanitic. No lumps are palpable but bowel sounds are exaggerated. The inheritance of the underlying condition is: a) Autosomal dominant b) Autosomal recessive c) X-linked dominant d) X-linked recessive e) Polygenic inheritance Answer: a) This is a young patient admitted with features of intestinal obstruction and peri-oral mucocutaneous pigmentation. The diagnosis is Peutz-Jegher’s syndrome, which has autosomal dominant inheritance

98

Mastering MRCP BOF: 224 A 24-year-old female presents with a history of passing large volumes of urine and excessive thirst. Clinical examination is unremarkable. The specific gravity of her urine is 1.003 and urine osmolality is 140 mOsm/L In this patient you arrange a water deprivation test. In the interpretation of the water deprivation test which of the following statements is true? a) A normal response is characterised by the maximum urine osmolality reaching the plasma osmolality b) In a normal response following injection of vasopressin the urine osmolality increases by > 50% c) Patients with nephrogenic diabetes insipidus do not concentrate their urine but show a marked response to vasopressin injection d) Patients with diabetes insipidus are unable to concentrate their urine and the response to vasopressin injection is an increase in concentration by 5 % e) Patient s with partial diabetes insipidus are able to concentrate their urine but show a response to vasopressin injection of > 9% Answer: e) In a normal response to the water deprivation test the maximum urine osmolality exceeds plasma osmolality and the urine osmolality does not increase > 5 % after administration of vasopressin. In diabetes insipidus, patients are unable to concentrate their urine to greater than plasma osmolality but after administration of vasopressin the urine osmolality increases by > 50 %. In partial diabetes insipidus, the patients are often able to concentrate their urine to over plasma osmolality but the response to vasopressin injection is > 9% Patients with nephrogenic diabetes insipidus are unable to concentrate their urine and they show no response to administration of vasopressin

BOF: 225 A 63-year-old male is admitted with a history of cough and fever. He has a cough productive of purulent, blood-stained sputum and a swinging temperature. On examination he is febrile, clubbed, tachycardic, tachypnoeic. The trachea is in the midline; respiratory movements are diminished in the right lower zone. Vocal fremitus is increased at the right base; percussion note in this area is dull. The breath sounds are amphoric in nature, vocal resonance is increased and whispering pectoriloquy is demonstrated. Localised crepitations (crackles) are audible in this area and there is a pleural rub. In this patient which of the above physical signs is indicative of lung abscess: a) Clubbing b) Tachypnoea c) Whispering pectoriloquy d) Increased vocal resonance e) Amphoric breath sounds Answer: e) Amphoric breath sounds are caused by a bronchus communicating with a large cavity or pneumothorax. The sound is similar to that made if one were to blow across the top of an open bottle. This physical sign will suggest that the patient has a lung abscess

99

Mastering MRCP BOF: 226 A 56-year-old female presents with a history of weight loss, tremor and increased sweating. On examination she is underweight, has a tachycardia, a fine tremor and her palms are warm and moist. Investigations reveal: Plasma thyroxine 300 nmol/L TSH < 0.1 mU This patient is treated with radioactive iodine. The common adverse effect of radioactive iodine therapy is: a) Goiter b) Hyperthyroidism c) Hypothyroidism d) Carcinoma of the thyroid e) Activation of latent thyroid eye disease Answer: c) The major complication of treatment with radioactive iodine is the progressive incidence of hypothyroidism. If the patient has dysthyroid eye disease there is a likelihood that this may worsen and this represents a relative contraindication although this may be prevented by administration of steroids. Discomfort in the neck and immediate worsening of hyperthyroidism may sometimes occur. The risk of cancer is increased although it remains low in absolute terms

BOF: 227 A 68-year-old male presents with a history of progressive tiredness and breathlessness. On examination there is pallor, his pulse rate is 60 beats per minute, regular rhythm, low volume, slow rising. The JVP is not elevated. His apex beat is at the 5th left intercostal space mid-clavicular line and it is heaving in nature. There is a systolic thrill in the 2nd right intercostal space just lateral to the sternum and there is a systolic murmur in the aortic area radiating to the neck. Investigations reveal: Hb 7.0 MCV 68 fL MCHC 28 g/dL Upper GI endoscopy is normal In further investigation of this patient’s anaemia the most useful test would be: a) Colonoscopy b) Flexible sigmoidoscopy and barium enema c) Mesenteric angiography d) CT colography e) Capsule endoscopy Answer: a) The patient has the clinical features of aortic stenosis and he is anaemic. In a patient in this age group colon cancer is a possibility and imaging the large bowel in important in investigating this patient. However, aortic stenosis may be associated with angiodysplasia of the colon and the test that is likely to demonstrate this is colonoscopy. Mesenteric angiography will only demonstrate active bleeding or large arterio-venous malformations

100

Mastering MRCP BOF: 228 A 28-year-old female of Irish descent presents with a history of mouth ulcers, abdominal pain and bloating, diarrhoea and loss of weight. On examination she looks underweight, and has aphthous ulcers in her mouth. Immunology: Anti-endomysial antibodies positive Her blood film shows: Microcytes, macrocytes, hypersegmented polymorphs and Howell-Jolly bodies This suggests the patient has: a) Iron deficiency, folate deficiency, hypersplenism b) Iron deficiency, folate deficiency, hyposplenism c) Iron deficiency, vitamin B12 deficiency, hypersplenism d) Iron deficiency, vitamin B 12 deficiency, hyposplenism e) Pyridoxine deficiency Answer: b) The history and positive immunology suggest the patient has coeliac disease . In coeliac disease folic acid and iron deficiency occur and splenic atrophy is a complication. The blood film thus shows features of iron and folate deficiency and features of hyposplenism

BOF: 229 With regard to the conducting system of the heart, what is the position of the Purkinje fibers ? a) Subendocardial b) Subepicardial c) Transmural d) Runs in the middle of the myocardium e) Runs at varying levels in the myocardium Answer: a) The Purkinje fibers run in a subendocardial position. This results in depolarisation of the heart running from endocardium to epicardium. This is important in understanding the principles of electrocardiography

BOF: 230 In relation to the conducting system of the heart conduction of electrical activity is: a) Faster in the left bundle than in the right b) Faster in the right bundle than in the left c) Equally fast in both bundle branches d) Begins with the left bundle branch conducting faster and later the right bundle conducts faster e) Begins with the right bundle branch conducting faster and later the left bundle branch conducts faster Answer: a) The left bundle branch conducts faster than the left and this results in depolarisation of the interventricular septum occurring from left to right and is important in understanding principles of electrocardiography

101

Mastering MRCP

RA (mean) RV PA LV

Pressure mm Hg 0 24/0-2 24/12 220/0-9

LA

7

Femoral

90/60

Aorta

170/100

In this patient the chest x-ray feature that is most likely to lead to a diagnosis is: a) Left ventricular enlargement b) Dilated ascending aorta c) Pulmonary plethora d) Pruning of the pulmonary vasculature e) Rib notching Answer: e) The history of hypertension and the pressure data point to a diagnosis of coarctation of the aorta. In this condition enlarged intercostal collaterals cause notching of the ribs. The other features mentioned are not diagnostic of coarctation of the aorta

BOF: 231 In relation to the conducting system of the heart conduction of electrical activity is: a) Faster in the left bundle than in the right b) Faster in the right bundle than in the left c) Equally fast in both bundle branches d) Begins with the left bundle branch conducting faster and later the right bundle conducts faster e) Begins with the right bundle branch conducting faster and later the left bundle branch conducts faster Answer: a) The left bundle branch conducts faster than the left and this results in depolarisation of the interventricular septum occurring from left to right and is important in understanding principles of electrocardiography

BOF: 232 A 17-year-old boy has chronic anaemia. He is short and has an abnormal facies with frontal and parietal bossing, enlargement of the malar (maxillary) bones and protruding teeth. On examination of the abdomen he has hepatosplenomegaly. Investigations are as follows: Hb 7.0 MCV 58 In this patient which therapeutic intervention listed below would be helpful in treating his anaemia?

102

Mastering MRCP a) Oral iron b) Parenteral iron c) Folic acid d) Ascorbic acid e) Vitamin B12 Answer: c) The history of chronic anaemia and the features on examination suggest a chronic haemolytic anaemia with extramedullary erythropoiesis. This together with the microcytic anaemia should suggest the diagnosis of thalassaemia. In thalassaemia folic acid supplementation is useful in treatment of anaemia as there is increased demand for folic acid. Iron is not required as the problem in these patients is iron overload for which desferrioxamine and ascorbic acid are used. Vitamin B12 deficiency is not a feature

BOF: 233 A 50-year-old female has been diagnosed as having rheumatoid arthritis. She has been started on a nonsteroidal anti-inflammatory drug (NSAID). Where pain relief is concerned which of the following statements is true? a) Full analgesic effect is achieved within 24 hours of starting the drug b) Full analgesic effect is achieved within 48 hours of starting the drug c) Full analgesic effect is achieved within 72 hours of starting the drug d) Full analgesic effect is achieved within 1 week of starting the drug e) Full analgesic effect is achieved within 3 weeks of starting the drug Answer: d) Pain relief begins soon after starting the drug and full analgesic effect is normally achieved within one week. If an appropriate response is not obtained after this time another agent may be tried, as there is considerable variation in individual patient tolerance and response to different agents

BOF: 234 A 57-year-old lady has been started on a non-steroidal anti-inflammatory agent for treatment of rheumatoid arthritis. In this patient which of the following statements is true regarding the antiinflammatory effect of NSAIDs? a) Full anti-inflammatory effect is achieved within 24 hours b) Full anti-inflammatory effect is achieved within 48 hours c) Full anti-inflammatory effect is achieved within 72 hours d) Full anti-inflammatory effect is achieved within 1 week e) Full anti-inflammatory effect is achieved within 3 weeks Answer: e) Full anti-inflammatory effect of NSAIDs may not be achieved for up to 3 weeks and this length of time should be allowed to elapse before another agent is considered. Patients who do not respond to one agent may respond to another as there is as there is considerable variation in patient tolerance and response to different agents

103

Mastering MRCP BOF: 235 A 22-year-old female is admitted with a history of loss of weight, diarrhoea and secondary amenorrhoea. Her mother is known to have hypothyroidism and is on thyroxin. On examination she is thin, there are no thyroid eye signs, her palms are warm and sweaty, there is a fine tremor of her hands and she has a resting tachycardia. Her thyroid gland is not palpable; there is no tenderness over her thyroid. Pemberton’s sign cannot be elicited. There is evidence of proximal myopathy. Chest x-ray is unremarkable. Investigations show raised levels of T4 and T3 and TSH levels are undetectable. In this patient which of the following features favours the diagnosis of thyrotoxicosis factitia? a) No ophthalmopathy b) No thyroid acropachy c) No thyroid tissue palpable d) Secondary amenorrhoe e) Proximal myopathy Answer: c) Thyrotoxicosis factitia refers to surreptitious ingestion of thyroid hormone in supra-physiological doses. Patients become clinically thyrotoxic but do not have the eye signs of Grave’s disease (lid retraction due to sympathetic over activity may occur) Due to suppression of TSH shrinkage of the thyroid gland occurs and no thyroid tissue is palpable. In Grave’s disease, toxic adenoma and toxic nodular goiter, thyroid tissue is palpable. In DeQuervain's thyroiditis the thyroid gland is tender. In silent thyroiditis a firm painless thyroid gland may be felt

BOF: 236 A 32-year-old male presents following an injury to his right upper limb. On examination of the right upper limb there is incomplete and defective pronation. The wrist flexors are paralyzed when examined against resistance. When this is tested the tendon of flexor carpi ulnaris stands out and the hand becomes ulnar deviated. Flexion of the ulnar two fingers is possible although it is weaker than normal. The index finger cannot be flexed and the middle finger can only be flexed incompletely. Flexion at the metacarpophalangeal joints is possible. Flexion of the terminal interphalangeal joint of the thumb is not possible if the proximal phalanx of the thumb is immobilized. Abduction and opposition of the thumb is defective. There is sensory loss over the lateral three and a half digits of the hand and the lateral aspect of the palm. In this patient the injury is at the: a) Median nerve at the wrist b) Median nerve at the elbow c) Ulnar nerve at the elbow d) Ulnar nerve at the wrist e) Musculocutaneous nerve Answer: b) An injury to the median nerve at the elbow causes weakness of pronator teres, radial flexors of the wrist, the long finger flexors except the ulnar half of the deep flexors, most of the muscles of the thenar eminence

104

Mastering MRCP and the two radial lumbricals. Sensory loss occurs over the lateral three and a half digits of the hand and the lateral aspect of the palm. Vasomotor and trophic changes may occur causing reddening of the skin, which also becomes dry and atrophic. The pulp of the affected finger becomes atrophic and ulceration develops at the tips of the fingers. The nails may become dry and atrophic

BOF: 237 You are called to see a patient in the burns unit. The patient has been admitted with severe burns the previous day and has now become dyspnoeic. On examination the patient is cyanosed, tachpnoeic, has intercostal and subcostal indrawing and bilateral basal fine crepitations. Blood gas analysis shows a respiratory alkalosis, low PaO2 and low PaCO2. Chest x-ray shows bilateral alveolar infiltrates and a normal cardiac silhouette. In this patient one of the factors involved in pathogenesis is reduced production of surfactant by: a) Alveolar macrophages b) Type I pneumocytes c) Type II pneumocytes d) Goblet cells e) Ciliated columnar epithelial cells Answer: c) The patient has developed the acute (adult) respiratory distress syndrome. One of the factors involved in the pathogenesis of this condition is reduced production of surfactant by type 2 pneumocytes

BOF: 238 A 28-year-old male presents with abdominal pain mainly in the right hypochondrium. The pain is aggravated by movement and is referred to the right shoulder. He also has a swinging fever with chills and sweats. He gives a history of diarrhoea a few weeks prior to this presentation and has been on holiday in the Far East and admits to not been careful regarding food hygiene. On examination he is febrile, looks unwell. The liver is enlarged and tender and there is marked intercostal tenderness. Ultrasound shows a fluid-filled cavity in the right lobe of the liver and aspiration reveals thick, chocolate brown fluid. He is started on treatment for this and two days later develops tachycardia, hypotension, and muscular weakness. Which of the following drugs is the likely cause of these latter features? a) Metronidazole b) Diloxanide furoate c) Dehydroemetine d) Chloroquine e) Iodoquinol Answer: c) The clinical features suggest the patient has developed an amoebic liver abscess.

105

Mastering MRCP Aspiration of thick chocolate brown fluid is highly suggestive. Trophozoites are rarely seen in aspirate. Amoebic fluorescent antibody would be an useful test as it is positive in 90% of patients with amoebic liver abscess. Treatment of these patients is with metronidazole but sometime emetine or dehydroemetine may be needed. Emetine and dehydroemetine are toxic and may cause tachycardia, hypotension, muscular weakness, marked gastrointestinal effects and dermatoses. Emetine is contraindicated in pregnancy, renal and cardiac disease. Chloroquine is usually combined with emetine when treating amoebic abscesses. Diloxanide and iodoquinol are used to eradicate luminal forms

BOF: 239 A 65-year-old male who has reversible airways obstruction has been started on a long acting theophylline. Improvement has not occurred and you arrange for theophylline levels to be performed to try and optimise treatment. You are aware that a considerable variation in plasma theophylline levels may occur and that theophylline levels are reduced in: a) Cirrhosis of the liver b) Heart failure c) Elderly patients d) Viral infections e) Chronic alcoholism Answer: e) Theophylline is metabolized in the liver. Considerable variation in plasma levels occurs. The plasma theophylline levels are increased in heart failure, cirrhosis of the liver, viral infections, elderly patients, and by drugs that inhibit metabolism of theophylline. The plasma levels of theophylline are decreased in smokers, chronic alcoholism and by drugs that induce liver metabolism

BOF: 240 A 67-year-old female who has mitral valve disease and atrial fibrillation is being treated with amiodarone. Her GP has checked her cholesterol levels and started her on simvastatin. She comes to see you at clinic and you realize that this combination of drugs increased the risk of developing: a) Myopathy b) Diffuse pulmonary alveolitis c) Optic neuritis d) Corneal microdeposits e) Alopecia Answer: a) All of the above are side-effects that may occur when patients are treated with amiodarone. However, the combination of amiodarone and simvastatin increases the risk of developing myopathy

106

Mastering MRCP BOF: 241 A 36-year-old female has been on long-term steroids for management of Crohn’s disease. She complains of pain in her right groin radiating down the anteromedial thigh. She has an antalgic gait and on examination of the hip there is decreased range of movement especially flexion, abduction and internal rotation. In this condition which of the following statements is true? a) An x-ray of the hip will show early changes b) CT scan is useful to show early changes c) Bone scanning is very sensitive and specific and should be used to detect early change d) MRI is specific but not sensitive and hence not useful for early disease e) CT scan is useful to rule out advanced disease if duration is not clear Answer: e) In a patient on long term steroids presenting with pain in the groin radiating to the thigh associated with an antalgic gait and decreased range of movement of the hip, the most likely diagnosis is avascular necrosis of the femoral head. In avascular necrosis of the hip, MRI is the most sensitive and specific technique and is useful for early diagnosis before collapse of bone occurs. X-ray findings may not become apparent for months to years. CT scan and x-ray are useful to rule out advanced disease if duration is not clear. Bone scanning is more sensitive than x-ray but is non-specific and MRI is preferable

BOF: 242 A 34-year-old male, who is homosexual, presents with a history of retrosternal discomfort on eating and dysphagia. He has also lost approximately 10 kgs in weight. In this patient which of the following tests would you arrange? a) HIV test b) Barium swallow c) Oesophago-gastro-duodenoscopy d) Oesophageal pH and motility studies e) Chest x-ray Answer: c) Presentation with retrosternal discomfort and dysphagia could indicate oesophagitis, which in this patient may be due to candidiasis, cytomegalovirus or herpes simplex as this patient may well have AIDS. It could also be due to severe reflux oesophagitis. The other possibilities are benign or malignant oesophageal stricture or an oesophageal dysmotility disorder. To differentiate between these conditions it would be best to visualize the oesophagus and take biopsies if required. In any case a patient presenting with dysphagia and weight loss requires urgent upper GI endoscopy. HIV testing would be performed after endoscopy if any of the infective causes of oesophagitis are detected

BOF: 243 A 28-year-old Chinese architect presents with painless lumps in the neck. This has been present for the last 7 weeks. He also says he has lost about 8 kgs in weight over the last six months and he has had fever with

107

Mastering MRCP night sweats. He smokes 20 cigarettes a day and consumes 24 units of alcohol a week. No relevant past illnesses or family history. On examination there are several enlarged lymph nodes in the right supraclavicular fossa. No other abnormal clinical findings. Investigations are as follows: Hb 10.9 g/dL MCV 80 fl WBC 15.0 x 109/L Neutrophils 56% Lymphocytes 35 % Eosinophils 9% ESR 54 mm in 1st hour Biochemistry normal In this patient which of the following tests would you arrange in order to make a diagnosis? a) Mantoux test b) Chest x-ray c) Toxoplasma serology d) Lymph node biopsy e) Serum angiotensin converting enzyme levels Answer: d) The most likely diagnosis in this patient who has cervical lymphadenopathy and B symptoms is lymphoma. An excision lymph node biopsy would be the best method of arriving at this diagnosis

BOF: 244 A 30-year-old obese female presents with a history of headache. She also reports menstrual irregularities. On examination the only abnormality detected is bilateral papilloedema. Investigations show a normal CT scan and MRI. EEG is normal. Lumbar puncture reveals elevated CSF pressure but the constituents of CSF are normal. In this patient which of the following would suggest that a complication has occurred? a) Enlarged blind spot b) Bitemporal hemianopia c) Homonymous hemianopia d) Upper quadrantopia e) Unilocular visual loss Answer: e) The history and examination findings and the results of investigations suggest the patient has benign intracranial hypertension (pseudotumour cerebri). In this condition papilloedema may result in an enlarged blind spot The complication that may occur in this condition is infarction of the optic nerve consequent on severe and long standing papilloedema. This results in uniocular visual loss

108

Mastering MRCP BOF: 245 One of your colleagues is going on an expedition to the Himalayas. He asks you about acclimatization and the role played by 2, 3 diphosphoglycerate (DPG). He asks you what stimulates production of 2, 3-DPG. Your answer is: a) Direct effect of hypoxia b) Direct effect of hypocarbia c) Increased pH d) Erythropoietin e) Carbonic anhydrase Answer: c) At high altitude the total barometric pressure falls. The low barometric pressure causes a reduction in alveolar Po2. Production of 2, 3-DPG is secondary to an increase in pH, which occurs as a consequence of hyperventilation caused by hypoxic stimulation of chemoreceptors

BOF: 246 A 56-year-old male presents with a painful swollen left knee. There is no history of trauma. He gives a history of pain and stiffness of both hands. He is impotent and he has been investigated for abdominal pain in the past but no cause was found. On examination his skin is pigmented. There is tenderness and swelling of the 2nd and 3rd metacarpophalangeal joints of both hands, his liver is enlarged 4 fingerbreadths. His knee is swollen and aspiration of the joint yields turbid, straw colored fluid. In this patient which of the following tests would identify the underlying condition? a) Plasma growth hormone levels b) Fasting blood sugar levels c) Serum urate levels d) Serum ferritin e) Blood culture Answer: d) The history of abdominal pain together with impotence, skin pigmentation, and hepatomegaly suggest the patient has haemochromatosis. Haemochromatosis is associated with chondrocalcinosis, which commonly affects the 2nd and 3rd metacarpophalangeal joints. The acute arthropathy is pyrophosphate crystal arthritis. Out of the tests given a raised serum ferritin would identify the fact that the patient has haemochromatosis and lead one to arrange confirmatory tests.

BOF: 247 A 72-year-old male is admitted with a history of headache. The headache is present when he wakes up in the morning and is accompanied by nausea. He also gives a history of collapse a few weeks prior to admission. He had been playing cards with his friends and had asked them whether anyone noticed an unusual smell and then fell and lost consciousness. He had been seen at the accident and emergency department, nothing abnormal had been found and he had been discharged.

109

Mastering MRCP On examination of the patient’s fundus, the optic cups were filled and the medial margins of the discs were blurred. Apart from this no abnormality could be found on clinical examination. The most likely cause of this patient’s condition is: a) Subdural hematoma b) Posterior fossa tumour c) Frontal lobe tumour d) Obstructive hydrocephalus e) Benign intracranial hypertension (pseudotumour cerebri) Answer: c) The history of headache worse in the morning and accompanied by nausea indicates increased intracranial pressure. This is confirmed by the fact the optic discs are filled and that the medial margins of the discs are blurred. The history of a probable seizure preceded by an aura of a strange smell suggests a focal lesion in the frontal lobe. CT scan in this patient showed a ring-enhancing lesion in the frontal lobe

BOF: 248 A 27-year-old male presents with fever and loss of weight. He has spent several months travelling across the countries around the Mediterranean, Africa and Asia. He admits that he did not bother about protecting himself from insect bites and was frequently bitten. On examination he appears to have lost weight, his skin is pigmented and he has lymphadenopathy and hepatosplenomegaly. Haematology shows pancytopaenia. Aspirates are taken from bone marrow and Giemsa stained smears of these aspirates show amastigotes. In this patient the infecting organism is likely to be: a) Leishmania donovani b) Leishmania tropica c) Leishmania aethiopica d) Leishmania major e) Leishmania garnhami Answer: a) The patient has clinical features of visceral leishmaniasis and aspirates have demonstrated amastigotes confirming the diagnosis. Visceral leishmaniasis is caused by parasites of the Leishmania donovani complex: L. donovani, L. infantum, L. chagasi

BOF: 249 A 32-year-old female has been on long-term steroids to suppress activity of Crohn’s disease. She has been started on azathioprine and disease control appears to be satisfactory. The consultant who saw her in clinic has instructed one of your juniors to tail off this patient’s steroids. He has asked the junior to reduce steroids to physiological dosage and maintain the patient on that dosage until she is next seen in clinic. The junior rings you to ask what dosage of prednisolone should be considered physiological. You reply that physiological dosage of corticosteroids is a daily dose of prednisolone: a) 15 mgs b) 10 mgs

110

Mastering MRCP c) 7.5 mgs d) 5 mgs e) 2.5 mgs Answer: c) During withdrawal of corticosteroids the dose may be rapidly reduced to physiological dosage, which is equivalent to 7.5 mgs of prednisolone daily after which reduction should be more gradual

BOF: 250 A 16-year-old male is admitted with a history of passage of blood in his urine and swelling of his face. On examination there is facial and ankle oedema. On examination of his hands there is evidence of infected scabies. His blood pressure is marginally elevated. Investigations show haematuria and red cell casts. Blood urea is elevated. C 3 and C 4 levels are reduced. Abdominal ultrasound shows normal sized kidneys. In this patient which of the following interventions would not be beneficial? a) Low salt diet b) Treatment with penicillin c) Diuretics d) Steroids e) Fluid restriction Answer: d) The clinical features together with the results of investigations suggest the patient has post-streptococcal glomerulonephritis (acute nephritic syndrome). Steroids and immunosuppressive therapy are not of benefit in these patients. Spontaneous improvement usually occurs and the aim of treatment is to prevent the patient dying of complications such as pulmonary oedema, uremia or hypertensive encephalopathy whilst awaiting spontaneous improvement in renal function. As there is evidence of active infection treatment with penicillin would be beneficial

____________________________________

_________________________________

BOF: 2.1 A 65-year-old patient presents with a history of palpitations and breathlessness. She gives a history of Sydenham’s chorea as a child. On examination she has a pulse rate of 120 beats per minute rhythm; irregularly irregular, small volume. JVP not elevated, apex beat not displaced, tapping in nature. There is left parasternal heave, the first and second heart sounds are loud and there is an opening snap and a lowpitched rumbling mid-diastolic murmur. Which of the following physical signs suggest that a complication of her condition has occurred? a) Loud first heart sound

111

Mastering MRCP b) Loud second heart sound c) Opening snap d) Mid-diastolic murmur e) Tapping apex Answer: b) The history and physical signs suggest the patient has mitral stenosis. The tapping apex beat and loud second sound reflect thickening of the mitral valve. They are caused by the thickened valve leaflets not floating up with ventricular filling in diastole and the valves shutting from a greater distance than normal, thus producing a louder sound. The opening snap reflects thickening of the valve and its presence also indicates that the valve is pliable. The mid-diastolic murmur is due to stenosis of the mitral valve. The loud second heart sound is caused by pulmonary hypertension and reflects development of this complication of mitral stenosis

BOF: 2.2 A 59-year-old male presents with a history of cough, fever and pleuritic chest pain. He had been feeling unwell for several months, complaining of feeling tired and breathless. He had consulted his GP about this and was noted to have splenomegaly but this had been attributed to tropical splenomegaly syndrome as he had spent several years in India and had had malaria several times. No investigations had been performed as his symptoms of feeling tired and breathless had been attributed to stress at work. He was not on any drugs; he smoked heavily; 20 cigarettes a day for the last 30 years. On examination he looked unwell, he was febrile, there was no clubbing, there was consolidation in the base of his left lung and he had hepatosplenomegaly. Investigations: Chest x-ray confirmed consolidation in the base of his left lung. Hb 7.6 g/dL WBC 56.9 x 109/l Neutrophils 41% Metamyleocytes 8% Myleocytes 21% Promyelocytes 18% Blast cells 9% Nucleated red cells 3% Platelets 49x109/l The features are likely to be the result of a complication of: a) Tropical splenomegaly syndrome b) Bacterial pneumonia c) Bronchial carcinoma d) Acute myeloid leukemia e) Chronic myeloid leukemia Answer: e) The clinical features are not in keeping with tropical splenomegaly syndrome and the patient would have had haematological features of hyposplenism

112

Mastering MRCP If this were a leukaemoid reaction due to severe bacterial pneumonia or due to marrow replacement by malignant cells from a bronchial cancer infiltrating the bone marrow, this would not explain long-standing splenomegaly. The absence of clubbing would not favor bronchial carcinoma. Acute myeloid leukemia would not be supported by the fact that mature forms are seen and there is a high neutrophil count. Chronic myeloid leukemia in its accelerated phase with blast transformation is thus the most likely diagnosis and would explain all the clinical features in this patient

BOF: 2.3 A 70-year-old male presents with a history of syncope. On examination he has a pulse rate of 60 beats per minute, small volume, slow rising. His jugular venous pressure is not elevated; apex beat is at the 5th left intercostal space, mid-clavicular line, heaving in nature. The second heart sound is single, there is an ejection systolic click and there is a harsh loud, ejection systolic murmur radiating to the neck. In this patient which of the following physical signs relates to the severity of the lesion? a) Heaving apex beat b) Presence of an ejection click c) Intensity (loudness) of the murmur d) Duration of the murmur e) Radiation of the murmur into the neck Answer: a) The history and physical signs suggest the patient has aortic stenosis. A heaving apex beat suggests that the heart is contracting against resistance. This indicates that the degree of stenosis is severe. An ejection click reflects thickening of the valve or dilatation of the aorta and is not an indicator of severity of stenosis. Loudness and duration of the murmur bear no relationship to the degree of stenosis. Similarly radiation of the murmur to the neck does not bear a relationship to the degree of stenosis

BOF: 2.4 A 47-year-old male has been referred by his GP for investigation of abnormal liver function tests. He uses approximately 50 units of alcohol per week and is an intravenous drug abuser having picked up the habit when he was in prison for two years. He had just been released from prison. There were no past illnesses of note; in particular he had never received a blood transfusion or any blood products. On examination there were no stigmata of chronic liver disease and there was no hepatomegaly. Investigations revealed raised aminotransferases and he was anti-HCV antibody positive and HCV-RNA positive genotype 1. In this patient the risk of progression of liver disease is not increased by: a) Age at which he acquired infection b) Alcohol consumption c) Male sex d) Continuing drug abuse e) Genotype 1 Answer: d)

113

Mastering MRCP The patient is hepatitis C positive. In patients with hepatitis C the risk of progression of liver disease is increased by acquiring the illness after the age of 40. In this patient the history suggests he acquired the infection whilst in prison when he began intravenous drug abuse. Alcohol consumption affects the course of the illness adversely. The patient should be advised to abstain. Male patients are at greater risk of more rapid development of fibrosis. Genotype 1 and 4 are also associated with more rapid development of fibrosis. The rate of progress of liver disease has not been shown to be adversely affected by continuing drug abuse although this will affect his overall health.

BOF: 2.5 A 35-year-old male is known to have ulcerative colitis. It is a pan-colitis. He also has enteropathic arthritis. He has been in long-term remission. He and his wife are being investigated for subfertility and he has been found to have a low sperm count. Which of the following drugs is he likely to be on? a) Mesalazine b) Balsalazide c) Sulphasalazine d) Azathioprine e) Mercaptopurine Answer: c) Patients with ulcerative colitis can be maintained in remission by using aminosalicylates, azathioprine or mercaptopurine. Sulphasalazine can cause oligospermia. On the whole sulphasalazine has more side effects than the other aminosalicylates preparations and is less often used. However, selected patients such as those with reactive arthritis may benefit. The thiopurines (azathioprine and mercaptopurine) have been used in both male and female parents and there have been no differences in miscarriage, congenital abnormality or infection rate in a study comparing the thiopurine group and a control group.

BOF: 2.6 A 56-year-old male presented with crushing central chest pain associated with nausea, vomiting and sweating. His ECG showed ST elevation and T inversion in leads II, III and AVF. He is a known diabetic and has proliferative retinopathy and in view of this thrombolysis was withheld. A few hours later he develops a tachycardia, he becomes hypotensive, his JVP is elevated. On auscultation heart sounds are soft with a fourth heart sound, lung fields are clear. In this patient which of the following leads will show the location to which the infarction has extended? a) Lead I b) AVL c) AVR d) V4 R e) V6 Answer: d)

114

Mastering MRCP The patient presented with an inferior infarction. The clinical features that are obtained when he becomes hypotensive suggest right ventricular infarction, which is indicated by ST segment elevation in the rightsided lead, V4R

BOF: 2.7 A 16-year-old male presents with a history of passing blood in his urine. He gives a history of tonsillitis 3 weeks prior to the onset of haematuria. On examination there is periorbital oedema and ankle oedema and his blood pressure is elevated. Investigations reveal proteinuria and uremia. In this patient, which of the following features seen on light microscopy will confirm the diagnosis? a) Hyaline casts b) Fine granular casts c) Red cell casts d) White blood cells e) Bacteria Answer: c) The clinical features suggest the patient has glomerulonephritis. Casts are formed by precipitation of Tamm-Horsfall protein. Tamm-Horsfall protein is a glycoprotein, which is excreted by the thick ascending loop of Henle and the first part of the distal tubules. Hyaline casts and fine granular casts may be seen in normal urine particularly after exercise. Red cell casts indicate that the patient has glomerulonephritis. White blood cells indicate an inflammatory reaction within the urinary tract, most commonly infection. Bacteria in a clean-catch, mid-stream urine suggest infection

BOF: 2.8 A 24-year-old female who has primary generalised epilepsy and is on sodium valproate attends the antenatal clinic for her first consultation. The registrar who sees her rings you up to ask you for advice regarding the risk of teratogenecity of this drug. You reply that the baby is at risk of developing: a) Atrial septal defect b) Cleft lip and palate c) Tracheo-oesophageal fistula d) Neural tube defects e) Renal agenesis Answer: d) There is an increased risk of teratogenecity associated with the use of antiepileptic drugs. The risk is reduced if treatment is limited to one drug. The risk of neural tube defects is increased with valproate and is also associated with the use of carbamazepine, oxcarbazepine and phenytoin. Antenatal screening for neural tube defects can be performed by measuring alpha-foetoprotein levels and second trimester ultrasound scan.

115

Mastering MRCP BOF: 2.9 A 30-year-old female who is known to have ileo-caecal Crohn’s disease has been treated with oral steroids for several months. She has been admitted under the care of a surgical team with intestinal obstruction and a decision has been made to proceed to surgery. The house surgeon has been instructed to change the patient’s treatment from oral prednisolone to intravenous hydrocortisone in the peri-operative period. He rings you to ask how much hydrocortisone is equivalent to prednisolone 30 mgs. Your reply is: a) 60 mg b) 90 mg c) 120 mg d) 150 mg e) 180 mg Answer: c) The equivalent anti-inflammatory doses of Prednisolone 5 mgs are: Hydrocortisone 20 mg Cortisone acetate 25 mg Methylprednisolone 4 mg Deflazacort 6 mg Betamethasone 750 micrograms Dexamethasone 750 micrograms Triamcinolone 4 mg

BOF: 2.10 A 30-year-old female is admitted for investigation of headache. On examination of her pupils you notice that her left pupil is semi-dilated and reacts slowly to both light and accommodation. Her visual acuity is normal, there is no field defect, no ptosis and eye movements are normal with no nystagmus or diplopia. Examination of the fundus does not reveal papilloedema. The pupillary abnormality in this patient is due to: a) Holmes-Adie syndrome b) Argyll-Robertson syndrome c) Horner’s syndrome d) 3rd nerve palsy e) Relative afferent pupillary defect Answer: a) The Holmes-Adie syndrome refers to a myotonic pupil, which reacts very slowly (or there is no reaction) to light and convergence. It is commoner in young females. It may be unilateral or bilateral. It may be associated with absence or depression of the tendon reflexes and rarely anhidrosis of the limbs. It is due to denervation of the ciliary ganglion. The cause is not known

BOF: 2.11 A 56-year-old male who has hypertension with evidence of end-organ damage has been started on captopril for control of his hypertension. Shortly after this he develops a dry irritating cough. Clinical

116

Mastering MRCP examination of his chest is unremarkable and his chest x-ray is clear. In this patient the dry cough is likely to be due to inhibition of breakdown of: a) Bradykinin b) Histamine c) Serotonin d) Kallikrein e) Endothelin Answer: a) Breakdown of bradykinin is inhibited by ACE inhibitors and this is thought to be the reason why patients develop a persistent dry cough. The kinins are peptides, which cause vasodilatation by relaxing vascular smooth muscle through the action of NO (nitric oxide). They cause contraction of other visceral smooth muscle. There are two types: Bradykinin Lysylbradykinin They are formed from precursors; high molecular weight kininogen and low molecular weight kininogen. The conversion from kininogens to kinins is caused by proteases known as kallikreins. Bradykinin and lysylbradykinin are inactivated by kininase I and kininase II. Kininase II is the same enzyme as angiotensin II converting enzyme

BOF: 2.12 A 45-year-old female presents with a 2-month history of slowly progressive pain, stiffness lasting more than half an hour in the morning and swelling of the joints of her fingers and wrists. She feels generally unwell with early afternoon fatigue and malaise. On examination she appears to have lost weight, there is swelling of her metacarpophalangeal joints and the proximal interphalangeal joints. Investigations reveal Hb 10.0 g/dL MCV 95 fL ESR 70 mm in the 1st hour Rheumatoid Factor: 50 k IU/L The features that would suggest that this patient has a worse than average prognosis are: a) Female patient, gradual onset over a few months, positive IgM rheumatoid factor, anaemia b) Early afternoon fatigue and malaise, loss of weight, high ESR, normocytic anaemia c) Involvement of metacarpophalangeal joints, morning stiffness, positive rheumatoid factor, high ESR, anaemia d) Loss of weight, involvement of metacarpophalangeal joints, positive IgM rheumatoid factor, anaemia, high ESR e) Fatigue and malaise, positive IgM rheumatoid factor, anaemia, high ESR Answer: a) The clinical features suggest the patient has rheumatoid arthritis. In rheumatoid arthritis a worse than average prognosis may be predicted with 80 % accuracy if the patient is female, a gradual onset over a few months, positive IgM rheumatoid factor and or anaemia within 3 months of onset

117

Mastering MRCP BOF: 2.13 An 80-year-old male presents with a history of dyspnoea and fatigue. On examination you find that the patient has a collapsing pulse with positive Corrigan’s sign. The pulse pressure is wide. The JVP is not elevated, his apex beat is displaced and thrusting in nature. On auscultation the second heart sound is soft, there is a third heart sound and an ejection click, a soft ejection systolic murmur and an early diastolic murmur. The first heart sound is not loud and there is no opening snap. At the mitral area a mid-diastolic murmur is audible. Lungs are clear on auscultation. In this patient which of the following is not an indicator of severity? a) Soft second heart sound b) Ejection click c) Third heart sound d) Mid-diastolic murmur at the apex e) Wide pulse pressure Answer: b) The patient has clinical features of aortic regurgitation. The signs that correlate with severity are: Wide pulse pressure Soft second heart sound Presence of a third heart sound Duration of the early diastolic murmur Presence of an Austin Flint murmur Signs of left ventricular failure An ejection click suggests that the patient has a bicuspid aortic valve and it is not related to severity The mid-diastolic murmur at the apex is likely to be an Austin Flint murmur and not due to mitral stenosis as there is no opening snap and the first heart sound is not loud

BOF: 2.14 A sixty-year-old male presents with a history of difficulty in walking. On examination of his eyes you note that his pupils are small and irregular. They do not react to light but react to accommodation. He also has bilateral ptosis and has wrinkling of his forehead. The abnormality that this patient has is: a) Holmes-Adie pupil b) Argyll-Robertson pupil c) Horner’s syndrome d) Relative afferent pupillary defect e) Myasthenia gravis Answer: b) The Argyll-Robertson pupil is thought to be due to a lesion in the tectum of the midbrain in proximity to the oculomotor nuclei. The pupils are small and irregular and do not react to light but react to accommodation. Bilateral ptosis and wrinkling of the forehead due to overcompensation by the frontalis muscle suggests tabes dorsalis.

118

Mastering MRCP BOF: 2.15 A 54-year-old male presents with abdominal pain and diarrhoea. He had been taking diclofenac for backache. Clinical examination was unremarkable. He underwent endoscopy, which showed multiple duodenal ulcers, CLO test was negative. He was treated with a proton pump inhibitor but symptoms continued. He was admitted for further investigations which were as follows: Hb 11.0 MCV 77 fl Calcium 3.0 mmol/L Phosphate 0.7 mmol/L The likely diagnosis is: a) Crohn’s disease b) Non-steroidal induced ulcers and colopathy c) Multiple endocrine neoplasia d) Chronic pancreatitis e) Hyperparathyroidism Answer: c) The patient has diarrhoea, abdominal pain and hypercalcaemia. The ulceration of his duodenum is not associated with Helicobacter pylori and does not respond to proton pump inhibitors. This suggests hypergastrinaemia due to Zollinger-Ellison syndrome. Hypercalcaemia with low phosphate suggests hyperparathyroidism. The combination of Zollinger-Ellison syndrome and hyperparathyroidism suggests the patient has Type I multiple endocrine neoplasia (MEN)

BOF: 2.16 A 60-year-old female presents with a history of fatigue. On examination she has a regular pulse, rate 80 beats per minute, jerky in character. The JVP is not elevated her apex beat is displaced and thrusting in nature. On auscultation there is a 3rd heart sound and there is a pan systolic murmur at the apex radiating to the axilla. Lung bases are clear. In this patient which of the following physical signs suggests that the mitral regurgitation is severe? a) Jerky pulse b) Thrusting apex c) 3rd heart sound d) Pan systolic murmur at the apex e) Murmur radiating to the axilla Answer: c) This physical sign suggests that the patient has mitral regurgitation. The clinical signs that suggest severity of mitral regurgitation are the size of the left ventricle and the presence of a 3rd heart sound. A soft first heart sound also suggests severe mitral regurgitation

BOF: 2.17 A 56-year-old male lawyer has persistent atrial fibrillation. He has had direct current cardioversion twice and has been on amiodarone and warfarin but has gone into atrial fibrillation again.

119

Mastering MRCP A decision has been made to refer him for radiofrequency ablation treatment. This treatment involves radiofrequency ablation of the ostium of the: a) Superior vena cava b) Inferior vena cava c) Coronary sinus d) Pulmonary veins e) Coronary arteries Answer: d) Atrial fibrillation is one of the most common arrhythmias and is the strongest independent risk factor for embolic stroke. Classification of atrial fibrillation Atrial fibrillation may be: • Paroxysmal (where the episode terminates spontaneously) • Persistent (where electrical or chemical cardioversion is required to terminate it) • Chronic (where cardioversion is unsuccessful or not indicated) Aims of treatment The aims of treatment are: • Rhythm control • Rate control • Reduction in thromboembolic risk In older patients no mortality or morbidity benefits have been shown in rhythm control over rate control. Anti-coagulation 1. Warfarin Should be started at least 3 weeks before attempted cardioversion and continued for at least 4 weeks after successful cardioversion. It may be necessary to continue anticoagulation for longer periods if there is an increased risk of reverting to atrial fibrillation or if there are ongoing paroxysms of atrial fibrillation. Continuous anticoagulation is required if the patient is being managed by controlling rate 2. Ximelagtran This is a new drug, which acts as a direct thrombin inhibitor. It does not need monitoring. It is not in routine use yet. Rhythm control • Direct current cardioversion External Using monophasic or more recently biphasic defibrillators Internal Using transvenous atrial defibrillation • Pharmacological cardioversion The established agents are: Sotalol, flecainide, amiodarone, propafenone Other drugs: Dofetilide (available in the US) Azimilide (undergoing evaluation) Pacing

120

Mastering MRCP Dual chamber pacing with anti-atrial fibrillation algorithms may be used for those who have an additional indication for pacing Radiofrequency ablation of the AV node Interrupt conduction of the AV node followed by pacemaker implantation Maze procedure Intra-atrial incisions to form anatomical barriers and reduce the number of circulating wavelets that cause atrial fibrillation. This may be used if the patient has another indication for cardiac surgery Radiofrequency ablation of the ostium of the pulmonary veins This isolates the pulmonary veins by ablating the electrical connections at the ostia and has shown high success rates. Rate control The drugs commonly used are digoxin, beta-blockers and calcium antagonists together with anticoagulation.

BOF: 2.18 A 40-year-old female who has end stage renal failure and is on renal replacement therapy presents with painful skin lesions that came on suddenly and are progressing rapidly. She also gives a history of recent , sudden weight loss. On examination she has lesions on her thighs. The lesions are star shaped, purpuric lesions with central ulceration. They are indurated and extremely tender. Biopsy of these lesions shows calcification of the media of the small and medium sized vessels with intimal hyperplasia and fibrosis. This is surrounded by a mixed inflammatory infiltrate. In this patient, which of the following forms of treatment would be of no benefit? a) Judicious use of antibiotics b) Surgical debridement c) Hyperbaric oxygen d) Parathyroidectomy e) Immunosuppression Answer: e) The clinical features and biopsy features suggest the patient has calciphylaxis. The first four methods of treatment are of benefit in calciphylaxis. Immunosuppression may be a trigger for this condition Calciphylaxis calcific uremic arteriolopathy (CUA) is a syndrome of vascular calcification, thrombosis and skin necrosis. It is seen almost exclusively in patients with Stage 5 chronic kidney disease. It is dangerous disease due to deposition of calcium form blood vessels into the skin.

BOF: 2.19 A 17-year-old boy presents complaining that he has not begun to shave unlike his friends. There is no previous medical history of note nor is there a significant family history. On examination he is tall. The upper segment of his body is longer than the lower segment. His arm span is more than 5 cms longer than his height in meters’. There is no hair on his beard and moustache area and there is no hair over his body. Pubic and axillary hair is not present. His penis is small and the testes are small as well. Apart from anosmia no other abnormalities can be detected on examination of his other systems.

121

Mastering MRCP In this patient, which of the following features suggests that he has hypogonadotrophic hypogonadism? a) Eunuchoidal proportions b) Small penis c) Small testes d) Absence of facial and body hair e) Anosmia Answer: e) The patient has features of hypogonadism. Eunuchoidal proportions (lower segment longer than upper segment, arm span more than 5 cms greater than height) small genitalia; decreased body and facial hair all suggest hypogonadism. Anosmia would suggest that this is due to Kallman’s syndrome or hypogonadotrophic hypogonadism

BOF: 2.20 An 18-year-old male from Thailand presents with a history of syncope. No other symptoms, no significant past medical history. He does not smoke or drink and does not abuse drugs. There are no abnormalities detectable on clinical examination. His ECG shows right bundle branch block with J point elevation and concave ST elevation in leads V1-V3. The diagnosis is: a) Antero-septal myocardial infarction b) Pericarditis c) Ventricular aneurysm d) Brugada syndrome e) Wolfe-Parkinson-White syndrome Answer: d) Treatment should be with: a) Implantable cardioverter defibrillator b) Sotalol c) Thrombolysis d) High dose steroids e) Repair of the aneurysm Answer: a) The patient has the features of Brugada syndrome (Sudden Unexpected Death Syndrome). A previously well asymptomatic young man is unlikely to have had a myocardial infarction, pericarditis or ventricular aneurysm. The ECG changes are not suggestive of WPW syndrome. Treatment of Brugada syndrome is with an implantable cardioverter defibrillator.

BOF: 2.21 A 28-year-old male presents with pain and weakness of his right shoulder. He gives a history of a recent upper respiratory tract infection. On examination there is wasting and weakness of his supraspinatus and infraspinatus muscles and passive movements of the shoulder are painful.

122

Mastering MRCP The likely diagnosis is: a) Polymyositis b) Neuralgic amyotrophy c) Proximal spinal muscular atrophy d) Erb’s palsy e) Thoracic outlet syndrome Answer: b) The clinical features described are very much in favor of neuralgic amyotrophy . Proximal spinal muscular atrophy causes slowly progressive usually symmetrical wasting and weakness. Erb’s palsy affects nerve roots C5, C6 and thoracic outlet syndrome affects C8 and T1. Polymyositis usually affects people in their 4th and 5th decades of life and causes proximal weakness with aching muscles, which are sometimes tender and indurated

BOF: 2.22 Your junior calls you up to ask advice regarding a 50-year-old male who has been admitted with a history of cough and breathlessness. He has had an upper respiratory infection for a few days prior to presentation. On the day of presentation he developed a single shaking chill followed by fever. He also complained of cough productive of purulent sputum, right-sided pleuritic chest pain and breathlessness. On examination he looked unwell, was febrile, had a tachycardia and tachypnoea. On palpation the trachea was in the midline, respiratory movements were diminished at the right base. There were fine crepitations at the right base. Chest x-ray did not show any abnormality. Your advice to your junior would be: a) Arrange a right lateral chest x-ray b) Repeat the chest x-ray c) Arrange a ventilation perfusion scan d) Arrange a CT pulmonary angiogram e) Arrange a high resolution CT scan Answer: b) The clinical features are suggestive of pneumococcal pneumonia. In patients with pneumococcal pneumonia, the chest x-ray findings may be minimal or undetectable during the first several hours

BOF: 2.23 A 45-year-old female presents with a history of headache. She has had a thyroidectomy in the past. On examination she has a partial ptosis on the right, enophthalmos on the right side, the right pupil is smaller than the left and the cilio-spinal reflex is absent on the right hand side. Her vision is unimpaired, visual fields are normal and eye movements are full with no diplopia. The lesion described is: a) Third nerve palsy b) Holmes-Adie pupil c) Horner’s syndrome d) Grave’s ophthalmopathy

123

Mastering MRCP e) Myasthenia gravis Answer: c) Horner’s syndrome consists of partial ptosis, enophthalmos, small pupil, absent cilio-spinal reflex and absent sweating on the side of the lesion. Causes: The causes of Horner’s syndrome are: Cerebral lesions: • Massive cerebral infarction • Herniation of the temporal lobe Brain stem lesions: • Pontine glioma • Lateral medullary syndrome Cord lesions: • Syringomyelia • Cord tumours Chest lesions: • Pancoast tumour • Apical tuberculosis • Aortic aneurysm Neck lesions: • Congenital • Cervical rib • Thyroid, laryngeal surgery • Cervical sympathectomy • Brachial plexus trauma • Neoplasms • Carotid artery aneurysm BOF: 2.24 A 65-year-old male presents with intermittent dysphagia. There is no loss of weight. He is referred for endoscopy and the endoscopist reports a tight peptic stricture, which is dilated successfully. Biopsies from the area do not reveal a malignancy. In the long term management of this patient the most important step would be: a) Long term acid suppression with a proton pump inhibitor b) Regular endoscopic surveillance c) Regular dilatation to prevent recurrence d) Insertion of an oesophageal stent e) Formal surgical correction of the stricture Answer: a) The patient has a benign oesophageal stricture. This is a consequence of reflux oesophagitis and the most important step in management is to prevent reflux by using long-term acid suppression.

124

Mastering MRCP Endoscopic surveillance is of no benefit. Repeat dilation would be required if the patient were to become symptomatic but regular dilatation is not undertaken. Stents are used only for narrowing of the oesophagus caused by cancer. Surgery is rarely required for benign oesophageal stricture

BOF: 42 (2.25) A 55-year-old male who is known to have cirrhosis of the liver as a consequence of hereditary haemochromatosis presents with a rapid development of weight loss, anorexia, fever, ache in the right hypochondrium and ascites. On examination of the abdomen an enlarged, tender liver with an irregular edge is palpable. Ultrasound scanning of the liver shows filling defects but serum alpha-foetoprotein is not raised. In this patient; a) Hepatocellular carcinoma is unlikely as the alpha-foetoprotein levels are not raised b) Ultrasound guided liver biopsy should be used to confirm the diagnosis of hepatocellular carcinoma c) Ascitic fluid cytology is likely to confirm the diagnosis of hepatocellular carcinoma d) Alpha-foetoprotein levels may be normal in patients with hepatocellular carcinoma e) Radio-isotope scanning should be used to confirm the diagnosis of hepatocellular carcinoma Answer: d) The rapid development of weight loss, anorexia, fever and ache in the right hypochondrium and ascites in a patient with cirrhosis should make one suspect that the patient has hepatocellular carcinoma. Serum alpha-foetoprotein may be raised but could be normal in at least a third of patients. Ultrasound scans show filling defects in 90% of patients. Liver biopsy in not used as there is a risk of seeding and imaging techniques show characteristic appearances

BOF: 2.26 A 40 year old male presents who is known to have ulcerative colitis which is in remission presents with fever, abdominal pain, pruritus and jaundice. An ERCP is performed and this shows intra-hepatic and extrahepatic biliary strictures. In the long-term this patient is at increased risk of: a) Hepatocellular carcinoma b) Pancreatic carcinoma c) Cholangiocarcinoma d) Adenocarcinoma of the oesophagus e) Lung cancer Answer: c) The clinical features are suggestive of cholangitis and the ERCP findings are those of primary sclerosing cholangitis, which is a known complication of ulcerative colitis. In this condition cholangiocarcinoma can occur in up to 20 % of patients

125

Mastering MRCP BOF: 2.27 Which of the following skin manifestations is pathognomonic of sarcoidosis? a) Erythema nodosum b) Lupus pernio c) Sarcoid nodules d) Sarcoid plaques e) Scar sarcoid Answer: b) 20-30 % of patients with systemic sarcoidosis have skin disease. Cutaneous sarcoid may occur without evidence of systemic disease. The extent of the skin disease does not correlate to the extent of systemic disease. The skin manifestations of sarcoid are: Erythema nodosum Lupus pernio, which is pathognomonic of sarcoidosis and is usually associated with chronic fibrotic sarcoid of the respiratory tract (upper). The response of the respiratory lesions to treatment in this situation is poor. Sarcoid papules Sarcoid plaques Sarcoid nodules Scar Sarcoid

BOF: 2.28 A 70-year-old male presents with thickening and hyperpigmentation of the skin in the axillae, neck, genitals and groin. On further inspection there is involvement of the mucous membranes as well and on palpation the lesions in the skin have a velvety texture. The palms of the patients hands appear thicken as well. Which type of acanthosis nigricans is this likely to be? a) Benign b) Hereditary benign c) Pseudo d) Malignant e) Naevoid Answer: d) The main types of acanthosis nigricans are: Benign which is associated with various syndromes where relative insulin resistance occurs Hereditary benign which is an autosomal dominant condition that is not associated with endocrine abnormalities Pseudo, which is a reversible complication of obesity Malignant which is usually secondary to adenocarcinoma. The features, which should make one suspect malignancy, are onset in the elderly, rapid evolution, severe involvement with thickening of the palms, mucous membranes and mucocutaneous junctions Naevoid which is isolated and localised and is not associated with endocrine abnormalities

126

Mastering MRCP BOF: 2.29 A 65-year-old male presents with a history of difficulty in walking. On examination there is weakness of the proximal muscles of the lower limbs. The tendon reflexes are depressed but no other neurological deficit is detected. On further questioning the patient also admits to having a dry mouth, constipation and impotence. He has no diplopia. In this patient the investigation that is most specific for the condition that he suffers from is: a) Anti acetylcholine receptor antibodies (Anti AChR) b) Anti Muscle Specific Kinase antibodies (Anti MuSK) c) Anti voltage gated calcium channel antibodies d) Tensilon test e) Chest X-ray Answer: c) The history of proximal muscle weakness mainly affecting the lower limbs, depressed tendon reflexes and absence of diplopia together with autonomic symptoms suggests that the patient has the Lambert-Eaton myasthenic syndrome. In this syndrome antibodies against voltage gated calcium channels are present in the serum of most patients and these antibodies are highly specific

BOF: (2.30) A 35-year-old Japanese male presents with anterior uveitis. There is a history of recurrent oral ulceration and he has also had ulcers on his scrotum that have resulted in scarring. To further investigate the patient’s condition a 20 G needle is inserted into the skin of his forearm. A positive result in this test is: a) A blister appearing at the site if injury in 24-48 hours b) A weal appearing at the site of injury immediately c) Multiple petechiae appearing at the site of injury within 5 minutes d) A papule or pustule appearing at the site of injury within 24-48 hours e) An erythematous nodule appearing at the site of injury within 24-48 hours Answer: d) The clinical features suggest that the patient has Behcet’s disease. In this condition pathergy is often demonstrable. Pathergy refers to an excessive inflammatory reaction to non-specific injury. In a positive test, insertion of a 20 G needle into the skin of the forearm results in a papule or pustule within 24-48 hours

BOF: 2.31 A 56-year-old female presents with a history of dry eyes and dry mouth. She has bilateral swelling of her parotid glands. The typical renal manifestation of this condition is: a) Acute nephritic syndrome b) Nephrotic syndrome c) Microscopic haematuria d) Recurrent urinary tract infections

127

Mastering MRCP e) Renal tubular acidosis Answer: e) The clinical features suggest the patient has Sjögren's syndrome . In this condition the typical renal lesions is interstitial and causes tubular dysfunction and renal tubular acidosis

BOF: (2.32) A 32-year-old female has a slowly progressive difficulty in climbing stairs and getting up from the seated position. On examination there is evidence of proximal muscular weakness affecting the pelvic girdle more than the shoulder girdle. She undergoes many investigations and ultimately a muscle biopsy is performed and this shows large vacuoles, which stain positive for periodic acid-Schiff stain. This patient has: a) Becker muscular dystrophy b) Duchenne muscular dystrophy c) Polymyositis d) Polymyalgia rheumatica e) Acid maltase deficiency Answer: e) Acid maltase deficiency is an autosomal recessive condition. In the adult form onset is in the second and third decade of life. It affects proximal muscles rather than the distal and the pelvic girdle is affected more than the shoulder. The large vacuoles that are seen on muscle biopsy are lysosomes full of glycogen. Becker and Duchenne muscular dystrophies are X-linked conditions and hence not likely to affect females. Polymyositis causes inflammation of muscles and polymyalgia rheumatica is a disease that predominantly affects more elderly patients.

BOF 2.33 A 26-year-old female presents with abdominal pain, nausea and vomiting and constipation. She complains of discolouration of her urine. There is a previous history of epilepsy. On examination she is in pain, pulse rate of 120 beats per minute, BP 160/90 abdomen soft not distended, no lumps or tender areas, bowel sounds normal. Investigations reveal: Hb12.5 g/dL White cell count 5.5 x 10 9 /L Neutrophils 3.5 x 10 9 /L Lymphocytes 1.5 x 10 9 /L Monocytes 0.7 x 10 9 /L Eosinophils 0.3 x 10 9 /L Na 120 mmol/L K 4.0 mmol/L Urea 3.0 mmol/L Serum creatinine 75 mmol/L Your initial management of this patient would be:

128

Mastering MRCP a) Low carbohydrate diet b) High protein diet c) Infusion of haem arginate in dilute albumin solution through a peripheral vein d) Infusion of haem arginate through a central vein e) Infusion of tin porphyrin Answer: d) The patient has acute intermittent porphyria. She has pain, nausea, vomiting and constipation with tachycardia m hypertension, discolouration of urine, epilepsy and hyponatraemia. These are all suggestive features. A high carbohydrate diet is a useful adjunct to treatment of an acute attack and may even abort an incipient attack. Haem arginate acts by inhibiting ALA synthetase and is used in the management of acute attacks. It is irritant and should be administered through a central vein. Administration in dilute albumin solution further reduces its local irritant effect. Tin porphyrin is a haem oxygenase inhibitor, which enhances the efficacy of haem preparations, but they are not at present recommended for use currently.

BOF: 2.34 A 50-year-old male presents with a blistering rash on the dorsum of his hands on exposure to sunlight. He drinks 30 units of alcohol per week and smokes 15 cigarettes per day. On examination of his hands there are blisters, erosions scars and milia. The skin is pigmented and fragile. There are no signs of chronic liver disease and his liver and spleen are not palpable. Investigations show normal haematology and routine biochemistry. Serum ferritin 1200 micrograms per liter Genetic testing shows homozygosity for C282Y In this patient the most effective treatment of the skin condition would be: a) Desferrioxamine b) Venesection c) Low dose chloroquine d) Penicillamine e) Dapsone Answer: b) The clinical features suggest the patient has porphyria cutanea tarda. This is essentially a liver disorder that presents with skin manifestations. It is caused by an endogenous inhibitor of the hepatic uroporphyrinogen decarboxylase enzyme. The inhibitor forms in the presence of iron and an oxidative environment. The porphyrins that are formed become toxic when exposed to sunlight. Where iron overload is the cause of the problem the treatment of choice is venesection both for liver damage and the porphyria.

BOF: 2.35 A 42-year-old female presents with a blistering rash on her hands on exposure to sunlight. She is an intravenous drug abuser, drinks 24 units of alcohol per week and smokes 10 cigarettes per day.

129

Mastering MRCP On examination of her hands there are blisters, erosions scars and milia. The skin is pigmented and fragile. She looks emaciated she has clubbing palmar erythema and spider naevi. There is tense ascites and the liver and spleen are not palpable. Investigations reveal a macrocytic anaemia and disordered liver function. She is hepatitis-C PCR positive and her ferritin levels are within the normal range. In this patient treatment of the skin condition should be with: a) Desferrioxamine b) Venesection c) Low dose chloroquine d) Penicillamine e) Dapsone Answer: c) The clinical features suggest the patient has porphyria cutanea tarda. This is essentially a liver disorder that presents with skin manifestations. It is caused by an endogenous inhibitor of the hepatic uroporphyrinogen decarboxylase enzyme. The inhibitor forms in the presence of iron and an oxidative environment. The porphyrins that are formed become toxic when exposed to sunlight. Where iron overload is not the cause of the problem treatment with low dose chloroquine (125 mg twice weekly) is effective treatment.

BOF: 2.36 A 60-year-old female presents with pain in her knees. The pain increases after use and is relieved by rest. There are no systemic symptoms. On examination she is obese. On examination of the hands there are Heberden’s nodes and Bouchard’s nodes. The knees show bony swelling; there is localised joint and periarticular tenderness. The range of movement is decreased and there is crepitus on movement. Investigations show normal haematology and biochemistry and inflammatory markers are not raised. In this patient, the pharmacological intervention of first choice should be treatment with: a) Paracetamol b) Codeine phosphate c) Non Steroidal Anti Inflammatory Drugs (NSAIDs) d) Tramadol e) Glucosamine sulphate Answer: a) The clinical features suggest the patient has osteoarthrosis. Paracetamol is the safest analgesics and is the recommended oral agent of first choice in osteoarthrosis. Codeine is useful in combination with paracetamol but constipation and central nervous system disturbance may be a problem. NSAIDs may be more effective than paracetamol but they are associated with renal and gastrointestinal toxicity. Tramadol has more side effects. Glucosamine sulphate is a nutritional supplement. In trials it has been shown to cause modest reductions in pain and slows joint space narrowing in osteoarthritis of the knee.

130

Mastering MRCP BOF 2.37 A 52-year-old female presents with pain and swelling of her wrists and hands of 6 weeks duration. She complains of about one and a half hours of stiffness in the mornings. She says her mouth is dry and her eyes feel gritty. She also complains of fatigue and being generally unwell. No past illnesses of note. On examination she has rheumatoid nodules, there is a bilateral, symmetrical arthropathy mainly affecting the metacarpophalangeal and proximal interphalangeal joints. Schirmer`s test is positive. Investigations reveal: Hb 10.5 g/dL MCV 90 fL ESR 60 mm in 1St hour Rheumatoid factor 60 k IU /L In this patient: a) The presence of rheumatoid nodules precludes the use of methotrexate b) Early introduction of DMARDs (Disease Modifying Anti Rheumatic Drugs) would enable an improved outcome c) NSAIDs and paracetemol should be given for an extended period to enable least exposure to toxic second line agents d) Sicca syndrome rules out the use of sulphasalazine e) Symptom control with low dose prednisolone would be the treatment of choice Answer: b) The patient has rheumatoid arthritis. Early treatment of this disease with DMARDs enables improved outcomes. The presence of rheumatoid nodules and sicca syndrome indicate a worse prognosis. Prednisolone has symptom controlling and disease modifying effects but is usually used in combination with other agents

BOF: 2.38 A 75-year-old female has had several episodes of delirium. There is also a history of intermittent falls. She is noted to have persistent visual hallucinations. She has memory impairment, visuospatial impairment and fluctuating performance. On examination of her limbs there are features of Parkinsonism. In this patient the likely diagnosis is: a) Alzheimer’s disease b) Lewy body dementia c) Cerebrovascular dementia d) Frontotemporal dementia e) Normal pressure hydrocephalus Answer: b) Lewy body dementia usually presents with early memory impairment and intermittent delirium. Visuospatial impairment is a feature as is fluctuating performance. Parkinsonism and recurrent falls are other features of Lewy body dementia

131

Mastering MRCP BOF: 2.39 A 16-year-old male presents with fever and a lump in the neck. The patient was well till a week prior to presentation when he developed fever and a tender lump in his neck was felt. No past history of significance, no recent sore throat. He has not been on any medications. No history of foreign travel. No ingestion of undercooked meat. He gave a history of being scratched when playing with the family pet (a cat). On examination he was febrile temperature 39° C. there was a single palpable lymph node in the left posterior triangle 6 cms in diameter. It was tender but no overlying erythema no fluctuation. No skin lesions. No hepatosplenomegaly. Other systems were normal on examination. Investigations revealed: Normal FBC and biochemistry. ESR 91 mm/h Serology positive for Toxoplasma, CMV EBV In this patient the likely causative organism is likely to be: a) Brucella melitensis b) Ricketsia prowazeki c) Mycobacterium marinum d) Streptococcus pyogenes e) Bartonella henselae Answer: e) The history of fever and lymphadenopathy following a cat scratch would suggest cat-scratch disease, which is caused by Bartonella henselae

BOF: 2.40 You see a 66-year-old female in the gastroenterology clinic. She is known to have Crohn’s disease and has had several courses of steroids but is now maintained in remission with the use of azathioprine. Your registrar suggests that you arrange a dual-energy x-ray absorptiometry (DXA) study on this patient to assess whether she has osteoporosis. Which of the following sites is the preferred site for diagnosis of osteoporosis and is also the site at which fracture causes the greatest personal and financial cost. a) Lumbar spine b) Thoracic spine c) Wrist d) Shaft of femur e) Hip Answer: e) The hip is the preferred site for diagnosis as in this area there is low precision error and there is adequate reference data. Spinal assessment in the elderly gives falsely elevated results because of degenerative disease. Femoral assessment gives the highest predictive value for fracture risk. The spine is the preferred site for monitoring treatment. Hip fractures are also thought to cause the greatest financial and personal cost

132

Mastering MRCP BOF: 2.41 In dual-energy x-ray absorptiometry (DXA), the T score refers to: a) The number of standard deviations the patient’s bone mineral density (BMD) is away from the mean values for BMD in a 30 year old of the same sex and ethnicity. b) The number of standard deviations the patient’s bone mineral density (BMD) is away from the mean values for BMD in age and sex matched controls c) The number of standard deviations the patient’s bone mineral density (BMD) is away from the mean values for the general population d) The number of standard deviations the patient’s bone mineral density (BMD) is away from the mean values for the local population e) The number of standard deviations the patient’s bone mineral density (BMD) is away from the mean values for local age and sex matched controls Answer a) Dual-energy x-ray absorptiometry provides accurate, rapid, convenient assessment of bone mineral density (BMD) The results are expressed as Z or T scores The Z score refers to the number of standard deviations the patient’s BMD values is away from age and sex matched controls The T score refers to the number of standard deviations the patient’s BMD is away from a young normal that is a 30 year old person of the same sex and ethnicity as the patient. At the age of 30 people have their peak bone mass. WHO definition of osteoporosis T >– 1.0 (Normal) BMD equal to or more than 1 standard deviation (SD) below the young adult mean T –1.0 to –2.5 (Osteopaenia) BMD 1.0to 2.5 SD below the young adult mean T < 2.5 (Osteoporosis) BMD more than 2.5 SD below the young adult mean T <2.5 with one or more associated fractures (Established /severe osteoporosis) BMD more the 2.5 SD below the young adult mean with one or more associated fractures

BOF: 2.42 A 23-year-old male university student is brought in to casualty by his flatmates. He says that he can hear people talking about him over the radio. He feels there is a plot against him and that the people in charge of the radio station are trying to turn his friends against him. At times he can hear voices in his head and he also feels that these people take control of his actions at times. His friends are concerned about his behavior and ask you whether drug or alcohol abuse could play a part in the aetiology of his condition. You reply that the agent that can be considered a risk factor for development of this condition is: a) Cigarettes b) Alcohol c) Heroin d) Cannabis

133

Mastering MRCP e) LSD Answer d) The patient has schizophrenia. The diagnosis is made on the basis of several first rank symptoms. The first rank symptoms of schizophrenia are: · Auditory hallucinations in the third person and or voices commenting on their behaviour · Thought withdrawal, insertion and broadcast · Primary delusion · Delusional perception · Somatic passivity and feelings (patients believe that their thoughts, feelings or acts are under the control of others) (The likelihood of a patient having schizophrenia is increased if they have more of these symptoms) Cannabis use is considered a risk factor for the development of schizophrenia

BOF: 2.43 A 46-year-old female has been on long-term treatment for schizophrenia. She has been referred to the medical clinic as she has put on a lot of weight and developed diabetes mellitus. You see her at the clinic and take a detailed drug history, as you are aware that the following drug could be responsible for her weight gain and the development of diabetes mellitus: a) Chlorpromazine b) Haloperidol c) Clozapine d) Risperidone e) Olanzapine Answer: e) The drugs used in the treatment of schizophrenia are: Antipsychotic Drugs (Neuroleptics) These drugs act by blocking the D1 and D2 groups of dopamine receptors. The drugs are: Phenothiazines Chlorpromazine Trifluoperazine Fluphenazine Butyrophenones Haloperidol Atypical Antipsychotics (Block D2 less than D1) Clozapine Risperidone Olanzapine (causes weight gain and increases risk of diabetes mellitus) Sulpiride Quietiapine Ziprasidone

134

Mastering MRCP BOF: 2.44 You are called in to see a patient on the surgical ward. He is an 84-year-old male who was admitted the previous day with an obstructed inguinal hernia for which he had surgery, which was uncomplicated and successful. He has become confused in the night and is shouting out that there are wild animals attacking him. He has been trying to run away and the nursing staff are having difficulty controlling him as he says that they are trying to steal his money. He is been nursed in a single room. He is afebrile but it is difficult to examine him thoroughly as he is so agitated. The surgical house officer wishes to sedate him and asks you whether he should use diazepam or pethidine. You reply that the most appropriate drug to use in this situation would be: a) Chlorpromazine b) Haloperidol c) Risperidone d) Clozapine e) Quietiapine Answer: b) The sudden onset of a confusional state in an elderly patient admitted to hospital is suggestive of delirium. The visual hallucinations that he appears to be experiencing and the delusions lend further support to this. In delirium psychoactive drugs should be avoided if at all possible as they too may exacerbate delirium but in this case they appear to be necessary and the most suitable drug would be haloperidol which may be administered intramuscularly if required

BOF: 2.45 A 27-year-old female is brought in by her parents. She is known to be schizophrenic and is on Trifluoperazine for this. The parents tell you that their daughter seems to be exceedingly restless and never seems to stay still. You observe that the patient is seated on a chair but is continuously crossing and uncrossing her legs and fidgeting with her clothes and touching her hair. On examination there is no tremor, the muscle tone is normal and there is no weakness of her muscles. The rest of the neurological examination is unremarkable. The condition the patient has is: a) Akathisia b) Chorea c) Parkinsonism d) Tardive dyskinesia e) Acute dystonia Answer: a) Akathisia refers to a sensation of restlessness, which manifests as an inability to stay still (Greek: kathesis sitting a without hence without sitting) Chorea is a type of dyskinesia and refers to a dance like quasi-purposive movement There are no features of Parkinsonism such as paucity of movement, tremor or rigidity. Tardive dyskinesias are late onset purposeless involuntary movements (lip smacking grimacing)

135

Mastering MRCP Acute dystonia causes changes in tone resulting in conditions such as torticollis, oculogyric crisis

BOF: 2.46 A 28-year-old male who has recently been started on trifluoperazine for treatment of schizophrenia presents complaining of a feeling of severe restlessness. He cannot sit still and is continuously fidgeting with his clothes and stamping his feet as if he is marching. On examination there is no neurological abnormality In managing this condition you would: a) Stop trifluoperazine and give him chlorpromazine instead b) Add on benzhexol c) Treat with tetrabenazine d) Ask him to take two units of alcohol every day e) Treat with propranolol Answer: e) The patient has akathisia, which can occur when a patient is treated with antipsychotic medication. In the management of this condition one may try changing to a newer class of antipsychotic (atypical antipsychotic) or try a lipophilic beta-blocker such as propranolol

BOF: 2.47 A 33-year-old chef has been referred for further evaluation. He is finding it difficult to continue working as he has a need to wash his hands every time he touches his apron or the tabletop. If he does touch either surface he has to go and wash his hands 6 times. When he returns to the work surface he has to line up any items he is chopping in a certain direction and then count to 25 before he can recommence chopping. This behaviour is making it difficult to continue working and his employers are considering termination of service. He has been referred for psychological treatment but this has been unsuccessful. Which one of the following classes of drugs would be considered the mainstay of treatment for his condition? a) Benzodiazepines b) Phenothiazines c) Butyrophenones d) Atypical antipsychotics e) Selective serotonin reuptake inhibitors Answer: e) The patient has Obsessive-Compulsive Disorder (OCD). In this condition abnormalities are thought to occur in the frontal lobes and the basal ganglia. Serotonin function is probably abnormal in these patients and serotonin reuptake inhibitors are effective drugs.

136

Mastering MRCP BOF: 2.48 A 42-year-old female presents with a history of extreme fatigue and headaches. She also complains that she has lost her enthusiasm for life and that she feels that life is devoid of any pleasure and she has no interest any more. She also complains of episodes where she goes in to a state of panic and feels extremely anxious. These symptoms have gone on for two months and are interfering with her work as a lawyer. In this patient which of the following features would suggest that she has a severe form of her illness? a) Second person auditory hallucinations b) Visual hallucinations c) Third person auditory hallucinations d) Obsessional symptoms e) Phobic symptoms Answer: a) The symptom she describes suggests that the patient is depressed. In severe forms of depression the patient may have delusions of guilt, bodily disease and persecution and second person auditory hallucinations insulting the patient or suggesting suicide may occur. Third person auditory hallucinations occur in schizophrenia and visual hallucinations usually suggest an organic brain syndrome. Phobic and obsessional symptoms occur in depression and do not suggest it is severe.

BOF: 2.49 A 36-year-old female is on citalopram for treatment of depression. She presents with a history of shivering, anxiety, dizziness, headache and nausea. These symptoms are the result of: a) Overdose of citalopram b) Withdrawal of citalopram c) Combination of citalopram and St. John’s Wort d) Concomitant alcohol abuse e) Excessive consumption of cigarettes in combination with citalopram Answer: b) Citalopram is a SSRI (selective serotonin reuptake inhibitor). These drugs may be associated with a specific withdrawal syndrome known as the discontinuity syndrome, which is characterised by shivering, dizziness, anxiety, nausea and headaches. SSRIs should be gradually reduced and patients should be counseled that they should not leave out doses. St. John’s wort is an herbal antidepressant, which interacts with serotoninergic drugs in particular

BOF: 2.50 A 40 year old female who is known to be depressed has recently been started on citalopram as she had shown no response to the dosulepin that she had been taking earlier.She presents with a history of diarrhoea and she is found to be febrile, tremulous, tachycardic, hypertensive, confused and agitated. This state is likely to be due to:

137

Mastering MRCP a) Co-existent phaeochromocytoma b) Concomitant use of alcohol c) Combination of dosulepin and citalopram d) Sudden withdrawal of treatment e) Thyrotoxic crisis precipitated by the drugs Answer: c) The serotonin syndrome is a toxic serotonergic state that may be induced by the combination of two or more drugs that increase serotonin levels. This may be a selective serotonin reuptake inhibitor combined with a monoamine oxidase inhibitor, tricyclic antidepressant or dopaminergic drug. Treatment for this condition is supportive. In this patient the likelihood is that she used the two drugs in combination

BOF: 2.51 A 26-year-old female has been brought to the clinic by her parents as her behaviour is causing concern. She has run up enormous debts, as she has been spending money lavishly. She says that this is not a problem as she is going to earn large sums of money by selling her paintings (she is an artist) and soon she will be world famous. She has been having many boyfriends and has problems with relationships as she has become very promiscuous. On examination she seems excitable talks very fast and rapidly moves from one subject to another. She admits to difficulty sleeping and she has lost some weight. In this patient which of the following tests would be useful in predicting an increased risk of complications with first line treatment. a) Thyroid autoantibodies b) Serum osmolality c) Urine osmolality d) ECG e) Serum calcium Answer: a) The patient has the clinical features of hypomania. This condition is usually treated with lithium or antipsychotic (neuroleptic) drugs. The common adverse effects of lithium are thyroid disorders and renal disease. Patients should be screened by checking T4 TSH and thyroid autoantibodies together with urea, creatinine and 24-hour urinary volume. The presence of thyroid autoantibodies increase the risk of developing thyroid disease (hypothyroidism)

BOF: 2.52 A 30-year-old female has been seen at the antenatal clinic. She is on lithium for treatment of hypomania and has just found out she is pregnant. The registrar in the clinic rings you up to ask you what the likelihood of fetal abnormalities is in this situation.

138

Mastering MRCP You reply that in the first trimester treatment with lithium increases the likelihood of the patient developing: a) Atrial septal defect b) Ventricular septal defect c) Fallot’s tetralogy d) Ebstein’s anomaly e) Coarctation of the aorta Answer: d) Lithium use is not advised in the first trimester of pregnancy as it can increase the likelihood of fetal malformation such as Ebstein’s anomaly (atrialisation of the right ventricle)

BOF: 2.53 A 42-year-old male who is known to have sarcoidosis presents with progressive breathlessness. His chest xray shows bilateral hilar lymphadenopathy with interstitial infiltrates. Which stage of pulmonary sarcoidosis is this? a) Stage 0 b) Stage1 c) Stage 2 d) Stage 3 e) Stage 4 Answer: c) The chest x-ray staging of pulmonary sarcoidosis is as follows: Stage 0: Normal Stage 1: Bilateral hilar lymphadenopathy Stage 2: Bilateral hilar lymphadenopathy with interstitial infiltrates Stage 3: Interstitial infiltrates only Stage 4: Diffuse fibrosis

BOF: 2.54 A 63-year-old male is admitted with a history of low-grade fever and feeling generally unwell. He had had an endoscopy two months ago and was found to be Helicobacter pylori positive and was given a course of eradication treatment. As he was having fever his general practitioner also gave him a week's course of ciprofloxacin but this did not settle his fever .The course of antibiotics was completed a week prior to admission. On examination he has a temperature of 38.4º C. On examination of his hands there are erythematous macules on his palms and tender erythematous nodules in the pulp of his fingers. His pulse rate is 70 beats per minute, regular with a slow rising pulse. There is brachio-radial delay of the pulse. The JVP is not elevated, the apex beat is in the 5th intercostal space in the mid-clavicular line and it is heaving in nature. The second heart sound is soft and single and there is an ejection systolic murmur in the aortic area conducted to the neck. On examination of his abdomen the spleen tip is palpable.

139

Mastering MRCP Investigations reveal normocytic, normochromic anaemia, elevated CRP and he has microscopic haematuria. In this patient your next line of management would be to take blood cultures and: a) Treat with penicillin 1.2 g 4-hourly with gentamicin 80 mgs 12-hourly b) Treat with vancomycin 1 g 12- hourly with gentamicin 80 mgs 12- hourly c) Treat with ampicillin 2 g 4-hourly with gentamicin 80 mgs 12-hourly d) Treat with flucloxacillin 2g 4-hourly with gentamicin 80 mgs 12-hourly e) Await results of blood cultures Answer: e) The clinical features suggest the patient has aortic stenosis and together with the rest of the history, examination and investigations you would suspect infective endocarditis. However, to conclusively prove a patient has infective endocarditis, Duke’s criteria should be fulfilled and in this case they have not yet been fulfilled. In addition, this patient has had two courses of antibiotics recently. Hence, one would delay antibiotic treatment until cultures are available. This is possible, as the patient does not have hemodynamic instability. Duke’s Criteria Infective endocarditis (IE) may be diagnosed if one of the following conditions is fulfilled: · 2 major criteria · 1 major and 3 minor · 5 minor criteria Summary of major and minor criteria Major criteria · Positive blood cultures: Expected organisms Blood cultures > 12 hours apart 3/3 or 3/ 4 positive with > 60 mins between 1st and last · Echocardiographic support: Oscillating mass Abscess New partial dehiscence of valve New valve regurgitation Minor criteria · Fever · Predisposition to IE (heart condition or IVDA intravenous drug abuse) · Echocardiogram consistent with IE · Immune phenomena: Roth spots, Osler’s nodes, glomerulonephritis, rheumatoid factor · Microbiological evidence of IE (positive blood culture but do not meet major criteria, serological support for infection) · Vascular phenomena (emboli, mycotic aneurysm, septic pulmonary infarcts, conjunctival haemorrhage, intracranial haemorrhage, Janeway lesions)

140

Mastering MRCP BOF: 2.55 A 67-year-old male from Thailand was admitted with a history of severe crushing central chest pain. His ECG showed ST elevation in the anterior chest leads and he was treated with tPA, which resulted in resolution of the ECG changes. Over the next few days he had several episodes of pulmonary oedema, his blood pressure began to rise and he was noted to have a petechial rash over his feet. His renal function began to deteriorate and he was ultimately commenced on haemodialysis. His blood tests showed an elevated eosinophil count of 1.13×109/l (normal range <0.40) In this patient underlying condition is: a) Polyarteritis nodosa b) Henoch-Schonlein purpura c) Cholesterol embolisation syndrome d) Beurger’s disease e) Tropical pulmonary eosinophilia Answer: c) The cholesterol embolisation syndrome is a serious multisystem disorder that arises in individuals with generalised atheroma. It is caused by destabilisation of cholesterol plaques, which leads to release of cholesterol crystals. These crystals circulate and get lodged in small blood vessels with consequent infarction of tissue. The crystals also provoke an inflammatory response in the affected vessel with consequent fibrosis of the vessels. This may occur months after the in initial insult. The inflammatory response is thought to account for the eosinophilia, which is often seen in this condition. Characteristics of cholesterol embolisation syndrome: · Livedo reticularis · Acute renal failure · Eosinophilia Risk factors for cholesterol embolism: · Hypertension · Diabetes mellitus · Aortic aneurysm. Precipitating factors: · Angiography · Anticoagulation · Thrombolysis · Vascular surgery (including coronary artery bypass grafting)

BOF: 2.56 An 18-year-old male is admitted as an emergency. He has ingested a large number of his mother’s propranolol tablets following a family dispute. On admission he had a generalised seizure and is now very drowsy responding only to painful stimuli. He has a severe bradycardia and is hypotensive. In this situation which of the following would be the most suitable agent to use? a) Atropine b) Adrenaline c) Noradrenaline

141

Mastering MRCP d) Glucagon e) Sodium bicarbonate Answer: d) Glucagon acts by bypassing the blocked beta-receptors. It activates adenyl cyclase thus forming cyclic AMP from ATP. AMP results in beta stimulation directly

BOF: 2.57 A 32-year-old female is admitted as an emergency having taking an overdose of dosulepin. In this condition which one of the following correlates with the severity of the poisoning? a) Plasma drug concentration b) Width of the QRS complex on ECG c) Degree of metabolic acidosis d) Degree of respiratory acidosis e) Urinary retention Answer: b) Dosulepin (a tricyclic antidepressant) is toxic in overdose. The main mechanism of toxicity is cardiac sodium channel blockade. This increases the duration of the cardiac action potential and refractory period and delays atrioventricular conduction. ECG changes in overdose are as follows · Prolongation of the PR interval · Prolongation of the QRS interval · Prolongation of the QT interval · Non-specific ST segment and T wave changes · Atrioventricular block · Right axis deviation of the terminal 40ms vector of the QRS complex in the frontal plane (T 40ms axis) · Brugada pattern (downsloping of the ST segment elevation in leads V1-V3 with right bundle branch block) Maximal changes in the QRS duration and the T 40ms axis usually occur within 12 hours of ingestion. These changes may take up to a week to resolve. Sinus tachycardia is the most common arrhythmia. This is due to anticholinergic activity and inhibition of norepinephrine (noradrenalin) uptake by tricyclic antidepressants. Bradyarrhythmias (due to atrioventricular block) and tachyarrhythmias (supraventricular and ventricular) may occur. Torsade de pointes may occur uncommonly. Hypotension occurs as a result of decreased myocardial contractility and reduced systemic vascular resistance caused by alpha-adrenergic blockade. Life-threatening arrhythmias and death due to tricyclic antidepressant poisoning usually occur within 24 hours of ingestion. Rapid deterioration is common. Predictors of Severity · Level of consciousness at presentation is the most sensitive clinical predictor of serious complications.

142

Mastering MRCP · A QRS duration >100ms and a rightward T 40ms axis appear to be better at predicting cardiovascular toxicity than the plasma tricyclic drug concentration although they have moderate sensitivity and specificity for predicting the onset of complications

BOF: 2.58 A 36-year-old female has been admitted as an emergency having taken an overdose of dosulepin. She is drowsy and has dilated pupils. Whilst you are examining her you notice that the ECG monitor show supraventricular tachycardia. Which one of the following would you administer to this patient? a) Adenosine b) Verapamil c) Lidocaine d) Amiodarone e) Sodium bicarbonate Answer: e) In poisoning with tricyclic antidepressants, the onset of supraventricular or ventricular tachycardia is an indication for treatment with sodium bicarbonate 50 mmol (50 mL of 8.4%) over 20 minutes. This should be given even if there is no acidosis

BOF: 2.59 3 groups of volunteers with similar levels of raised blood cholesterol were followed over the course of 24 months to compare the effects of different lipid lowering agents. Choose the most appropriate statistical analysis to determine any quantitative serological differences as a result of the different treatments. a) Regression analysis b) F test (ANOVA) c) Correlation analysis d) Chi-square test e) t test Answer: b) The F test (ANOVA) is used to compare the means of 3 or more samples to determine statistical significance of an observed finding. The student’s t test compares the means of 2 samples. The information in this stem is limited with no indication of the number of volunteers or whether they are age and sex matched. These rules out the use of the other tests.

BOF: 2.60 In a prospective epidemiological study the efficacy of 2 different chemotherapeutic agents for carcinoma cervix were being compared. Cases were selected from one group of patients who have had periodic PAP smears for 3 years or more prior to diagnosis. The other group had no PAP smear history. What limitation may result from case selection? a) Confounding

143

Mastering MRCP b) Interviewer bias c) Berksonian bias d) Lead time bias e) Recall bias Answer: d) A more favourable response by the first group may be attributed to the fact that they are likely to have entered a course of treatment earlier than their counterparts. This “Lead time” gained by the first group results in a form of non random or systematic error which can be avoided through appropriate study design.

BOF: 2.61 A new surrogate lifestyle-marker for cardiovascular risk is being developed. The researcher wishing to pilot this in a high-risk population chooses to assess lifestyle using a multiple-choice questionnaire. The data from this approach alone would be questionable due to issues with which of the following? a) Reliability b) Validity c) Accuracy d) Sensitivity e) Specificity Answer: b) The answers on a multiple-choice questionnaire may not reflect actual behaviour / lifestyle, which is best, recorded through direct observation. However, due to practicality it would be better to use a questionnaire-based interview involving recall of activities as well as direct responses. Accuracy may also be an issue in this example but it is of higher importance to develop tools, which are first of all valid.

BOF: 2.62 In a cohort study, the occurrence of infertility was recorded for smokers and non-smokers in the reproductive age group, over a decade. The difference in incidences between the 2 groups was reported to be significant at the p > 0.05 level. Which is the most appropriate interpretation that can be made from the results of this study? a) The null hypothesis can be rejected; results may have occurred by chance up to 5% of the time b) A significant difference between the 2 groups may occur 5% of the time or less c) The null hypothesis may be accepted as the difference between the 2 groups is only 5% of the time d) The alternate hypothesis may be rejected e) A causal relationship between exposure and outcome may be established at significance level 0.05 Answer: a) Most researchers use the 0.05 significance value to reject the null hypothesis. This is arbitrary but universally accepted as the probability that differences observed are purely due to chance is less than 5% at this significance level. The larger the numbers and longer the study (especially for observational studies such as a cohort study) the more meaningful the results if p < 0.05.

144

Mastering MRCP BOF: 2.63 A regional audit showed that 62 women who with deep vein thrombosis whilst in hospital a total of 45 women had used oral contraception for varying periods during their lifetime. The odds ratio can be best expressed as which of the following? a) 45/62 b) 17/62 c) 62/17 d) 45/17 e) 62/45 Answer: c) The odds ratio is an estimate of relative risk and is significant (in the absence of experimental error) at values above 1. In this case the information available is that of oral contraceptive positive and negative DVTs (i.e. cells A and C of an ABCD 2x 2 tables). Since we do not know the number of women who did not have a DVT we cannot determine cells B and D of a 2x2 table. Odds ratio is usually calculated as AD/BC. However since we do not know the values of B and D, the best answer is A/C.

BOF: 2.64 A 64-year-old male who is known to have insulin dependent diabetes mellitus presents with recurrent episodes of hypoglycemia. This has necessitated reduction in his daily insulin dosage. He also has decreased renal function on account of a combination of diabetic nephropathy and bilateral renal artery stenosis. In this patient the recurrent hypoglycemic episodes are likely to be a consequence of: a) Non Alcoholic Fatty Liver Disease resulting in decreased liver glycogen stores b) Autonomic neuropathy resulting in decreased absorption of sugar c) Decreased catabolism of insulin by the kidney d) Activation of the renin angiotensin system e) Decreased insulin resistance Answer: c) The kidney is one of the sites of catabolism of small molecular weight proteins. This is done by endocytosis of these proteins in the brush border of the proximal tubular cells. In renal failure this process is reduced and the half-life of these proteins is increased resulting in reduced insulin requirements in these patients. Other hormones that are metabolized by the kidneys are: PTH and calcitonin

BOF: 2.65 A 26year old female presents with sudden onset of retrosternal chest pain. On direct questioning she reveals that the pain came on shortly after smoking marijuana. On examination she has a pulse rate of 110 beats per minute, BP 110/60, JVP not elevated, trachea midline, apex not displaced. On auscultation a crunching sound is heard in systole.

145

Mastering MRCP Respiratory and abdominal examination is unremarkable. No abnormality is seen on the plain x-ray of her chest, ECG shows T inversion in the anterior chest leads. Which of the following investigations is likely to confirm your diagnosis? a) Coronary angiogram b) Upper gastrointestinal endoscopy c) VQ scan d) Chest CT scan e) Gastograffin swallow Answer: d) The sudden onset of chest pain with the finding of a pericardial crunch (Hamman’s sign) suggests the patient has developed a spontaneous pneumomediastinum. The normal respiratory examination and normal plain x-ray of the chest will exclude pneumothorax. Lateral chest x-ray is likely to show air in the mediastinum but CT of the chest will be more accurate. Spontaneous pneumomediastinum is caused by alveolar rupture with the released air subsequently dissecting the tissues and travelling proximally to reach the mediastinum It may be caused by any maneuver that increases alveolar pressure such as: Forceful coughing, crying, or shouting Vomiting, defecation, and Valsalva maneuvers Strenuous athletic activity, diving, flying, playing woodwind instruments, and childbirth Spirometry. COPD Respiratory tract infections Foreign body aspiration Mechanical ventilation Inhalation of drugs may such as marijuana and cocaine may also cause spontaneous pneumomediastinum Management is conservative Rarely mediastinoscopy to relieve life threatening complications and placement of mediastinal drainage tubes

BOF: 2.66 A 59-year-old male is admitted to your ward with a history of jaundice. The patient has had an aortic valve replacement with a metal valve and is on warfarin on account of this. He is also known to have alcoholic liver disease with cirrhosis of the liver, portal hypertension and oesophageal varices. A transjugular intrahepatic portosystemic stent shunt (TIPSS) has been inserted to reduce portal pressure and decompress the varices. He is on warfarin, allopurinol and propranolol He admits to drinking 10 units of alcohol per week. On examination, he was deeply jaundiced, afebrile and had a flapping tremor. Pulse 80 beats per minute regular, BP 110/70, JVP not raised, heart sounds native first heart sound, closing click heard instead of a second heart sound, soft systolic murmur at the aortic area, lungs clear abdomen was normal. Investigations were as follows: HGB 8.4 L g/dl (13-18) RBC 2.36 L x10^12/l (4.5-6.0)

146

Mastering MRCP HCT 0.251 L (0.4-0.52) MCV 106.4 H fl (80-96) MCH 35.6 H pg (27-32) MCC 33.5 g/dl (31.5-36.0) WBC 4.3 x10^9/l (4-11) NE 2.6 x10^9/l (2.0-7.5) LY 1.0 L x10^9/l (1.5-4.0) MO 0.6 x10^9/l (0.2-0.8) EO 0.16 x10^9/l (0-0.4) BA 0.0 x10^9/l (0-0.1) PLT 46 L x10^9/l (150-400) Serum Folate 5.3 ug/l (2.0-20.0) B12 1410 H ng/l (160-900) ALT 34 U/L (13-43) BILIRUBIN 299 >> umol/L (0-17) ALKALINE PHOSPHATASE 129 U/L (45-130) ALBUMIN 21 << g/L (36-50) CALCIUM 2.00 < mmol/L (2.10-2.55) CALCIUM (CORRECTED) 2.45 mmol/L (2.10-2.55) C-REACTIVE PROTEIN 8 > mg/L (up to 5) Urinary Haemosiderin POS Abdominal Ultrasound Scan The liver is small in size. The TIPSS appears patent with normal blood flow. The IVC is also patent with normal blood flow. The gallbladder is distended and contains thick bile sludge. No gallstones identified. The spleen is large in size measuring 14.5cm in maximum diameter. Kidneys and pancreas are unremarkable. The urinary bladder is distended to the level of the umbilicus. In this patient the cause of the jaundice is most likely to be: a) Haemolysis across the prosthetic aortic valve b) Acute alcoholic hepatitis c) Carcinoma head of pancreas d) Infective endocarditis e) Haemolysis across the TIPSS Answer: e) The scenario and the results would suggest the patient has haemolytic jaundice. Haemolysis may occur across prosthetic heart valves but this is usually a low-grade haemolysis and does not cause severe jaundice. There were no features to suggest endocarditis. The fact that he has porto-systemic encephalopathy indicates that the TIPSS is causing complications and the jaundice is also likely to be a complication of the shunt. Transjugular intrahepatic portosystemic stent shunt (TIPSS) is a procedure in which a shunt is created between the portal vein and the hepatic veins. It thus reduces portal pressure. It is mainly used to treat variceal bleeding from oesophageal and gastric varices. It is also used to treat resistant ascites, hepatic hydrothorax, hepatorenal syndrome and Budd-Chiari syndrome. The main complication that results from a TIPSS is porto-systemic encephalopathy but haemolysis is also a recognised complication.

147

Mastering MRCP BOF: 2.67 A 62-year-old male has had an aortic valve replacement. He presents 5 months after surgery with malaise, anorexia and low-grade fever. On examination he has splinter haemorrhages in his nails; there are tender nodules on the pulps of his fingers and erythematous macules over his palms. His spleen is just palpable. You arrange for blood cultures to be taken on this patient Which of the following is the most likely infecting organism? a) Staphylococcus aureus b) MRSA c) Streptococcus viridians d) Staphylococcus epidermidis e) Streptococcus bovis Answer: d) The patient has clinical features suggestive of infective endocarditis (fever, anoxia, splinter haemorrhages, Osler’s nodes, Janeway lesions and a palpable spleen). Prosthetic valve endocarditis is divided into two categories: Early prosthetic valve endocarditis (less than 1 year) is usually due to contamination during surgery and in these cases the most common organism is Staphylococcus epidermidis Late prosthetic valve endocarditis is due to haematogenous spread and the most common cause is Streptococcus viridians There are different views as to the cut off point for dividing into early and late prosthetic valve endocarditis (PVE) some authorities say 60 days and others say 6 months

BOF: 2.68 A 53-year-old male presents with an itchy rash on the anterior aspect of his wrists. On examination you note several purplish, flat -topped papules. There are papules that have arisen in areas of trauma and you also note fine, white, lacy lines on the lesions. Oral involvement is also seen. In this patient which one of the following is characteristic of this condition? a) Purple colour b) Papules arising in areas of trauma c) Oral involvement d) Flat topped papules e) Fine white lacy lines on the lesion Answer: e) The clinical features described suggest the patient has Lichen planus. The fine white lacy lines on the lesions (these are known as Wickham's striae) are characteristic of this condition Lesions arising over areas of trauma (Koebner’s phenomenon) may occur in other conditions such as psoriasis, lichen nitidus and vitiligo. Oral involvement is common in this condition and is induced by trauma to the mucosa overlying the bite margin BOF: 2.69 A 28-year-old male who is known to be an intravenous drug abuser is admitted with a history of fever with chills and rigors. On examination he is febrile, has multiple puncture marks and thrombosed veins in his skin. He has a pulse rate of 120 beats per minute, his jugular venous pressure is elevated with a

148

Mastering MRCP predominant systolic wave and there is a pan systolic murmur at the left sternal edge, which increases with inspiration. The liver is enlarged and pulsatile. You arrange for blood cultures in this patient. Which of the following organisms is the most likely to be isolated in this patient? a) Streptococcus viridans b) Staphylococcus aureus c) Staphylococcus epidermidis d) Coxiella burnetii e) Candida albicans Answer: b) In an intravenous drug abuser presenting with fever, the diagnosis to consider is acute infective endocarditis (IVDA IE). The raised JVP with a V wave, the systolic murmur at the left sternal edge that increases on inspiration and the pulsatile liver suggest the patient has tricuspid regurgitation which is a consequence of acute infective endocarditis The most common infecting organism in IVDA IE is Staphylococcus aureus

BOF: 2.70 A 53-year-old female who was on amiodarone presents with weight loss despite a good appetite, heat intolerance, tremors and excessive sweating. On examination she has exophthalmos and a palpable goiter. Pulse rate was 120 beats per minute regular and she had a fine tremor of her hands Thyroid function tests showed the following values: thyroid stimulating hormone (TSH), < 0.03 mIU/L (normal, 0.5-5.0 mIU/L); thyroxine (T4), 15.3 µg/dL (normal, 4.0-10.5 µg/dL); triiodothyronine (T3), 256 ng/dL (normal, 80-200 ng/dL); free thyroxine index (FT4I), 19.4 (normal, 4.7-10.0). She was positive for thyroid peroxidase (“microsomal”) antibodies, and thyroid stimulating immunoglobulin. In this patient the most suitable initial treatment would be a combination of: a) Steroids and azathioprine b) Carbimazole and radioactive iodine c) Potassium perchlorate and carbimazole d) Carbimazole and surgery e) Steroids and carbimazole Answer: c) The clinic features suggest the patient has thyrotoxicosis. Amiodarone may induce thyrotoxicosis. There are two types of amiodarone-induced thyrotoxicosis: Type I caused by excess iodine in amiodarone (usually occurs where there is latent disease in the thyroid; Graves’ disease or nodular goiter) Type 2 which is a thyroiditis induced by amiodarone (normal thyroid) Treatment of the two types is different Treatment of amiodarone-induced thyrotoxicosis Type 1 Thionamide (methimazole, carbimazole, propylthiouracil) Potassium perchlorate Type 2 Prednisolone Hence it is important to differentiate the two types before commencing treatment. This can be difficult.

149

Mastering MRCP In this patient the features of goiter, exophthalmos suggest she has pre-existing thyroid disease and hence would favor type I amiodarone induced thyrotoxicosis. The presence of thyroid antibodies lends further weight to this diagnosis.

BOF: 2.71 A 64-year-old female has been admitted to your ward with a deep vein thrombosis. She has been started on treatment with warfarin and is about to be discharged. You speak to her about warfarin and she asks you whether there are any precautions she should take regarding her diet. Which one of the following would you ask her to avoid whilst taking warfarin: a) Orange juice b) Tomato juice c) Grapefruit juice d) Apple juice e) Cranberry juice Answer: e) Cranberry may enhance the action of warfarin and should be avoided whilst taking it.

BOF: 2.72 A 26-year-old male is admitted with a 2-day history of finding it difficult to walk. There is a history of a diarrheal illness 2 weeks prior to presentation. On examination he is found to have a flaccid paraparesis with absent knee and ankle jerks and equivocal sensory loss over his feet. Lumbar puncture is performed and the cerebrospinal fluid shows proteins 5.7 g/L In this patient which one of the following would you use as treatment? a) Iv immunoglobulin b) iv methylprednisolone c) Iv cyclophosphamide d) iv cyclosporin e) iv infliximab Answer: a) The clinical features of a rapid onset of flaccid paraparesis with absent reflexes and minimal sensory loss would suggest that the patient has developed the Gullain-Bare syndrome. The high proteins in the CSF would be further evidence in favor of this. Guillain-Barre syndrome could be treated with immunoglobulins iv or with Plasmapheresis

BOF 2.73 A 22-year-old female is admitted to your ward with a significant paracetamol overdose. Treatment is commenced with N-acetyl cysteine infusion. Which one of the following compounds does N acetyl cysteine replenish? a) Glutathione b) Methionine c) Phenyl alanine d) Cystine

150

Mastering MRCP e) Arginine Answer: a) In the liver paracetamol is mostly converted to inactive compounds via phase II metabolism by conjugation with sulphate and glucuronide, with a small portion being oxidized via the cytochrome P450 enzyme system Cytochromes P450 2E1 (CYP2E1) and 3A4 (CYP3A4) convert paracetamol to a highly reactive intermediary metabolite, N-acetyl-p-benzo-quinone imine (NAPQI). In health, NAPQI is detoxified by conjugation with glutathione. In paracetamol overdose, the sulphate and glucuronide pathways become saturated, and more paracetamol is shunted to the cytochrome P450 system to produce NAPQI. As a result of this excess production of NAPQI, hepatocellular supplies of glutathione become exhausted and NAPQI is free to react with cellular membrane molecules, resulting in widespread hepatocyte damage and death, which leads to acute hepatic necrosis. Studies performed on animals suggest that 70 % of hepatic glutathione must be lost before hepatotoxicity occurs. N-acetylcysteine provides sulphydryl groups that increase hepatic glutathione levels

BOF: 2.74 A 53-year-old alcoholic has been admitted to the medical admissions unit. He has presented to the accident and emergency department with a history of fits. He lives in a hostel and has not been managing to look after himself. The nursing staff tells you that he is confused and was unsteady on his feet when trying to walk to the toilet. On examination he looks unkempt and emaciated. He is confused, ataxic and has nystagmus and a 6th nerve palsy. In this patient your next step in management would be: a) Arrange an urgent CT scan of his head b) Treat with diazepam c) Treat with intravenous thiamine d) Treat with intramuscular haloperidol e) Check his blood glucose levels Answer: c) The patient is an alcoholic who is malnourished. The triad of features ophthalmoplegia, ataxia and confusion are classical manifestations of Wernicke’s encephalopathy. Treatment would be with intravenous thiamine If untreated the patient is likely to develop Korsakoff’s psychosis and there is a risk of death

BOF: 2.75 A 53-year-old male presents with painful joints. He complains of feeling tired and lethargic and on examination you notice that he has pain, swelling, deformity and limitation of movement of his 2nd and 3rd metacarpophalangeal joints. Which of the following tests would indicate the nature of the arthropathy? a) Uric acid levels b) ESR

151

Mastering MRCP c) Anti nuclear factor d) Rheumatoid factor e) Ferritin levels Answer: e) The feeling of tiredness and lethargy should alert one to the fact that there is a systemic disturbance in this patient. The 2nd and 3rd metacarpophalangeal joints are commonly affected in the chondrocalcinosis associated with haemochromatosis and hence this is the most likely diagnosis in this middle-aged male. The serum ferritin levels would indicate this

BOF: 2.76 A 26-year-old male is transferred to your ward from the short stay unit of the accident and emergency department because of ongoing pyrexia. He was initially admitted having become unconscious following an alcoholic binge. He had been brought in covered in vomitus but had no haematemesis. On examination he was conscious and alert. Body temperature 39 ºCelsius, pulse rate 120 beats per minute respiratory rate 32 breaths per minute. His trachea was in the midline, respiratory movements were diminished on the right side of his chest, percussion note was dull in the right mid zone anteriorly and over this area the breath sounds were amphoric in nature. In this area there were fine crepitations, vocal fremitus and vocal resonance were increased and there was whispering pectoriloquy. In this patient which of the following antibiotic regimes would you use? a) Cefotaxime and gentamicin b) Amoxycillin and clarithromycin c) Co-amoxiclav and clarithromycin d) Co-trimoxazole e) Ceftriaxone and metronidazole Answer: e) The history of loss of consciousness and vomiting should immediately make one think of aspiration pneumonia. The physical signs suggest there is cavitation in the right lung Treatment of aspiration pneumonia is with broad-spectrum antibiotics including cover for anaerobic organism. Hence the combination of a cephalosporin with metronidazole would be the best of the choices listed.

BOF: 2.77 A 42-year-old male who is known to have Down's syndrome is admitted by his general practitioner for management of an in tractable itch. The itch is worse at night. He is a long term resident in an institution for the mentally handicapped On examination of his skin you notice multiple vesicles, pustules, papules, burrows and excoriations especially over the webs of his fingers and in the flexures. What is the most likely diagnosis? a) Polycythaemia rubra vera b) Lichen planus c) Scabies d) Pityriasis rosea e) Dermatitis herpetiformis

152

Mastering MRCP Answer: c) Scabies is an infestation by the parasite Sarcoptes scabiei var hominis. The itch is classically worse at night and the infestation is more common in families or in people in institutions. A burrow is a short elevated serpiginous (S-shaped) track in the superficial epidermis and this is pathognomonic of scabies infestation. A small vesicle or papule may appear at the end of the burrow or occur independently. Excoriations occur due to the itch and secondary infection may occur

BOF: 2.78 A 48-year-old male presents complains of feeling tired most of the time. He feels sleepy during the day and says he does not feel refreshed when he wakes up in the morning. His wife says he snores a lot. He says because of the daytime sleepiness he almost crashed his car on two occasions. He also complains of having to get up at night to pass urine. He has decreased libido and becomes irritable. On examination he is grossly overweight. Which of the following is associated with the condition the patient has? a) Hypertension b) Hypothyroidism c) Hypoglycaemia d) Hyperuricaemia e) Hyperlipidaemia Answer: a) The clinical features of the case would suggest the patient has obstructive sleep apnoea. There is an independent association between obstructive sleep apnoea and hypertension

BOF: 2.79 You are teaching one of your juniors how to insert a subclavian line. To emphasis the importance of knowing the anatomy of this region, you ask the junior which one of the following structures lies anterior to the scalenus anterior muscle near its insertion to the first rib: a) Subclavian vein b) Subclavian artery c) Phrenic nerve d) Brachial plexus e) Pleura Answer: a) The subclavian vein lies anterior to the scalenus anterior muscle near its insertion to the first rib

BOF: 2.80 A patient who is known to have hyperlipidaemia has been started on simvastatin. You see him in the cardiovascular risk clinic and he asks you about diet as he has recently started eating a lot of fruit and taking a lot of fruit juice as he has heard that this is good for him Which of the following would you ask him to avoid a) Orange juice b) Tomato juice

153

Mastering MRCP c) Cranberry juice d) Pineapple juice e) Grapefruit juice Answer: e) Statins such as simvastatin and atorvastatin are metabolized by CYP3A4. Grapefruit juice is a CYP3A4 inhibitor. Ingesting grapefruit juice may increase levels of these drugs thereby increasing the risk of dose related side effects such as myopathy and rhabdomyolysis. Patients taking simvastatin should avoid grapefruit juice as even modest amounts can increase drug levels. Patients taking atorvastatin should avoid drinking large quantities of grapefruit juice

BOF: 2.81 A 54-year-old male who is known to have alcoholic liver disease and cirrhosis has been admitted to your ward with tense ascites. After draining the ascites you start the patient on spironolactone. One of the medical students asks you why you used spironolactone instead of furosemide. You begin to explain how spironolactone counteracts the effects of aldosterone and he asks you in which part of the kidney aldosterone acts. You reply that aldosterone acts primarily on the: a) Proximal convoluted tubule b) Ascending loop of Henle c) Descending loop of Henle d) Collecting ducts e) Macula densa Answer: d) Mineralocorticoids act primarily on the principal cells (P cells) of the cortical collecting ducts absorbing sodium and exchanging it for potassium. This results in a decrease in quantity of sodium in the blood and an increase in the quantity of sodium in the urine. Conversely, the amount of potassium increases in the blood and decreases in the urine. Spot urine would therefore show higher sodium and lower potassium. This may be used to monitor treatment

BOF 2.82 A 75-year-old female has been admitted to the orthopaedic unit with a fracture neck of femur. She has had a DEXA scan earlier in the year and this had shown osteoporosis with a high risk of fracture. You decide that the patient should be started on calcium and vitamin D supplements and a bisphosphonate. The mechanism of action of bisphosphonates is: a) Inhibits the action of osteoclasts b) Increases the action of osteoblasts c) Increases deposition of calcium in the bone matrix d) Suppresses the action of parathyroid hormone e) Increases the action of 1,25 dihydocholecalciferol Answer: a) Although the detailed mechanism of action of bisphosphonates has not been elucidated, it is clear that at the tissue level bisphosphonates inhibit bone resorption, bone turnover, thereby decreasing bone loss. The effect is caused by a decrease in the formation of new bone remodelling units and a decrease in the depth

154

Mastering MRCP of the erosion cavities. At the cellular level, bisphosphonates inactivate osteoclastic bone resorption directly and or indirectly

BOF: 2.83 A 52-year-old male presents with severe headache, nausea and vomiting. He also complains of double vision. On examination, he was afebrile, his pulse rate was 120 beats per minute and his blood pressure was 85/50. There was a complete ptosis of his right eye and the right pupil was dilated and the eye was deviated inferiorly and laterally. There was neck stiffness. Investigations: Hb 14.2 g/dL WCC 15.2 Na 120 mmol/l (137-144 mmol/L) K 5.3 mmol/L (3.5-4.9 mmol/L) TSH 1.0 mU/L (0.4-5 mU/L) Free T4 6 pmol/L (10-22 pmol/L) Serum cortisol 150 nmol/L (200-700 nmol/L) In this patient which one of the following conditions is most likely? a) Meningitis b) Cavernous sinus thrombosis c) Pituitary apoplexy d) Subarachnoid haemorrhage e) Brain stem stroke Answer: c) Pituitary apoplexy is caused by acute haemorrhage or infarction of the pituitary gland resulting in sudden onset of headache, visual symptoms, altered mental status, and hormonal dysfunction. Usually there is an existing pituitary adenoma. The visual problems could be visual acuity and visual field impairment from involvement of the optic nerve or chiasma and motility dysfunction of the eye caused by involvement of the cranial nerves traversing the cavernous sinus. Most patients develop hypopituitarism

BOF: 2.84 A 56-year-old male is admitted with a 3-hour history of severe, crushing central chest pain, which radiates to his shoulder tip and is associated with sweating and nausea. You arrange an ECG on this patient. Which one of the following would be an indication for thrombolysis? a) 1 mm ST elevation in one chest lead b) 1 mm ST elevation in one limb lead c) Right bundle branch block d) Dominant R wave with ST depression in leads V1 to V3 e) 3 mm ST depression in 3 limb leads Answer: d) Indications for thrombolysis: Presentation with chest pain of less than 12 hours duration with one of the following ECG changes: ♥ ST elevation > 2 mm in 2 or more chest leads

155

Mastering MRCP ♥ ♥ ♥

ST elevation > 1mm in 2 or more limb leads Dominant R waves and ST depression in V1-V3 (posterior infarction) New onset left bundle branch bloc

BOF: 2.85 A 65-year-old male is to have a colonoscopy. He has had an aortic valve replacement with a metal valve. He is penicillin allergic. Which one of the following would you prescribe as antibiotic prophylaxis? a) Amoxycillin and gentamicin b) Ceftriaxone and gentamicin c) Teicoplanin and gentamicin d) Ciprofloxacin and gentamicin e) Clarithromycin and gentamicin Answer: c) Patients who have prosthetic heart valves should have prophylactic antibiotics when undergoing endoscopy. Patients not allergic to penicillin and who have not had penicillin more than once in the previous month should have amoxycillin and gentamicin Patients allergic to penicillin or who have had penicillin more than once in the previous month should have either: Vancomycin and gentamicin Or Teicoplanin and gentamicin BOF: 2.86 A 67-year-old male who is known to have hypertension and is a heavy smoker is admitted with a crushing central chest pain radiating to his shoulder tip. It is accompanied by sweating, nausea and vomiting. After looking at his ECG you take blood for estimation of troponin T levels. A medical student who is clerking on your firm asks you what troponin T is. You reply that troponin T is a: a) Cardiac enzyme b) Cell membrane protein c) Contractile protein d) Structural protein e) Ribosomal protein Answer: c) Troponin is a complex of three proteins that are involved in muscle contraction in skeletal muscle and cardiac muscle but not in smooth muscle. Troponin consists of 3 subunits troponin T, troponin I and troponin C. Troponin I and T are sensitive and specific indicators of damage to cardiac muscle.

BOF: 2.87 A 67-year-old male presents with a history of chronic cough. He smokes 20 cigarettes a day and has done so for 40 years. On examination he has a partial ptosis of his right eyelid and a constricted pupil on the right side. There is reduced sweating on the right side of his face. On examination of his hands you note that there is wasting of all the small muscles of his right hand.

156

Mastering MRCP Examination of the chest shows that the trachea is deviated to the right and there are bronchial breath sounds over the right apex with fine crepitations in this region. In this patient the wasting of the small muscles of his hand is due to the lesion affecting nerve root: a) C8 b) C7 c) T1 d) T2 e) C6 Answer: c) The small muscles of the hand are supplied by nerve root T1 The physical signs suggest that the patient has a right apical lung cancer complicated by a T1 lesion and Horner’s syndrome

BOF: 2.88 A 75-year-old male presents with anorexia and loss of weight. He has an endoscopy, which shows a malignant looking ulcer in the body of the stomach, biopsies show adenocarcinoma. He has had a rash over his trunk for several months prior to presentation Which of the following skin rashes is associated with the condition he has: a) Erythema annulare b) Migratory necrolytic erythema c) Erythema marginatum. d) Erythema gyratum repens e) Erythema chronicum migrans Answer: d) Erythema gyratum repens is defined as a figurate erythema. Figurate means having a definite form or figure. Other figurate erthemas are erythema annulare centrifugum, erythema migrans, and erythema marginatum. Erythema gyratum repens is believed to be a paraneoplastic process being associated with malignancy in almost 80% of patients. The lung is the commonest organ affected followed by the breast, urinary bladder, uterus, cervix, gastrointestinal tract (stomach), and prostate. The rash often precedes diagnosis of the malignancy and is present at an average of 9 months before the diagnosis of the malignancy (range of 1-72 months). The features of the rash are: It has the appearance of wood-grain. This appearance is created by concentric, mildly scaling bands of flat or raised erythema. It mainly affects the trunk and the extremities It migrates fairly rapidly There is an intense itch The course of the rash mirrors the course of the underlying illness with clearance of the lesion within 6 weeks of resolution of the underlying illness

BOF: 2.89 Which of the following statements best describes the action of gastrin? a) Simulates the G cells in the gastric pits to produce hydrochloric acid

157

Mastering MRCP b) Stimulates enterochromaffin-like cells to produce histamine c) Stimulates the G cells in the gastric pits to produce histamine d) Decreases bicarbonate production e) Stimulates the G cells to reabsorb bicarbonate Answer: b) Gastrin is synthesized in the G cells, which are located in gastric pits that are found primarily in the antral region of the stomach. Gastrin binds receptors found predominantly on parietal and enterochromaffin-like cells When stimulated by gastrin (along with histamine and acetylcholine) parietal cells secrete acid Stimulation of ECL cells by gastrin results in histamine release. Histamine binds to H2 receptors on parietal cells causing acid secretion.

BOF: 2.90 A 78-year-old male presents with falls. On examination you note that he has a pill-rolling tremor of his fingers, there is bradykinesia, cogwheel rigidity of his limbs and a positive glabella tap. He is also depressed. You discuss drug therapy with the consultant and he asks you for your opinion on the use of selegeline in this case and whether it will be useful in order to alleviate the depression. You know that the mechanism of action of selegeline is: a) It is a monoamine oxidase inhibitor b) It is a tricyclic antidepressant c) It is a dopaminergic drug d) It is an anticholinergic e) It inhibits catechol-o- methyl transferase. Answer: a) Selegeline is a monoamine oxidase inhibitor (MAOI)

BOF: 2.91 A 60-year-old female is admitted with a history of melaena. She has been taking ibuprofen tablets for pain relief as she has osteoarthritis. On examination she is pale, pulse rate 110 beats per minute, blood pressure 90/60 Hb 9.3 g/dL She undergoes endoscopy and the endoscopist reports that there was a chronic anterior duodenal ulcer with a visible vessel. He has injected the ulcer with 1: 10,000 adrenaline and used heater probe coagulation on the vessel. Which of the following treatment regimes would prevent rebleeding a) Helicobacter pylori eradication b) Omeprazole infusion for 72 hours c) Intravenous pantoprazole daily bolus d) Ranitidine infusion e) Oral omeprazole in high dose Answer: b) Following endoscopic treatment of a bleeding peptic ulcer, an infusion of omeprazole for 72 hours following an initial bolus of 80 mgs has been shown to decrease the incidence of rebleeding

158

Mastering MRCP BOF: 2.92 An 18-year-old male is admitted with a history of progressive drowsiness. The previous day he had been seen in the accident and emergency department where he had presented with a history of a head injury after being involved in a brawl in a nightclub. As he had been well with no neurological signs, he had been discharged with instructions to take paracetamol if he developed a headache. On examination he was drowsy but there were no localizing signs. What is the likely cause of this patient’s decreased level of consciousness? a) Extradural haemorrhage b) Subdural haemorrhage c) Subarachnoid haemorrhage d) Intracerebral haemorrhage e) Intraventricular haemorrhage Answer: a) Extradural haemorrhage is usually caused by fractures of the parietal or temporal bone causing tears of the middle meningeal artery and vein. The typical history is that of a head injury followed by a lucid interval after which progressive deterioration in conscious level occurs. The lucid interval may last from a few hours to a few days

BOF: 2.93 A 30-year-old male has been investigated for weight loss and was found to have mediastinal lymphadenopathy. He had endoscopic ultrasound and biopsy of the lymph nodes and this showed caseating granulomas .A diagnosis of tuberculosis was made and the patient was started on isoniazid, rifampicin and pyrazinamide. He presents with breathlessness and has stridor. Your next course of action would be: a) Treat with steroids b) Add streptomycin to the above regime c) Tracheostomy d) Intubation and ventilation e) Mediastinoscopy and drainage Answer: a) Following initial treatment of tuberculosis an increase size of the lymph nodes may occur and this is thought to be due to exaggeration of the inflammatory response. Treatment with steroids would be the first course of action

BOF: 2.94 A 54-year-old male has an endoscopy for investigation of dyspepsia. The endoscopist reports and unusual gastritis and takes multiple biopsies. Histology is reported as gastric B cell lymphoma (MALTOMA) low grade Your next course of action would be: a) Long term acid suppression using a proton pump inhibitor b) Helicobacter pylori eradication therapy c) Partial gastrectomy

159

Mastering MRCP d) Total gastrectomy e) Chemotherapy Answer: b) A MALT lymphoma (MALTOMA) refers to a lymphoma of the mucosa associated lymphoid tissue. Gastric maltoma are low-grade gastric B-cell lymphoma Low grade gastric MALTOMA is a neoplasia with a very indolent course and the prognosis is usually excellent. It has a tendency to remain localized to the gastric wall and seldom involves lymph nodes and bone marrow. H pylori is suggested to be associated with low-grade gastric MALTOMA Complete remission may be achieved in up to 80% of patients with early stage MALTOMA following eradication of the bacteria In patients who fail to respond to H pylori eradication or have low grade gastric MALTOMA without H pylori infection, radiotherapy, chemotherapy or surgery may be tried

BOF: 2.95 A 57-year-old male who is known to have chronic obstructive pulmonary disease presents with swelling of his ankles On examination he is in sinus rhythm with a pulse rate of 100 beats per minute. His JVP is elevated with a predominant V wave. He has bilateral pitting ankle oedema In this patient which of the following auscultatory signs would suggest to you that the patient has developed pulmonary hypertension? a) Reversed splitting of the second heart sound b) Pan systolic murmur at the left sternal edge c) 3rd heart sound d) 4th heart sound e) Loud 2nd heart sound Answer: e) A loud 2nd hear sound occurs in pulmonary hypertension and this may be palpable. Reversed splitting of the second hear sound would occur in severe aortic stenosis, left bundle branch block, right ventricular pacing, patent ductus arteriosus A pan systolic murmur at the left sternal edge in this patient would suggest tricuspid regurgitation Right ventricular S3 could be heard in right ventricular failure and tricuspid regurgitation A S4 would suggest reduced compliance of the ventricular due to hypertrophy or cardiomyopathy

BOF: 2.96 A 26-year-old female who is in the second trimester of her first pregnancy presents with fever, loin pain and dysuria. On examination she is febrile and is tender in her loins. Urine shows many pus cells and her white blood count shows a neutrophilia. Which of the following underlying conditions is she likely to have? a) Horseshoe kidney b) Duplex ureters c) Uterovaginal prolapse

160

Mastering MRCP d) Reflux nephropathy e) Renal stone disease. Answer: d) Reflux nephropathy is caused by persistent reflux of sterile or infected urine from the bladder to one or both kidneys via the ureters. Sterile vesico-ureteric reflux may cause renal scarring but the appearance of renal scarring or the extension of established renal scars is thought to require infection. The common pathogen that causes infection is Escherichia coli. . The greater the degree of reflux, the greater the likelihood of development of new or progressive scarring in association with infection. Intrarenal reflux (extension of vesico-ureteric reflux into the collecting tubules of the nephrons) allowing urinary microorganisms access to the renal parenchyma, is thought to be particularly important in the development of renal scarring Although vesico-ureteric reflux occurs at a similar rate in boys and girls; girls are at greater risk of developing reflux nephropathy because they have an increased incidence of urinary tract infection Reflux nephropathy is one of the renal diseases encountered frequently in women of childbearing age. Patients with severe bilateral atrophy are the most likely to develop proteinuria, hypertension, focal glomerular sclerosis and progressive chronic renal failure Those with persistent vesico-ureteric reflux are the most likely to suffer recurrent pyelonephritis. Clinically latent disease often first manifests in pregnancy, usually by urinary tract infection but also by proteinuria, hypertension, pre-ecclampsia or renal failure. Pregnancy is successful and uneventful whenever renal function is normal or near normal and hypertension is absent at conception. Urinary tract infection accounts for frequent morbidity but rarely results in fetal mortality. If renal function is significantly impaired and if hypertension is also present, there is a high risk of fetal growth retardation or intrauterine death. In addition there is a striking risk of rapid worsening of renal function and hypertension, with accelerated progression towards end-stage renal failure. Women with reflux nephropathy should attempt to conceive before renal function has begun to deteriorate Patients with poor renal function should be informed of the high risk for the pregnancy and the fact that renal function may deteriorate further

BOF: 2.97 A 70-year-old male is admitted to the ward for investigation of a change in bowel habit. On examination he looks well, he is not in respiratory distress, pulse rate 80 beats per minute regular, BP 140/60; no abnormality was detected on examination of his chest and abdomen. He has a chest x-ray as part of the routine work up in your unit. The chest x-ray shows a calcified pleural plaque located in the diaphragmatic pleura on the right hand side. Which of the following is likely in this patient? a) Healed pulmonary tuberculosis b) Mesothelioma c) Asbestosis d) Asbestos exposure

161

Mastering MRCP e) Active pulmonary tuberculosis Answer: d) Calcified pleural plaques located in the diaphragmatic pleura are indicators of asbestos exposure. However, this is not required for a diagnosis of asbestosis. Asbestosis refers to pulmonary fibrosis caused by asbestos. The patient does not have any evidence of pulmonary fibrosis Mesothelial cells line the body cavities; the pleura, peritoneum, pericardium, and the tunica vaginalis around the testis. Malignant mesothelioma refers to malignancies involving mesothelial cells Pulmonary tuberculosis usually affects the upper zones

BOF: 2.98 A 22-year-old female who is known to have Crohn's disease is seen by you at the inflammatory bowel disease clinic. She has pain in her joints and she asks you what type of arthritis occurs in people with Crohn's disease You reply that in Crohn's disease females are more likely to develop: a) Sacroiliitis b) Spondyloarthropathy c) Peripheral oligoarthritis d) Enthesitis e) Distal symmetrical small joint arthropathy Answer: c) In inflammatory bowel disease patients may develop either a peripheral arthropathy or an axial arthropathy. Peripheral arthropathy: This occurs in 10-20% of patients with IBD It is more common in Crohn’s disease than ulcerative colitis Usually coincides with inflammatory bowel disease activity. It mainly affects the lower limbs. It is an oligoarthritis. It is asymmetric, relapsing. , migratory Usually lasts 2-6 weeks. HLA-B27 association-uncommon Radiographic changes are rare. Axial arthropathy Axial arthritis (sacroiliitis and spondylitis) occurs in 2-7% of patients with inflammatory bowel disease May precede the onset of GI symptoms Chronic course independent of the activity of the inflammatory bowel disease 2-7% of patients with inflammatory bowel disease HLA-B27 association 50-70%.

162

Mastering MRCP BOF: 2.99 A 42-year-old businessman presents with severe crushing central chest pain associated with nausea and sweating. The pain radiates down his left arm He admits to have snorted cocaine whilst at a nightclub. His pain would be the result of cocaine causing: a) Vasospasm in the coronary arteries b) Accelerated atherosclerosis c) Atheromatous plaque rupture d) Coronary artery dissection e) Myocarditis Answer: a) Cocaine induces release of catecholamines and blocks presynaptic reuptake. This results in tachycardia, hypertension, and increased myocardial oxygen consumption. Increased alpha-adrenergic stimulation causes arterial vasospasm, which includes the coronary arteries. Cocaine promotes platelet aggregation, decreases prostacyclin production and release, and increases thromboxane A production. Local increase in levels of platelet-derived serotonin may lead to vasospasm sufficient to provoke distal myocardial ischaemia or myocardial infarction (MI) Chronic cocaine results in accelerated atherosclerosis. Dopamine depletion that results from chronic cocaine use can lead to coronary vasoconstriction. Cocaine-related myocardial insults could be caused by coronary atherosclerosis and or coronary spasm Tachycardia and hypertension increase myocardial work. Direct toxic effects of cocaine on the cardiac muscle include focal myocarditis, fibrosis, and hypertrophy. These histological changes act as foci for dysrhythmias (area of slower conduction leading to re-entry tachycardia) during a catecholamine surge. Cocaine has quinidine-like effects. This can resultant in intraventricular conduction delays leading to QTc prolongation and QRS complex widening. Large doses of cocaine can cause negative inotropy

BOF: 2.100 A 26-year-old male presents with a history of haematemesis. He has been to a nightclub and consumed 1 bottle pf vodka after which he vomited several times and during the last episode of vomiting he brought up a cupful of blood He was referred for endoscopy. What is the likely finding at endoscopy? a) Mallory- Weiss tear b) Oesophageal varices c) Oesophageal ulcer d) Oesophagitis e) Duodenal ulcer Answer: a) Mallory-Weiss syndrome refers to upper gastrointestinal bleeding secondary to longitudinal mucosal lacerations at the gastro-oesophageal junction or gastric cardia.

163

Mastering MRCP Mallory-Weiss tears may occur after any event that provokes a sudden rise in intragastric pressure or gastric prolapse into the oesophagus.

BOF: 2.101 A 70-year-old female presents with acute pulmonary oedema. She is known to have mitral stenosis and when last seen in clinic 6 months prior to presentation she had been in sinus rhythm. She hast fast atrial fibrillation and pulmonary oedema. Which of the following physical signs are you likely to elicit in this patient? a) “a” wave in JVP b) 4th heart sound c) 3rd heart sound d) Presystolic accentuation of mid-diastolic murmur e) Mid-diastolic murmur Answer: e) The physical signs that one may elicit in the cardiovascular system are as follows: Pulse Ñ Small volume pulse: Low output Ñ Sinus rhythm: Initially Ñ Atrial fibrillation: Later, due to left atrial dilatation JVP Ñ Elevated: Right heart failure Ñ V wave: Tricuspid regurgitation Apex Ñ Tapping apex: Loud 1st heart sound Ñ Left parasternal heave: Right ventricular hypertrophy Ñ Mid-diastolic thrill Auscultation Ñ Loud first heart sound Ñ Opening snap Ñ Mid diastolic rumble: Best heard at the apex or just medial to the apex Increases in expiration Ñ Pre-systolic accentuation Ñ Loud P2: Pulmonary hypertension Ñ Early diastolic murmur: · Graham-Steele murmur:

164

Mastering MRCP Pulmonary incompetence as a consequence of pulmonary hypertension in mitral stenosis The “a” wave is caused by atrial contraction and will be lost in atrial fibrillation. Similarly there will be no presystolic accentuation of the murmur because this is due to forceful atrial contraction, which will be lost in atrial fibrillation. A fourth heart sound occurs when there is forceful atrial contraction against a stiff ventricle. In atrial fibrillation there can be no fourth heart sound and anyway as flow across the mitral valve is decreased a fourth heart sound does not occur in mitral stenosis. A 3rd heart sound will not occur as stenosis of the mitral valve will prevent the rush of blood into the left ventricle in diastole which is responsible for the 3rd heart sound.

BOF: 2.102 A 70-year-old male has a history of recurrent falls. On examination he has a mask like facies, a “pill rolling” tremor, cogwheel rigidity of his limbs and bradykinesia. Your consultant suggests treatment with ropinirole. What is its mode of action? a) Aromatic amino acid decarboxylase inhibitor b) Catechol-O-methyl transferase inhibitors: c) Monoamine oxidase B inhibitors: d) Dopamine receptor agonists: e) Anticholinergic Answer: d) The drugs that may be used in Parkinsonism are as follows: • Laevodopa • Aromatic amino acid decarboxylase inhibitors: Benserazide Carbidopa • Catechol-O-methyl transferase inhibitors: Entacapone • Monoamine oxidase B inhibitors: Selegeline • Dopamine release: Amantadine • Dopamine receptor agonists: Bromocriptine Lisuride Pergolide Cabergoline Pramipexole Ropinirole • Anticholinergics: Benzhexol Orphenadrine • Antioxidants: Vitamins C and E

165

Mastering MRCP BOF: 2.103 A 45-year-old Asian male presents with a history of progressive breathlessness. His chest x-ray shows curvilinear calcification of the right side of the heart. In this patient which of the following physical signs are you likely to elicit: a) 4th heart sound b) Pericardial crunch c) Pericardial rub d) Tumour plop e) Diastolic knock Answer: e) Curvilinear calcification of the right side of the heart would suggest pericardial calcification. 50-75% of patients with pericardial calcification have constrictive pericarditis. In constrictive pericarditis sudden arrest of ventricular filling occurs in diastole and this gives rise to a diastolic knock

BOF: 2.104 A 24-year-old male presents with a history of double vision, and difficulty in walking. He gives a history of a diarrheal illness a few weeks prior to the onset of these symptoms. He has been fit and well otherwise, he does not take any medication and does not drink nor does he smoke. On examination he is found to have ophthalmoplegia, ataxia and areflexia but there is no limb weakness. In this patient the most likely way to reach a diagnosis would be by: a) MR of the posterior fossa b) MR angiogram c) CT scan d) Give intravenous thiamine and assess response e) Lumbar puncture Answer: e) The patient has the Miller Fisher variant of Gullain Barre Syndrome. This consists of ophthalmoplegia, ataxia, areflexia with little or no limb weakness. Lumbar puncture would reveal high protein levels in the CSF The other tests would not reveal any abnormalities in this condition. There is no history of alcoholism and he has been well until the acute onset of his illness and this makes Wernicke’s encephalopathy unlikely

BOF: 2.105 A 66-year-old female presents with a history of a lump in the neck that is gradually increasing in size. She is on thyroxine replacement, as she is known to have Hashimoto’s thyroiditis. On examination she is found to have a nodule in her thyroid gland. Fine needle aspiration performed and the results would suggest: a) Lymphoma b) Papillary carcinoma c) Medullary carcinoma d) Follicular carcinoma

166

Mastering MRCP e) Anaplastic carcinoma Answer: a) There is an increased incidence of lymphoma of the thyroid in patients with Hashimoto’s thyroiditis

BOF: 2.106 A 50-year-old female presents with a history of diarrhoea. The diarrhoea is associated with bloating and passage of a lot of wind. She has a long-standing history of Raynaud’s phenomenon. On examination you note that she has a beak like nose, a small mouth with perioral puckering of the skin (pseudorhagades). On examination of her hands you note sclerodactyly and nail fold capillary dilatation. In this patient which of the following investigations is likely to reveal the diagnosis? a) Distal duodenal biopsy b) Small bowel follow through c) Colonoscopy d) Enteroscopy e) Glycocholic acid breath test Answer: e) The clinical features suggest the patient has scleroderma. In scleroderma patients develop small bowel bacterial overgrowth and this can cause diarrhoea. The glycocholic acid breath test will demonstrate small bowel bacterial overgrowth.

BOF: 2.107 A 20-year-old male has spent his gap year in Sri Lanka. He presents with a history of breathlessness and feeling tired. He is found to have iron deficiency anaemia. Stools are sent for microscopy and ova of Ancylostoma duodenale are demonstrated. Which of the following drugs would you prescribe? a) Albendazole b) Diethylcarbamazine c) Praziquantel d) Diloxanide furoate e) Emetine Answer: a) Hookworms (Ancylostoma duodenale and Necator americanus) can cause iron deficiency anaemia and is probably the commonest cause of iron deficiency worldwide. Treatment is with an anthleminthic such as albendazole or mebendazoleBOF: 2.108 A 22 year old who is known to have bronchial asthma but who does not comply with treatment is admitted with acute severe asthma. Which of the following would be an indication for intubation and ventilation? a) Respiratory rate > 25 /min b) Pulse rate >110 beats per minute c) Peak expiratory flow rate < 50% of predicted or best d) Hypoxia e) Hypercapnoea Answer: e)

167

Mastering MRCP Indications for intubation and ventilation in life threatening asthma are as follows: · Tired, drowsy, confused · Hypercapnoea · Hypoxic despite optimal therapy · Comatose · Respiratory arrest

BOF: 2.109 A 72-year-old male is admitted to the coronary care unit with a transmural anterior myocardial infarction. One day after admission he has a cardiac arrest and resuscitation is unsuccessful. A post-mortem examination is performed. Which of the following histopathological processes are likely to be seen in this patient? a) Death of fibers and intercellular oedema b) Coagulative necrosis of myocytes, neutrophil infiltration of necrotic tissue c) Dead myocytes begin to disintegrate, resorbed by macrophages and enzymatic proteolysis d) Granulation tissue forms, necrotic tissue replaced by a fibrous scar e) No change Answer: b) The histopathological changes that occur after myocardial infarction are as follows: Ñ Within 1 hour Death of fibers and intercellular oedema Ñ 12 to 72 hours: Coagulative necrosis of myocytes Neutrophil infiltration of necrotic tissue Ñ 3 to 7 days Dead myocytes begin to disintegrate Resorbed by macrophages and enzymatic proteolysis Ñ 7-10 days Granulation tissue forms Necrotic tissue replaced by a fibrous scar

BOF: 2.110 Which of the following hormones produced by the gut stimulate appetite? a) Ghrelin b) Glucagon like peptide-1 c) Oxyntomodulin d) Peptide YY (PYY) e) Cholecystokinin Answer: a) Gut hormones, that regulate appetite, function as follows: Stimulate appetite:

168

Mastering MRCP Ghrelin (is a hormone produced mainly by P/D1 cells lining the fundus of the human stomach and epsilon cells of the pancreas that stimulates hunger ,Its levels rose sharply shortly before and fell shortly after every meal. Inhibit, appetite Glucagon like peptide-1 Oxyntomodulin Peptide YY (PYY) Cholecystokinin Pancreatic polypeptide

BOF: 2.111 A 68-year-old female presents with a history of problems with vision. On confrontation perimetry she is noted to have a left homonymous hemianopia. She has a formal visual field test performed and this shows that the hemianopia is incongruous. Where is the lesion likely to be? a) Right optic nerve b) Right optic tract c) Right lateral geniculate body d) Right optic radiation e) Right occipital lobe Answer: b) A congruous hemianopia means the field defect in each eye is the same in size and shape In an incongruous hemianopia the field defects in the two eyes are different in size and shape The visual pathways become more organized as they proceed from the optic nerve backwards Thus lesions that are more posterior demonstrate a greater degree of congruity An incongruous hemianopia indicates an anterior lesion Lesions in the optic nerve will cause unilateral blindness

BOF: 2.112 Which one of the following may be detected using the technique of Northern Blotting? a) DNA b) RNA c) Immunoglobulins d) Human leukocyte antigens e) Enzymes Answer: b) Northern blotting is used to detect RNA

BOF: 2.113 A 60-year-old female presents with a history of episodic double vision, blurring of vision and palpitations. These symptoms are relieved by taking sugary drinks. There is a history of weight gain

169

Mastering MRCP Which of the following investigations would you arrange in this patient? a) EEG b) 24 hour ECG monitoring c) 72 hour fast d) Oral glucose tolerance test e) 24 hour urinary catecholamines Answer: c) The history of episodic symptoms relieved by glucose would point to the diagnosis of insulinoma The history of weight gain too is in favor of this diagnosis The diagnosis is made on the basis of hypoglycemia in the face of inappropriately elevated levels of insulin Prolonged fasting (up to 72 h) with associated: · Serum insulin levels elevated · Glucose levels low · C-peptide levels elevated · Proinsulin levels elevated · Screening for sulphonylureas negative

BOF: 2.114 Which of the following is an adverse prognostic marker in relation to three-year survival in patients with small cell lung cancer? a) Clubbing of fingers b) Proximity of lesion to hilum c) Elevated blood urea d) Elevated serum lactate dehydrogenase e) High ESR Answer: d) The factors that are associated with adverse prognosis in relation to three-year survival in patients with small cell lung cancer are: · · · ·

Poor performance status Extensive disease Elevated serum lactate dehydrogenase Elevated alkaline phosphatase

BOF: 2.115 A 30-year-old male presents with a history of cough and flu like symptoms. He has been losing weight for several months and his GP has recently treated him with nystatin for oral candidiasis. His chest X-ray shows a cavitating lesion in the right apex. The likely cause of the lesion is: a) Amoebiasis b) Pneumocystis carinii pneumonia c) Legionella pneumophilia d) Histoplasmosis

170

Mastering MRCP e) Brucellosis Answer: d) The history suggests the patient is immunocompromised (weight loss, candidiasis) Histoplasmosis is caused by the fungus Histoplasma capsulatum. This disease primarily affects the lungs although disseminated infection may occur. It is common in immunocompromised hosts.

BOF: 2.116 A 58-year-old male presents with a history of falls and difficulty in performing common tasks like opening doors. He also gives a history of choking when he eats. The history is of several months duration and he feels the problems are steadily getting worse There is no other significant history, no family history of note and he is not on any regular medication. On examination you note weakness of the muscles of his shoulder and pelvic girdle and distal weakness, especially affecting the finger flexors. There is asymmetric wasting of the muscles. Tendon reflexes are preserved. There is no sensory deficit. In this patient the most likely diagnosis is: a) Polymyositis b) Dermatomyositis c) Inclusion body myositis d) Amyotrophic lateral sclerosis e) Syringomyelia Answer: c) The patient gives a history of slowly progressive symptoms. Polymyositis and dermatomyositis are usually more rapidly progressive. They also affect the proximal muscles and distal weakness is not a feature. Reflexes would be exaggerated in amyotrophic lateral sclerosis Syringomyelia would cause sensory features and the weakness usually affects the small muscles of the hand. Skeletal muscle inflammatory myopathies are: · Polymyositis · Dermatomyositis · Inclusion body myositis The first two are usually more rapidly progressive and mainly affect the proximal muscles. Distal weakness especially weakness of the finger flexors is a feature of inclusion body myositis. In addition dysphagia and choking when eating are also features of inclusion body myositis. Wasting is a feature of inclusion body myositis and it is asymmetric. Muscle biopsy is necessary to make the diagnosis of inclusion body myositis Treatment is supportive

BOF: 2.117 Which of the following is a feature of Pendred’s syndrome? a) Anosmia b) Cataract

171

Mastering MRCP c) Sensorineural deafness d) Retinitis pigmentosa e) Albinism Answer: c) Pendred’s syndrome is an autosomal recessive condition It is characterised by goiter and congenital sensorineural deafness Most patients are euthyroid but they have impaired organification of iodine (incorporation into thyroxine)

BOF: 2.118 On examination of a patient’s eyes, small pupils may be seen in: a) 3rd nerve palsy b) Holmes-Adie pupil c) Cocaine use d) Amblyopia e) Pontine haemorrhage Answer: e) The causes of a constricted pupil are: • Iritis • Horner’s syndrome • Argyll-Robertson syndrome • Pontine lesions • Opiate use • Organophosphate poisoning BOF: 2.119 Which of the following drugs is used in the treatment of the long QT syndrome? a) Atenolol b) Digoxin c) Amiodarone d) Flecainide e) Quinidine Answer: a) Beta-blockers are the drugs of choice for patients with long QT syndrome · Adrenergic blockade reduces the risk of cardiac arrhythmias · Beta-blockers may also reduce the QT interval in some patients Implanted cardioverter-defibrillators are the most effective therapy for high-risk patients who are defined as those with: · Aborted cardiac arrest Or · Recurrent cardiac events (syncope or Torsades de pointes) Despite use of a beta-blocker alone

172

Mastering MRCP BOF: 2.120 In left atrial myxoma auscultation of the heart may reveal an added sound in: a) Mid-diastole b) Late diastole c) Early diastole d) Ejection systole e) Mid-systole Answer: c) In left atrial myxoma the pedunculated tumour may prolapse through the mitral valve as it opens in diastole resulting in a sound similar to the opening snap of mitral stenosis

BOF: 2.121 Which of the following congenital heart defects is the most common in Down’s syndrome? a) Endocardial cushion defect b) Ventricular septal defect c) Septum secundum atrial septal defect d) Tetralogy of Fallot e) Patent ductus arteriosus Answer: a) Congenital heart defects are common in patients with Down’s syndrome They are more likely to be seen in patients who are hospitalised In decreasing order of frequency the defects that are likely to occur are: · Endocardial cushion defect · Ventricular septal defect · Septum secundum atrial septal defect · Tetralogy of Fallot · Patent ductus arteriosus

BOF: 2.122 An 18-year-old male who is known to have sickle cell disease presents with a history of breathlessness on exertion. On examination he looks pale Investigations reveal: Hb 4.7 g/dL Reticulocyte count 6 (normal 25-85x 109/L) What is the likely cause of the anaemia? a) HIV infection b) HSV infection c) Epstein-Barr virus infection d) Hepatitis C infection e) Parvovirus infection Answer: e) Severe anaemia in a patient who is known to have sickle cell disease could be due to:

173

Mastering MRCP · Acute sequestration · Haemolysis · Marrow aplasia The age of the patient would be against acute sequestration as this usually occurs in childhood before splenic infarctions occur The low reticulocyte count indicates marrow aplasia and this is usually due to viral infections, especially infections with parvovirus

BOF: 2.123 A 24-year-old female medical student who is on an elective in Sri Lanka, asks her consultant for advice regarding a rash she has developed. The rash is not itchy. On examination she has a marked suntan and there are several hypopigmented, scaly macules on her upper arms and upper trunk. What is the likely diagnosis? a) Psoriasis b) Tuberculoid leprosy c) Lepromatous leprosy d) Pityriasis versicolor e) Pityriasis rosea Answer: d) Pityriasis versicolor (Tinea versicolor) is caused by Malassezia furfur (Pityrosporon orbiculare, Pityrosporon ovale, Malassezia ovalis) It occurs more commonly in warmer environments Psoriasis would cause plaques Leprosy does not occur acutely Pityriasis rose is itchy

BOF: 2.124 A 16 year old male who has been treated for acute lymphoblastic leukemia and who is in remission presents with an insidious onset of pain in the left hip. On examination hip movements are restricted to a minimal extent but there are no other abnormalities on physical examination. Which of the following tests is most likely to confirm your diagnosis? a) Plain x-ray of the hip b) Bone scan c) MRI of the hip d) Serum uric acid e) Bone marrow aspiration Answer: C) In this patient the likely diagnosis is avascular necrosis of bone as a consequence of treatment for leukemia. He is in remission so it is unlikely that he has gout

174

Mastering MRCP Plain x- ray and bone scan may also be used to investigate avascular necrosis of bone but the changes are delayed and less specific MRI is sensitive, specific and may be used in the early stages

BOF: 2.125 A 42-year-old male presents with a history of involuntary movements of his upper limbs. The tremor resolves during sleep but is aggravated by certain positions and is decreased if he takes alcohol On examination you note a positional tremor tone power and reflexes are normal. Which one or the following features would rule out Parkinsonism? a) Tremor decreasing during sleep b) Family history c) Asymmetry d) Tremor affecting head e) Normal tone Answer: e) The cardinal features of Parkinsonism are tremor rigidity and bradykinesia Essential tremor is s syndrome that is characterised by slowly progressive postural and or kinetic tremor it usually affects the upper extremities. Tone and reflexes are normal in this condition. Essential tremor is familial in 50-70 % of patients, transmission being autosomal dominant with incomplete penetrance

BOF: 2.126 A 22-year-old male university student presents to casualty following an alcoholic binge. No previous medical history. On examination he is in atrial fibrillation but there are no other abnormalities detected. ECG confirms fast atrial fibrillation. No specific treatment is given. He is transferred to a short stay ward where he sobers up repeat ECG shows that he has reverted to sinus rhythm. Which one of the following would be useful in this patient? a) Warfarin b) Amiodarone c) Sotalol d) Digoxin e) Lifestyle adjustments Answer: e) Alcohol excess can cause atrial fibrillation and in this young patient with no underlying heart disease this is the most likely cause of atrial fibrillation. Avoiding alcohol abuse is the most useful intervention in this patient

BOF: 2.127 A 27-year-old male presents with a history of backache of several month duration. The pain is slowly progressive and he has also noted loss of weight, he feels generally unwell and has night sweats On examination he looks unwell, appears to have lost weight and there is tenderness over his thoracic spine He has normocytic normochromic anaemia and his ESR is raised 80 mm in the 1st hour Which of the following tests would most likely confirm your diagnosis?

175

Mastering MRCP a) Spinal x-ray b) White cell scan c) Bone scan d) MRI of the spine e) Mantoux test Answer: d) The history of backache, tenderness over the spine and systemic symptoms would suggest the patient has vertebral osteomyelitis, which would be best confirmed by MRI of the spine the other tests would be nonspecific With the history of weight loss and night sweats, tuberculosis’s would be likely but the Mantoux test would not be diagnostic

BOF: 2.128 A 65-year-old male presents with a history of difficult climbing stairs and combing his hair of a few months duration. He has no major medical problems, has not been on any medication. He smokes 20 cigarettes a day. He has recently retired, he was a taxi driver. On examination he is found to have no wasting, normal tone, weakness of the muscles of his shoulder and pelvic girdle reflexes are normal Chest x-ray reveals a shadow in his right lower zone In this patient what is the likely diagnosis? a) Polymyositis b) Lambert-Eaton syndrome c) Myasthenia gravis d) Pancoast syndrome e) Polymyalgia rheumatica Answer: b) Lambert Eaton myasthenic myopathic syndrome is a rare, non-metastatic manifestation of small cell lung cancer caused by defective release of acetylcholine at the neuromuscular junction

BOF: 2.129 A 50-year-old male is referred by his GP for investigation of palpitations. He is found to have episodic hypertension and further investigation reveals a left sided adrenal phaeochromocytoma. In this patient with which of the following drugs would you initiate treatment? a) Phenoxybenzamine b) Propranolol c) Atenolol d) Verapamil e) Reserpine Answer: a) Treatment of phaeochromocytoma is with alpha blockade initially followed by beta blockade

176

Mastering MRCP BOF: 2.130 A 35-year-old female presents with pain and restriction of movement of her right shoulder of several months duration. This is interfering with her life as she finds it difficult to get dressed and brush her hair. She is known to have insulin dependent diabetes mellitus and is a regular attendee at the diabetic clinic. On examination she looks generally well. On examination of her right shoulder there is restriction of both active and passive movements. All other joints appear normal and there is no restriction of movement of these joints. In this patient what is the most likely diagnosis? a) Adhesive capsulitis b) Rheumatoid arthritis c) Osteoarthritis d) Haemachromatosis e) Septic arthritis Answer: a) Adhesive capsulitis or frozen shoulder is a condition that is often associated with diabetes mellitus. Osteoarthritis rarely affects the shoulder joint primarily, the patient is quite young. Rheumatoid arthritis usually affects the small joints first. In a female haemochromatosis will present later in life as menstruation protects against iron overload. In addition classically haemochromatosis affects the 2nd and 3rd metacarpophalangeal joints. In septic arthritis the patient will be septic with fever and feeing unwell.

BOF: 2.131 A 40-year-old female presents with a history of heartburn. There is a long-standing history of Raynaud’s phenomenon. On examination she is noted to have a small mouth, telangiectasia over her face, dilated nail fold capillaries. In this condition which of the following complications may occur? a) Pulmonary hypertension b) Erosive arthropathy c) Spondyloarthropathy d) Myositis e) Amyloidosis Answer: a) The clinical features suggest the patient has limited cutaneous scleroderma (formerly CREST) The major complication of this condition is development of pulmonary hypertension

BOF: 2.132 A 72-year-old female presents with a history of severe left temporal headache. She gives a history of feeling generally unwell and also complaints of problems with eating. You arrange investigation of this patient Which one of the following drugs would you use in this patient whilst awaiting confirmation of the diagnosis? a) Steroids b) Antiviral c) Anticonvulsants

177

Mastering MRCP d) Sumatriptan e) NSAIDs Answer: a) In an elderly patient with a history of headache, systemic features and the possibility of jaw claudication one should consider temporal arteritis as the most likely diagnosis. In this situation one would use steroids in order to prevent blindness, which is the most serious complication of this condition

BOF: 2.133 A 37-year-old male presents with a history of pain and swelling of his knees. He also has a low backache.. He also complains of pain and redness of his eyes and he has a discharge per urethra. 2 weeks prior to the onset of the illness he had diarrhoea for several days. This was self-limiting. He is not on any medications, smoke 5 cigarettes a day and takes 12 units of alcohol per week. He is married has 2 children and is an accountant. On examination of his knees there are bilateral effusions and limitation of movement. Aspiration of the joint shows that the effusion is sterile. X-ray of his pelvis shows sacroiliitis. What is the likely diagnosis? a) Gonococcal arthritis b) Enteropathic arthropathy c) Rheumatoid arthritis d) Ankylosing spondylitis e) Reactive arthritis Answer: e) This patient has presented with urethritis, conjunctivitis and arthritis following a diarrheal illness. The most likely diagnosis is reactive arthritis

BOF: 2.134 A 57-year-old male presents to the clinic for investigation of hypertension. He is an immigrant and as such there are no medical records. He appears to be deaf in one ear and you note several scattered skin nodules. What is the likely diagnosis? a) Alport’s syndrome b) Sturge-Weber syndrome c) Tuberous sclerosis d) Neurofibromatosis e) Paget’s disease of bone Answer: d) With the clinical features of deafness and skin nodules one should consider neurofibromatosis as the diagnosis (type 2)

BOF: 2.135 A 20-year-old female presents to the clinic. She is concerned about some disfiguring lumps that have begun to appear on her neck. There is a family history of ischaemic heart disease at an early age. On examination you note small yellow papules a few of which appear to have coalesced to from plaques.

178

Mastering MRCP What is the likely diagnosis? a) Pseudoxanthoma elasticum b) Ehlers Danlos syndrome c) Tangier disease d) Remnant hyperlipidaemia e) Heterozygous familial hypercholesterolemia Answer: a) The description of the skin lesions will fit in with Pseudoxanthoma elasticum Hyperlipidaemia may also present with early ischaemic heart disease and skin lesions

BOF: 2.136 A 71-year-old male is admitted with a history of progressive confusion, difficulty in walking and incontinence of urine. On examination he is found to be demented, he has gait apraxia, but there is no abnormality of tone, power or reflexes, coordination is normal there is no sensory abnormality. A CT scan shows dilated ventricles but no obstruction to CSF flow. Lumbar puncture reveals normal pressure with normal microscopy and biochemistry. Routine haematology and biochemistry are normal. In this patient which of the following would help in deciding further management? a) MRI b) Treponema pallidum haemagglutination titer c) Chest X-ray d) Glucose tolerance test e) Reassessment after lumbar puncture and drainage of CSF Answer: e) The triad of dementia, gait apraxia and urinary incontinence would suggest normal pressure hydrocephalus, which was confirmed by CT scan and lumbar puncture. If the patient improves after lumbar puncture and drainage of CSF, a ventriculo-jugular shunt may be considered

BOF: 2.137 A 22 year old female presents with fever in the 2nd trimester of her 1st pregnancy she gives history of dysuria and loin pain. On examination she is febrile and has bilateral tenderness in the loins. She gives a history of recurrent urinary tract infections in childhood. There is a family history of hypertension and she has been told that her mother has kidney disease. Urine examination shows pyuria and culture of urine demonstrates a significant growth of E. coli. In this patient which of the following conditions could be the underlying cause of her presentation? a) Reflux nephropathy b) Chronic glomerulonephritis c) Adult polycystic kidney disease d) Utero-vaginal prolapse e) Urolithiasis Answer: a) Vesico-ureteric reflux predisposes to recurrent urinary tract infections. This leads to renal scarring and may progress to renal failure

179

Mastering MRCP BOF: 2.138 A 66-year-old male presents with a history of loin pain and passage of blood in his urine. He says he has been feeling unwell for some time with weight loss, fever and night sweats. On examination apart from an elevated blood pressure you cannot find any clinical signs. Investigations reveal an elevated ESR and hypercalcaemia. In this patient what is the most likely diagnosis: a) Chronic pyelonephritis b) Renal tuberculosis c) Renal cell carcinoma d) Prostatic carcinoma e) Multiple myeloma Answer: c) Renal cell carcinoma commonly presents with loin pain, haematuria and a palpable mass in the abdomen. Systemic symptoms are common with weight loss, fever, hypertension, hypercalcaemia, night sweats and malaise Varicocele, usually left sided, may occur due to obstruction of the testicular vein Renal cell carcinoma is frequently associated with paraneoplastic syndromes, including hypercalcaemia, erythrocytosis, hepatic dysfunction (Stauffer syndrome). Polyneuromyopathy, amyloidosis, anaemia, fever, cachexia, weight loss, dermatomyositis, increased erythrocyte sedimentation rate, and hypertension may also occur Cytokine release by the tumour (IL-6, erythropoietin, nitric oxide) is the mechanism by which these paraneoplastic effects occur Successful treatment of the primary metastatases may result in resolution of symptoms or biochemical abnormalities

BOF: 2.139 A middle-aged homeless male is brought into casualty having being found unconscious in the street. On examination he is unkempt and is very drowsy and is just rousable to painful stimuli. He is noted to have papilloedema but apart from this there is no gross neurological deficit, he is afebrile, there is no rash and no abnormality of his other systems. Investigations reveal normal full blood count, urea and electrolytes and liver function tests. Blood glucose 6.2 Blood gases show pH 7.20, pCO2 3.3 kPa, pO2 12 kPa, and bicarbonate 11 mmol/L In this patient which of the following would lead to a diagnosis: a) CT scan of head followed by lumbar puncture b) Serum methanol level c) Trial of intravenous thiamine d) CT scan of abdomen e) Paracetamol level in blood Answer: b) When faced with a situation where no history is available but the patient is a homeless, individual who looks as if he is neglecting himself, the possibility of poisoning should come to mind.

180

Mastering MRCP Severe metabolic acidosis is one of the consequences of methanol poisoning and the clinical features of optic disc oedema are in favor of this diagnosis. Hence serum methanol level is the best way of defining the diagnosis although this test is not freely available in many hospitals. Methanol poisoning commonly follows ingestion of contaminated alcohol beverages or 'methylated spirits'. Methanol is metabolized by alcohol dehydrogenase to formaldehyde, which is itself rapidly oxidized to the toxin, formic acid. The specific antidote is ethanol given intravenously

BOF: 2.140 In which one of the following conditions could sacroiliitis occur? a) Rheumatoid arthritis b) Gout c) Ulcerative colitis d) Behcet’s disease e) Haemachromatosis Answer: c) Ulcerative colitis may be complicated by sacroiliitis, ankylosing spondylitis and peripheral arthropathy The other conditions usually cause peripheral arthropathy

BOF: 2.141 A 27-year-old male attends the gastroenterology clinic as he has a presumed diagnosis of irritable bowel syndrome. There are no abnormalities on clinical examination of the patient. Investigations reveal serum bilirubin 35 micromoles per liter. His full blood count and the remainder of his liver function tests are within normal limits. This patient is most likely to have: a) Hepatitis C b) Haemachromatosis c) Alcoholic cirrhosis d) Autoimmune hepatitis e) Gilbert’s syndrome Answer e) Isolated mild hyperbilirubinaemia in an otherwise fit and healthy individual should make one suspect Gilbert’s syndrome. Gilbert’s syndrome: · Autosomal recessive condition · Characterised by intermittent jaundice · Absence of haemolysis · Absence of underlying liver disease · May be precipitated by Dehydration Fasting Menstrual periods Stress

181

Mastering MRCP BOF: 2.142 An obese 19-year-old female presents with a history of swelling of her abdomen and feet. She has a blood pressure of 140/90, is noted to have proteinuria and her haemoglobin level is 10.6 g /dL. In this patient which one of the following is likely to lead to a diagnosis? a) Plasma protein electrophoresis b) 24 urinary protein estimation c) Renal biopsy d) Urinary beta human chorionic gonadotrophin e) Abdominal paracentesis Answer: d) In any female in the childbearing age, swelling of the abdomen and ankles should make one think of pregnancy. Anaemia is in keeping with this and the proteinuria and hypertension should make one suspect pre-ecclamptic toxaemia

BOF: 2.143 An 80-year-old male presents in a state of confusion. His carriers say he has been complaining of backache and he had been constipated for some time. He has also been increasingly thirsty. On examination he is confused and disoriented. He appears dehydrated. Investigations reveal Hb 8.0 g/dL ESR 90 mm in 1st hour Urea 12.7 mmol/L Creatinine 130 mmol/L Blood glucose 6.0 mmol/L Chest X-ray shows multiple rib fractures Spinal X-ray shows wedge fractures In this patient which one of the following investigations is essential in initial management of this patient? a) Plasma protein electrophoreses b) Bone marrow examination c) Urine for Bence-Jones protein d) Renal ultrasound e) Serum calcium Answer: e) In an elderly patient with backache, rib fractures, vertebral fractures anaemia and a high ESR, myeloma is the likely diagnosis. The patient has presented with dehydration, constipation and increased thirst. This is likely to be due to hypercalcaemia and the first step in management would be to correct the dehydration and hypercalcaemia. Hence serum calcium would be the most important first investigation.

BOF: 2.144 A 20 year old female presents with a history of perioral tingling and tingling of her arms and feet and muscle cramps. No other symptoms. No previous illnesses of note.

182

Mastering MRCP Investigations reveal Corrected calcium 1.74 mmol/L Urea 80 mmol/L Creatinine 90 mmol/L In this patient which one of the following investigations would you arrange next? a) Chest x-ray b) Parathyroid hormone concentration c) Vitamin D assay d) Coeliac screen e) Urinary calcium concentration Answer: b) When investigating a patient with hypocalcaemia, the most important step is to know whether hypocalcaemia is associated with low PTH (parathyroid disease) or with a high PTH level (parathyroid resistance, vitamin D deficiency and resistance). Hence this would be your first investigation in this patient

BOF: 2.145 A 30-year-old female who is known to have multiple sclerosis is found to have abnormal eye movements. On lateral gaze (bilaterally) she is found to have nystagmus of the abducting eye and failure of adduction of the other eye. This abnormality is caused by a lesion in the: a) Superior colliculus b) Inferior colliculus c) Frontal eye fields d) Red nucleus e) Median longitudinal fasiculus Answer: e) The patient has bilateral internuclear ophthalmoplegia, which is caused by a lesion in the median longitudinal fasiculus

BOF: 2.146 A 27-year-old male who recently returned from holiday in Spain presents with a history of bloody diarrhoea, cramping abdominal pain, loss of weight and a painful swelling of his right knee. What is the likely diagnosis? a) Giardiasis b) Whipple’s disease c) Ulcerative colitis d) Crohn’s disease e) Campylobacter infection Answer: e) An acute onset after a holiday would be more in keeping with an infective cause rather than idiopathic inflammatory bowel disease. Campylobacter infection is a cause of acute diarrhoea and is also one of the precipitating causes of reactive arthritis

183

Mastering MRCP Whipple’s would follow a more chronic course

BOF: 2.147 A 74-year-old male presents with a pain in his right eye. He also complains of nausea and vomiting. The pain is of a few hours duration. On examination the eye looks red, is hard on palpation and the pupil is dilated. Which of the following is the likely diagnosis? a) Conjunctivitis b) Acute angle closure glaucoma c) Iritis d) Keratitis e) Papillitis Answer: b) In angle closure glaucoma, intra-ocular pressure increases as a result of an impairment of aqueous outflow Contact of the peripheral iris with the trabecular meshwork at the entrance to Schlemm's canal obstructs aqueous outflow Angle closure glaucoma presents acutely as a painful red eye and is an ophthalmological emergency as delay in treatment may result in blindness

BOF: 2.148 A 65 year old male who has uncontrolled hypertension presents with backache which is followed by weakness of his lower limbs. On examination he is found to have a flaccid paraplegia with a sensory level at T10 with sparing of the dorsal columns. In this patient which of the following investigations is likely to lead to a diagnosis? a) Vitamin B12 levels b) CT scan of the chest c) Lumbar puncture d) Myelogram e) Nerve conduction studies Answer: b) Dissecting aneurysm of the descending thoracic aorta can cause interference with the blood supply to the anterior spinal artery and cause the infarction of the anterior aspect of the spinal cord (anterior spinal artery syndrome) In a patient with uncontrolled hypertension and backache this is the most likely diagnosis and would be revealed by CT scanning of the chest

BOF: 2.149 A 65-year-old male lifelong smoker presents with haemoptysis. Investigations reveal a diagnosis of squamous carcinoma of the bronchus. In this patient which one of the following would be a contraindication to surgery? a) Haemoptysis b) Productive cough

184

Mastering MRCP c) Hypertrophic pulmonary osteoarthropathy d) Hypercalcaemia e) Superior vena cava obstruction Answer: e) Superior vena cava obstruction will indicate compression or invasion of the superior vena cava and this would make the tumour unresectable. Treatment of superior vena cava obstruction · Chemotherapy · Radiotherapy · Stent insertion · Steroids (effectiveness has been questioned)

BOF: 2.150 A 36-year-old male presents with a history of severe, agonising pain on the left side of his face. The pain comes on suddenly and lasts for a very short while. The pain is sometimes related to washing his face or shaving. This is most likely to be: a) Temperomandibular joint syndrome b) Migraine c) Sinusitis d) Trigeminal neuralgia e) Impacted molar Answer: e) Trigeminal neuralgia (tic douloureux) · Pain syndrome recognised by history alone · Pain often accompanied by a brief facial spasm or tic · Unilateral · Sensory distribution of cranial nerve V (typically maxillary mandibular area) Treatment · Carbamazepine · Baclofen

BOF: 2.151 A 40-year-old male has long standing backache. The x-ray of his spine shows ankylosis. As you speak to the patient you notice that his ears look pigmented. What is the likely diagnosis? a) Homocystinuria b) Ankylosing spondylitis c) Alkaptonuria d) Phenylketonuria e) Haemachromatosis Answer: c)

185

Mastering MRCP Alkaptonuria (ochronosis) is an autosomal recessive disease, which results in a defect in the catabolic pathway of tyrosine with accumulation of homogentisic acid, which deposits in cartilage and other tissues

BOF: 2.152 A 53-year-old female presents with a rash over the anterior aspect of both her lower limbs, below the knee. On examination you note that in addition to the rash she looks underweight, her pulse rate is 110 beats per minute and on examination of her eyes you note lid retraction and lid lag. In this patient the rash is most likely to be: a) Erythema nodosum b) Pretibial myxoedema c) Pyoderma gangrenosum d) Necobiosis lipoidica diabeticorum e) Acanthosis nigricans Answer: b) Pretibial myxoedema presents as a shiny, red non-tender infiltration on the front of the shins. In the chronic form it may look like localised elephantiasis. This is due to the deposition of hyaluronic acid in the dermis and subcutis usually as a component of thyroid disease Local corticosteroid is the mainstay of treatment

BOF: 2.153 A 65-year-old male is admitted for investigation of palpitations. Biochemical investigations show that he has hypomagnesaemia Which of the following is likely to be the cause of his hypomagnesaemia? a) Hypothyroidism b) Chronic renal failure c) Diuretic therapy d) Antacid therapy e) Elevated phosphate levels Answer: c) Diuretic therapy causes increased excretion of magnesium and may result in hypomagnesaemia Other causes of hypomagnesaemia GIT · Prolonged nasogastric suction · Chronic vomiting · Malabsorption syndromes · GI fistulae · Short-bowel syndrome · Chronic diarrhoea · Pancreatitis · Refeeding syndrome Toxins · Alcoholism

186

Mastering MRCP E&M · Hypokalaemia · Hypercalcaemia · Hyperparathyroidism · Hyperthyroidism · Conn's · Diabetes ketoacidosis KUS · Renal tubular acidosis · ATN diuretic phase RAG · Lactation Drugs · Diuretics · Aminoglycosides · Carbenicillin · Ticarcillin · Amphotericin B · Cisplatin · Cyclosporin

BOF: 2.154 A 26-year-old male presents with a history of fever and a rash. He has recently returned from a trip to Thailand. This has been his 3rd trip there over the last 2 years. On examination the patient has a widespread petechiae, his blood pressure is 80/60. Investigations reveal: Hb 19.0g /dL WBC 3.5x109/L Platelets 45 x 109/L In this patient which of the following would you administer immediately? a) Acyclovir b) Ribavirin c) Metronidazole d) Steroids e) Saline Answer: e) The history of fever and rash after a return from Thailand should make one consider an infective aetiology for the condition. The findings of petechial haemorrhages with low blood pressure with history of fever should make one think of a haemorrhagic fever. The findings of haemoconcentration, low white count and low platelets would make dengue fever the most likely diagnosis Treatment is with large volumes of crystalloid and colloidal solutions intravenously with platelet infusions and fresh frozen plasma

187

Mastering MRCP BOF: 2.155 A 24-year-old female is admitted 15 hours after ingesting 30 grams of paracetamol. On examination she looks pale, sweaty, has a pulse rate of 110 beats per minute and blood pressure 90/60 In this patient your next step in management would be to: a) Administer activated charcoal orally b) Start up a 10 % dextrose drip c) Start N-acetylcysteine intravenously d) Perform gastric lavage e) Give vitamin K intravenously Answer: c) Gastric lavage is not considered necessary if paracetamol is the only substance ingested, in any case at 12 hours it is too late Activated charcoal is used within 4 hours of ingestion (some say 1 hour) Vitamin K is not given, as the prothrombin time is very important in monitoring progress in paracetamol overdose After 15 hours N-acetylcysteine should be commenced without awaiting paracetamol levels

BOF: 2.156 Which of the following is most likely to show up on chest X-ray as a cavitating lesion? a) Squamous carcinoma b) Large cell carcinoma c) Adenocarcinoma d) Alveolar cell carcinoma e) Small cell carcinoma Answer: a) Of the lung cancers, squamous carcinoma is the most likely to occur as a cavitating lesion

BOF: 2.157 A 40-year-old male who is otherwise fit and healthy presents with pain and swelling of his right index finger. He says he pricked the finger on a bougainvillea thorn in his garden a few days previously. On examination the finger is swollen, red, warm and tender. In this patient which of the following organisms is likely to be causing this condition? a) Streptococcus pyogenes b) Escherichia coli c) Klebsiella d) Pseudomonas aeruginosa e) Enterobacter Answer: a) This patient has community acquired cellulitis and has no risk factors (diabetes mellitus, immunosuppressed, ulcerated lesions) The common organisms that cause infection in this situation are: group A streptococci, staphylococcus aureus, streptococcus pneumoniae

188

Mastering MRCP BOF: 2.158 A 71-year old male was admitted to hospital with oliguria Investigations revealed hyponatraemia, low serum osmolality and concentrated urine Which of the following could account for this? a) Lithium b) Prednisolone c) Paroxetine d) Carbenoxolone e) Bendroflumethiazide Answer: c) The patient has SIADH (syndrome of inappropriate ADH secretion) SSRIs are known to cause SIADH

BOF: 2.159 A 24-year-old male prisoner is brought in to the outpatient department with a history of itching, which is increased at night. On examination you note papules, vesicles and a few burrows mainly in the webs of his fingers but also in his groins. In this patient which of the following would you prescribe ? a) Chlorpheniramine b) Calamine lotion c) Cream E45 d) Benzyl benzoate e) Erythromycin Answer: d) The clinical features would suggest the patient has scabies. The burrows are characteristic of this condition. Treatment is with a topical scabicide such as Permethrin, Malathion, Benzyl benzoate, Lindane Oral ivermectin is used for Norwegian scabies

BOF: 2.160 Which of the following is a risk factor for developing prostate cancer? a) Asian ethnicity b) High alcohol intake c) Smoking d) Strong family history of breast cancer e) Sedentary occupation Answer: d) The following are risk factors for development of prostate cancer · Increasing age · Family history of prostate cancer · Strong family history of breast cancer · African men The following reduce the risk of prostate cancer:

189

Mastering MRCP ·

Foods containing lycopenes and selenium

BOF: 2.161 Which of the following may occur in quinine overdose? a) Hypertension b) Bradycardia c) Blindness d) Hyperglycemia e) Polyuria Answer: c) The features of quinine overdose are as follows: GIT · Nausea, vomiting CNS · Tremor · Tinnitus, deafness · Blurred vision-may proceed to complete blindness within a few hours · Pupils become dilated, unresponsive to light · Initially narrowing of the retinal arterioles · After 3 days retinal oedema may appear · Coma · Convulsions CVS · Tachycardia · Dysrhythmias · Hypotension · ECG conduction abnormalities · Cardiac arrest HS · Intravascular haemolysis

BOF: 2.162 A 32-year-old male who suffers from recurrent oral and genital ulcers and who has also had erythema nodosum, presents with a swollen right leg. What is the likely cause? a) Cellulitis b) Lymphoedema c) Polymyositis d) Thrombophlebitis e) Hypoalbuminaemia Answer: d) The history of recurrent oral and genital ulcers should make one suspect Behcet’s disease. Erythema nodosum is a feature of Behcet’s

190

Mastering MRCP Thrombophlebitis of the leg veins does occur in Behcet’s disease

BOF: 2.163 A 45 year old female who is known to have diffuse cutaneous scleroderma presents with headache and blurring of vision. On examination her blood pressure is 240/140. Ophthalmoscopy shows thickened vessels, arteriovenous nipping, haemorrhages and soft exudates and papilloedema. Investigations reveal a high blood urea and serum creatinine. In this patient which of the following would you use to control her hypertension? a) Nitroprusside b) Furosemide c) Beta blocker d) Sublingual calcium antagonist e) Angiotensin converting enzyme inhibitor Answer: e) This patient has presented with acute renal failure and accelerated hypertension (hypertension, grade 4 hypertensive retinopathy) Scleroderma renal crisis: · Complication of systemic sclerosis · Acute renal failure · Hypertension Treatment is with angiotensin converting enzyme inhibitors

BOF: 2.164 Which of the following is used in surveillance following surgical treatment of colorectal cancer? a) PSA b) CA 19.9 c) CA 125 d) BRCA1 e) CEA Answer: e) Carcinoembryonic antigen · Normally produced during fetal development · Not usually present in the blood of healthy adults · identify recurrences after surgical resection of colorectal cancer

BOF: 2.165 A 74-year-old female complains of difficulty in opening screw top bottles. She also complains of pain in her knees and hips. There are no systemic symptoms. On examination of her hands she is noted to have squaring of her carpus bilaterally. Investigations reveal: Hb 12.5 g/dL

191

Mastering MRCP ESR 28 mm in 1st hour Rheumatoid factor 40 IU/L This patient is most likely to have: a) Rheumatoid arthritis b) Gout c) Osteoarthritis d) Chondrocalcinosis e) Psoriatic arthropathy Answer: c) Squaring of the carpus occurs due to subluxation of the carpometacarpal joint of the thumb in osteoarthritis Rheumatoid factor may be positive in normal individuals, the incidence of this increases with age

BOF: 2.166 In massive pulmonary embolism which of the following may occur? a) Elevated troponin levels b) Hypercarbia c) Normal D-dimers d) Acidosis pH <7.1 e) Low plasma lactate levels Answer: a) In large pulmonary embolism with right ventricular strain and dilatation, myocardial injury occurs and this causes an increase in troponin levels

BOF: 2.167 A 57-year-old male is admitted to hospital with pneumonia. Which of the following would be associated with a poor outcome? a) Age 57 years b) Blood pressure of 110/70 c) Respiratory rate of 20 d) Confusion e) Pulse rate of 90 Answer d) In determining the prognosis of pneumonia the CURB-65 score may be used · Confusion · Urea > 7mmol/L · Respiratory rate ³30 /min · Blood pressure systolic < 90 diastolic £ 60 m of Hg · Age ³ 65 years 1 point for each of the above · 3 or more High risk of death · 2

192

Mastering MRCP Increased risk · 1 or 0 Low risk

BOF: 2.168 A 64-year-old male is admitted with swelling of his body. He is found to have massive proteinuria and hypoalbuminaemia. In this patient which of the following would indicate the cause of his condition? a) Decreased plasma antithrombin III level b) Bence-Jones proteinuria c) Increase in alpha and beta globulin of plasma protein electrophoresis d) Hypercholesterolemia e) Increased fibrinogen levels Answer: b) The patient has nephrotic syndrome Bence-Jones proteinuria indicate myeloma which is a cause of the nephrotic syndrome All the other features are effects of the nephrotic syndrome

BOF: 2.169 A 24-year-old male presents with a history of headache. He has a long-standing skin condition. On examination he is noted to have bilateral papilloedema. CT scanning of his head does not show a space-occupying lesion. Which of the following skin conditions may have relevance to the headache? a) Psoriasis b) Seborrhoeic dermatitis c) Acne d) Erythema nodosum e) Pityriasis versicolor Answer: c) Headaches and papilloedema with no lesion seen on CT scanning in a young patient would suggest benign intracranial hypertension. Acne vulgaris may be treated with long-term tetracycline antibiotics, which can cause benign intracranial hypertension Isotretinoin, which is a vitamin A derivative, may also cause benign intracranial hypertension

BOF: 2.170 Which of the following would account for an ataxic gait? a) Frontal lobe lesion b) Cerebellar vermis lesion c) Internal capsule lesion d) Proximal myopathy e) Cerebellar hemisphere lesion

193

Mastering MRCP Answer: e) An ataxic gait is caused by a cerebellar hemisphere lesion. This may be unilateral or bilateral. Examples are phenytoin toxicity, alcohol toxicity, multiple sclerosis, cerebrovascular disease, tumour.

BOF: 2.171 A 72-year-old male complains of backache and pain in his legs. He also says that he has difficulty in hearing and says he found that he had to buy a new cap as the previous one he had had become too small. His shoes and gloves still fit. On examination you note bowing of both legs. What is the likely diagnosis? a) Acromegaly b) Prostatic cancer with bone metastases c) Paget’s disease of bone d) Osteomalacia e) Myeloma Answer: c) The patient has an abnormality of bone that is causing deformity and pain. It has affected his skull (change in hat size, deafness) and the lower limbs but his extremities have not been affected (no change in shoe and glove size) In an elderly patient with these features one should consider Paget’s disease of bone as the likely diagnosis. Paget’s disease of bone is a progressive focal disorder of bone remodelling in which normal bone is removed and replaced by abnormal bone

BOF: 2.172 A 46-year-old female presents with abdominal pain and diarrhoea. She has been on diclofenac for a long time and has also been taking omeprazole long term for dyspepsia. Investigations reveal: Hb 12.5 g/dL Corrected calcium 2.94 mmol/L ESR 20 mm in 1st hour Endoscopy shows multiple duodenal ulcers, biopsies from the antrum of the stomach and the fundus do not show Helicobacter pylori on histology In this patient what is the likely diagnosis? a) Thyrotoxicosis b) Crohn’s disease c) Non-steroid induced colopathy d) Coeliac disease e) Multiple endocrine neoplasias Answer: e) This patient has hypercalcaemia and also has persistent duodenal ulcers despite treatment with a proton pump inhibitor, H. pylori negative

194

Mastering MRCP In this situation one would consider MEN as this could cause parathyroid adenoma or hyperplasia to account for the hypercalcaemia and an islet cell tumour secreting gastrin to account for the PPI resistant ulcers

BOF: 2.173 A 60-year-old male presents with a history of weakness of his upper limbs. On examination you note that there is wasting and weakness of the small muscles of his hands and the forearm muscles. You note fasiculations. There is weakness. The reflexes are exaggerated. In the lower limbs there is no wasting or fasiculations, there is mild weakness, the reflexes are exaggerated, there is ankle clonus and the plantars are upgoing. There is no sensory loss. The likely diagnosis is: a) Syringomyelia b) Subacute combined degeneration of the cord c) Motor neurone disease d) Pseudobulbar palsy e) Tabes dorsalis Answer: c) When you note wasting it could be due to lower motor neuron disease, myopathy or disuse Fasciculation is due to lower motor neuron damage and may occur in a number of conditions Exaggerated reflexes and clonus would indicate damage to the upper motor neuron as well Now you know that this patient has lower and upper motor neuron damage This makes motor neuron disease the most likely condition Pseudobulbar palsy does not cause lower motor neurone damage The other conditions mentioned cause sensory loss as well

BOF: 2.174 A 50-year-old male presents with a history of cough, productive of sputum, which has streaks of blood in it. He also gives a history of wheeze and diarrhoea. What is the likely cause of his condition? a) Alveolar cell carcinoma b) Small cell carcinoma c) Adenocarcinoma d) Squamous carcinoma e) Carcinoid tumour Answer: e) The symptoms of cough with haemoptysis should make one think of a bronchial tumour With the added features of wheeze and diarrhoea one should think about the carcinoid syndrome

BOF: 2.175 A 50-year-old female presents with acute onset of breathlessness. Her blood gases show: pH 7.47, paO2 8.2 kPa, paCO2 4.2 kPa, bicarbonate 20 mmol/L

195

Mastering MRCP

Which of the following is the likely diagnosis? a) Narcotic overdose b) Anxiety c) Pulmonary embolism d) Severe kyphoscoliosis e) COPD Answer: c) The patient is hypoxic and has a respiratory alkalosis Pulmonary embolism by causing reflex hyperventilation can give this picture

BOF: 2.176 A 22-year-old female is admitted with a history of taking an overdose of her grandmother’s medication. Investigations show: Serum potassium 6.3 mmol/L Which of the following tablets is she likely to have ingested? a) Propranolol b) Prednisolone c) Digoxin d) Bendroflumethiazide e) Verapamil Answer: a) Beta adrenergic stimulation increases cellular entry of potassium and conversely this is blocked by propranolol and may result in hyperkalaemia

BOF: 2.177 A 36-year-old female who is married and has 2 children and has completed her family, presents with a history of amenorrhoea and galactorrhoea. She has no visual abnormality. Investigations reveal a raised prolactin level and MRI shows a 1.4 cm pituitary tumour. In this patient which of the following lines of management would you follow? a) Surgery b) Surgery combined with radiotherapy c) Cabergoline therapy d) Radiotherapy alone e) Surgery together with cabergoline Answer: c) This patient has a prolactinoma. Prolactinomas are usually treated medically with dopamine agonists. There are no pressure effects, which would have been an indication for surgery. The patient has completed her family and this would have been the only concern as these tumours enlarge during pregnancy and surgery would have to be considered pre-pregnancy

196

Mastering MRCP BOF: 2.178 Which of the following is a common adverse effect of treatment with cyclosporin? a) Nephrotoxicity b) Hypokalaemia c) Alopecia d) Bone marrow toxicity e) Hypotension Answer: a) Adverse effects of cyclosporin therapy are: · Nephrotoxicity · Hyperlipidaemia · Hypertension · Hyperkalaemia · Hyperuricaemia · Hypophosphataemia · Hirsutism · Hepatotoxicity · Haemolytic uremic syndrome · Diabetes mellitus · Lymphoma · Tremor · Gingival hypertrophy Bone marrow problems are not a feature

BOF: 2.179 A 56-year-old female who is known to have rheumatoid arthritis presents with progressive breathlessness. In this patient which of the following physical signs would probably occur? a) Bronchial breathing b) Fine late inspiratory crackles (crepitations) c) Polyphonic rhonchi d) Whispering pectoriloquy e) Monophonic rhonchi Answer: b) Fibrosing alveolitis may occur in rheumatoid arthritis and this would cause progressive breathlessness. Respiratory complications of rheumatoid arthritis are: Cricoarytenoid joint involvement Obstructive bronchiolitis Fibrosing alveolitis Nodule Caplan’s syndrome Pleural effusion Pleural nodules

197

Mastering MRCP BOF: 2.180 A 60 year-old male presents with a 3-month history of recurrent abdominal pain localised to the left hypochondrium He also complained of fever. Investigations reveal Haemoglobin of 19.5 g dL White cell count 18 x109 /L Platelets 709 x 109 /L In this patient which of the following tests would you arrange next in order to reach a definite diagnosis? a) Blood gas analysis b) Abdominal ultrasound scan c) Bone marrow examination d) Serum erythropoietin level e) JAK 2V617F mutation analysis Answer: e) This patient has polycythaemia. The history of abdominal pain localised to the right hypochondrium would suggest that he has splenomegaly The chances are he has polycythaemia rubra vera (PRV). 95% of patients with PRV have this mutation in the Janus kinase 2 gene (JAK2). This mutation and other rarer mutations are highly specific for myeloproliferative disease and have thus altered the approach made in the investigation of these conditions

BOF: 2.181 A 34-year-old male who is known to be HIV positive complains of grittiness in his eyes and a dry mouth. On examination he is noted to have bilateral parotid enlargement and hepatomegaly. Investigations: Anti Ro: negative Anti La: negative ANA: negative Rheumatoid factor: negative In this patient which of the following is the likely diagnosis? a) Sjögren’s syndrome b) Lymphoma c) Diffuse infiltrative lymphocytosis syndrome d) Mumps e) Salivary duct calculi Answer: c) Diffuse infiltrative lymphocytosis syndrome (DILS) Complication of HIV infection: · Persistent CD8+ lymphocytosis · Lymphocytic infiltration of various organs Presents with · Bilateral parotid gland enlargement · Sicca syndrome

198

Mastering MRCP · · ·

Pulmonary involvement Liver involvement Peripheral neuropathy

BOF: 2.182 Which of the following drugs increases the anticoagulant effect of warfarin? a) Clarithromycin b) Carbamazepine c) Phenytoin d) Rifampicin e) Cholestyramine Answer: a) Clarithromycin increases the anticoagulant effect of warfarin All the other dugs decrease the anticoagulant effect of warfarin

BOF: 2.183 A 26-year-old female presents with a history of weight gain and polyuria. On examination she is obese has marked abdominal striae, blood pressure 190/110 and urine testing shows glycosuria In this patient which of the following tests would you arrange in the first instance? a) 24 hour urinary free cortisol b) Pituitary MRI c) High dose dexamethasone suppression test d) CRH test e) Plasma ACTH levels Answer: a) Clinically the patient has Cushing’s syndrome the first step is to confirm Cushing’s and for this purpose out of the tests listed 24-hour urinary free cortisol would be the most appropriate The other tests are to determine the cause of Cushing’s

BOF: 2.184 A 68-year-old female complains of feeling tired all the time and also complains of breathlessness on exertion. She has had radiotherapy for carcinoma of the cervix in the past. Investigations reveal iron deficiency anaemia. She has an upper GI endoscopy and distal duodenal biopsy. Histology from the duodenal biopsy is reported as showing villous atrophy, crypt hyperplasia, increase in number of intraepithelial lymphocytes, infiltration of plasma cells and lymphocytes in the lamina propria. In this patient which of the following is the likely diagnosis? a) Whipple’s disease b) Radiation enteropathy c) Coeliac disease d) Crohn’s disease

199

Mastering MRCP e) Peptic duodenitis Answer: c) The histological features are in keeping with a diagnosis of coeliac disease

BOF: 2.185 A78 year old male presents with a history of feeling generally unwell. He says he has lost weight and puts it down to not eating properly as he has pain in his jaw when he eats. He has seen his dentist who has not found any abnormality. He also says he is finding it difficult to climb stairs and get out of bed and rise from the seated position and needs help from the carriers at the nursing home he lives in. Investigations reveal: Hb 10.6 /dL normocytic normochromic picture ESR 84 mm in the 1st hour Which of the following is the likely diagnosis in this patient? a) Polymyositis b) Polymyalgia rheumatica c) Sjögren’s syndrome d) Eaton Lambert syndrome e) Myeloma Answer: a) The history of pain on eating would fit in with jaw claudication. Difficulty in climbing stairs, getting out of bed and rising from chairs would be due to girdle involvement in polymyalgia

BOF: 2.186 A 56-year-old investment banker, who has recently returned from a conference abroad, presents with a history of cough, fever and breathlessness. His chest X-ray shows bilateral pulmonary infiltrates. In this patient which one of the following antibiotics would be the most appropriate? a) Amoxycillin b) Clarithromycin c) Penicillin d) Rifampicin e) Augmentin Answer: b) Given the history of a conference and the bilateral pulmonary infiltrates one would suspect Legionella as the causative organism and treat the patient with clarithromycin.

BOF: 2.187 A 26-year-old male presents with a history of bleeding gums. On examination you note s few petechial haemorrhages around the patient’s ankles. Investigations reveal: Hb 13.4 g/dl

200

Mastering MRCP WBC 6.2 x 109/L Platelets 36 x109/L In this patient bone marrow examination is likely or show: a) Increased number of megakaryocytes b) Decreased number of megakaryocytes c) Normal number of megakaryocytes d) Abnormal morphology of the megakaryocytes e) Degenerating megakaryocytes Answer: a) The clinical features and investigations suggest the patient has idiopathic thrombocytopenic purpura. This is caused by immune destruction of platelets. The megakaryocytes increase in number to try and compensate for the loss of platelets.

BOF: 2.188 A 26-year-old male who live in the south-western coastal region of Sri Lanka, presents with a history of cough and wheeze of a few months duration. He also complains of fever, feeling generally unwell and has lost weight. On examination he appears to have lost weight. Auscultation of his lungs reveals widespread rhonchi and crepitations. Chest X-ray shows miliary mottling. Hb 12.9 g/ dL Eosinophil count 36,000 /mm3 In this patient which of the following would you use to treat the patient? a) High dose oral steroids b) Rifampicin, Isoniazid, Ethambutol, Pyrazinamide c) Albendazole d) Praziquantel e) Diethylcarbamazine Answer: e) The patient has cough, wheeze and systemic features and a very high eosinophil count. This should make you think of a parasitic infestation especially as the patient is from Sri Lanka an area where filariasis is endemic The features are typical of tropical pulmonary eosinophilia and the treatment of choice is diethylcarbamazine

BOF: 2.189 A 52-year-old male complains of flushing of his face especially in relation to consuming alcohol. On examination of his face you note that his face is flushed, there are multiple telangiectasiae, papules and pustules. In this patient which one of the following agents would you use as treatment ? a) Oral metronidazole b) Steroid cream c) Benzoyl peroxide

201

Mastering MRCP d) Cyproterone acetate/ethinyl oestradiol e) Dapsone Answer: a) The clinical features suggest the patient has rosacea which may be treated with oral or topical antibiotics such as metronidazole or tetracycline or oral isotretinoin

BOF: 2.190 A patient from West Africa comes to your clinic with a complaint of noticing pale patches on his skin. On examination of the patient you notice several hypopigmented macules. On further examination you note loss of sensation over these hypopigmented areas. In this patient which one of the following agents would be the mainstay of treatment? a) Selenium sulphide b) Ketoconazole c) Monobenzone d) Intralesional triamcinolone e) Dapsone Answer: e) The finding of hypopigmented macules with loss of sensation over them should make one suspect leprosy The main agent used in treatment of leprosy is dapsone

BOF: 2.191 A 36-year-old male presents with a history of recurrent chest infections. He gives a history of long standing cough productive of sputum and halitosis. On examination you note that he has clubbing of his fingers and auscultation of his chest reveals coarse crepitations at both lung bases. In this patient which one of the following would you recommend to reduce the frequency of chest infections? a) Lung resection b) Long term antibiotics c) Inhaled steroids d) Postural drainage e) Inhaled bronchodilators Answer: d) The clinical features would suggest the patient has bronchiectasis. In this condition postural drainage would be the most effective way to prevent recurrent chest infections.

BOF: 2.192 Which of the following antibodies is associated with the development of fetal congenital heart block? a) Anti Ro b) Anti GBM c) Anti MA d) ANCA

202

Mastering MRCP e) ANA Answer: a) Anti Ro and anti La are associated with development of fetal congenital heart block

BOF: 2.193 A 72-year-old male presents with a history of feeling tired, weak and breathless. He also complains of bleeding gums. On examination he is found have lymphadenopathy and his spleen is just palpable. Investigations reveal: Hb 9.0 g/dL normocytic normochromic picture WBC 86,000 predominantly lymphocytes Platelets 85 x 109/L Coomb’s test negative In this patient which of the following treatment options would you choose? a) Observe with serial blood counts b) Chlorambucil c) Splenectomy d) Splenic irradiation e) Vincristine, prednisolone, doxorubicin, cyclophosphamide Answer: b) The patient has chronic lymphocytic leukemia and is symptomatic and hence should be treated. There is no evidence of haemolysis (Coomb’s negative) and there is only minimal enlargement of the spleen hence no necessity for splenectomy or splenic irradiation. The last option is treatment for acute lymphoblastic leukemia

BOF: 2.194 A 74-year-old male presents with anorexia and loss of weight. On examination you note a palpable lymph node in the left supraclavicular region. Endoscopy shows a large ulcer in the stomach with rolled edges. Histological examination reveals that it is an adenocarcinoma. In this patient which of the following would account for the palpable lymph node in the left supraclavicular fossa. a) Retrograde spread from the cisterna chyli b) Non- metastatic manifestation c) Haematogenous spread d) Antegrade spread from the right lymphatic duct e) Retrograde spread from the thoracic duct Answer: e) Gastric cancer spreads through the regional lymph nodes to the thoracic duct, which drains into the left subclavian vein. Retrograde spread from the thoracic duct can give a palpable lymph node in the left supraclavicular fossa (Virchow’s node). This is known as Troisier’s sign.

203

Mastering MRCP BOF: 2.195 Which one of the following does the antibody Rituximab act against? a) CD 20 b) CD 4 c) CD 5 d) IL 6 e) TNF alpha Answer: a) Rituximab is a monoclonal antibody that acts against the protein CD (cluster of differentiation) 20 It is used in the treatment of lymphoma, leukemia and autoimmune disorders.

BOF: 2.196 A 36-year-old male presents with a history of pain in his neck. On examination you note that his neck is twisted with the chin pointing towards the left shoulder. He is unable to move the neck normally. Which one of the following drugs may be responsible for his condition? a) Omeprazole b) Metoclopramide c) Clarithromycin d) Metronidazole e) Salbutamol Answer: b) The history of pain in the neck together with the abnormal posture should make one think of torticollis (twisted neck) This is a type of dystonia it could be a focal dystonia or could be due to drugs such as phenothiazines, metoclopramide, haloperidol, carbamazepine, and phenytoin

BOF: 2.197 You are sitting in with the neurology registrar at the epilepsy clinic and he asks you to examine a young male. The patient has leaf like hypopigmented macules on his trunk and on examination of his hands you note Periungual fibroma. This patient has: a) Neurofibromatosis b) Sturge Weber syndrome c) Tuberous sclerosis d) Refsum’s disease e) Fabry’s disease Answer: c) Tuberose sclerosis is a condition characterised by a variety of hamartomas. The cardinal features are epilepsy, mental retardation and cutaneous abnormalities.

204

Mastering MRCP BOF: 2.198 Which of the following is a risk factor for developing diabetic retinopathy? a) Smoking b) Alcohol abuse c) Obesity d) Lack of exercise e) Drug abuse Answer: a) The risk factors for development of diabetic retinopathy are as follows: · Duration of disease · Poor glycaemic control · Hypertension · High cholesterol · Pregnancy · African or Hispanic race · Smoking

BOF: 2.199 You are at the diabetic clinic and a patient who is an insulin dependent diabetic complains of a rash on the back of his hands. On examination you note small erythematous papules arranged in the form of a ring. This patient has: a) Diabetic dermaopthy b) Necobiosis lipoidica diabeticorum c) Granuloma annulare d) Carbuncles e) Cutaneous candidiasis Answer: c) In diabetes mellitus the skin may be affected by dermatoses and by infections Granuloma annulare is a granulomatous dermal infiltrate with foci of collagen degeneration as presents as clusters of small flesh colored or erythematous papules that are arranged in the form of a ring or part of a ring.They are distributed over the dorsum of the hands and feet

BOF: 2.200 You are teaching medical students how to analyze the jugular venous pulsation. One of the students asks you the relationship between the different waves and the waves on the ECG You tell the student that the “a” wave in the jugular venous pulsation corresponds to the following wave on the ECG: a) P wave b) Q wave c) R wave d) S wave e) T wave

205

Mastering MRCP Answer: a) The “a” wave coincides with atrial contraction and therefore corresponds to the p wave on the ECG

BOF: 2.201 A 27-year-old female has been referred to your clinic by her GP with a presumed diagnosis of irritable bowel syndrome. Which of the following features would make you think that this is not irritable bowel syndrome and warrants further investigation? a) Nocturnal defecation b) Passage of mucous c) Backache d) Pain relieved by defecation e) Incomplete evacuation of the bowel Answer: a) Rectal bleeding, steatorrhoea and nocturnal defecation are not features of irritable bowel syndrome and would warrant further investigation

BOF: 2.202 A 56-year-old female who is known to have breast cancer presents with a history of unsteadiness of gait. CT scanning shows multiple cerebral metastases with surrounding oedema. Which of the following would be most effective in reducing the cerebral oedema? a) Mannitol b) Furosemide c) Spironolactone d) Dexamethasone e) Metolazone Answer: d) Cerebral oedema surrounding brain metastases is thought to be vasogenic in origin and dexamethasone is the established treatment for this

BOF: 2.203 A 36-year-old male who plays squash regularly complains of pain in his wrist that is aggravated by turning the wrist, grasping anything or making a fist. The pain radiates towards his forearm. On examination there is no obvious deformity of his hand. There is swelling and tenderness in the dorsal compartment of his wrist. You ask the patient to make a fist with his thumb held inside the fingers and then you passively ulnar deviate his fist and this reproduces the pain. There are no abnormalities of motor or sensory function of the hand. In this patient the diagnosis is likely to be: a) Arthritis of the first carpometacarpal joint b) de Quervain’s tenosynovitis c) Carpal tunnel syndrome d) Reactive arthritis

206

Mastering MRCP e) Cervical radiculopathy Answer: b) De Quervain’s tenosynovitis refers to inflammation of the tendon sheath enclosing the abductor pollicis longus and the extensor pollicis brevis tendons at the lateral border of the anatomical snuffbox This occurs with repetitive use and injury The Finkelstein test where the patient makes a fist with the thumb inside the finger and the clinician applies passive ulnar deviation of the wrist, reproduces the dorsolateral wrist pain Treatment would involve the use of NSAIDs, local steroid injections and physiotherapy

BOF: 2.204 A 72-year-old male who is known to have aortic stenosis presents with a history of feeling generally unwell and a low-grade fever. He also gives a history of a change in bowel habit. On examination he is noted to have splinter haemorrhages, early clubbing and there is an early diastolic murmur at the left sternal edge, which has not been noted earlier. In this patient blood cultures are taken and a significant growth is noted. Which of the following organisms is likely to be isolated? a) Streptococcus viridans b) Staphylococcus aureus c) Streptococcus bovis d) Staphylococcus epidermidis e) Kingella kingae Answer: c) · The clinical features suggest the patient has infective endocarditis. · With the history of change in bowel habit one would suspect that this bears a relationship to the aetiology of the endocarditis. · There is a strong association between S bovis bacteraemia with or without endocarditis and underlying malignant or premalignant lesions of the colon · An association of bacteraemia (or endocarditis) and liver disease has also been established · Every patient with S bovis bacteraemia with or without endocarditis should be investigated for a gastrointestinal malignancy

BOF: 2.205 A 26-year-old male presents to the casualty unit with a history of severe nausea and vomiting and abdominal cramps. On further enquiry he says he ate fried rice 3 hours prior to the onset of symptoms. What is the organism responsible for his condition? a) Clostridium perfringens b) Bacillus cereus c) Staphylococcus aureus d) Bacillus subtilis e) Escherichia coli Answer: b) Bacillus cereus · Food borne illness

207

Mastering MRCP Caused by: · Survival of bacterial endospores when food is improperly cooked · Problem compounded when food is then improperly refrigerated, allowing the endospores to germinate Bacillus cereus · Two types of food-borne illnesses · "Short-incubation" or emetic form of the disease Characterised by nausea and vomiting and abdominal cramps Incubation period of 1 to 6 hours · "Long-incubation" or diarrhoeal form Manifested primarily by abdominal cramps and diarrhoea Incubation period of 8 to 16 hours Diarrhoea may be a small volume or profuse and watery Staphylococcus aureus may cause an illness similar to the short incubation form of disease The type of food involved is usually cooked meats and cream products Clostridium perfringens causes and illness similar to the long incubation form of the disease

BOF: 2.206 A 56-year-old male is admitted with breathlessness. He admits to smoking 30 cigarettes a day for 30 years. On examination you note flaring of his alae nasi, trachea was in the midline, cricosternal distance decreased, chest barrel shaped, vocal fremitus was reduced but equal on both sides, respiratory movements by palpation were equal, percussion note was hyper-resonant with decreased cardiac and liver dullness, breath sounds were vesicular but reduced in intensity, no added sounds In this patient which one of the following measures would improve the patient’s prognosis? a) Long term oxygen therapy b) Chest physiotherapy c) Long term antibiotics d) Inhaled steroid e) Inhaled salbutamol Answer: a) · The clinical features would point to a diagnosis of chronic obstructive pulmonary disease · Long-term oxygen therapy prolongs life of those with advanced COPD and severely reduced oxygen levels · Round-the-clock therapy is best But Using oxygen 12 hours a day also has benefits · Reduces secondary polycythaemia · Relieves cor pulmonale · Oxygen therapy may also improve breathlessness during exercise

BOF: 2.207 A 48-year-old female who has recently been diagnosed as having rheumatoid arthritis presents with upper abdominal discomfort and a history of passing a dark tarry stool. Which of the following drugs would have been likely to account for this?

208

Mastering MRCP a) Prednisolone b) Methotrexate c) Celecoxib d) Penicillamine e) Diclofenac Answer: e) The history of upper abdominal discomfort and the passage of dark tarry stools (melaena) would make you suspect peptic ulceration. The duodenal mucosa is protected from gastric acid and pepsin by: · Mucous layer · Bicarbonate secretion · Protective prostaglandins NSAIDs by reducing prostaglandin synthesis reduces this protective effect Selective COX 2 (cyclooxygenase 2) inhibition is thought to reduce the risk of peptic ulceration Diclofenac being a non-selective COX inhibitor would be more likely to cause peptic ulceration than celecoxib

BOF: 2.208 A 52-year-old male who is known to have cirrhosis of the liver presents with deteriorating renal function. Your consultant makes a diagnosis of hepatorenal syndrome and suggests treatment with salt free albumin and terlipressin. Terlipressin is beneficial in hepatorenal syndrome as it causes: a) Renal vasodilatation b) Splanchnic vasoconstriction c) Osmotic diuresis d) Stimulates the juxtaglomerular apparatus e) Inhibits anti diuretic hormone (ADH) Answer: b) · The initial event in hepatorenal syndrome is vasodilatation of the splanchnic circulation · Thereby pooling blood in the splanchnic circulation and reducing renal perfusion · Vasopressin V1 receptors, such as ornipressin and terlipressin predominantly act on the splanchnic circulation and cause vasoconstriction

BOF: 2.209 Which of the following conditions causes an increased secretion of erythropoietin? a) Renal haemangioblastoma b) Secondary deposits in the liver c) Choriocarcinoma d) Renal carcinoma e) Pontine glioma Answer: c) Renal carcinoma can cause an increased secretion of erythropoietin

209

Mastering MRCP BOF: 2.210 A 60-year-old female presents with sudden onset of breathlessness, pleuritic chest pain and syncope. She gives history of recently returning to the UK from a holiday in Australia. There are no past illnesses of note and she is not on any medication. On examination she looks pale, has a tachycardia of 120 beats per minute BP 80/50 JVP elevated predominant V wave, there is a triple rhythm with a loud second sound on auscultation of the heart. There is a pan systolic murmur at the left sternal edge; a pleural rub is heard at the left base of the lung In this patient which of the following would you use as therapy? a) Thrombolysis b) Intravenous diuresis c) Inotropes d) Intravenous fluids and vasoconstrictors e) Intra-aortic balloon counterpulsation Answer: a) The clinical features would suggest that the patient has had a massive pulmonary embolism. Thrombolysis for pulmonary emboli is recommended for massive PE with haemodynamic shock

BOF: 2.211 A 63-year-old female has osteoporosis. You suggest treatment with a bisphosphonate. The patient asks you whether to take the medication before or after meals and you reply that she should take the bisphosphonate: a) In the morning before having anything to eat or drink b) With her calcium and vitamin D supplement c) After her main meal d) Last thing at night e) After breakfast Answer: a) Bisphosphonates · Characterised by poor intestinal absorption · Highly selective localisation and prolonged storage in bone · Stable · Bisphosphonates are absorbed, stored and excreted unchanged Absorption of bisphosphonates · Intestinal absorption is very low and variable (1-10%) · Passive diffusion in the stomach and upper small intestine · Reduced if the drug is given with calcium or iron · Never given at meal times or with dairy products

BOF: 2.212 Which of the following drugs may cause prolongation of the QT interval? a) Atenolol b) Sotalol c) Bisoprolol

210

Mastering MRCP d) Carvedilol e) Propranolol Answer: b) Several drugs can cause prolongation of the QT interval: Alfuzosin Amantadine Azithromycin Chloral hydrate Clozapine Flecainide Foscarnet Fosphenytoin Indapamide Levofloxacin Lithium Moxifloxacin Nicardipine Octreotide Ofloxacin Ondansetron Oxytocin Quetiapine Risperidone Tacrolimus Tamoxifen Venlafaxine

BOF: 2.213 Which of the following is the most common cause of thrombophilia? a) Factor V Leiden b) Protein C deficiency c) Protein S deficiency d) Antithrombin deficiency e) Antiphospholipid syndrome Answer :a) Thrombophilia Most common · Factor V Leiden · Prothrombin G20210A · Homocystinaemia Less Common · Protein C deficiency · Protein S deficiency · Antithrombin deficiency

211

Mastering MRCP · ·

Rare thrombophilias associated with increased clotting factors Antiphospholipid syndrome

BOF :2.214 Which of the following viruses has been implicated in the pathogenesis of adult T cell leukemia ? a) EBV b) HHV c) HTLV1 d) HIV e) FeLV Answer: c) EBV Epstein Barr Virus HHV Human Herpes Virus HTLV1 Human T cell Lymphotropic Virus HIV Human Immunodeficiency Virus FeLV Feline Leukemia Virus Adult T-cell leukemia/lymphoma (ATLL) · Mature T-cell neoplasm of post-thymic lymphocytes · Aetiologically linked to the human T-cell lymphotropic virus (HTLV 1) · Distinct geographical distribution Disease manifestation · Leukemia · Lymphoma

BOF: 2.215 A 26-year-old male who is known to have hereditary spherocytosis presents with dyspnoea following a viral infection. He is found to be anaemic with a low reticulocyte count. Which of the following viruses is likely to have caused this condition? a) Parvovirus B19 b) Respiratory Syncytial Virus c) Dengue virus d) Epstein Barr virus e) Influenza B Answer: a) Anaemia with a low reticulocyte count in a patient with a known haemolytic anaemia would suggest an aplastic crisis Chronic haemolytic anaemia · Increased rate of erythropoiesis to maintain haemoglobin concentration · Parvovirus B19 can lead to a bone marrow hypoplasia by disrupting erythropoiesis · It will thus precipitate an aplastic crisis · Parvovirus B19 is associated with about 90% of aplastic crises in patients with underlying haemolytic disorders

212

Mastering MRCP BOF: 2.216 A 27-year-old female presents with a history of episodes of wheezing and breathlessness of 6 months duration. No past illnesses of note. She admits to smoking 20 cigarettes a day for 10 years. On examination no abnormalities are detected. Which of the following methods would you use to manage this patient? a) Arrange baseline FEV1 repeat following inhaled corticosteroids b) Arrange baseline FEV1 repeat following inhaled salbutamol c) Ask the patient to maintain a peak flow diary d) Arrange spirometry e) Give the patient a trial of salbutamol inhaler Answer: d) The history is suggestive of asthma or reversible airflow obstruction. However, this must be confirmed and confirmation of the diagnosis would be best achieved by maintaining a peak flow diary and documenting reversible airway obstruction.

BOF: 2.217 A 65-year-old male is admitted with a history of breathlessness and swelling of his ankles. He is known to have ischaemic heart disease and has had a myocardial infarct in the past. On examination there is peripheral oedema, his pulse rate is 100 beats per minute regular, he has an elevated JVP, displaced apex beat, a third heart sound, reversed splitting of the second heart sound and a soft systolic murmur at the apex. Reversed splitting of the second heart sound occurs in association with: a) Left bundle branch block b) First degree heart block c) Delta wave in lead V1 d) Right bundle branch block e) QT prolongation Answer: a) Reversed splitting of the second heart sound occurs in left bundle branch block

BOF: 2.218 A 74-year-old male presents with a history of episodes of sudden onset transient loss of consciousness. On examination you note pulse rate of 40 beats per minute, BP 110/60 JVP cannon a waves, heart sounds dual rhythm, varying intensity of the 1st heart sound, soft systolic murmur in the mitral area In this patient which of the following would you expect to see on ECG? a) Wenckebach phenomenon b) 1st degree heart block c) Complete heart block d) Right bundle branch block e) Left bundle branch block Answer: c) · Canon a waves occur when the atrium contracts against a closed atrioventricular valve

213

Mastering MRCP · Stokes-Adams attacks (transient asystole or ventricular fibrillation on a background of complete heart block) may occur in those with complete heart block

BOF: 2.219 A 30 year old female presents with swelling of her left leg. There is not past history of note she has not been on a long journey. On examination you note that her left leg is swollen, erythematous, there are dilated superficial veins, the area feels warm to touch and is tense. Investigations show Hb 13.5 g/dL WBC 7.5 x109/ Platelets 400 x109/L Prothrombin time 14 s (control 12 s) Activated Partial Thromboplastin Time 63 s (30-40s) In this patient what is the likely diagnosis? a) Factor V Leiden b) Protein C deficiency c) Protein S deficiency d) Antithrombin III deficiency e) Antiphospholipid syndrome Answer :e) The patient has the clinical features of a deep vein thrombosis. There is no predisposing cause. The blood tests show anticoagulant activity. In the setting of a thrombosis the presence of anticoagulant activity should make you think of lupus anticoagulant and the antiphospholipid syndrome. Lupus anticoagulant · Immunoglobulin (IgG or IgM) In vitro · Binds to phospholipids and prevents coagulation reactions from taking place on the platelet surface Paradoxically In vivo · Arterial and venous thrombosis · Recurrent spontaneous abortions Occurs in · 30% of patients with systemic lupus erythematosus · May be found in other autoimmune diseases · Response to drugs such as phenothiazine · Patients with infectious diseases such as AIDS · Often, no underlying condition Test · No direct test for the lupus anticoagulant (LA) · Inhibitory actions on coagulation · Binds to phospholipid on the platelet surface and interferes with the formation of the prothrombin activator complex Clotting times

214

Mastering MRCP · Increased · Not corrected by the dilution with normal plasma · Increased activated partial thromboplastin time · Increased prothrombin time False positive VDRL test

BOF: 2.220 A 46 year old female presents with a history of breathlessness and passing dark urine. There was s history of episodes of passing dark urine over the last one year. She also gave a history of an unexplained deep vein thrombosis in the past. On examination she was pale and looked jaundice but other than that there were no abnormalities detected. Investigations Hb 7.5 g/dL WBC 2.3 x109 /L Platelets 110 x109 /L Bilirubin 60 µmol/L Urine urobilinogen +++ haemosiderin ++ In this patient which one of the following complications is likely to occur? a) Vitamin B12 deficiency b) Folic acid deficiency c) Iron deficiency d) Hyposplenism e) Hypersplenism Answer :c) The patient has a haemolytic anaemia with intravascular haemolysis causing haemosiderinuria and deep vein thrombosis. This should make you think of paroxysmal nocturnal haemoglobinuria. In PNH iron deficiency may occur due to loss of iron in the urine as haemoglobinuria and haemosiderinuria

BOF: 2.221 A 63 year old male presents with sudden onset breathlessness. He is a known diabetic well controlled on oral hypoglycaemic agents. On examination he is noted to be breathless, orthopnoeic, sweating. He has a pulse rate of 120 beats per minute, his BP is 170/100, his JVP is raised, heart sounds are normal with a soft systolic murmur at the apex; on auscultation of his lungs bilateral basal crepitations are heard. The patient is treated with intravenous furosemide and makes a speedy recovery. Two days later the patient is changed over to oral furosemide and started on an ACE inhibitor. Investigations On admission Urea 8.6 mmol/L Creatinine 132 µmol/L Sodium 136 mmol/L Potassium 3.8 mmol/L

215

Mastering MRCP One week after admission Urea 16 mmol/L Creatinine 240 µmol/L Sodium 138 mmol/L Potassium 4.8 mmol/L In this patient which one of the following physical signs would point you to the cause of the deteriorating renal function? a) Abdominal bruit b) Absent left radial pulse c) Delayed femoral pulse d) Distended bladder e) Absent peripheral pulses Answer: a) Deterioration of renal function after commencement of an ACE inhibitor should make you suspect renal artery stenosis and the presence of a renal artery bruit would point you to the diagnosis. The patient is a diabetic and this increases his risk of developing atheroma and renal artery stenosis.

BOF: 2.222 A 22 year old female presents with a history of headaches of several weeks duration. The headaches are most severe in the morning and associated with nausea. She was on long term tetracycline for acne and was also using high doses of vitamin B complex supplements and vitamin C as she had heard that this carried health benefits. There is a strong family history of migraine .On examination of her visual fields you note enlargement of the blind spot, there was bilateral papilloedema and left lateral rectus palsy. A CT scan of her head does not show a mass lesion. In this patient which of the following would indicate the likely cause of the condition? a) Vitamin C supplementation b) Vitamin B supplementation c) Family history of migraine d) Treatment with tetracycline e) Lateral rectus palsy Answer: d) The history of headache in a young female and the clinical features of raised intracranial pressure with no mass lesion would suggest the patient has benign intracranial hypertension Treatment with tetracycline is one of the causes of this condition Vitamin A intoxication can cause benign intracranial hypertension but not water soluble vitamins B and C A family history of migraine would have no bearing on this condition Lateral rectus palsy or sixth nerve palsy is a false localizing sign

BOF: 2.223 A 32 year old female presents with a history of weight gain. There is no past history of note On examination you note that she has truncal obesity and has marked acne and is hirsute In this patient which one of the following clinical features would suggest the patient has Cushing’s syndrome?

216

Mastering MRCP a) Truncal obesity b) Acne c) Hirsutism d) Striae e) Waddling gait Answer: e) A waddling gait would suggest the patient has a proximal myopathy and this would indicate Cushing’s syndrome All the other features could occur in simple obesity and polycystic ovarian syndrome which are the other conditions that could present in a similar fashion

BOF: 2.224 A 36 year old male presents with a history of pain and weakness of his right shoulder. 10 days prior to the onset of symptoms he had an upper respiratory tract infection but apart from that there was no significant past medical history. He was not on any medication On examination you note wasting of his right supraspinatus and infraspinatus muscles. There is weakness of initiation of abduction and external rotation of his right shoulder. The rest of the neurological examination of the upper limb is unremarkable. Reflexes are normal and there is no sensory loss. In this patient which of the following investigations is the most likely diagnosis? a) Polymyositis b) Neuralgic amyotrophy c) Eaton Lambert syndrome d) Syringomyelia e) Polymyalgia rheumatica Answer: b) This young patient has presented with painful wasting and weakness of the muscles of the shoulder girdle following an upper respiratory tract infection This should make you think of neuralgic amyotrophy as the cause of the problem

BOF: 2.225 A 50 year old female presents with a history of progressive breathlessness of several months duration She has no significant past medical history and is not on any medication. On examination you note that she is breathless at rest and orthopnoeic. She has flaring of her alae nasi, her conjunctiva was dry and her mouth looked dry, pulse rate 110 beats per minute , BP120/70 respiratory rate 30 breaths per minute heart sounds were normal , breath sounds vesicular with fine end inspiratory crepitations at both lung bases . In this patient which one of the following complication is most likely? a) Acute myeloid leukemia b) Chronic myeloid leukemia c) Polycythaemia rubra vera d) Chronic lymphocytic leukemia e) Lymphoma Answer: e)

217

Mastering MRCP The patient has features of fibrosing alveolitis and also has dry eyes and mouth. This would suggest the patient has Sjogren’s syndrome. Patients with Sjogren’s syndrome have a 40 fold higher chance of developing lymphoma and the salivary glands are the main site of lymphomatous change

BOF: 2.226 A 65 year old male who is known to have valvular heart disease and chronic heart failure is admitted to the ward for treatment of cellulitis of his right leg. The patient is on digoxin, diuretics and warfarin. On examination of the patient you note that his pulse rate is 80 beats per minute irregular , not collapsing , BP 130/80, JVP elevated with a prominent V wave , apex displaced the 6th left intercostal space anterior axillary line , thrusting in nature , there was left parasternal heave , first heart sound was soft , there was a blowing pan systolic murmur at the apex and this radiated to the axilla, there was another pan systolic murmur at the left sternal edge which increased with inspiration and there was an early blowing decrescendo early diastolic murmur at the left sternal edge. In this patient which of the following physical signs would enable you to determine the valvular origin of the blowing decrescendo early diastolic murmur at the left sternal edge? a) Thrusting apex beat b) Soft first heart sound c) Pulse not collapsing d) V wave e) Displaced apex Answer: c) The patient has features of mitral and tricuspid regurgitation. The fact that mitral valve disease has caused right ventricular failure would suggest that there is pulmonary hypertension. This may result in pulmonary incompetence leading to the early diastolic murmur at the left sternal edge. The absence of a collapsing pulse and the normal pulse pressure would be against aortic incompetence and this would suggest that the early diastolic murmur is likely to be due to pulmonary regurgitation.

BOF: 2.227 A 63 year old female who has been on warfarin because of atrial fibrillation has been inadvertently prescribed erythromycin for an upper respiratory tract infection and is admitted with a few minor bruises on her lower limbs She has no other medical problems and her current therapy is warfarin and digoxin Her INR is 8.5 In this patient your management would be: a) Stop warfarin b) Stop warfarin, give vitamin K 0.5 mg c) Stop warfarin, give vitamin K 5 mg d) Stop warfarin, give vitamin K 10 mg e) Stop warfarin, give prothrombin complex concentrate or FFP and Vitamin K 5 mg iv Answer :a) The guidelines for management of over-anticoagulation with warfarin are set out below

218

Mastering MRCP Reduce warfarin or stop INR < 6.0 INR 6-8 No bleeding or little bleeding INR> 8.0 Little bleeding or minor bleeding

Major bleeding

Restart when INR <5.0 Stop warfarin Restart when INR <5.0 Stop warfarin Restart when INR < 5.0 If other risk factors for bleeding give Vitamin K 05-2.5 mg Stop warfarin Give prothrombin complex concentrate or FFP Vitamin K 5 mg iv

BOF: 2.228 A 63 year old male who is known to have chronic obstructive pulmonary disease presents with increasing breathlessness and cough productive of purulent sputum .On examination you note that he is breathless, tachycardic and auscultation of his chest reveals widespread rhonchi and crepitations. Blood gases show pH 7.32, PaCO2 8.9 kPa, PaO2 6.8 kPa, Bicarbonate 37 mmol/L In this patient which one of the following states would best describe his condition? a) Metabolic acidosis with respiratory compensation b) Respiratory acidosis with metabolic compensation c) Laboratory error d) Metabolic alkalosis with respiratory compensation e) Respiratory alkalosis with metabolic compensation Answer :b) The patient is acidotic. The high PaCO2 would suggest that the cause is respiratory. The increase in bicarbonate would suggest the kidneys are conserving bicarbonate to compensate for the acidosis

219

Mastering MRCP BOF: 2.229 A 76 year old male is being investigated for breathlessness due to probable valvular heart disease. Investigations are as follows: Chamber Right atrium Right ventricle

Oxygen saturation 64 66

Pressure in mm Hg% 6 24/5

Pulmonary artery Left ventricle Aorta

66 98 98

24/16 220/20 220/60

In this patient which physical sign are you likely to elicit in this patient in addition to the murmur heard due to the valvular lesion? a) Mid diastolic murmur b) Pan systolic murmur c) Loud second heart sound d) Fourth heart sound e) Diastolic knock Answer: a) The patient has aortic regurgitation. This can be concluded from the wide pulse pressure demonstrated. The patient would have a blowing early diastolic murmur at the left sternal edge due to this valvular lesion. In addition to this mid diastolic murmur may be heard in the mitral area in aortic regurgitation. This is known as the Austin Flint murmur and is caused by the regurgitant jet of blood pushing the anterior mitral valve leaflet backwards resulting in functional mitral stenosis

BOF: 2.230 A 65 year old female presents with a history of breathlessness on exertion and malaise. She has Parkinsonism and was on treatment for this but was unable to name the drugs she was on. On clinical examination she was noted to be pale and had features of Parkinsonism detected on examination of her nervous system. Investigations Hb 8.5 g/dL WBC 7. 5 x109/L Platelets 450 x109/L MCV 109 fl Reticulocyte count 8% In this patient which one of the following drugs is the most likely cause of her anaemia? a) Laevodopa b) Benzhexol c) Oprphenadrine d) Amantadine e) Lisuride Answer: a)

220

Mastering MRCP The patient has a haemolytic anaemia the evidence for this being the high reticulocyte count and the high MCV (the high MCV is due to immature red cells being larger than mature red cells. In haemolysis rapid production of red cells would mean that more immature red cells are released into the circulation. In addition if haemolysis continues increased demand could cause secondary folic acid deficiency. ) The patient has Parkinsonism and is likely to be on laevodopa which can cause a Coomb’s positive haemolytic anaemia

BOF: 2.231 A 34 year old male presents with a history of dull headache of a few weeks duration. In addition he had felt generally unwell with mild fever and lack of appetite. No past illnesses of note, he was not on any drugs. On examination you note that he is ataxic and there is a sixth nerve palsy on the right side. He has a CT scan which does not reveal a space occupying lesion and this is followed by a lumbar puncture CSF Opening pressure 300 mm H2O Protein 2.2 g/L Cells 60 /mL (90% lymphocytes) Glucose 0.5 mmol/L Blood Hb 13.5 g/dL WBC 11 x109/L Glucose 6.0 mmol/L In this patient what is the most likely diagnosis? a) Viral meningitis b) Viral encephalitis c) Bacterial meningitis d) Tuberculous meningitis e) Froin’s syndrome Answer: d) The patient has increased CSF pressure, a high lymphocyte count and low CSF glucose in relation to plasma glucose. This would suggest tuberculous meningitis. Froin's syndrome CSF with High protein content Xanthochromia Normal cell count Sign of blockage of CSF flow in the spine

BOF: 2.232 A 63 year old male who is known to be hepatitis B positive is admitted with a history of abdominal discomfort and distension. Apart from infection with hepatitis B the patient had no significant past illnesses. He was not on any drugs. On examination he was deeply jaundiced, there were multiple spider naevi on his upper chest, there was palmar erythema and he had a flapping tremor.

221

Mastering MRCP Examination of his abdomen revealed hepatomegaly 4 fingerbreadths, there was a hepatic bruit and he had significant ascites. Investigations Hb 12.5 g/dL WBC 7.2 x109/L Platelets 65 Bilirubin 300 µmol/L ALT 400 U/L ALP 350 U/L Albumin 32 g/L In this patient which one of the following is the most likely cause of the deterioration in his condition? a) Bleeding oesophageal varices b) Hepatoma c) Electrolyte imbalance d) Constipation e) Sedative drugs Answer: b) The patient has been admitted with deteriorating liver function on a background of chronic hepatitis B and clinical features of cirrhosis of the liver ACES for PACES page 305-306. There are many causes to account for his but the clue here is that the patient has a hepatic bruit. There are several causes of a hepatic bruit (see ACES for PACES page304) but in a patient who is known to have hepatitis B the possibility of hepatoma is the most likely There is nothing in the clinical scenario to support any of the other causes of decompensation of his condition

BOF: 2.233 The coronary sinus opens into the right atrium in the following position: a) Right atrial appendage b) To the left of the opening of the superior vena cava c) To the left of the opening of the inferior vena cava d) To the right of the fossa ovalis e) Above the anterior cusp of the tricuspid valve Answer: c) The coronary sinus lies in posterior part of the atrioventricular groove and opens into the posterior wall of the right atrium to the left of the opening of the inferior vena cava and above the septal cusp of the tricuspid valve

BOF: 2.234 A 42 year old male lifted a heavy object and felt severe pain in his right arm. He presents the next day with continuing pain. On examination you note a rounded swelling in the middle of his upper arm, anterior aspect. Further examination reveals failure to flex his elbow and failure to supinate the forearm The most likely diagnosis is: a) Bursitis

222

Mastering MRCP b) Rotator cuff syndrome c) Bicipital tendonitis d) Hematoma e) Ruptured biceps tendon Answer :e) Rupture of the proximal biceps tendon comprises 90-97% of all biceps ruptures Almost exclusively involves the long head Remainder occur distally at the insertion on the radial tuberosity Even less commonly, at the short-head insertion on the acromion Sudden pain Associated with an audible snap in the area of their shoulder Ruptured tendon retracts causing a bulge in their arm at the biceps muscle (retracted muscle bunched up in the arm)

BOF: 2.235 A 43 year old female who has ileocaecal Crohn’s disease attends the gastroenterology clinic and has blood tests done. Hb 31.5 g/dL RBC 4.32 x1012/L WBC 1.9 x109/L Platelets 88 x109/L The house officer looking through the results rings you to ask what he should do about them. You ask the house officer what medication the patient is on . Which one of the following would account for these results? a) Sulphasalazine b) Mesalazine c) Prednisolone d) Azathioprine e) Ciprofloxacin Answer: d) Immunosuppression with Azathioprine is regularly used to maintain remission in Crohn’s disease Azathioprine may be associated with myelosuppression

BOF: 2.236 A 22 year old male presented with a history of hypersalivation there was no significant past medical history but he remarked that his brother had liver disease and was on long term treatment for this Which of the following drugs is his brother likely to be taking? a) Desferrioxamine b) Ursodeoxycholic acid c) Ribavirin d) Azathioprine e) Penicillamine Answer :e)

223

Mastering MRCP Hypersalivation is a less common neurological manifestation of Wilson’s disease. Young adults are more likely to present with neurological manifestations of Wilson’s disease. Wilson’s disease is hereditary and the brother is likely to be on treatment for this with penicillamine

BOF: 2.237 A 56 year old male is admitted with loss of consciousness. He had no significant past illnesses. He is found to be dehydrated and hypertensive and investigations confirm a hyperosmolar non ketotic state. He is managed with intravenous fluids and low does insulin infusion and makes a gradual recovery. Following recovery he complains of blurring of vision. The most likely cause of his visual impairment is: a) Diabetic retinopathy b) Central retinal artery occlusion c) Osmotic changes in the lens d) Retinal detachment e) Central retinal venous occlusion Answer c) In diabetes mellitus visual blurring may develop as the lens changes shape with marked changes in blood glucose concentrations This is caused by osmotic fluxes of water into and out of the lens Usually occurs as hyperglycemia increases It also may be seen when high glucose levels are lowered rapidly Recovery to baseline visual acuity can take up to a month

BOF: 2.238 A 26 year old pregnant female presents has been referred by the obstetric registrar as he is worried about the skin lesions he has noticed on the patient. The patient has axillary freckles and multiple flesh colored papules distributed in a generalised fashion The likely diagnosis is: a) Acanthosis nigricans b) Melanoma c) Simple naevi d) Tuberose sclerosis e) Neurofibromatosis Answer: e) Axillary freckling in association with multiple flesh colored papules would suggest the patient has neurofibromatosis type I Revision Tip

BOF: 2.239 An 18 year old male presents with nephrotic syndrome. Renal biopsy is performed and this shows minimal change glomerular lesion. In this patient which one of the following would be useful in treating proteinuria?

224

Mastering MRCP a) High protein diet b) ACE inhibitors c) Angiotensin 2 receptor blockers d) Furosemide e) Prednisolone Answer: e) Minimal change glomerular lesion accounts for most cases of childhood nephrotic syndrome and about a quarter of adult nephrotic syndrome First line treatment is with steroids in high dose

BOF: 2.240 A 62 year old female presents with violent flinging involuntary movement of her right side of acute onset In this patient where is the site of the lesion? a) Caudate nucleus b) Subthalamic nucleus c) Red nucleus d) Substantia nigra e) Globus pallidus Answer: b) Ballism refers to violent flinging movements of the limbs . it is caused by a lesion in the subthalamic nucleus and is usually unilateral ; hemiballism

BOF: 2.241 A 56 year old patient, who is known to have rheumatoid arthritis and has been on steroids, presents with a history of pain in her left hip. On examination you note decreased range of movement of the hip joint. In this patient which one of the following investigations would give you the earliest indication of avascular necrosis of the hip? a) Plain X-ray b) CT scan c) MRI d) Bone scan e) Alkaline phosphatase Answer: c) MRI is sensitive, specific and may be used for very early diagnosis

BOF: 2.242 A 53 year old female presents with fever. There is no past history of note and she is not on any drugs. On examination you note that she is confused and that there is neck stiffness and Kernig’s sign is positive. A lumbar puncture is performed and the CSF shows Protein 0.6 g/L Cells 85 /mL-1

225

Mastering MRCP Neutrophils 90% Glucose 1.9 mmol/L (blood glucose 6.2 mmol/L) In this patient which one of the following would be the most likely causative organism? a) Streptococcus pneumoniae b) Staphylococcus aureus c) Streptococcus pyogenes d) Mycobacterium tuberculosis e) Listeria monocytogenes Answer :a) The clinical features suggest meningitis and the investigations point to bacterial meningitis. Streptococcus pneumoniae (pneumococcus) is the most common cause of bacterial meningitis is adults

BOF: 2.243 With which of the following substances does heparin bind? a) Antithrombin III b) Protein C c) Protein S d) Activated factor XII e) Activated factor X Answer: a) Heparin binds to Antithrombin III Heparin-AT complex inactivates a number of coagulation enzymes: Thrombin factor (IIa) Xa IXa XI XIIa

BOF: 2.244 A 63 year old female developed diarrhoea shortly after being treated with antibiotics for community acquired pneumonia. Stools were positive for Clostridium difficile toxin and she was treated with oral metronidazole for 2 weeks. She made a satisfactory recovery. 2 weeks later she presents with diarrhoea and stools are once more positive for Clostridium difficile toxin In this patient which one of the following would you use as treatment? a) Oral vancomycin b) Intravenous vancomycin c) Metronidazole d) Clindamycin e) Cholestyramine Answer :c) The patient has had a recurrence of Clostridium difficile induced diarrhoea which is quite common Recurrences are treated with a repeat course of oral metronidazole

226

Mastering MRCP BOF: 2.245 A 45 year old male is admitted with a left sided stroke. His BP is 180/120. ECG shows evidence of left ventricular hypertrophy. The CT scan shows an infarct in the right parietal region. In this patient when would you commence antihypertensive medication? a) Immediately b) 24 hours c) 48 hours d) 72 hours e) 1 week Answer: d) Following stroke antihypertensive medication is not commenced within 72 hours unless there is a hemorrhagic stroke or there is evidence of hypertensive end organ damage such as encephalopathy, pulmonary oedema, renal failure, dissection, acute coronary syndrome

BOF: 2.246 A 58 year old male presents with deafness. He is investigated further and is diagnosed as having nasopharyngeal carcinoma, Which one of the following viruses is implicated in the pathogenesis of nasopharyngeal cancer? a) Herpes Simplex Virus b) Human Papilloma Virus c) Retrovirus d) Paramyxovirus e) Flavivirus Answer :b) Human Papilloma Virus (HPV) Known to cause cervical cancer Implicated in the development of some cancers of the oral cavity, pharynx and larynx 30-40% of cases of oropharyngeal cancer the west Cancer of the tonsil particularly likely to be due to HPV infection HPV positive tumours better prognosis than those associated with smoking and alcohol

BOF: 2.247 A 32 year old male presents with abdominal pain and loose motions. Stools examination reveals ova of Strongyloides stercoralis Which of the following may be used in treatment of this condition? a) Praziquantel b) Albendazole c) Metronidazole d) Tinidazole e) Diethylcarbamazine Answer: b) Strongyloides stercoralis is a parasite that causes intestinal infections Treatment is with ivermectin, albendazole, and thiabendazole

227

Mastering MRCP

BOF: 2.248 A 47 year old female presents with a history of breathlessness, constipation and abdominal swelling. There is no significant past medical history and she is not on any medication. On examination you note that the patient looks thin, pulse 90 beats per minute regular, JVP elevated and the JVP is increased during inspiration. There is ascites. A chest X-ray is performed and this shows pericardial calcification. In this patient what is the likely underlying diagnosis? a) Hyperparathyroidism b) Hypoparathyroidism c) Hypothyroidism d) Rheumatic fever e) Tuberculosis Answer: e) The clinical features and X-ray findings would suggest the patient has constrictive pericarditis. There are many causes of constrictive pericarditis but out of the causes listed the most likely is tuberculosis

BOF: 2.249 A 56 year old female presents with a history of chronic diarrhoea. Colonoscopy reveals pigmentation of the colonic mucosa. Biopsies are taken and they show pigment laden macrophages in the lamina propria In this patient what is the likely diagnosis? a) Peutz-Jegher’s syndrome b) Colonic melanoma c) Colonic carcinoma d) Laxative abuse e) Collagenous colitis Answer :d) The patient has melanosis coli Melanosis Coli Deposition of a brown black pigment (lipofuscin) in the lamina propria of the colon Mucosal cell death or apoptosis resulting from laxative abuse (commonly anthraquinone) These cells are then phagocytosed by macrophages in lamina propria producing lipofuscin This gives a dark colour to the colonic mucosa

BOF: 2.250 A 32 year old male presents with a history of fever and cough. There is no significant past medical history and he is not on any drugs. He does not smoke cigarettes. On examination you note that he is febrile, he has herpes labialis, pulse rate 100 beats per minute. On examination of his chest you detect signs of consolidation of the right base of his lung. In this patient what is the most likely organism causing his condition? a) Pneumococcus b) Mycoplasma c) Haemophilus

228

Mastering MRCP d) Klebsiella e) Pneumocystis Answer: a) The patient has herpes labialis and pneumonia Herpes labialis is most commonly associated with pneumococcal pneumonia

BOF: 3. 1 A 30-year-old female presents with diplopia and weakness of her upper limbs that increases towards evening. On examination there is demonstrable fatiguability, reflexes are normal. There is no sensory loss. The patient is treated with pyridostigmine and there is resolution of her symptoms. She subsequently presents with increasing weakness that has begun to involve her limbs and she also has involvement of the muscles of respiration. There are no fasiculations, pulse rate is 90 beats per minute, pupils normal in size, equal and react to light and accommodation. In this situation the best mode of treatment would be with: a) Steroids b) Steroids and azathioprine c) Atropine d) Edrophonium e) Plasma exchange Answer: e) The clinical features suggest the patient has developed a myasthenic crisis. There are no cholinergic features. In this life-threatening situation plasma exchange is extremely helpful as an immediate measure. It is thought to work by removing circulating acetylcholine receptor antibody

BOF: 3.2 A 35-year-old female presents with difficulty in chewing her food, dysarthria, dysphonia, dysphagia and regurgitation of food through her nose. On examination there is ptosis on sustained upward gaze. This condition may be induced by: a) Suxamethonium b) Beta-blockers c) Aminoglycoside antibiotics d) Penicillamine

229

Mastering MRCP e) Anticonvulsants Answer: d) Penicillamine may occasionally induce myasthenia gravis. The other drugs worsen myasthenia gravis

BOF: 3.3 A 60 year old male who has been smoking 20 cigarettes a day for the last 40 years, presents with fatiguable weakness of the proximal limb muscles and trunk. There is no ptosis. He complains of a dry mouth, he has problems with micturition and he has erectile failure. In this patient the tendon reflexes would be: a) Absent and cannot be reinforced by brief maintained isometric contraction b) Absent but may be reinforced by brief maintained isometric contraction c) Exaggerated d) Normal e) Pendular Answer: b) The clinical features suggest the patient has Eaton-Lambert syndrome. In this condition the tendon reflexes are usually absent but can be reinforced by brief sustained isometric contraction. This is called post-tetanic potentiation and is an useful sign in making the diagnosis

BOF: 3.4 A 38-year-old male presented initially with impaired vision of 6 hours duration. He had been bumping into objects, had been unable to see complete faces and had difficulty reading. No history of headache, neck pain or trauma. He gave a history of attacks of migraine with aura and had sensorineural deafness. His mother aged 65 has non-insulin dependent diabetes mellitus and his father has ischaemic heart disease. He was not homosexual. On examination the positive findings were he was shorter than average, had sensorineural deafness and had a left homonymous hemianopia. There was no abnormality on examination of the cardiovascular system. No other positive findings. Investigations showed normal haematology and biochemistry, coagulation screen was normal, ECG, transthoracic echocardiogram, chest x-ray were normal. CT scan of his head showed low-density changes in the right parieto-occipital region. His hemianopia resolved within 72 hours, he was treated with aspirin and discharged. 8 months later he was re-admitted with a right-sided weakness and right homonymous hemianopia. MRI showed ischaemic changes in the both parieto-occipital regions and ischaemic changes in the left temperoparietal region. Which of the following would confirm your diagnosis? a) Transoesophageal echocardiography b) Homocysteine levels in blood c) Visual evoked potentials d) Lumbar puncture e) Muscle biopsy Answer: e)

230

Mastering MRCP The patient has MELAS (myopathy, encephalopathy, lactic acidosis, stroke like episodes). Muscle biopsy showed ragged red, succinate dehydrogenase positive and cytochrome oxidase positive and negative fibers. The diagnosis was confirmed by genetic analysis. MELAS is a mitochondrial disorder, which results in defective oxidative phosphorylation. The features are short stature, limb myopathy, seizures, episodes of lactic acidosis and stroke like episodes. The strokes do not usually conform to vascular territories and the parieto-occipital lobes are commonly affected. Patients have high lactate/pyruvate levels in blood and CSF. Other features are non-insulin dependent diabetes mellitus and sensori-neural deafness. Migraine with aura is also a feature

BOF: 3.5 A 40-year-old female had neck manipulation by a chiropractor for treatment of migraine. She presents shortly after with occipital headache, pain in the neck, pain and numbness in the right side of her face, hoarse voice, numbness of the right limbs and right side of her trunk, vertigo and difficulty in maintaining balance. On examination, a right-sided Horner's syndrome was noted. There was nystagmus (fast phase to the right), sensorineural hearing loss in the right ear and paresis of the soft palate on the right hand side. There was reduced tone and power in the upper and lower limbs on the right. Deep tendon reflexes were brisk on the right side and plantar response was extensor. Sensory examination revealed a crossed hemianaesthesia with involvement of the face on the right side. The likely lesion is dissection of the: a) Vertebral artery b) Carotid artery c) Basilar artery d) Anterior spinal artery e) Posterior spinal artery Answer: a) Vertebral artery dissection is a recognised cause of stroke in patients under 45 years of age. The primary lesion is an expanding haematoma in the vessel wall. Risk factors are: Spinal manipulation, judo, nose blowing, painting ceilings, yoga, neck trauma, intrinsic disease of the blood vessels, hypertension, oral contraceptive use, female sex

BOF: 3.6 A 78-year-old female patient presents with sudden onset violent, flinging movements involving the right side of her body. On examination she is conscious and alert, there is no speech defect and her vision is intact. The limbs are difficult to examine because of the abnormal movements, which are present at rest, but there does not appear to be an impairment of tone, power or reflexes. The site of the neurological lesion is likely to be: a) Red nucleus b) Substantia nigra c) Amygdala d) Subthalamic nucleus e) Globus pallidus

231

Mastering MRCP Answer: d) The patient has hemi-ballism, which is caused by a lesion in the subthalamic nucleus

BOF: 3.7 A 40-year-old male who is a heavy smoker and known to have emphysema and has had previous pneumothoraces is referred to the liver clinic for further investigation of abnormal liver function tests. On examination he has clinical evidence of cirrhosis of the liver. There is no history of neonatal jaundice. In this patient: a) The lack of a history of neonatal jaundice excludes alpha1 antitrypsin deficiency b) Hepatocellular carcinoma is a recognised complication of this condition c) Presentation in infancy is uncommon d) It is the MZ heterozygotes who develop cirrhosis as the others die early e) The neonatal hepatitis syndrome that occurs in this condition is invariably fatal Answer: b) Alpha 1 antitrypsin is a protease inhibitor. The normal phenotype is MM. The clinically important variant is designated Z and it is the ZZ homozygotes who develop disease. Liver disease usually presents in childhood with the neonatal hepatitis syndrome (conjugated hyperbilirubinaemia with pale stools dark urine and hepatomegaly). Those who survive (25% die in early childhood) usually undergo a long clinical course. Some paints may present with cirrhosis with no history of jaundice in infancy. In these patients primary hepatoma is a recognised complication

BOF: 3.8 A 78-year-old female is admitted with a history of fever, anorexia, malaise and weight loss. She looks thin, has a tachycardia and a soft systolic murmur at the apex. No other abnormalities are detectable. Investigations reveal a polymorphonuclear leucocytosis, an elevated ESR and a high serum alkaline phosphatase. Blood cultures show growth of Streptococcus milleri In this patient the definitive investigation would be: a) Abdominal ultrasound scan b) Transoesophageal echocardiogram c) ERCP d) Colonoscopy e) CT Chest Answer: a) Streptococcus milleri is a common infecting organism in pyogenic liver abscess, which is now more common in elderly patients as a complication of biliary tract disease or diverticulitis. It usually presents with abdominal pain, swinging fever, anorexia, malaise weight loss and hepatomegaly but, in elderly patients abdominal pain and hepatomegaly may not feature. Abdominal ultrasound will usually reveal the diagnosis and may be used to aspirate the abscess

232

Mastering MRCP BOF: 3.9 A 65-year-old male presents with a wound infection complicating repair of an inguinal hernia. The patient is septic with a neutrophil leucocytosis, normal haemoglobin and normal reticulocyte count and platelet count. The patient’s serum bilirubin is raised but serum transaminases and alkaline phosphatase are normal. This condition is due to: a) Cholestasis associated with severe sepsis b) Shocked liver c) Pyogenic liver abscess d) Enlarged lymph nodes at the porta-hepatis e) Haemolysis Answer: a) Cholestasis may occur in severe sepsis and is often associated with normal transaminases and alkaline phosphatase. The pathogenesis is unknown but gross liver damage does not occur. When treated with the appropriate antibiotics, rapid resolution occurs

BOF: 3.10 A 62-year-old male who has had a right hemicolectomy for colon cancer presents with malaise, weight loss, abdominal pain and distension of the abdomen. On examination there is evidence of weight loss and the liver is enlarged, hard in consistency and has a craggy edge and irregular surface. In this patient, which of the following features would indicate a poor prognosis? a) Elevated alkaline phosphatase b) Jaundice c) Malignant tissue on liver biopsy d) Elevated transaminases e) Ultrasound evidence of metastases Answer: b) The patient has liver metastases. In this condition jaundice is usually a late occurrence and indicates a poor prognosis

BOF: 3.11 A 78-year-old male who is known to have ischaemic heart disease is admitted with a history of severe sudden onset abdominal pain, which woke him up, from his sleep. This was followed by diarrhoea, which later became bloody. On examination of the patient’s abdomen there was mild peritonism. Plain x-ray of the abdomen showed thumb printing and a dilated colon. Which of the following areas of the colon is most likely to be affected by this condition? a) Caecum b) Ascending colon c) Hepatic flexure d) Transverse colon e) Splenic flexure Answer: e)

233

Mastering MRCP The patient has the typical clinical and radiological features of ischaemic colitis, which usually affects the watershed areas (splenic flexure and rectosigmoid area). Rarely the ascending colon and proximal transverse may be affected by ischaemia of the superior mesenteric artery

BOF: 3. 12 A 35-year-old female has had a right hemicolectomy and ileal resection for Crohn’s disease. She presents with diarrhoea. Investigations do not reveal recurrence of Crohn’s disease. A 75 SeHCAT scan shows low values. In this patient the diarrhoea is best managed by treating with: a) Codeine phosphate b) Budesonide c) Loperamide d) Cholestyramine e) Azathioprine Answer: d) Ileal resection causes bile acid malabsorption, which results in diarrhoea because the unabsorbed bile acids have a secretory effect on colonic epithelium. The 75 SeHCAT scan measures retention at 7 days of orally administered 75 Se labelled homocholic acid conjugated with taurine. Low values indicate bile acid malabsorption (BAM). Treatment is with bile acid sequestrants such as cholestyramine

BOF: 3.13 A 65-year-old male presents with a history of headaches and visual disturbance. On examination he has a distinctive facies with prominent supraorbital ridges, a large nose and prognathos. He has thickening of his skin and soft tissues. His hands are large and spade like and his feet too are large. On examination of his visual fields he is noted to have a bitemporal hemianopia. In this patient daytime somnolence and sleep apnoea could occur due to: a) Macroglossia b) Prognathos c) Thoracic kyphosis d) Enlargement of the paranasal sinuses e) Goiter Answer: a) In acromegaly macroglossia may lead to sleep apnoea and daytime somnolence. All the other features may occur in acromegaly but do not result in sleep apnoea

BOF: 3.14 A 29-year-old female presents for investigation of sub fertility. She has amenorrhoea following withdrawal of the oral contraceptive pill. She also complains of decreased libido. She has galactorrhoea, headaches and visual disturbance. In this patient which of the following features would suggest she has a macroadenoma? a) Amenorrhoea b) Galactorrhoea c) Subfertility

234

Mastering MRCP d) Decreased libido e) Headache Answer: e) The features suggest the patient has a prolactinoma. Local pressure effects such as headache, visual field defects and cranial nerve palsies suggest the patient has a macroprolactinoma and these features are often present at diagnosis

BOF: 3.15 A 32-year-old female presents with a history of tremor and heat intolerance of 3 months duration. She has lost weight and felt more tired than usual. Her periods were regular. She had episodes where she felt her heart racing. No serious past illnesses. There was family history of hypothyroidism. She smokes 10 cigarettes a day and takes alcohol occasionally. On examination she was thin, had a fine tremor of her hands pulse 110 beats per minute sinus rhythm. She had diffuse enlargement of her thyroid gland but had no eye signs. Reflexes were symmetrical and brisk. After being told the diagnosis she says she is planning to become pregnant and asks what effect pregnancy would have on this condition. Your reply would be that this condition: a) Improves during pregnancy but relapses in the puerperium b) Improves during pregnancy and the puerperium c) Deteriorates during pregnancy but improves during the puerperium d) Deteriorates during pregnancy and the puerperium e) Pregnancy has no effect on the condition Answer: a) Like most autoimmune conditions Grave’s disease often enters remission during pregnancy but relapses during the puerperium

BOF: 3.16 A 39-year-old Asian male is admitted with a history of vomiting and abdominal pain. The patient has been unwell and anorexic for some time prior to admission. On examination the patient looks unwell, is febrile, tachycardic and hypotensive. The skin is pigmented and lacks turgor and the eyes look sunken. The buccal mucosa, and skin creases are pigmented. The abdomen feels rigid. Investigations reveal an elevated blood urea with low serum sodium and high serum potassium. There is hypercalcaemia and hypoglycemia. The feature that will help to differentiate primary glandular failure from secondary glandular failure is: a) Hyperkalaemia b) Elevated eosinophil count c) Anaemia d) Fever e) Hypoglycaemia Answer: a) The patient has adrenal insufficiency. The features of mineralocorticoid deficiency (sodium and water depletion, hyperkalaemia and Hypercalcaemia) are usually due to primary adrenal failure and are absent or less marked in secondary adrenal failure

235

Mastering MRCP BOF: 3.17 A 30-year-old female who has had a recent pregnancy where labour was complicated by postpartum haemorrhage had failure of lactation and menstruation did not recommence. She feels giddy on standing and has postural hypotension. Which of the following features indicates pituitary insufficiency rather than adrenal insufficiency? a) Hyperkalaemia b) Low sodium c) Hypercalcaemia d) Pigmentation e) Hypoglycaemia Answer: e) Hypoglycaemia in adrenal insufficiency is due to glucocorticoid deficiency rather than mineralocorticoid deficiency, which is more common in primary adrenal failure. In addition growth hormone lack in pituitary insufficiency makes hypoglycemia more marked

BOF: 3.18 Which of the following thyroid cancers is more common in areas of high iodine intake? a) Papillary b) Follicular c) Anaplastic d) Medullary e) Lymphoma Answer: a) Over two thirds of thread cancers are papillary carcinomas. They are more common in areas of high iodine intake

BOF: 3.19 A 75-year-old male has been admitted to the emergency admitting unit. He has ischaemic heart disease with previous myocardial infarction and has been on treatment for chronic heart failure. He is on furosemide, ramipril, spironolactone and carvedilol. He recently developed pain, redness and swelling of his right metatarsophalangeal joint which came on acutely overnight. His GP initially treated him with diclofenac but when he did not respond he started him on colchicine. Following this he developed diarrhoea, which made him feel unwell. He stopped eating and drinking and took to his bed. The GP referred him to the hospital, as he was worried about the deterioration in his condition. On examination he looked unwell, his mucus membranes were dry, skin turgor was diminished and he was tachycardic and had low blood pressure with a significant postural drop. There was a pan systolic murmur at the apex, which radiated to the axilla but apart from this no other abnormalities were detected. Your house officer has arranged blood tests, the results are not available as yet but he calls you to see the patient’s ECG as he thinks it is abnormal. The ECG shows bradycardia, flattened p waves and a broad QRS complex. Your next step would be:

236

Mastering MRCP a) Urgent blood gas analysis b) 10 % Calcium chloride 10 ml over 5 minutes c) 50 % dextrose 50 ml with 10 units soluble insulin over 5 minutes d) Nebulised salbutamol 20 mg e) 1.26 % Sodium bicarbonate 500ml over 30-60 mins Answer: b) The patient is an elderly man with ischaemic heart disease and heart failure. He is probably a vasculopath. He is on multiple potentially nephrotoxic drugs. He has gout (sudden onset pain redness and swelling of his metatarsophalangeal joint) and this together with his vascular disease increases his risk of renal disease. In addition he has been given a NSAID, which would further increase the chances of damaging his kidneys. Following administration of colchicine he has developed diarrhoea and clinically there are features of salt and water depletion (“dehydration”). This raises the possibility of pre-renal renal failure. The possibility of developing renal failure has been brought up by a number of features of this case. Following this the ECG features are mentioned. These features suggest the patient has hyperkalaemia. When hyperkalaemia results in ECG changes it is imperative that quick action is taken. Treatment of hyperkalaemia, which is a life threatening condition, may be divided into 3 stages: ü Reduce the risk of an arrhythmia This may be achieved by the use of intravenous calcium chloride or calcium gluconate. Calcium chloride contains more calcium (Calcium chloride 6.8 mmol in 10ml, calcium gluconate 2.2 mmol in 10 ml) and hence is the preferred agent. ü Drive potassium into cells This may be achieved by the use of insulin and dextrose or nebulised salbutamol ü Remove potassium from the body This may be achieved by the use of calcium exchange resins such as calcium resonium or by the use of dialysis

BOF: 3.20 A 46-year-old Caucasian female has been referred by her GP for investigation of abnormal liver function tests. She has been found to have an elevated ALT. All other biochemical indices of liver function are normal. She is known to have type 2 diabetes mellitus (NIDDM). She takes 8 units of alcohol per week; she is not on any drugs. She is obese; height 1.60 meters weight 110 kgs. Her waist circumference is 90 cms. Her BP is 140/90. No other abnormalities are detectable clinically. She has had an ultrasound scan which is consistent with steatosis. She is negative for hepatitis B and C viruses; she is negative for smooth muscle antibodies and antimitochondrial antibodies and has normal levels of ferritin, caeruloplasmin and alpha 1 antitrypsin. Her GP wishes to start her on a statin and asks you whether it is safe to do so. Your reply is: a) The patient should have a liver biopsy first and depending on liver histology a decision may be made. b) Statins should not be used until liver biochemistry returns to within normal limits c) There is no evidence that this type of patient is at a higher risk of statin induced hepatotoxicity and statins may be used d) Statins should not be used until the patient has reduced her body weight to within normal limits e) Cardiovascular risk is not increased in these patients and hence statins are not required Answer: c)

237

Mastering MRCP The patient has abnormal liver function tests with the main abnormality being raised enzymes. She does not take alcohol in excess and is not on any drugs thus these are not aetiological agents. She is obese with abdominal obesity (waist circumference >80 cms in a Caucasian female), she has hypertension and type 2 diabetes mellitus. Thus she has features of the metabolic syndrome. The most likely cause of the raised enzymes is non-alcoholic fatty liver disease (NAFLD). The ultrasound findings together with the negative liver screen and the risk factors for NAFLD should be enough evidence to point to this diagnosis without the need for a liver biopsy. In patients with the metabolic syndrome (see later) the risk of cardiovascular death is increased and if required treatment with statins should be given, as there is no evidence that these patients are at increased risk of statin induced hepatotoxicity. Metabolic Syndrome Abdominal obesity (waist > 90 cms male > 80 cms female-Caucasian) Together with 2 or more of the following § Hypertension § Abnormal glucose tolerance § Hypertriglyceridaemia § Low LDL cholesterol (low density lipoprotein cholesterol)

BOF: 3.21 A 35-year-old male is seen in the follow up clinic. He has an inherited condition for which he attends yearly follow up. On examination, he has 8 café-au-lait macules of over 15 mm in greatest diameter on his trunk, he has multiple discrete skin nodules distributed over his trunk and limbs and there is freckling in the axillae. The patient is worried about his child; a boy of 6. He was told that examination of the child could conclusively exclude inheritance of this condition in the child but he has forgotten what the exclusion criteria were. He asks you to reiterate them and your reply is that this condition may be confidently excluded if: a) The child does not have any café-au-lait macules b) If the child has fewer than 6 café-au-lait spots c) If the child has no axillary freckling d) If the child has no skin nodules e) If no member of the mother’s family have been affected Answer: a) This patient has an inherited condition. There are several conditions where localised hyperpigmentation is associated with systemic disease. Multiple café-au-lait spots are seen in neurofibromatosis Skin nodules are seen in many conditions; neurofibromatosis is one of them. Axillary freckling too is a feature of neurofibromatosis NF1 The diagnosis of neurofibromatosis type 1 (NF1) may be made in the presence of 2 or more of the following features: · Six or more café-au-lait spots of more than 5 mm in greatest diameter in pre-pubertal individuals and more than 15 mm in greatest diameter in post-pubertal individuals. · 2 or more peripheral neurofibroma or one plexiform neurofibroma · Freckling in the axillary or inguinal regions

238

Mastering MRCP · Optic glioma · 2 or more iris hamartoma (Lisch nodules) · An osseous lesion such as sphenoid dysplasia or thinning of the long bone cortex with or without pseudoarthrosis · A first degree relative (parent, sibling or offspring) with NF1 Peripheral neurofibroma may be skin nodules (dermal neurofibroma) or nodular neurofibroma on major peripheral nerve trunks. Neurofibromatosis is inherited as an autosomal dominant condition. Inheritance is 100 % and in children of affected parents, the absence of café-au-lait spots by the age of 5 excludes the condition. Rare families have less than six café-au-lait spots and 10% of the general population have one or two spots hence having less than 6 spots does not exclude it. Dermal neurofibromas are unusual until the late teens or early twenties and hence their absence does not exclude this condition. Axillary freckling usually develops after age 7 As it is an autosomal dominant condition and one parent is affected the other parent need not be affected

BOF: 3. 22 A 27-year-old female has been referred by her GP as he suspects she has chronic fatigue syndrome. On examination you note that she has a bilateral ptosis. There is frontal balding and wasting of the facial muscles. There is wasting of the masseter and temporalis muscles. The sternocleidomastoid muscles are weak bilaterally. On examination of her limbs there is grip myotonia and percussion myotonia demonstrable in the thenar eminence. There is weakness of the distal limb muscles. The patient wishes to know what the chances are of this condition being transmitted to her children. Your reply would be: a) 0% b) 25% c) 50% d) 75% e) 100% Answer: c) The patient has bilateral ptosis which has multiple causes ACES for PACES Page 480 Wasting of the facial muscles is seen in dystrophia myotonica ACES for PACES Page 490 Wasting of the temporalis and masseters may occur in a few conditions, one of them being myotonic dystrophy Bilateral weakness of the sternocleidomastoids may occur in a few conditions one of them being dystrophia myotonica Grip myotonia and percussion myotonia occur in dystrophia myotonica Myotonic dystrophy usually affects the distal muscles of the limbs ACES for There are a few causes of myotonia In association with wasting the diagnosis is myotonic dystrophy. The fact that the patient has frontal balding is further evidence in favor of this.

239

Mastering MRCP Myotonic dystrophy is inherited as an autosomal dominant condition hence there is a 50% chance of a child being affected if a parent has the condition. Myotonic dystrophy is a multisystem disorder. The other systems that may be affected are: CVS conduction abnormalities, arrhythmias GIT dysphagia, delayed gastric emptying, bile acid malabsorption, small bowel bacterial overgrowth, raised liver enzymes, gallstones RS nocturnal hypoventilation E&M diabetes mellitus RAG testicular atrophy, male subfertility IS frontal balding EYES cataracts

BOF: 3.23 A 35-year-old female presented with a history of lethargy and jaundice. She also complained of joint pains and mild pain in the right upper quadrant. She did not drink and did not take any medications including over the counter preparations or herbal remedies. On examination the patient was not obese, she had mild jaundice and the liver was enlarged 3 fingerbreadths below the costal margin and was tender. Investigations revealed normal haematology raised ALT, ALP and bilirubin. Gammaglobulin levels were raised and she was positive for ANA and SMA antibodies. She was negative for hepatitis B and C viruses and screening for haemochromatosis, Wilson’s disease and alpha antitrypsin deficiency was negative. Ultrasound scanning did not reveal a focal lesion. Liver biopsy was performed and this showed a lymphoplasmacellular infiltrate in the portal tracts with necrosis affecting the limiting plate. In this patient: a) Selective elevation of IgG G antibody is typical b) Selective elevation of IgA A antibody is typical c) Selective elevation of IgM M antibody is typical d) Positive anti SMA is diagnostic of this condition e) Positive anti LKM antibody is diagnostic of this condition Answer: a) The clinical features point to the patient having chronic hepatitis (lethargy, jaundice, pain in the right hypochondrium hepatomegaly) Alcohol and drugs are not the cause as there is no relevant history. Other common causes have been excluded and what remain are elevated gammaglobulins and positive ANA and SMA antibodies. This point to autoimmune hepatitis (AIH), which is confirmed by liver biopsy. The diagnostic criteria for AIH are the following: Major Criteria Hypergammaglobulinaemia (increased IgG) Autoantibodies (ANA, SMA, SLA/LP, LKM) Absence of viral hepatitis Portal hepatitis (with lymphoplasmacellular infiltrate) on histology

240

Mastering MRCP Minor Criteria Personal or family history of other autoimmune disease Spontaneously fluctuating course Arthralgia Presence of HLA-DR3 or DR 4 In AIH typically there are high levels of IgG. IgA suggests a toxic cause such as ethanol IgM is a feature of primary biliary cirrhosis ANA, SMA and LKM may be found in other liver disease SLA/LP antibodies are highly specific for AIH and thus are diagnostic if detected (present in about 20%) ANA antinuclear antibody SMA smooth muscle antibody LKM liver-kidney microsomal antibody SLA/LP soluble liver antigen antibody

BOF: 3.24 A 15-year-old male is found to have systemic hypertension. Which of the following would be the most likely underlying cause? a) Coarctation of the aorta b) Phaeochromocytoma c) Conn’s syndrome d) Renal artery stenosis e) Renal parenchymal disease Answer: e) Renal parenchymal disease is the most common cause of hypertension in children. The most common nonrenal cause is thought to be coarctation of the aorta. Essential hypertension is recognised in childhood. The renal parenchymal causes of hypertension would include chronic glomerulonephritis and scarring due to vesico-ureteric reflux

BOF: 3.25 An 18-year-old male has been referred to the clinic by his GP. He had attended a medical examination when he applied to join the army and was told he had a hole in his heart. Which of the following physical signs would have made the examining doctor suspect that the patient has a ventricular septal defect (VSD)? a) Fixed splitting of the second heart sound b) A pan systolic murmur best heard at the left sternal edge c) A pan systolic murmur best heard at the apex d) An early diastolic murmur best heard at the left sternal edge e) A mid diastolic murmur best heard at the apex Answer: b) Pan systolic murmurs may be heard in VSD, MR and TR Murmurs best heard at the left sternal edge are VSD, HOCM, AR and TR Thus a pan systolic murmur best heard at the left sternal edge would make one suspect that the patient has a VSD.

241

Mastering MRCP Fixed splitting of the second sound is a feature of ASD Apical murmurs may originate from the mitral valve or the aortic valve Early diastolic murmurs may be due to AR or PR Mid diastolic murmurs may be due to multiple causes Remember VSD too can result in a mid diastolic murmur at the apex due to increased flow across the mitral valve

BOF: 3.26 You are called to see a patient in the surgical ward. He is recovering from surgery. He has had emergency surgery for dissecting aneurysm of the descending aorta. The postoperative period was complicated by pneumonia and pseudomembranous colitis. He has been transferred from the ITU to the surgical wards two days prior to you been called to see him. The patient is not moving his right lower limb normally and the surgical registrar thinks he may have had a stroke. On examination, the patient is conscious and alert and has no abnormality in relation to speech. His cranial nerves are intact and there is no neurological deficit in the upper limbs. On examination of his lower limbs there is weakness of dorsiflexion and eversion of his right foot and there is weakness of toe extension. Sensation is impaired over the lateral aspect of the right lower leg and over the dorsum of the right foot. The lesion in this patient is: a) Anterior spinal artery thrombosis b) Lacunar infarction of the left internal capsule c) Femoral nerve palsy d) Tibial nerve palsy e) Common peroneal nerve palsy Answer: e) A compression neuropathy of the common peroneal nerve may occur after prolonged bed rest. Associated loss of weight with reduction in the size of the protective fat pad may also contribute. Anterior spinal artery thrombosis would result in paraparesis and a characteristic pattern of sensory loss, bilateral loss of pain and temperature sensation with preserved joint position sense and vibration sense Lesions of the internal capsule would result in hemiparesis or hemiplegia BOF: 3.27 A 16-year-old male has been referred by his GP for investigation of a murmur. On examination he has a soft systolic murmur in the pulmonary area and fixed splitting of the second heart sound. In which of the following conditions does fixed splitting of the second heart sound occur: a) Atrial septal defect b) Patent ductus arteriosus c) Right bundle branch block d) Left bundle branch block e) Pulmonary stenosis Answer: a) Fixed splitting of the second heart sound is a feature of atrial septal defect Varying splitting of the second heart sound may occur in right bundle branch block and pulmonary stenosis Reversed splitting of the second heart sound may occur in left bundle branch block and patent ductus arteriosus

242

Mastering MRCP BOF: 3.28 You are called to see a 60-year-old female on the oncology ward. She has breast cancer with bone metastases. Her serum calcium level is 3.8 mmol/l. Which of the following features are likely to occur in this patient? a) Hyporeflexia on neurological examination b) Prolonged QT interval on the ECG c) Tachycardia d) Tall peaked T waves e) Diarrhoea Answer: a) Hypercalcaemia may result in the following features in relation to the CNS: Personality changes, malaise, headache, hallucinations, depression, irritability confusion, depressed sensorium, unsteady gait, proximal muscle weakness, hyporeflexia The ECG changes that may occur are: Shortened QT interval, bradycardia, coving of ST-T wave, widened T wave The features in relation to the gastrointestinal system are: Hypercalcaemia may cause paralytic ileus, with resultant abdominal cramping, constipation, anorexia, nausea, and vomiting. Ectopic calcification may lead to features of pancreatitis, with epigastric pain and vomiting. Increased gastric acid secretion may produce features of gastritis

BOF: 3.29 You are called to see a patient who has been admitted to your ward earlier in the day. She is known to have breast cancer with bone metastases. She has been complaining of thirst, polyuria and constipation. She is irritable and in pain. Her serum calcium is now available and is 3.9 mmol/l. Your first line of action in this patient would be: a) NSAIDs for treatment of her pain b) Treatment with intravenous bisphosphonates c) Treatment with calcitonin d) Rapid infusion of saline e) Treatment with high dose steroids Answer: d) NSAIDs should not be used in hypercalcaemia as they decrease renal blood flow and thus inhibit urinary calcium excretion. Treatment of hypercalcaemia of malignancy is initial saline diuresis; rapid infusion of saline and once the patient is fluid replete furosemide may be used to decrease tubular reabsorption of calcium. Bisphosphonates are the treatment of choice in hypercalcaemia of malignancy but they do not act immediately. Calcitonin accelerates the fall in calcium produced by bisphosphonates. Steroids are effective in the treatment of the hypercalcaemia of sarcoidosis, vitamin D intoxication, lymphomas and Addison’s disease

243

Mastering MRCP BOF: 3.30 A 46-year-old female who is known to have rheumatoid arthritis presents with progressive breathlessness. On examination of her chest she is noted to have fine late inspiratory crepitations at both lung bases. Which of the following drugs that she has taken may be responsible for this condition? a) Prednisolone b) Diclofenac c) Penicillamine d) Sulphasalazine e) Azathioprine Answer: d) The history of progressive breathlessness in a patient with rheumatoid arthritis may have multiple causes. However, the finding of fine late inspiratory crepitations at both bases points to a diagnosis of pulmonary fibrosis ACES for PACES page 264 Many drugs cause pulmonary fibrosis and sulphasalazine is one of them. The other drugs used in rheumatoid arthritis that may cause pulmonary fibrosis are gold and methotrexate. Nalidixic acid does not cause pulmonary fibrosis but nitrofurantoin does do so

BOF: 3.31 A patient who is known to have restrictive lung disease and a chest x-ray that shows reticular shadowing has an open lung biopsy. The pathologist reports the presence of cells with pentalaminar inclusions in their cytoplasm (Birbeck granules, X bodies) This is diagnostic of: a) Sarcoidosis b) Idiopathic fibrosing alveolitis c) Asbestosis d) Lymphangioleiomyoatosis e) Langerhans’ cell histiocytosis Answer: e) Isolated pulmonary involvement may occur in Langerhans’ cell histiocytosis (Histiocytosis X) and is confirmed by the finding of Birbeck granules within the Langerhans’ cells. Langerhans’ cell histiocytosis refers to a clonal proliferation of Langerhans’ cells which are histiocytes found in the dermis

BOF: 3.32 A 63-year-old male presents with a 6-day history of diarrhoea and lower abdominal pain. He is known to have COPD and has had several courses of antibiotics recently for what his GP thought were infective exacerbations. On examination the patient looks unwell, he is febrile (temp 39° C) dehydrated, tachycardic and his blood pressure is low. His abdomen is distended and tender, no lumps palpable, bowel sounds are heard and not exaggerated. Which of the following investigations would be most appropriate in this patient? a) Plain x-ray abdomen b) Stool microscopy and culture c) CT scan of the abdomen

244

Mastering MRCP d) Colonoscopy e) Unprepared flexible sigmoidoscopy Answer: e) The history and examination suggest the patient has an acute inflammatory condition of his bowel. With the history of antibiotic treatment the likely cause is pseudomembranous colitis due to Clostridium difficile rather than an infective colitis. Idiopathic inflammatory bowel disease such as ulcerative colitis is unlikely as the history is too short. Ischaemic colitis usually has a much more dramatic onset. The best way to make a quick diagnosis would be to perform an unprepared flexible sigmoidoscopy and directly view the mucosa. As the patient has diarrhoea it is likely that the distal colon is involved. This should be sufficient to make a diagnosis and biopsies could be taken to provide histological confirmation. Colonoscopy would be unwise in the setting of an acutely inflamed colon, as this would increase the risk of perforation. CT scanning would show inflammation of the colon but the type of colitis would be unclear. Plain x-ray would be useful in detecting toxic mega colon and may sometimes show thumb printing in colitis but these changes are not specific. Stool microscopy and culture are unhelpful but toxin assay would point to the diagnosis

BOF: 3.33 A 69-year-old male who is known to have ulcerative colitis has been found to be mildly anaemic and has a high ESR. His colitis is quiescent with a normal bowel habit and his last surveillance colonoscopy was normal. He is not jaundiced nor does he have fever. There are no abnormalities found on clinical examination. Investigations reveal: Hb 11.5 g/L WCC 4.7 PLT 168 Corrected Calcium 2.60 IgG 26 (6-13) IgA 1.6(0.9-3) IgM 1.5 (0.4-2.2) Which of the following investigation would you arrange next? a) Colonoscopy b) Magnetic Resonance Cholangio Pancreatography c) Isotope bone scan d) Plasma protein electrophoresis e) Chest x-ray Answer: d) The patient has quiescent ulcerative colitis and does not appear to have any complication in relation to this. In an elderly patient with an elevated ESR, elevated calcium, immunoglobulins and mild anaemia one would consider a diagnosis of multiple myeloma and hence plasma protein electrophoresis would be the investigation of choice

245

Mastering MRCP BOF: 3.34 A 30-year-old male has been diagnosed as having pulmonary tuberculosis. He was started on a therapeutic regime that contains isoniazid. He has been tested and been found to be a fast acetylator. Which of the following may occur as a consequence of his acetylator status? a) Increased risk of drug resistance b) Increased risk of hepatotoxicity c) Increased risk of peripheral neuropathy d) Increased risk of megaloblastic anaemia e) Increased risk of convulsions Answer: a) Enzymatic acetylation of isoniazid by N-acetyl transferases reduces the therapeutic effectiveness of the drug. This results in chronic underdosing, which gives inadequate therapy and may give rise to the development of drug resistance

BOF: 3.35 A 23-year-old female presents with red, tender lumps over the anterior aspect of her lower legs. She also complains of joint pains. A chest x-ray is performed and this shows bilateral hilar lymphadenopathy but the lung fields are clear. In this patient which of the following options would be most appropriate? a) CT scan of chest and abdomen b) Mediastinoscopy with lymph node biopsy c) Review in 3 months with repeat chest x-ray d) Biopsy of the skin nodules e) Serial measurement of serum angiotensin converting enzyme levels Answer: c) This presentation represents a benign form of sarcoidosis. No histological confirmation is required. The prognosis is excellent. It is known as Lofgren’s syndrome BOF: 3.36 A 36-year-old female presents with joint and muscle pains of several years duration and an erythematous eruption over her nose spreading to the cheeks in a “butterfly” distribution. Investigations reveal renal involvement as well. After initial treatment she goes into remission. Which of the following may cause an acute relapse in this patient? a) Use of sunscreen b) Pregnancy c) Progesterone only contraceptive pill d) Hydroxychloroquine e) Penicillamine Answer: b) The clinical features of the patient suggest the patient has systemic lupus erythematosus. The measures that may be taken to prevent a relapse are avoidance of sunlight, avoidance of non-essential drugs and prompt treatment of infections. Thus sunscreens should be used by the patient.

246

Mastering MRCP Pregnancy may cause deterioration of the condition but progesterone only pills are safe. Hydroxychloroquine is used in the treatment of this condition. Drugs such as penicillamine, hydralazine, isoniazid, chlorpromazine, methyldopa, procainamide may cause drug-induced lupus but this condition is distinct from systemic lupus erythematosus and these drugs do not exacerbate systemic lupus erythematosus

BOF: 3.37 A 36-year-old female presents with a history of intense pruritus and a blistering eruption affecting her elbows, knees and buttocks. On examination there are excoriations and vesicles some of which have ruptured leaving a crust. Which of the following conditions is the most likely cause of these clinical features? a) Pemphigus b) Pemphigoid c) Porphyria cutanea tarda d) Scabies e) Dermatitis herpetiformis Answer: e) Itch may be caused by a number of conditions There are many causes of vesicles The combination of vesicles and pruritus suggests the patient has dermatitis herpetiformis. The distribution of the rash too fits in with this condition Scabies causes burrows Porphyria cutanea tarda causes a bullous eruption on exposure to sunlight Pemphigus and pemphigoid also cause bullae

BOF: 3.38 A 75-year-old female has been admitted as an emergency having been found collapsed on the floor at home. She is known to have hypothyroidism but the bottle of tablets found by the ambulance crew appears unopened. On examination she is drowsy, has a core temperature of 30oC. Her pulse rate is 40 beats per minute and blood pressure 90/60. There are fine inspiratory crepitations at the left base of her lungs. ECG shows sinus bradycardia with low voltage complexes. In this patient your most important course of action would be: a) Rapid infusion of intravenous saline b) Bolus of 50 % dextrose 50 mls intravenously c) Rapid external rewarming d) Intravenous broad spectrum antibiotics after taking blood cultures e) Intravenous thyroxine Answer: e) The patient has myxoedema coma. This is caused by decompensation of undiagnosed or poorly controlled hypothyroidism. The fact that the bottle of tablets was unopened suggests the patient was non-compliant with treatment.

247

Mastering MRCP The main features of myxoedema coma are hypothermia, altered mental state and the presence of a precipitating cause. In this case the patient has left basal pneumonia as a precipitating cause. Where treatment is concerned this is an emergency situation and one should not await blood tests to confirm the diagnosis. Intravenous thyroxine should be used. Care should be taken about fluid resuscitation, as these patients may be fluid overloaded. Rapid external rewarming may cause cardiovascular collapse and gradual rewarming with space blankets should be instituted. Broad-spectrum antibiotics should be used after taking cultures and intravenous steroids would be necessary as well. However, the question is regarding your most important course of action and hence the answer is giving intravenous thyroxine

BOF: 3.39 A 32-year-old male presents to the emergency department with a history of nausea, diarrhoea and abdominal cramps. He is a known drug abuser. On examination he is irritable, has rhinorrhoea and excessive lacrimation. His skin is flushed; there is gooseflesh (piloerection). He has a pulse rate of 125 beats / minute, his blood pressure is 170/110. He has dilated pupils, involuntary movements and when questioned he admits to having muscle pains. Which of the following would be effective in treating this condition? a) Naloxone b) 50 % dextrose 50 mls bolus c) Diazepam d) Flumazenil e) Methadone Answer: e) The clinical features suggest the patient is suffering from opiate withdrawal and the most effective way of alleviating these features would be the use of methadone

BOF: 3.40 A 46-year-old male presents with chest pain. He has a chest x-ray and the radiology registrar who reports on it tells you that the only abnormality he can see on it is a “reverse comma sign” in the right apex. Your house office asks you what this is and your reply is that the reverse comma is caused by failure of migration of the azygos vein during development and it is made up of: a) Fibrous tissue and the azygos vein b) Parietal pleura and the azygos vein c) Visceral pleura and the azygos vein d) Visceral and parietal pleura and the azygos vein e) Azygos vein Answer: d) The patient has an azygos lobe, which is a normal variant that may occur in less than 5 % of the population. It is caused by failure of migration of the azygos vein over the apex of the lung during fetal life. It courses through the lung instead and drags down two layers of parietal and two layers of visceral pleura with the

248

Mastering MRCP vein lying at the inferior aspect. The four layers of pleura make up the azygos fissure and this together with the vein at its inferior aspect give “the reverse coma sign”. The vein seen end on may sometimes be mistaken for an upper lobe pulmonary mass

BOF: 3.41 A 32-year-old male has been brought into the emergency department by the police. He had been arrested the previous night on a charge o disturbing the peace at a nightclub. Whilst in police custody the patient became irritable, confused and had several generalised seizures. On examination the patient is not orientated in place and time and is having visual hallucinations. He is febrile but has a tachycardia and tachypnoea. There are no other physical signs. The most likely cause of his condition is the withdrawal effects of: a) Barbiturates b) Marijuana c) Cocaine d) Heroin e) Ecstasy Answer: a) Barbiturates may be used as anticonvulsants, sedatives, and general anaesthetics. They can also cause effects similar to alcohol intoxication (euphoria, elation, and uninhibited behaviour) hence they may be abused. A withdrawal syndrome appears 12-20 hours after the last dose. The features are anxiety, irritability, elevated heart and respiratory rate, muscle pain, nausea, tremors, hallucinations, confusion, and seizures. Death may occur if the condition is left untreated. The features of withdrawal of the other drugs are as follows: Marijuana irritability, anxiety, physical tension, decreases in appetite and mood Cocaine agitation, depression, intense craving for the drug, extreme fatigue, anxiety, angry outbursts, lack of motivation, nausea, vomiting, shaking, irritability, muscle pain, disturbed sleep Ecstasy depression, anxiety, panic attacks, sleeplessness, de-personalization, de-realisation, paranoid delusions Heroin, yawning, loss of appetite, nausea, vomiting stomach cramps, diarrhoea muscle cramps, insomnia, shaking, tremors, panic, chills, profuse sweating, irritability, jitteriness dilated pupils, piloerection (gooseflesh) increased lacrimation, rhinorrhoea

BOF: 3.42 A 55-year-old male presents with a cough productive of large volumes of clear, watery sputum. He has never smoked and has been always careful to avoid passive inhalation of cigarette smoke. He is an accountant by profession. Clinical examination is unremarkable. Chest x-ray shows a single, irregular shadow in the periphery of his right lung with an air bronchogram within the tumour opacity. This is most likely to be: a) Squamous carcinoma b) Adenocarcinoma c) Small cell carcinoma

249

Mastering MRCP d) Large cell carcinoma e) Alveolar cell carcinoma Answer: e) Alveolar cell carcinoma is an uncommon tumour (1-2 % of all primary pulmonary tumours). It is not related to cigarette smoking. It occurs in both sexes equally and is most common after the age of 50. Bronchorrhoea (production of copious volumes of clear sputum) is a feature. The x-ray features are as described.

BOF: 3.43 A 36-year-old female is known to have migraine. She is usually able to get relief by using paracetamol. She had given up smoking for years but has begun to smoke 20 cigarettes a day and now complains that paracetamol no longer relieves her headaches. This is because: a) Smoking reduces cerebral blood flow b) Nicotine is a competitive inhibitor of paracetamol c) Smoking induces liver enzymes d) Smoking reduces blood flow to the intestine and thereby reduces absorption of paracetamol e) Smoking increases renal clearance of paracetamol Answer: c) Cigarette smoking reduces the efficacy of certain drugs or makes drug therapy unpredictable. Pharmacokinetic interactions that cause smokers to require a larger dosage of certain drugs are: § An increase in plasma clearance § A decrease in absorption § An induction of cytochrome P450 enzymes § A combination of these factors

BOF: 3.44 In which of the following people would the oxygen-haemoglobin dissociation curve be shifted to the left? a) A person living at high altitude b) An insulin dependent diabetic who has been admitted with signs of salt and water depletion and Kussmaul’s respiration c) A patient brought into casualty having attempted suicide by shutting himself in his garage and turning on the car engine d) A person who has been engaging in unaccustomed strenuous physical exercise e) A patient in the febrile phase of vivax malaria Answer: c) The oxygen haemoglobin dissociation curve is the curve relating percentage oxygen saturation of haemoglobin to the partial pressure of oxygen. It is an S shaped curve. The factors that cause the curve to shift to the right and thereby increase oxygen delivery to tissues are: § Increased 2,3 PG levels, which occur in people living at high altitude and in people within 60 minutes of engaging in exercise. The rise may not occur in trained athletes.

250

Mastering MRCP § Acidosis causes the curve to shift to the right and this would be the cause of the shift to the right in the diabetic patient who has salt and water depletion and Kussmaul’s respiration or deep sighing respiration, which indicates metabolic acidosis. These features suggest the patient has diabetic ketoacidosis. § High temperature too causes the curve to shift to the right and this would occur in the patient who has high fever in vivax malaria. The factors that cause the curve to shift to the left are as follows: § Carboxyhaemoglobin causes the curve to shift to the left and this would have occurred in the patient who attempted suicide, as he would have carbon monoxide poisoning. § Methaemoglobin § Haemoglobin F § High pH § Low temperature § Low 2,3 DPG levels

BOF: 3.45 A 28-year-old female is admitted to the obstetric unit. She is in her 35th week of gestation. She complains of pain in her right upper quadrant, nausea vomiting and headache. Since her 28th week of gestation she has had hypertension, oedema and proteinuria. On examination there is tenderness in the right upper quadrant, no other physical signs. Investigations reveal: Hb 9.5 g /dL The blood film shows deformed red cells Platelet count 75 x 109/l LDH 650 u/l AST 165 IU/l Serum bilirubin 55 mmol/L In this patient the best course of action would be: a) Urgent delivery of the baby by caesarean section b) Low dose aspirin c) Corticosteroids d) Plasma volume expansion e) Vasodilatation Answer: a) This patient has per-ecclampsia. The definition of pre-ecclampsia is the development of hypertension, oedema and proteinuria following the 20th week of gestation. This patient has developed haemolysis, elevated liver enzymes and low platelets on a background of preecclampsia. She has the HELLP syndrome (Haemolysis, Elevated Liver enzymes, Low Platelets). This usually resolves after delivery of the baby hence the mainstay of treatment is prompt delivery. If the period of gestation is not advanced medical treatments may be used to try and gain time for the fetus to mature and thus improve fetal outcome. Regimens used have been plasma volume expansion, low dose aspirin, vasodilatation and corticosteroids. There is little trial based evidence to support these regimes.

251

Mastering MRCP If the HELLP syndrome is advanced or complicated by organ failure then treatment of these complications with intensive care facilities should precede delivery. Hypoxia, seizures, fluid imbalance, hypertension and coagulopathy should be corrected prior to delivery. Dexamethasone may be used to promote maturity of the fetal lung and alleviate the HELLP syndrome. This is based on experience not on trials

BOF: 3.46 A 33-year-old male presents with an itchy rash over his scalp and on his face. The rash came on quite suddenly is very severe, and extensive and is not responding to simple local treatment that he was given by a pharmacist. On examination, on the scalp there are widespread thick adherent crusts. The skin shows greasy scaling over red inflamed skin. This involves the forehead, the eyebrows, the lash line, the nasolabial folds, the beard, and the post auricular skin. In this patient which of the following tests is most likely to determine the underlying cause of the disease? a) Skin scraping b) Skin biopsy c) Patch testing d) HIV test e) TPHA Answer: d) The patient has seborrhoeic dermatitis which is a scaly rash involving the areas of skin where sebaceous glands are plentiful. A severe explosive onset as seen in this patient is usually a marker of HIV infection regardless of the age of the patient and should be considered the underlying cause in this patient

BOF: 3.47 A 38-year-old primigravida presents to the obstetric unit with a history of vomiting and abdominal pain. She is in her 36th week of gestation and she is carrying twins. She also complains of intense thirst and frequent passage of large volumes of urine. On examination there is epigastric tenderness but no other physical signs. Investigations reveal: Hb 11.5 g/dL WBC 14 x 109/L Neutrophils10 x 109/L Platelets 160 x 109/L AST 80 U/L Bilirubin 60 mmol/L Urea 8.8 mol/L Creatinine150 mmol/L Random Blood Glucose 5mmol/L Abdominal ultrasound scan shows ascites Which of the following features would help to differentiate the condition this patient has from the HELLP syndrome? a) Elevated bilirubin

252

Mastering MRCP b) Elevated transaminases c) Elevated creatinine d) Leucocytosis e) Vomiting Answer: d) The patient has Acute Fatty Liver of Pregnancy (AFLP). This is a condition in which the hepatocytes get swollen with microvesicular fat deposition. The risk factors are: § Older maternal age § Primiparity § Multiple pregnancies § Male fetus § Pre ecclampsia § Previous AFLP The features of the disease are six or more of the following: § Vomiting § Abdominal pain § Polyuria/polydypsia § Encephalopathy § Leucocytosis § Coagulopathy § Ascites (USS) § Elevated bilirubin § Hypoglycaemia § Elevated transaminases § Elevated ammonia § Renal impairment § Elevated urate § Microvesicular fat on liver biopsy The differentiating features are as follows: AFLP Polyuria/ polydypsia

HELLP Platelets very low

Leucocytosis

Anaemia

Elevated prothrombin time

Normal prothrombin time

Hypoglycaemia

Abnormal blood film Increased ammonia

253

Mastering MRCP BOF: 3.48 A 63-year-old female presents with dizziness, slurring of speech and incoordination of movement of 4-6 weeks duration. She was admitted numerous investigations were performed and she was discharged. Results were as follows: Routine haematology and biochemistry were normal. Antibody screen had shown her to be positive for anti Yo antibody MRI scan of her head had been normal. EEG showed no abnormality CSF examination following lumbar puncture normal Your colleague rings you up to ask you about these tests and your advise would be that this lady is likely to have: a) Small cell carcinoma of the lung b) Ovarian carcinoma c) Hodgkin’s lymphoma d) Non-Hodgkin’s lymphoma e) Pancreatic cancer Answer: b) The patient presented with a cerebellar syndrome. The normal MRI excludes a structural lesion of the cerebellum; normal CSF makes a demyelinating lesion unlikely. The chances of this being a paraneoplastic syndrome are high. Anti Yo antibodies occur in paraneoplastic cerebellar syndromes and suggest cancers of the breast or ovary as the underlying lesion. In males anti Yo antibodies occur in patients having cancers of the parotids, oesophagus or stomach. Anti Hu antibodies occur in small cell carcinoma of the lung

BOF: 3.50 A 26-year-old male presents complaining of a rash over his upper trunk and arms. He is an Asian who has recently arrived in the UK. On examination there are sharply demarcated depigmented macules over the upper trunk and arms with fine scaling. There is no loss of sensation over the macules, no loss of sweating, no lymphadenopathy, no thickened nerves are palpable and the patient is afebrile. Your next line of action in dealing with this patient would be to: a) Take skin scrapings and examine them for fungi b) Do a TPHA test c) Take a skin biopsy d) Examine a slit skin smear for acid fast bacilli e) Give local treatment with topical imidazole Answer: a) In a young otherwise well individual presenting with depigmented macules with fine scaling, the most likely diagnosis is pityriasis versicolor. This is caused by the yeast Pityriasis orbiculare (also known as Malssezia furfur). It is more common in the tropics and subtropics and commonly affects young adults. The diagnosis is made on skin scrapings, which would show spherical yeasts and short mycelia. Treatment is with local applications of selenium sulphide, local imidazoles or topical imidazoles.

254

Mastering MRCP Leprosy presents with hairless, hypopigmented, anaesthetic lesions and they show absence of sweating, nerves may be thickened. This is an unlikely diagnosis in this case and hence biopsy and slit skin smear are not required. In secondary syphilis the patient is bacteraemic and hence systemically unwell with low-grade fever, malaise, headache, lymphadenopathy. The rash consists of papules or plaques with scaling. The palms, soles and face are commonly involved. This is unlikely in this case and hence a TPHA test is not required

BOF: 3.49 A 27-year-old Asian female attends the antenatal clinic complaining of intense itching of her skin. This is generalised but the palms and soles of her feet are affected to a greater degree. The pruritus is increased at night. She is in her 34th week of gestation and has had no complications and is otherwise well. On direct questioning she reveals that her mother had similar problems during pregnancy and had been told that this was a normal occurrence. On examination she looked well and there were no abnormalities clinically. Investigations showed elevated transaminases but all other investigations were normal. The registrar in obstetrics who sees this patient rings you up for advice regarding this patient. Your advice would be: a) Treat with ursodexoycholic acid and plan to deliver the baby between 36-38 weeks b) Admit the patient for MRI scanning to localize the site of biliary obstruction c) Admit the patient and arrange an ERCP d) Deliver the baby by caesarean section immediately to prevent stillbirth e) Admit the patient and have her seen by the infectious disease team as the patient probably has viral hepatitis Answer: a) This patient probably has obstetric cholestasis (OC). This is a condition in which there is pure cholestasis with no necrosis of cells and no inflammation. Oestrogens and progestogens are thought to play the primary part in this condition. It is most common when levels of these hormones are high that is in late pregnancy and multiple pregnancies. The condition resolves after delivery. The condition usually presents at 30-36 weeks of gestation (may start in the first trimester). The presenting feature is pruritus, which is generalised but tends to affect the palm and soles of the feet in particular and is worse at night. A positive family history may be obtained in 30%. There are no abnormalities on physical examination and the only abnormality usually found on investigation is elevated transaminases. In a patient presenting with pruritus in pregnancy, the finding of elevated transaminases with no other abnormality (clinically or on laboratory testing) is diagnostic of obstetric cholestasis. Serum bile acid levels are a more specific marker of obstetric cholestasis. The fetus is at greater risk in these patients, the complications being prematurity, fetal distress, meconium staining, stillbirth and perinatal death. It is best to deliver the baby between 36-38 weeks as the risk of stillbirth increases after 36 weeks of gestation. Ursodexoycholic acid reduces bile acid levels and may ameliorate pruritus. It has not been shown to improve fetal outcome

255

Mastering MRCP BOF: 3.51 A 60-year-old female has been on long-term low dose steroids and azathioprine for treatment of rheumatoid arthritis. Which of the following would be most suitable to prevent osteoporosis in this lady? a) Calcium and vitamin D supplements b) Thiazide diuretics c) Calcitonin d) Hormone replacement therapy e) Bisphosphonates Answer: e) All of the above agents may be used in the prevention of osteoporosis but bisphosphonates would be the most suitable

BOF: 3.52 A 63-year-old female is readmitted to the ward with a history of fever, anorexia and jaundice. Eleven days prior to this the patient had been transfused with 4 units of packed cells, as she had been anaemic, Investigations reveal: Haemoglobin 8.6 g/dL (11.5 – 16.5 g/d) Serum total bilirubin 112 μmol/L (1 – 22 μmol/L) Which of the following investigations is likely to indicate the cause of the patient’s current condition? a) Presence of haemoglobinuria b) Presence of haemosiderinuria c) Positive Coombs test d) Increased reticulocyte count e) Level of conjugated bilirubin Answer: c) Delayed haemolytic transfusion reactions are caused by extravascular haemolysis occurring 3 to 21 days after transfusion. . It is the result of production of antibody causing accelerated removal of donor erythrocytes Delayed haemolytic reactions are usually mild and may not be recognized. The features of delayed transfusion reaction are: Post-transfusion haematocrit rapidly declines Or Does not stay elevated for as long as it was expected to Or The patient develops hyperbilirubinaemia and or bilirubinuria. The most common clinical features are fever, anorexia and jaundice. A positive direct Coombs test occurs

BOF: 3.53 A 64-year-old male presents with a history of fever and swelling of his body. He gives a history of taking diclofenac for pan relief from osteoarthritis. On examination his face looks puffy and he has ankle oedema. . His blood pressure is 210/110 mm Hg. Urine show s heavy proteinuria. 24-hour urine proteins 3.7 g/24 hours. His serum albumin is 24 g/'L

256

Mastering MRCP Which of the following is likely to have caused his present condition? a) Interstitial nephritis b) Retroperitoneal fibrosis c) Chronic glomerulonephritis d) Chronic pyelonephritis e) Amyloidosis Answer: a) Nephrotic syndrome may be defined as a triad of oedema, proteinuria and hypoalbuminaemia. By definition the patient has nephrotic syndrome NSAIDS and the Kidney NSAIDs are commonly used drugs. Despite the introduction of selective COX-2 inhibitors, nephrotoxicity is still a concern. NSAIDs inhibit prostaglandin synthesis from arachidonic acid by non-specific blocking of the enzyme cyclooxygenase This leads to vasoconstriction and reversible mild renal impairment in volume-contracted states It may lead to acute tubular necrosis and acute renal failure NSAIDs also produce interstitial nephritis with or without nephrotic syndrome secondary to minimal change disease. This presents as acute renal failure, but it can progress in some cases to chronic renal failure. Papillary necrosis has been incriminated in the development of chronic renal failure secondary to NSAIDs. Subclinical renal dysfunction with reduced creatinine clearance and impaired urine concentrating ability has been shown to be present in patients on long term NSAIDS This sub-clinical dysfunction is reversible on withdrawal of NSAIDS However, a persistent residual dysfunction may occur Interstitial Nephritis Acute tubulointerstitial nephritis usually begins abruptly and manifests as acute renal failure. Drug induced acute tubulointerstitial nephritis occurs within days of exposure to the offending drug However, sometimes acute tubulointerstitial nephritis begins after several months of exposure to the offending agent (NSAIDS) Patients commonly present with rash, fever, eosinophilia, eosinophiluria, and elevated immunoglobulin E (IgE) levels (exceptions rifampin and NSAIDS) Clinical presentation may be with tubular function abnormalities, such as Fanconi’s syndrome (aminoaciduria, glycosuria, renal tubular acidosis). Patients may also present with rash and haematuria. Proteinuria is usually absent or modest Urinalysis may show microscopic haematuria and or sterile pyuria (with or without eosinophils) The clinical presentation is often sufficient to make the diagnosis Renal biopsy is required to make a definitive diagnosis Patients with acute tubulointerstitial nephritis caused by NSAIDS typically present with heavy proteinuria, often in the nephrotic range Cessation of the offending drug usually results in complete recovery. Acute tubulointerstitial nephritides are usually the result of hypersensitivity reactions to drugs and are not mediated by direct toxicity NSAID induced acute tubulointerstitial nephritis More common in elderly people (higher incidence of arthritic disorders)

257

Mastering MRCP Allergic interstitial nephritis caused by NSAIDS may present with nephrotic syndrome Massive proteinuria with hypoalbuminaemia and oedema are present in addition to the typical features of acute interstitial nephritis Kidney biopsy would show features of minimal change nephritis in addition to the features of interstitial nephritis

BOF: 3.54 A young female is admitted to your ward with chronic diarrhoea. On examination you note that she is of short stature, she has a round face, short neck, short fourth and fifth metacarpals and metatarsals. Investigations reveal: Hb 12.8 g/dl Serum Calcium 1.62 mmol/l Serum Phosphate 2.9 mmol/l Creatinine 44 micromoles/l Albumin 38 g/l Plasma parathyroid hormone 2.9 pmol/L (normal 0.9 – 5.4 pmol/L) After fluid resuscitation and correction of electrolyte imbalance her blood tests are repeated and come back as: Plasma parathyroid hormone 9.3 pmol/L (normal 0.9 – 5.4 pmol/L) In this case, correction of which one of the following would have accounted for this change? a) Sodium b) Potassium c) Chloride d) Phosphate e) Magnesium Answer: e) The patient has the somatic features of pseudohypoparathyroidism. The low calcium levels are in keeping with this but one would have expected the parathyroid hormone level to be elevated Parathyroid hormone secretion is inhibited in patients with hypomagnesaemia. Once this is corrected the parathyroid gland will secrete parathyroid hormone in excess in response to the hypocalcaemia. Magnesium Magnesium is the fourth most common cation in the body It is a central electrolyte in a large number of cellular metabolic reactions, including DNA and protein synthesis, neurotransmission, and hormone-receptor binding It is a component of GTPase a cofactor for Na+/K+–ATPase, adenylate cyclase, and phosphofructokinase Magnesium is necessary for the production of parathyroid hormone. Magnesium deficiency affects many body functions. Magnesium is absorbed in the small bowel by active and passive transport mechanisms. Absorption takes place mainly in the ileum. It is excreted in stool and urine Regulation of serum magnesium is under renal control Renal reabsorption of magnesium occurs in the proximal tubule and the thick ascending limb of the loop of Henle. Causes of hypomagnesaemia

258

Mastering MRCP GIT • •

Malnutrition Malabsorption Malabsorption syndromes Radiation enteritis Bowel resection Small bowel bypass • GI Loss Chronic diarrhoea Laxative abuse Inflammatory bowel disease Neoplasms • Cirrhosis of the liver Extracellular volume expansion KUS Diuretic phase of acute tubular necrosis Post-obstructive diuresis Osmotic diuresis o Diabetic patients, o Alcoholics (osmotic diuresis from alcohol) Renal tubular acidosis Drugs Diuretics Cisplatin. Pentamidine Some antibiotics cause renal magnesium wasting Fluoride poisoning Intravenous fluids (Extracellular volume expansion) E&M Primary aldosteronism decreases magnesium levels by increasing renal flow. Hypoparathyroidism Hungry bone syndrome (Unusual complication of the postoperative period of primary hyperparathyroidism. It is a syndrome is characterized by hypocalcaemia, hypophosphataemia and hypomagnesaemia, due to an excessive bone remineralisation) Hyperthyroidism may cause renal wasting Insulin causes redistribution of magnesium into cells resulting in lower magnesium levels RAG Excessive lactation may create a significant amount of magnesium loss Pregnant women may be magnesium depleted (those in preterm labour)

259

Mastering MRCP BOF: 3.55 A 26-year-old male is brought into accident and emergency by his friends. He has had a seizure whilst at a nightclub. On direct questioning his friends admit that the patient had used ecstasy in addition to drinking several units of alcohol. On examination he is disorientated and drowsy. In this patient which one of the following may occur? a) Hyponatraemia b) Hypokalaemia c) Metabolic acidosis d) Metabolic alkalosis e) Hyperglycemia Answer: a) 3,4 methylenedioxymethamphetamine (MDMA), also known as ecstasy, is an amphetamine derivative Its action is similar to the stimulant methamphetamine and the hallucinogen mescaline. It causes catecholamine release from presynaptic vesicles It is also is a selective serotonergic neurotoxin that causes massive release of serotonin and is thought to inhibit its uptake. Hyponatraemia Hyponatraemia may occur after MDMA use. It is caused by a combination of: • Increased water intake • Excessive sweating due to physical exertion • Vasopressin release leading to the syndrome of inappropriate antidiuretic hormone secretion (SIADH) In severe hyponatraemia, patients can develop cerebral oedema with consequent seizures and coma

BOF: 3.56 A 30-year-old male presents with a history of feeling tired and passing dark urine. Five days prior to presentation his general practitioner had treated him with co-trimoxazole, as he had been complaining of a sore throat. Investigations reveal: Haemoglobin 8.4 g/dL (11.5 – 16.5 g/d) Serum total bilirubin 107 μmol/L (1 – 22 μmol/L) The blood film shows multiple bite cells The most likely cause of this patient’s present problem is: a) Paroxysmal cold haemoglobinuria b) Autoimmune haemolytic anaemia c) Glucose-6-phosphate dehydrogenase deficiency d) Cold agglutinin disease e) Paroxysmal nocturnal haemoglobinuria Answer: c) Bite Cells Bite cells are red cells from which denatured, precipitated masses of haemoglobin have been pitted by the spleen.

260

Mastering MRCP The precipitation is the result of oxidative injury to haemoglobin by drugs or by the denaturation of unstable mutant haemoglobins. If a large amount of precipitation occurs, the spleen is unable to pit all the precipitate and red cells with precipitate may appear in the peripheral blood. The precipitate within the red cell is known as a Heinz body. The patient has a haemolytic anaemia precipitated by treatment with a sulpha containing antibiotic The most likely underlying cause is glucose-6- phosphate dehydrogenase deficiency G6PD catalyses reduction of nicotinamide adenine dinucleotide phosphate (NADP) to NADPH NADPH protects cells from oxidative damage Erythrocytes do not generate NADPH in any other way Hence they are more susceptible than other cells to destruction from oxidative stress.

BOF: 3.57 A 67-year-old male presents to the accident and emergency department with a history of headache, vomiting and unsteadiness on his feet. He also complains of dizziness and has slurred speech He is a known hypertensive and is on atenolol and bendroflumethiazide. He is transferred to the medical admissions unit and you are called to see him. You find him to be drowsy and unable to cooperate with a full neurological examination. His blood pressure is 220/130 The most likely cause of this patient’s condition is: a) Cerebellar haemorrhage b) Intracerebral haemorrhage c) Pontine haemorrhage d) Intraventricular haemorrhage e) Subarachnoid haemorrhage Answer: a) In a known hypertensive presenting with headache one should consider intracranial haemorrhage The symptoms of vomiting adds further weight to this diagnosis. Dizziness slurred speech and unsteadiness on the feet should make one consider the cerebellum as the site of the lesion

BOF: 3.58 A 27-year-old female presents with a history of an itchy, blistering rash. On examination you note multiple small, clustered vesicles over the elbows, knees and buttocks. There are multiple excoriations around the lesions. You arrange a skin biopsy. The skin biopsy is likely to reveal: a) IgG deposition on the surface of the keratinocytes in and around lesions b) Granular IgA deposits in dermal papillae of perilesional skin observed by direct immunofluorescence c) IgG and complement C3 deposition in a linear band at the dermal-epidermal junction d) Immunoglobulin G and complement in a linear or granular bandlike pattern at the basement membrane zone e) Infiltration of lymphocytes at the dermal-epidermal interface with accompanying exocytosis and spongiosis in the epidermis Answer: b)

261

Mastering MRCP The clinical features of the case would suggest the patient has dermatitis herpetiformis. In this condition granular IgA deposits in dermal papillae of perilesional skin are observed by direct immunofluorescence In pemphigus direct immunofluorescence usually shows IgG deposited on the surface of the keratinocytes in and around lesions In bullous pemphigoid direct immunofluorescence usually demonstrates IgG (70-90% of patients) and complement C3 deposition (90-100% of patients) in a linear band at the dermal-epidermal junction In bullous systemic lupus erythematosus direct immunofluorescence of perilesional skin reveals deposition of immunoglobulin G (with and without immunoglobulin M and IgA) and complement in a linear or granular bandlike pattern at the basement membrane zone Erythema multiforme is characterized by infiltration of lymphocytes at the dermal-epidermal interface with accompanying exocytosis and spongiosis in the epidermis

BOF: 3.59 A 64-year-old male presents with a history of breathlessness. Clinical examination reveals signs of a rightsided pleural effusion and this is confirmed on chest x-ray. Pleural aspiration is performed and analysis of the pleural fluid reveals the following: Protein 43 g/L, pH 7.26, Glucose 1.3 mmol/L What is the most likely cause of the effusion? a) Bronchial cancer b) Mesothelioma c) Post pneumonic effusion d) Tuberculosis e) Rheumatoid disease Answer: e) Pleural effusion occurs in about 5% of patients with rheumatoid disease. The majority of patients with rheumatoid pleural effusions are men although the disease commonly affects women rather than men. Analysis of pleural fluid in rheumatoid disease usually shows high protein levels, low sugar, low pH, low C4 complement levels and high rheumatoid factor levels. A pleural glucose level of less than 3.3 mmol/l is found in exudative pleural effusions secondary to empyema, rheumatoid disease, lupus, tuberculosis, malignancy, or oesophageal rupture .The lowest glucose concentrations are found in rheumatoid effusions and empyema. Rheumatoid arthritis is unlikely to be the cause of an effusion if the glucose level in the fluid is above 1.6 mmol/L

BOF: 3.60 A 47-year-old male who is known to have alcoholic liver disease with established cirrhosis is admitted to your ward with worsening ascites. He has a paracentesis and is subsequently commenced on spironolactone 200 mgs daily and furosemide 40 mgs daily. There was no evidence of spontaneous bacterial peritonitis on the ascitic fluid samples sent to the laboratory. Over the next few days his urine output begins to decrease and there is a rise in his serum creatinine levels to 240 μmol/L (normal range serum creatinine 60 – 110 μmol/L). Abdominal ultrasound scan has shown normal sized kidneys and no evidence of obstructive uropathy. Urine dipstick examination does not show

262

Mastering MRCP proteins or blood. You stop the diuretics, ensure that there is no salt and water depletion and start the patient on infusions of albumin. In addition to the above which of the following is likely to be of benefit to the patient? a) Intravenous furosemide b) Dopamine infusion c) Dobutamine infusion d) Octreotide infusion e) Intravenous terlipressin Answer: e) Hepatorenal syndrome is a complication of end-stage liver disease. It is characterized by functional renal failure due to renal vasoconstriction in the absence of underlying renal pathology. The pathogenesis is extreme underfilling of the arterial circulation secondary to an arterial vasodilatation in the splanchnic circulation. This triggers a compensatory response with activation of vasoconstrictor systems leading to intense renal vasoconstriction. The hallmark of HRS is renal vasoconstriction There are 2 types of hepatorenal syndrome Type 1 HRS Rapid and progressive renal impairment (most commonly precipitated by spontaneous bacterial peritonitis SBP) median survival of patients with type 1 HRS is less than 2 weeks (without treatment) and virtually all patients die within 10 weeks after the onset of renal failure. Type 2 HRS More steady deterioration in renal function and commonly occurs in patients with relatively preserved hepatic function. Patients are often diuretic-resistant Median survival of 3-6 months Treatment is with intravenous albumin And Agonists of vasopressin V1 receptors (terlipressin), which predominantly act on the splanchnic circulation causing splanchnic vasoconstriction

BOF: 3.61 A 47-year-old male who is obese and is a known hypertensive presents with a history of passing a lot of urine and waking up in the night to pass urine. He also complains of feeling thirsty all the time. He feels lethargic and feels he may have lost some weight although he has not weighed himself. His general practitioner has arranged a fasting blood sugar and the value is as follows: Fasting venous glucose 12.3 mmol/L In this patient the diagnosis is: a) Impaired glucose tolerance b) Impaired fasting glucose c) Diabetes mellitus d) Metabolic syndrome e) Cushing’s syndrome Answer: c)

263

Mastering MRCP The diagnosis of diabetes mellitus may be made on the basis of 2 sets of criteria 1) Symptoms of hyperglycemia (polyuria, polydypsia, unexplained weight loss, visual blurring, genital thrush, lethargy) And Raised venous fasting glucose = or > 7 mmol/L or raised random venous glucose = or > 11.1 mmol/L 2) Raised venous glucose on 2 separate occasions: Fasting venous glucose = or > 7mmol/L Random venous glucose = or >11.1 mmol/L Oral glucose tolerance test 2 h values = or > 11.1 mmol/L Oral glucose tolerance test: ü Fast overnight ü 75 g glucose 300 mls water ü Venous plasma glucose at 0 hours 2 hours The patient meets the criteria for the diagnosis of diabetes mellitus. There is not enough information given to diagnose metabolic syndrome or Cushing’s syndrome Impaired Glucose Tolerance (IGT) Fasting plasma glucose < 7mmol/L And Oral glucose tolerance test 2 h glucose =or > 7.8mol/L but <11.1mmol/L Impaired Fasting Glucose (IFG) Fasting plasma glucose =or > 6.1 mmol/L but <7 mmol/L These patients should have an oral glucose tolerance test to exclude diabetes mellitus IGT and IFG are abnormalities of glucose regulation The risk of progressing to diabetes mellitus is lower in IFG than in IGT Management is by lifestyle adjustments (diet and exercise)

BOF: 3.62 A 52-year-old obese female who is on treatment for hypertension has had an oral glucose tolerance test. The results are as follows: Fasting plasma glucose 6.8 mmol/L 2 h Glucose 10.3 mmol/L Which of the following interventions will prevent this patient progressing to develop overt diabetes mellitus? a) Treatment with metformin b) Treatment with an angiotensin converting enzyme inhibitor (ACEI) c) Treatment with pioglitazone d) Lifestyle adjustment (diet and exercise) e) Treatment with acarbose Answer: d) On the basis of the tests the patient has impaired glucose tolerance (IGT) This is managed by lifestyle adjustments (diet and exercise) and regular review

264

Mastering MRCP BOF: 3.63 A 62-year-old male is admitted with breathlessness. On examination he has bilateral ankle oedema and sacral oedema. His pulse rate is 120 beats per minute irregularly irregular. His jugular venous pressure is raised to the level of his ear lobe and the dominant wave is a V wave. The apex beat is displaced to the 6th left intercostal space anterior axillary line, it is thrusting in nature and there is a pan systolic murmur at the apex, which radiates to his axilla. Auscultation of the lung bases reveals bilateral basal crepitations. The liver is enlarged 3 fingerbreadths below the coastal margin and it is pulsatile. The patient has a fasting sample of blood analysed and his fasting venous plasma glucose is 6.8 mmol/L. He has an oral glucose tolerance test and this excludes overt diabetes mellitus. In this patient treatment with which one of the following drugs will prevent progression to diabetes mellitus? a) Metformin b) Gliclazide c) Pioglitazone d) Angiotensin converting enzyme inhibitor e) Beta blocker Answer: d) The clinical features allow one to make a diagnosis of heart failure and atrial fibrillation. The plasma glucose level with an oral glucose tolerance test excluding overt diabetes mellitus shows that the patient has impaired fasting glucose (IFG) .It has been shown that treating patients, who have heart failure and IFG, with angiotensin converting enzyme inhibitors (ACEI) can prevent progression to overt diabetes mellitus

BOF: 3.64 A 47-year-old female presents with a history of recurrent episodes of blurring of vision, confusion and drowsiness. These episodes come on when the patient has been without food for several hours and they are accompanied by feelings of anxiety hunger, sweating and palpitations. She has started eating more frequently to try and prevent these episodes occurring and she has gained a lot of weight. In this patient which one of the following tests would you arrange? a) 72 hour fast with measurement of glucose, insulin and C- peptide levels b) Magnetic resonance imaging of the pancreas c) Computerised tomogram of the pancreas d) Endoscopic ultrasound of the pancreas e) 5 h oral glucose tolerance test Answer: a) An insulinoma is a neuroendocrine tumour derived from pancreatic islet cells .It produces excessive amounts of insulin The diagnosis of insulinoma should be suspected in a patient with symptomatic fasting hypoglycemia Whipple’s triad is the features that should be demonstrated to make a diagnosis of true hypoglycemia. These features are: I. Symptoms and signs of hypoglycemia II. Concomitant plasma glucose level of 2.5 mmol/L or less III. Reversibility of symptoms with administration of glucose.

265

Mastering MRCP Diagnosis of Insulinoma Normally, endogenous insulin production is suppressed in the setting of hypoglycemia. If insulin levels fail to suppress during a 72-hour fast the presence of an insulin-secreting tumour is likely Test During the test, the patient may have calorie-free and caffeine-free liquids Capillary blood glucose is measured every 4 hours until values 3.3 mmol/L are obtained The frequency of blood glucose measurement is increased to every hour until values are 2.7 mmol/L At this point, or when the patient has symptoms of hypoglycemia, blood is taken for estimation of glucose, insulin, proinsulin, and C-peptide levels. The fast is stopped at this point, and hypoglycemia treated with either intravenous dextrose or caloriecontaining food or drink

BOF: 3.65 A 63-year-old male is admitted to the coronary care unit with severe, crushing central chest pain accompanied by sweating and vomiting. The ECG shows ST elevation in the anterior chest leads. The patient is treated with thrombolysis. Random blood sugar levels in this patient are found to be elevated with levels in excess of 17 mmol/L You would treat this patient with: a) Metformin b) Gliclazide c) Pioglitazone d) Insulin e) Acarbose Answer: d) Strict glycaemic control improves outcome after acute myocardial infarction

BOF: 3.66 A 23-year-old female presents with a history of headaches. She says that the headache begins with a sensation of seeing flashing lights and she then develops a one-sided headache, which is throbbing in nature and accompanied by nausea and vomiting. The headache is severe and makes her want to lie in bed in a darkened room as light increase the intensity of the headache. The headache may last for several hours and sometimes may go on for 2 days. She is incapacitated by these attacks but they come on infrequently (about every 3-6 months) In this patient which of the following drugs would you consider using: a) Rizatriptan b) Pizotifen c) Propranolol d) Valproate e) Amitryptiline Answer: a) Migraine headaches are recurrent headaches that may be unilateral or bilateral They may occur with or without a prodrome

266

Mastering MRCP The aura of migraine may consist of neurological symptoms, such as dizziness, tinnitus, scotomas, photophobia, or visual scintillations Two types of migraine Classic migraine or migraine with aura Common migraine or migraine without aura Pathophysiology Evidence supports the role of neurogenic peptides, such as serotonin and dopamine, in the brain being involved in the pathophysiology of migraine These vasoactive neuropeptides stimulate an inflammatory cascade, which results in vasodilatation and a perivascular reaction The serotonin receptor (5-HT) is believed to be the most important receptor in the headache pathway. Migraine specific medications: ü Triptans Sumatriptan, rizatriptan, zolmitriptan, naratriptan, almotriptan, eletriptan, frovatriptan ü Ergot alkaloids Ergotamine, dihydroergotamine Migraine prophylaxis Pizotifen Propranolol Amitryptiline Valproate, gabapentin, topiramate, NSAIDS Indications for migraine prophylaxis ü Frequent disabling attacks, >2 per month ü Acute treatment fails, contraindicated, unacceptable side effects, overused ü Hemiplegic migraine or attacks that may cause permanent damage ü Very high attack frequency >2 per week ü Progressively worsening attacks ü Patient requests preventive therapy

BOF: 3.67 A 25-year-old male presents with a history of passing blood in his urine. He also complains of fever and a sore throat and on examination you note that he has tonsillitis. Which of the following are you likely to see on renal biopsy in this patient? a) Subendothelial immune deposits b) Pauci-immune segmental crescentic necrotising glomerulonephritis c) Linear deposits of immunoglobulin G (IgG) and complement (C3) along the glomerular basement membranes. d) Deposits of immunoglobulin G and C3 in a diffuse granular pattern along the glomerular capillary wall and mesangium e) IgA deposited in a diffuse granular pattern in the mesangium Answer: e) IgA nephropathy is a highly variable clinical and pathological entity Clinical features range from asymptomatic haematuria to rapidly progressive glomerulonephritis

267

Mastering MRCP IgA nephropathy is most often associated with microscopic haematuria or recurrent macroscopic haematuria Recurrent macroscopic haematuria, usually associated with an upper respiratory tract infection (gastroenteritis less commonly) is the most frequent clinical presentation The association of pharyngitis and macroscopic haematuria is known as synpharyngitic nephritis Spontaneously reversing acute renal failure can occur It can sometimes lead to chronic renal insufficiency as well A spectrum of glomerular lesions may be observed pathologically but mesangial proliferation with prominent IgA deposition is observed in almost all biopsies

BOF: 3.68 A 23-year-old male is admitted by his general practitioner as he has found him to be anaemic The patient had contracted infectious mononucleosis and was apparently recovering from this when he contacted his GP saying he felt extremely tired and lethargic On examination he looked pale and he had splenomegaly Blood tests showed: Hb 7.3 g/dL Blood film showed spherocytosis Serum total bilirubin 96 µmol/L (Normal 1 – 22 µmol/L) Which of the following tests would indicate the diagnosis? a) Reticulocyte count b) Abdominal ultrasound scan c) Bone marrow examination d) Coombs test e) Red cell osmotic fragility Answer: e) Hereditary spherocytosis is a familial haemolytic anaemia characterised by the presence of spherocytes, which are sphere shaped cells rather than the normal biconcave discs. It is caused by a molecular defect in one or more of the membrane proteins of the red blood cell which results in instability of the cytoskelton (ankyrin, spectrin, band 3 deficiency, protein 4.2 (pallidin) deficiency This defect in the cell skeleton causes the cell to contract to its most surface-tension efficient and least flexible configuration (sphere) Presentation is with anaemia, jaundice, and splenomegaly. Clinical features may be highly variable. Intercurrent illness may exacerbate clinical features or unmask clinically latent disease The most sensitive test to detect hereditary spherocytosis is the osmotic fragility test Abnormalities in spectrin, ankyrin, pallidin, or band 3 may be tested in research laboratory Treatment options: Blood transfusion Exchange transfusion Splenectomy

268

Mastering MRCP BOF: 3.69 A 72-year-old male attends clinic with his daughter with whom he lives. She says that she has noticed that he is becoming confused and his memory is failing. She says that he keeps seeing things that are not there. He also has periods of time where he becomes drowsy. On examination you note that his movements are slow, there is a pill-rolling tremor of his hands, there is cogwheel type of rigidity of his upper limbs. His gait is slow and there is lack of arm swing when he walks Which of the following is the patient likely to have? a) Parkinson’s disease b) Lewy body dementia c) Multi System Atrophy d) Progressive supranuclear palsy e) Cortico basal ganglion degeneration Answer: b) Lewy Body Dementia This is a common type of progressive dementia Features The features of Lewy body dementia are: ü Progressive cognitive decline Combined with: Three additional defining features: ü Pronounced fluctuations in alertness and attention (Frequent drowsiness, lethargy, lengthy periods of time spent staring into space, or disorganized speech) ü Recurrent visual hallucinations, ü Parkinsonian motor symptoms Clinical Features ü Features of dementia: confusion, memory loss, decreased or irregular awareness or reasoning (cognition), delusions such as false ideas about another person or situation, depression, fluctuating alertness ü Features of Parkinsonism ü Visual hallucinations (Seeing colours, shapes, animals or people) Another feature of Lewy body dementia is an increased susceptibility to an adverse reaction to neuroleptics

BOF: 3.70 An 18-year-old male presented with a 3-month history of lethargy followed by a 2-week history of headache, nausea and vomiting. No history of joint pains, rash, cough or haemoptysis. On examination, he was pale, blood pressure 180/110 mmHg. Investigations: Serum potassium 7.6 mmol/L Urea 48 mmol/L Creatinine 1726 µmol/L Haemoglobin 6.7 g/dL Urinalysis demonstrated red blood cells with granular casts 24-urine albumin 1600 mg/L Renal ultrasound demonstrated bright kidneys of normal size with loss of cortico-medullary differentiation

269

Mastering MRCP Renal biopsy showed a severe crescentic necrotising glomerulonephritis with intense linear deposition of IgG in the glomerular capillary walls. No immune deposits on electron microscopy and the glomerular basement membrane was of normal thickness. In this patient which one of the following tests is most likely to be positive? a) Anti glomerular basement membrane antibody b) Anti streptolysin O titer c) Anti neutrophil cytoplasmic antibody d) Anti nuclear antibody e) Rheumatoid factor Answer: a) Goodpasture’s syndrome This refers to the clinical entity of acute glomerulonephritis and pulmonary alveolar haemorrhage Goodpasture’s disease refers to the pulmonary renal syndrome specifically associated with anti glomerular basement membrane (anti GBM) antibodies Anti GBM antibodies result in a characteristic linear deposition along the glomerular basement membrane This helps to differentiate this condition from Wegener’s granulomatosis Anti GBM disease is defined as the triad of glomerulonephritis (usually rapidly progressive or crescentic), pulmonary haemorrhage and the presence of anti GBM antibody Patients with anti GBM disease may present with range of conditions varying from pulmonary haemorrhage with little or no renal involvement to full-blown renal failure with little or no pulmonary involvement Serology Antistreptolysin O (ASO) titers and complement studies are usually normal Anti GBM titers are elevated in more than 90% of patients. ANCA titers are elevated in 20-30% of patients with anti-GBM disease (perinuclear antineutrophil cytoplasmic autoantibody (p ANCA) i.e. antimyeloperoxidase ANA and rheumatoid factor usually negative

BOF: 3.71 A 27-year-old female who is known to have Crohn's disease affecting the terminal ileum and caecum presents with a one-week history of diarrhoea. She complains of passing green watery stools. There is no history of abdominal pain, passage of blood or mucous. She has no fever and there has been no loss of weight. She is on long-term azathioprine for Crohn's disease. There is no recent travel history and she has not eaten meals that may have been contaminated. She gives a history of swimming in the local swimming pool, which is frequented by many schoolchildren. On examination she does not look unwell and she is not tachycardic or dehydrated. She is afebrile and on examination of her abdomen there are no significant findings. Routine haematology and biochemistry are unremarkable. Stools are sent off for microscopy, culture and sensitivity. Which of the following organisms is likely to be detected in the stools? a) E coli O 157 b) Salmonella c) Shigella d) Campylobacter

270

Mastering MRCP e) Cryptosporidium Answer: e) Cryptosporidiosis is caused by infection with protozoans of the genus Cryptosporidium. Common species are: ü Cryptosporidium hominis (humans only natural host) ü Cryptosporidium parvum (infects bovines as well as humans) Cryptosporidiosis mainly affects children. It causes a self-limited diarrhoeal illness in healthy individuals. More severe illness in immunocompromised patients Cryptosporidium oocysts are highly infectious They are resistant to harsh conditions, including chlorine (levels usually applied in water treatment) Cryptosporidium causes diarrhoea by: ü Increasing intestinal permeability ü Chloride secretion ü Malabsorption In patients who are not immunocompromised, the infection is usually limited to the small intestine In immunocompromised patients, the biliary tract may be involved

BOF: 3.72 A 57-year-old female presents with a history of painful joints of several months duration. The pain mainly affects the small joints of both hands, they are swollen and stiff and the stiffness is worse in the mornings. She also complains of feeling generally unwell, fatigued, feverish and has lost weight. On examination you note that there is symmetrical swelling of her wrists, the metacarpophalangeal joints and the proximal interphalangeal joints. There are nodules at both elbows. In this patient which one of the following x-ray changes may be seen on x-rays of her hands? a) Subperiosteal resorption b) Periosteal reaction c) Looser’s zones d) Osteophyte formation e) Juxta-articular osteopaenia Answer: e) The clinical features suggest that the patient has rheumatoid arthritis. The x-ray changes that occur in rheumatoid arthritis are as follows: Early changes: ü Soft tissue swelling ü Juxta-articular osteopaenia ü Decreased joint space Later changes ü Bony erosions ü Subluxation ü Cysts ü Destruction of bone ends ü Complete joint destruction

271

Mastering MRCP BOF: 3.73 A 47-year-old female who is known to have rheumatoid arthritis presents with pain and reddening of her left eye. Her visual acuity is normal. There is no abnormality detected on examination with the ophthalmoscope. What is the likely diagnosis? a) Episcleritis b) Scleritis c) Anterior uveitis d) Acute closed angle glaucoma e) Keratitis Answer: b) The ocular complications of rheumatoid arthritis are: ü Scleritis ü Episcleritis ü Dry eyes The clinical features of the conditions listed in the question are as follows: Episcleritis ü Affects the tissue overlying the sclera ü Patients complain of irritation rather than pain ü Discrete areas of redness (dilation of conjunctival and episcleral vessels) are visible, these areas may be slightly raised sometimes Scleritis ü Causes a dull aching pain that may keep the patient awake at night. ü Vision may be reduced (contrast to episcleritis always normal) ü The injected sclera may be sectorial with a raised nodule, or there may be diffuse redness ü Often tender to touch through the closed eyelid Anterior uveitis ü Inflammation of the anterior uveal tract (iris and ciliary body) ü Painful red eye ü Photophobia ü Slight reduction in acuity ü Pupil may have an irregular shape due to adhesions between the inflamed iris and the lens (posterior synechiae) ü Inflammatory cells on the posterior cornea and in the anterior chamber Acute closed angle glaucoma ü Patient often feels unwell, and may have abdominal pain, nausea, or vomiting. ü The eye is painful ü Visual acuity often markedly reduced ü The conjunctiva is injected ü The cornea may have a hazy appearance. ü The pupil is fixed and in a mid-dilated position Keratitis ü Pain ü Photophobia ü Reduced vision

272

Mastering MRCP BOF: 3.74 A 26-year-old nurse suffers a needle stick injury on her right index finger. The patient is a 49-year-old male who has been admitted for investigation of pyrexia of unknown origin. You are asked for advice immediately. What first step should the nurse take? a) Suck on the finger b) Immerse the finger in bleach c) Press on it to stop bleeding d) Wash in running water e) Administer zidovudine, lamivudine and indinavir immediately Answer: d) The immediate step should be to wash well in running water. Bleeding should be encouraged rather than stopped. The patient should not suck on the wound Immersing in bleach is not advised Antiretroviral therapy should be started only if there is a risk of infection

BOF: 3.75 From which one of the following sites is BNP (brain natriuretic peptide, B-type natriuretic peptide) secreted? a) Floor of the 4th ventricle b) Right atrium c) Cardiac ventricles d) Arch of the aorta e) Coronary sinus Answer: c) Brain natriuretic peptide (BNP, also known as B-type natriuretic peptide ) is a cardiac neurohormone secreted by the cardiac ventricles as a response to ventricular volume expansion, pressure overload and increased wall tension Brain natriuretic peptide was initially identified in extracts from the porcine brain In humans it is produced mainly in the cardiac ventricles. The physiological actions of BNP includes decrease in systemic vascular resistance and central venous pressure as well as an increase in natriuresis BNP levels may be helpful in the diagnosis of heart failure and in selecting patients for further evaluation

BOF: 3.76 An 18-year-old male presents with redness, warmth, pain, tenderness and purulent discharge from the dorsum of his right hand. He says his hand had been bitten by his neighbour’s dog but he did not seek any medical treatment at the time. Which of the following is likely to be the infecting organism? a) Streptococcus viridans b) Bartonella henselae c) Staphylococcus aureus

273

Mastering MRCP e) Clostridium perfringens Answer: d) Pasteurella multocida is a small, gram-negative, non spore-forming coccobacillus with bipolar staining It often exists as a commensal in the upper respiratory tract of livestock, poultry, and domestic pets Infection in humans is often caused by an animal bite, scratch, or lick Local wound infections from animal bites are the most common human infections caused by P multocida

BOF: 3.77 A 34-year-old male presents with a history of a burning sensation in the mouth, facial flushing, headache, sweating, nausea, abdominal pain and diarrhoea. He says these problems started about an hour after he had a meal of sushi. When questioned further he said that the fish in the sushi was tuna. Which of the following toxins is responsible for his symptoms? a) Ciguatoxin b) Tetrodotoxin c) Enterotoxin d) Cytotoxin e) Scrombotoxin Answer: e) Scrombotoxin fish poisoning is caused by the bacterial spoilage of scrombroid fish; Scombridae family (tunas and mackerels) Histamine and other amines are formed by the action of the bacterial decarboxylase enzymes on histidine and other amino acids These toxins are resistant to high temperatures and cannot be destroyed by cooking

BOF: 3.78 What is the most common cause of death in patients with untreated acromegaly? a) Heart failure b) Colon cancer c) Prostate cancer d) Pituitary apoplexy e) Stroke Answer: a) Growth hormone plays an integral role in the development and maintenance of the structure and function of the heart. Acromegalic cardiomyopathy initially manifests as concentric left ventricular hypertrophy and diastolic dysfunction Untreated, it progresses to systolic heart failure Heart failure from acromegalic cardiomyopathy is one of the most common causes of death in acromegaly. Treatment to lower elevated growth hormone levels improves symptoms, improves exercise tolerance and results in echocardiographic improvement with regression of left ventricular hypertrophy and diastolic dysfunction

274

Mastering MRCP BOF: 3.79 Your house officer calls you to ask for advice. A patient on the ward has intractable nausea and vomiting following cytotoxic chemotherapy. You advise him to treat the patient with ondansetron. He asks you what the mechanism of action of ondansetron is. You reply that ondansetron is a: a) 5 HT3 receptor antagonist b) Dopamine receptor antagonist c) Histamine H1 receptor antagonist d) Histamine H 2 receptor antagonist e) Selective serotonin reuptake inhibitor Answer: a) Ondansetron is a selective 5 HT 3 receptor antagonist

BOF: 3.80 You are in the clinic with your consultant and you see a patient who is on flecainide for Wolff-ParkinsonWhite syndrome. Your consultant asks you what the mechanism of action of flecainide is. You reply that flecainide acts by: a) Blocking beta-adrenergic receptors b) Acting as a calcium antagonist c) Blocking sodium channels d) Blocking potassium efflux e) Increasing vagal activity Answer: c) Singh-Vaughan Williams Classification Classification of antiarrythmic drugs developed by Singh and Vaughan Williams based upon the drug's electrophysiological mechanisms of action Class I Act by blocking the sodium channel Subdivided into 3 subgroups, IA, IB, and IC based on their effects on repolarisation and potency towards blocking the sodium channel Subclass IA High potency as sodium channel blockers (prolong QRS interval) Prolong repolarisation (prolong QT interval) through blockade of potassium channels (usually) Subclass IB Lowest potency as sodium channel blockers, produce little change in action potential duration (no effect on QRS interval) in normal tissue, shorten repolarisation (decrease QT interval) Subclass IC Most potent sodium channel blocking agents (prolong QRS interval), little effect on repolarisation (no effect on QT interval) Class II Block beta-adrenergic receptors (slow sinus rhythm, prolong PR interval, no effect on QRS or QT intervals) Class III Prolong repolarisation (increase refractoriness) by blocking outward potassium conductance (prolong QT interval) Little effect on the rate of depolarisation (no effect on QRS interval)

275

Mastering MRCP Class IV Relatively selective AV nodal L-type calcium-channel blockers (slow sinus rhythm, prolong PR interval, no effect on QRS interval) Miscellaneous Digoxin, adenosine, magnesium, alinidine (chloride channel blocker) Actions don't fit the standard four classes

BOF: 3.81 A 65-year-old male presents with a history of fatigue, breathlessness on exertion on exertion, dizziness, fever and bleeding gums. On examination he is found to have hepatosplenomegaly. Investigations reveal anaemia and thrombocytopaenia. The blood film shows circulating blasts. His lactate dehydrogenase level is elevated and he also has an elevated uric acid level. Bone marrow aspiration shows 30% blasts in the bone marrow. In this patient the best determinant of prognosis is: a) Cytogenetic studies b) Lactate dehydrogenase levels c) Number of blasts in the bone marrow d) White cell count at time of diagnosis e) Degree of anaemia at time of diagnosis Answer: a) The clinical features and investigations suggest the patient has acute myeloid leukemia. In AML cytogenetic studies performed on bone marrow provide important prognostic information. Cytogenetics is the study of the structure of chromosome material

OF: 3.82 An 18-year-old male presents with a history of feeling tired and having bleeding gums. On examination he is noted to have hepatosplenomegaly. Investigations reveal anaemia with a high white cell count, predominantly lymphocytes with an excess of lymphoblasts. Bone marrow examination shows multiple lymphoblasts. In this patient which of the following is indicative of a poor prognosis? a) Presence of Philadelphia chromosome b) White cell count > 50x 109/L c) Severe anaemia d) T cell disease e) Platelet count < 50 x109/L Answer: a) In acute lymphoblastic leukemia (ALL) the indicators of a poor prognosis are: ü Adult ü Male ü Philadelphia chromosome present ü Presentation with CNS signs ü White cell count > 100 x 109 /L ü B-cell ALL

276

Mastering MRCP ü

Poor response to initial treatment

BOF: 3.83 A 70-year-old male is on goserelin for treatment of carcinoma of the prostate. What is the mechanism of action of goserelin? a) Oestrogen antagonist b) Testosterone antagonist c) Testosterone agonist d) Analogue of luteinising-hormone-releasing hormone e) Luteinising hormone antagonist Answer: d) Goserelin is a synthetic analogue of luteinising-hormone-releasing hormone (LHRH). Administration of goserelin causes an initial increase in luteinising hormone (LH) and follicle stimulating hormone (FSH). Following this chronic administration of goserelin results in sustained suppression of pituitary gonadotrophins. Serum testosterone levels fall. The levels reached are comparable to surgical castration. The mechanism by which this effect is achieved is unknown. It may be related to changes in the control of LH or down-regulation of LH receptors

BOF: 3.84 In organ transplantation, which of the following processes is implicated in hyperacute rejection? a) Presence of complement fixing antibodies b) T cell mediated immunity c) Presence of non complement fixing antibodies d) Ischaemia and reperfusion injury e) Infection Answer: a) Rejection of solid organs This may be: ü Hyperacute ü Accelerated ü Acute ü Chronic (late) Hyperacute rejection ü Within 48 h of transplantation ü Pre-existing complement-fixing antibodies to graft antigens (pre-sensitisation) Small-vessel thrombosis and graft infarction Accelerated rejection ü 3 to 5 days after transplantation ü Caused by pre-existing non complement-fixing antibodies to graft antigens Cellular infiltrate with or without vascular changes

277

Mastering MRCP Acute rejection ü 6 days to 3 months after transplantation ü T cell–mediated delayed hypersensitivity reaction to allograft histocompatibility antigens Mononuclear cell infiltration, varying degrees of haemorrhage, oedema, and necrosis Vascular integrity usually maintained However, vascular endothelium appears to be a primary target. Chronic rejection ü Graft dysfunction (without fever) ü Months to years after transplantation (sometimes weeks) Causes: ü Early antibody-mediated rejection ü Peri-procedural ischemia and reperfusion injury ü Drug toxicity ü Infection ü Vascular factors (hypertension, hyperlipidaemia) Proliferation of neointima consisting of smooth muscle cells and extra-cellular matrix (transplantation atherosclerosis) gradually occludes vessel lumina, resulting in patchy ischaemia and fibrosis of the graft

BOF: 3.85 A 76-year-old male has been admitted for a knee replacement. A routine ECG shows a narrow complex bradycardia with complete dissociation of P waves and the QRS complex. In this patient which of the following physical findings is likely to occur? a) Loud first hear sound b) Soft first heart sound c) Varying intensity of the first heart sound d) Split first heart sound e) Closing click Answer: c) The ECG changes would suggest the patient has complete heart block. In this condition the intensity of the first heart sound may vary

BOF: 3.86 A 53 -year-old female presents with a history of painful joints of several months duration. The pain mainly affects the small joints of both hands, they are swollen and stiff and the stiffness is worse in the mornings. She also complains of feeling generally unwell, fatigued, feverish and has lost weight. On examination you note that there is symmetrical swelling of her wrists, the metacarpophalangeal joints and the proximal interphalangeal joints. There are nodules at both elbows. Which of the following is a recognised respiratory complication of this condition? a) Apical fibrosis b) Apical cavitation c) Ankylosis of the costovertebral joints d) Bronchiectasis e) Fibrosing alveolitis

278

Mastering MRCP Answer: e) The clinical features of the case suggest the patient has rheumatoid arthritis. The respiratory complications of rheumatoid arthritis are: ü Fibrosing alveolitis ü Obliterative bronchiolitis ü Pleural effusions

BOF: 3.87 A 32-year-old female who is known to have ulcerative colitis presents with a painful ulcer on her left leg. On examination you note a large, deep ulcer superior to the left medial malleolus. The skin surrounding the ulcer is discolored, bluish black in colour and the edges of the ulcer are undermined. A biopsy is taken and this demonstrates an intense infiltration of neutrophils and vasculitis. In this patient which of the following medications would you use as first line treatment? a) Prednisolone b) Colchicine c) Dapsone d) Tacrolimus e) Azathioprine Answer: a) The clinical features and histology suggest the patient has pyoderma gangrenosum; the first line treatment for a large lesion is systemic steroids

BOF: 3.88 One of the medical students on your ward has been asked to write an essay on vancomycin resistant enterococci. He asks you which one of the following antibiotics is most likely to result in vancomycin resistance? a) Clarithromycin b) Ampicillin c) Ciprofloxacin d) Gentamicin e) Ceftriaxone Answer: e) Broad-spectrum antibiotics, particularly third-generation cephalosporins, have been identified as independent risk factors for VRE colonization. Other antibiotics that present a high risk are: Vancomycin, teicoplanin

BOF: 3.89 A 26-year-old male presents with a history of gradual onset of low back pain, which is increased at night. He has stiffness of his spine, which is worse in the morning. The stiffness is relieved by exercise. Magnetic resonance imaging of his spine reveals irregularities, erosions and sclerosis of his sacroiliac joints. In this patient which of the following is likely to be the earliest objective sign of spinal involvement? a) Loss of lateral flexion of the lumbar spine

279

Mastering MRCP b) Positive Trendelenburg test c) Increased lumbar lordosis d) Positive straight leg raising test e) Positive femoral stretch test Answer: a The clinical features and MRI changes suggest the patient has ankylosing spondylitis. Loss of lateral flexion of the lumbar spine is the earliest objective sign of spinal involvement in ankylosing spondylitis

BOF: 3.90 A 26 year old male who is a known asthmatic presents with acute onset of breathlessness and wheeze. On examination he is unable to complete sentences when speaking, he has a pulse rate of 120 beats per minute, his respiratory rate is 30 per minute, he finds it difficult to perform peak flow. You treat him with high dose oxygen, hydrocortisone 100 mg intravenously and prednisolone 50 mgs orally and you give him nebulised salbutamol 5 mgs and nebulised ipratropium bromide 0.5 mgs. 15 minutes later he is still breathless and tachycardic and unable to complete sentences. You repeat the nebuliser and as there is no improvement you treat him with: a) Magnesium sulphate intravenously b) Intravenous aminophylline c) Methyl prednisolone intravenously d) Intravenous amoxicillin and clarithromycin e) Intravenous salbutamol Answer: a) The patient has features of a severe attack of asthma. Treatment is with oxygen, nebulised salbutamol and ipratropium and steroids. If this does not work the next step would be intravenous magnesium. Next, one would consider an aminophylline infusion and intensive care

BOF: 3.91 Which one of the following is a common manifestation and often the earliest feature of metastatic intestinal carcinoid? a) Right sided cardiac valvular disease b) Cutaneous flushing c) Diarrhoea d) Asthma e) Abdominal crises Answer: b) The carcinoid syndrome is characterised by hepatomegaly, diarrhoea, flushing right heart valvular disease and asthma Clinical Features ü Cutaneous flushing, common manifestation, often the earliest sign of the syndrome ü Flushing can occur spontaneously, typically in the head and neck may be triggered by excitement, exercise, some types of food, or alcohol

280

Mastering MRCP ü Diarrhoea is the most common feature ü Malabsorption (occasionally) ü Pellagra (dementia, dermatitis, and diarrhoea) due to excessive diversion of dietary tryptophan to serotonin ü Carcinoid abdominal crisis (rare acute abdominal syndrome characterized by severe abdominal cramping without a mechanical bowel obstruction) due to intestinal ischemia caused by vasoactive substances elaborated by the tumour together with decreased mesenteric blood supply due to a perivascular fibrosis. ü Right-sided cardiac valvular disease (develops only after many years) ü Asthma (bronchoconstriction, attributed to serotonin, bradykinin or substance P)

BOF: 3.92 Which one of the following presentations is most likely in non-metastatic carcinoid tumour of the appendix? a) Cutaneous flushing b) Asthma c) Diarrhoea d) Right sided valvular heart disease e) Acute appendicitis Answer: e) Most carcinoid tumours are asymptomatic until metastases occur Carcinoid tumours of the appendix may present as acute appendicitis secondary to obstruction

BOF: 3.93 Which one of the following would result in downbeat nystagmus? A lesion at the level of the: a) Floor of the 4th ventricle b) Pontine tegmentum c) Anterior cerebellum d) Foramen magnum e) Medial vestibular nucleus Answer: d) Downbeat nystagmus refers to nystagmus where the fast phase is downwards. It occurs in lesions at the level of the foramen magnum such as herniation of the cerebellar tonsil (Arnold-Chiari malformation)

BOF: 3.94 A 35 year old male presents with sudden onset of loss of vision in his right visual field whilst reading. He has no significant medical history and he neither smokes nor uses alcohol or illicit drugs. Physical examination reveals a right homonymous hemianopia but no other abnormalities. Blood pressure is normal and cardiovascular examination does not reveal any abnormality. Magnetic resonance imaging reveals acute left occipital infarction and normal head and neck vessels.

281

Mastering MRCP Routine haematology and biochemistry are normal, there is no increase in inflammatory markers and biochemistry including lipid profiles are within the normal range. In this patient which of the following would you arrange next to determine the underlying cause of the patient’s condition? a) Contrast (bubble) echocardiography b) Lumbar puncture c) Blood cultures d) Cardiac catheterisation e) Carotid angiography Answer: a) In a young patient presenting with a stroke it is essential to try and determine the underlying cause of the stroke. Patent foramen ovale by causing paradoxical embolism may cause stroke

BOF: 3.95 A 78-year-old male is in your ward. He has metastatic bronchial cancer and is in severe pain with bone metastases. He is known to have chronic obstructive pulmonary disease and has also has ischaemic heart disease and Parkinson’s disease. He is conscious rational and alert. In this patient which of the following will influence your decision to make this patient not for resuscitation? a) Wishes of the family b) Futility of treatment c) Co-morbidity d) Patient’s wishes e) Age of the patient Answer: d) In a mentally competent patient, a do not resuscitate decision should be discussed with the patient and the decision should be in accordance with the patient’s wishes

BOF: 3.96 A healthy 47-year-old male was found to have serum potassium of 6.5 mmol/L when he had blood sent for a medical checkup by his GP. The blood sample was not haemolysed. He was admitted to the emergency admission unit but on admission his serum potassium was found to be 4.5 mmol/L There were no ECG changes; he was not on any medication. What is the most likely cause of this phenomenon? a) Pseudohyperkalaemia b) Laboratory error c) Acute renal failure d) Chronic renal impairment e) Excessive dietary potassium intake Answer: a) Normally there is some potassium release form lysis of white cells and platelets when blood clots Artefactually high potassium may be seen in: a) Delayed centrifugation (especially if blood is collected the previous day)

282

Mastering MRCP b) Uncentrifuged blood stored in a refrigerator (slows down Na-K ATPase c) Very high white cell and or platelet counts d) Abnormally fragile red cells (this will usually manifest as haemolysis) e) Familial pseudohyperkalaemia An useful test is to collect paired plasma and serum samples to check if potassium is significantly lower in the former. Persistent modest Hyperkalaemia for no apparent reason may also be seen in hyporeninaemic hypoaldosteronism or type 4 renal tubular acidosis although there is often some evidence of renal impairment

BOF: 3.97 A 37-year-old female undergoes radioiodine therapy for Grave’s disease. Her thyroid function tests before and after treatment are as follows: Before: Free T4: 31 pmol/L (9-22) Total T3: 3.5 nmol/L (0.92.6) TSH: < 0.02 mU/L (0.44) 2 weeks post radioiodine therapy: Free T4: 12pmol/L Total T3: 1.4 nmol/L TSH> <0.02 mU/L Why is the TSH still suppressed? a) Laboratory error b) Incomplete radio-iodine therapy c) Patient has been started on thyroxine d) Patient is still on carbimazole e) TSH usually takes about 6 weeks to stabilise to new level Answer: e) TSH should not be repeated before 6-8 weeks

BOF: 3.98 A consultant anaesthetist is about to induce a 68 year old male for an elective total knee replacement when he notices that on the blood tests done in the morning of the operation all are normal apart from a serum amylase of 570U/L (0-100). There is no history of abdominal pain. The serum amylase was requested inadvertently by a new junior doctor. Other biochemistry results are normal with normal urea and electrolytes but a high amylase has been noted previously but the patient had not had abdominal pain at that time either. The anaesthetist refuses to induce the patient and requests an urgent abdominal ultrasound scan which is normal. What is the likely cause of the high amylase in this patient? a) Macroamylase b) Undetected pancreatitis c) Renal failure

283

Mastering MRCP d) Contamination of the blood sample e) Duodenal perforation Answer: a) Certain enzymes such as amylase, CK, AST and hormones like prolactin may combine with Ig molecules. The resultant molecule is too large to be filtered by the glomeruli. This results in a benign condition referred to as macroenzyme (in this case macroamylase). Macroenzyme is usually a diagnosis of exclusion and its levels may fluctuate. The laboratory can do special tests to confirm its presence. Certain tumours may also produce enzymes, which may lead to misdiagnoses see

BOF: 3.99 A 26-year old known Type 1 diabetic was admitted to the A&E with confusion, dehydration and smelling of alcohol or ‘pear drops’. Her admission bloods showed plasma glucose of 35.7 mmol/L and arterial blood gases revealed a significant metabolic acidosis. Her urine showed only traces of ketones, though was 3+ for glucose on dipstick testing. Treatment was initiated with normal saline and insulin and within a few hours the blood glucose fell to 12.1 mmol/l.. Which of the following statements is most likely to be true on retesting the urine with dipstick? a) 3+ glucose, no ketones b) 1+ glucose, 2+ ketones c) No glucose, no ketones d) 1+ glucose, no ketones e) No change from the pre-treatment result Answer: b) This patient has Diabetic ketoacidosis. Urine ketones are an important bedside test in such patients but can be misleading. Urine dipstick for ketones uses a nitroprusside reagent. Ketones found in the urine are acetoacetate, acetone and beta-hydroxybutyrate. Both acetoacetate and acetone react with nitroprusside, but beta-hydroxybutyrate does not (or does so very slightly). In the presence of acidosis ketones exist mainly as beta-hydroxybutyrate, which gets converted to acetoacetate as the acidosis is corrected. Hence urine may test only mildly positive for ketones in the acute stage of the illness. Correction of the acid-base abnormalities may result in a paradoxical increase in urine ketones on dipstick testing

BOF: 3.100 Which of the following is NOT a cause of high anion gap metabolic acidosis a) Lactic acidosis b) Methanol poisoning c) Renal failure d) Renal tubular acidosis e) Diabetic ketoacidosis Answer: d) Renal tubular acidosis results in hyperchloraemic metabolic acidosis with a normal anion gap

284

Mastering MRCP BOF: 3.101 An 18-year-old male has learning difficulties. On examination he is very tall with long upper and lower limbs, his face looks thin and elongated, he as long spidery fingers. On examination of his eyes he is noted to have dislocation of his lens. What is the most likely diagnosis? a) Marfan’s syndrome b) Homocystinuria c) Klinefelter’s syndrome d) Kallman’s syndrome e) Gigantism Answer: b) In all of the above conditions the individual would be very tall. In all the conditions apart from gigantism they would also have eunuchoid proportions with very long upper and lower limbs. Dislocation of the lens would be seen in both Marfan’s syndrome and homocystinuria. However, learning difficulties would occur only in homocystinuria

BOF: 3.102 A young male has been referred to the clinic for investigation of haematuria and proteinuria. On examination he is noted to have high frequency high frequency nerve deafness, conical bulging of the lens of the eye and perimacular retinal flecks. What is the common mode of inheritance of this condition? a) Autosomal dominant b) X-linked c) Mitochondrial genetic disorder d) Trinucleotide repeat disease e) Sex chromosome disorder Answer: b) There are many causes of proteinuria and haematuria. When accompanied by deafness, lenticonus (conical bulging of the lens of the eye ) and perimacular retinal flecks one should think of Alport’s syndrome

BOF: 3.103 A middle-aged female who is known to have scleroderma presents with a history of explosive diarrhoea. Investigations reveal a macrocytic anaemia and she is found to have vitamin B12 deficiency. Which of the following is likely to have caused this? a) Crohn’s disease b) Infestation with Diphyllobothrium latum c) Treatment with cholestyramine d) Small bowel bacterial overgrowth e) Gastritis Answer: d) In patients with scleroderma small bowel bacterial overgrowth may occur and this can cause diarrhoea. Vitamin B12 deficiency occurs because the bacteria utilise vitamin B12 and reduce its availability for absorption.

285

Mastering MRCP Terminal ileal Crohn’s disease can cause B12 deficiency but that is postulating another illness. The same for infestation with the fish tapeworm Diphyllobothrium latum which can also cause B12 deficiency (postulation of another illness) Cholestyramine is a bile acid sequestrant, which adsorbs bile salts and thereby reduces diarrhoea caused by malabsorption. It does not interfere with B12 absorption. In pernicious anaemia gastritis does occur but gastritis per se does not cause B12 deficiency

BOF: 3.104 Which of the following is useful in treating mumps encephalitis? a) Ribavarin b) Acyclovir c) Interferon d) Amantadine e) Supportive treatment Answer: e) No antiviral agent (at present) is known to be of use in mumps encephalitis

BOF: 3.105 A patient is admitted to your ward with a community-acquired pneumonia. He is started on intravenous antibiotics. Subsequently he complains of chest pain and your house officer performs an ECG, which shows a prolonged QT interval. Which of the following antibiotics is likely to have caused this? a) Amoxycillin b) Erythromycin c) Augmentin d) Penicillin e) Gentamicin Answer: b) The following drugs are known to cause QT prolongation: Antibiotics Erythromycin Antidepressants Tricyclic Neuroleptics Phenothiazines Antiarrythmics Quinidine Amiodarone Antihistamines Non-sedating

286

Mastering MRCP BOF: 3.106 A 56-year-old male presents with epistaxis. On examination you note a bruit over the right base of his lungs. Investigations reveal iron deficiency anaemia. What is the likely diagnosis? a) Wegener’s granulomatosis b) Tuberculosis c) Colorectal cancer d) Osler-Rendu-Weber syndrome e) Pseudoxanthoma elasticum Answer: d) There are many causes of epistaxis However, bruits over the lung are not common and indicate an arteriovenous malformation (aces for paces page 267). These occur in hereditary haemorrhagic telangiectasia

BOF: 3.107 A 23-year-old female presents with an erythematous blistering eruption over her entire body She also has oral ulcers and complains of a burning sensation when she passes urine Her GP has treated her with antibiotics for a sore throat What is the likely diagnosis? a) Pemphigus vulgaris b) Pemphigoid c) Erythema nodosum d) Erythema marginatum e) Stevens-Johnson syndrome Answer e) Stevens Johnson syndrome or erythema multiforme major may occur as a drug reaction It involves the mucous membranes and may be associated with systemic symptoms Pemphigus and pemphigoid usually occur in older people They are not related to antibiotic therapy Erythema nodosum and erythema marginatum do not blister

BOF: 3.108 A 52-year-old female is on regular haemodialysis for end stage renal failure. She complains of feeing extremely tired and breathless on exertion. Her blood pressure prior to dialysis is 190/105 Her blood results before dialysis show: Hb 8.6 g/dl Serum calcium 1.9 mmol/l (corrected) Serum potassium 7.0 Creatinine 1500 In this patient which of the following would you consider of greatest benefit? a) Sevelemar b) Calcitriol

287

Mastering MRCP c) Erythropoietin (EPO) d) Ramipril e) Calcium carbonate Answer: c) All the above may be used in patients with chronic renal failure. However, in this patient the main concern are the symptoms of tiredness and breathlessness on exertion, which are probably related to her anemia. In this case correction of anemia by using erythropoietin would be most beneficial

BOF: 3.109 A 36 year old female presents with a history of nausea, vomiting and abdominal pain. She had lost weight over the month prior to presentation. On examination she looked unwell, was tachycardic and her blood pressure was 90/60 supine, 70/40 erect. Investigations reveal: Blood urea 9.8 mmol/L Sodium 127 mmol/L Potassium 5.6 mmol/L Glucose 2.1 mmol/L Free thyroxine 50 mmol /L (70-140 mmol/L) TSH 8 mu/L (0.5 –5 mu/L) In this patient which one of the following would you give as initial treatment? a) Intravenous antibiotics b) Intravenous omeprazole c) Intravenous hydrocortisone d) Intravenous thyroxine e) Glucagon Answer: c) The clinical and biochemical feature suggest the patient has acute hypoadrenalism (Addisonian crisis) An elevated TSH together with low T4 and T3 can occur in hypoadrenalism. If co-existent hypothyroidism is suspected it is best to treat with steroids first and once cortisol replacement is established the patient should be treated with T3 as this has a shorter half life and may be conveniently discontinued and TSH monitored later. Very rarely hypoadrenalism may present with features of hypothyroidism. Under these circumstances it is prudent to check cortisol levels before giving thyroxine as treating with steroids will exacerbate the glucocorticoid deficiency

BOF: 3.110 A 40-year-old female presents with a history of problems with her legs at night. She feels she needs to move her legs and is unable to keep them still. She relieves symptoms by getting up and pacing around the room. She does not have any symptoms during the day. She does not get a good night's sleep on account of these symptoms and feels tired during the day. Which of the following would you use to treat her? a) Ropinirole b) Amitryptiline

288

Mastering MRCP c) Carbamazepine d) Citalopram e) Beta blockers Answer: a) The patient has restless legs syndrome The criteria for diagnosing restless legs syndrome are as follows: 1) Compelling urge to move the legs · Associated with paraesthesiae /dysaestheiae 2) Motor restlessness · Floor pacing, tossing and turning in bed, rubbing legs 3) Symptoms worse at rest or present exclusively at rest · Relieved by activity 4) Circadian variation · Symptoms increased in the evening or at night Associated features (not required for diagnosis): · Sleep disturbances · Daytime fatigue · Normal neurological exam · Involuntary, repetitive, periodic, jerking limb movements, either in sleep or while awake and at rest Ropinirole: non ergoline dopamine agonist, approved for the treatment of restless legs syndrome, pramiprexole is also approved for this condition

BOF:3.111 Which one of the following may be detected using the technique of Southern Blotting: a) DNA b) RNA c) Immunoglobulins d) Human leukocyte antigens e) Enzymes Answer: a) Southern blotting is used to detect DNA

BOF:3.112 A 26-year-old female who has had a right hemicolectomy and terminal ileal resection for Crohn’s disease presents with watery diarrhoea . She does not pass blood or mucous , there is no abdominal pain and she has not lost any weight . She is not on any drugs and there is no history of any illness other than Crohn’s disease . On examination there are no abnormalities detected and investigations reveal normal haematology and biochemistry with no increase in inflammatory markers. Her coeliac screen is negative . Which of the following would you use in order to reduce her diarrhoea? a) Loperamide b) Steroids c) Infliximab

289

Mastering MRCP d) Codeine phosphate e) Colestyramine Answer: e) Bile acids are normally reabsorbed in the terminal ileum Bile acid malabsorption may occur as a result of · Disease or partial resection of the terminal ileum · Rarely a genetic defect in the distal ileum Bile acid malabsorption Is associated with: · Diarrhoea with or without steatorrhoea · Gallstone disease · Hyperoxaluria Bile acid malabsorption is diagnosed using · Selenohomocholic acid taurine test (SeHCAT scan) Management of bile acid induced diarrhoea · Low-fat diet · Cholestyramine (moderate bile acid diarrhoea) · Cholylsarcosine (replacement therapy, severe bile acid malabsorption)

BOF: 3.113 A 17 year old female is seen at the ophthalmology clinic. The registrar who sees her notices lens displacement. Which one of the following causes displacement of the lens? a) Ehlers-Danlos syndrome b) Homocystinuria c) Dystrophia myotonica d) Cystinosis e) Oxalosis Answer :b) Downward displacement of the lens occurs in homocystinuria

BOF: 3.114 A 66 year old male presents with an ulcer on his cheek. He says he has had an ulcer on his cheek for some time but recently he has noticed that it seems to be becoming larger. He has no significant past medical history apart from mild hypertension controlled with a thiazide diuretic. On examination you note an ulcer with an irregular border. It is of variegate colour. What is the likely diagnosis? a) Malignant melanoma b) Basal cell carcinoma c) Kaposi’s sarcoma d) Squamous cell carcinoma e) Pyoderma gangrenosum Answer: a)

290

Mastering MRCP The recent change in size should alert you to the possibility of malignancy and the variegate colour would suggest malignant melanoma Mnemonic for Malignant Melanoma ü Asymmetrical lesion. ü Border irregular. ü Colour multiple colours ü Diameter: >6 mm more likely

BOF: 3.115 A 56 year old male who is known to have chronic obstructive pulmonary disease is admitted with an infective exacerbation of his condition. His condition deteriorates and he is admitted to ICU where he requires mechanical ventilation. He survives the ICU stay and is to be discharged home. Which of the following would increase the chances of survival in this patient? a) Antibiotics b) Steroids c) Venesection d) Diuretics e) NIV ( non invasive ventilation ) Answer: e) COPD patients who require mechanical ventilation and survive after an ICU stay: ü NIV is an independent factor associated with a better long-term survival ü Especially in those with a documented cause of respiratory failure

BOF: 3.116 A 46 year old female attends the hypertension clinic for follow up. You examine her eyes and note that the left pupil is 8 mm and the right pupil is 4mm. The left pupil reacts sluggishly to both light and accommodation. The likely diagnosis is: a) Argyll Robertson pupil b) Holmes Adie pupil c) Hippus d) Morphine abuse e) Horner’s syndrome Answer :b) Holmes-Adie pupil ü Large and irregular ü Reaction to light and or accommodation slow and incomplete ü Once the pupil has constricted it remains small for a long time (tonic pupil) Holmes-Adie syndrome Association of a Holmes-Adie pupil with absent deep tendon jerks. Clinical features ü patients often young women ü unilateral (80% )

291

Mastering MRCP ü ü ü ü ü ü

dilated pupil decreased consensual and direct light reflex tonic pupil decreased accommodation reflex hypersensitivity to pilocarpine solution decreased tendon reflexes

BOF: 3.117 A 24 year old female presents with recurrent attacks of dizziness and sweating. During these episodes she is noted to be hypoglycemic. Her father is known to be diabetic. She undertakes a prolonged fast and during this period she is noted to have hypoglycemia with elevated insulin levels and C-peptide levels. The likely diagnosis is: a) Insulinoma b) Sulphonylurea abuse c) Insulin abuse d) Dumping syndrome e) Hysteria Answer :a) Insulinoma ü Neuroendocrine tumour derived from pancreatic islet cells ü Produce excessive amounts of insulin Diagnosis ü Glucose levels low ü Serum insulin levels high ü C-peptide levels elevated ü Proinsulin levels elevated ü Screening for sulphonylurea negative

BOF: 3.118 The hilum of the left kidney is separated from the peritoneum by: a) Body of the pancreas b) Duodenum c) Spleen d) Splenic flexure of the colon e) Jejunum Answer: a) The hilum of the left kidney is separated from the peritoneum by the body of the pancreas and the splenic vessels

292

Mastering MRCP BOF: 3.119 A 42-year-old man presents severe crushing retrosternal chest pain radiating to his left arm and into his neck. There is no significant past illnesses but he admitted to snorting cocaine prior to the onset of the pain. On examination you note the patient’s skin is cold and clammy , his pulse rate is 110 beats per minute , BP 100/60 JVP not elevated , heart sounds normal no murmurs , lungs are clear. ECG shows ST segment elevation in the anterior chest leads. In this patient which one of the following methods of treatment would you institute? a) Percutaneous coronary intervention b) Thrombolysis c) Beta blockade d) Low molecular weight heparin e) Glycoprotein IIb/IIIa inhibition Answer: a) Cocaine-induced myocardial infarction Pathophysiology ü Vasospasm ü Thrombosis ü Increased myocardial oxygen demand Percutaneous coronary intervention allows: ü Mechanical reperfusion ü Local delivery of vasodilators

BOF: 3.120 A 63 year old male presents to the clinic with a history of abnormal vision. He says that everything he sees has a blue tinge The patient is known to have ischaemic heart disease, well controlled atrial fibrillation, hypertension and diabetes mellitus. Which one of the following may account for the visual disturbance? a) Digoxin b) Amiodarone c) Metformin d) Sildenafil e) Ramipril Answer: d) Sildenafil ü Selective phosphodiesterase type 5 (PDE5) inhibitor ü Treat erectile dysfunction PDE5 inhibitors Direct effects on the retina ü Blue-tinged vision ü Light sensitivity

293

Mastering MRCP BOF: 3.121 You are in the rheumatology clinic with a medical student. The last patient seen was a young man with ankylosing spondylitis and the consultant had recommended starting the patient on anti TNF antibodies The medical student asks you what the main site of production of TNF (tumour necrosis factor) is. You reply that TNF is mainly produced by: a) Lymphocytes b) Fibroblasts c) Macrophages d) Mast cells e) Endothelial cells Answer: c) Tumour necrosis factor (TNF, cachexin or cachectin) ü Cytokine ü Involved in systemic inflammation ü Stimulates the acute phase reaction TNF production: Mainly by ü Macrophages But also produced by ü Lymphoid cells ü Mast cells ü Endothelial cells ü Cardiac myocytes ü Adipose tissue ü Fibroblasts ü Neuronal tissue

BOF: 3.122 A 26 year old male is seen in your clinic. You note that he has multiple café au lait patches on his skin, there is axillary freckling and there are multiple flesh coloured papules and nodules distributed all over his body. In his eyes you note small circular pigmented hamartoma. In this patient the chromosomal abnormality is on: a) Chromosome 17 b) Chromosome 22 c) Chromosome 19 d) Chromosome 15 e) Chromosome 21 Answer: a) The clinical features fit in with neurofibromatosis type 1 (NF) or peripheral neurofibromatosis. The abnormality is on chromosome 17

294

Mastering MRCP BOF: 3.123 You see a patient in clinic. He is a 30 year old male with, frontal balding, ptosis, bilateral cataract, wasting of facial and sternomastoid muscles. When you shake hands with him he finds it difficult to let go of your hand. You arrange an ECG on this patient. Which one of the following is common in this condition? a) ST segment elevation anterior chest leads b) T inversion in lateral leads c) T inversion in anterior chest leads d) Atrioventricular conduction disturbance e) Q wave in inferior leads Answer: d) The clinical features would suggest that the patient has dystrophia myotonica This is a multi-system condition and the heart may be involved with cardiomyopathy or conduction disturbances ECG findings include: ü Atrioventricular block ü Left anterior hemi block ü Intraventricular conduction delay

BOF: 3.124 A 54 year old male presents with a complaint that his ears have become very large. No other history of note. On examination, his ears do not look larger than normal. You look through his notes and see that he has seen numerous doctors with the same complaint and all have made a note that there does not seem to be any abnormality in the size or shape of his ears. In this patient the most likely diagnosis is: a) Acromegaly b) Hydrophobia c) Dysmorphophobia d) Tomophobia e) Trichophobia Answer :c) Dysmorphophobia (body dysmorphic disorder, BDD) ü Fixation on an imaginary flaw in the physical appearance

BOF: 3.125 A 72 year old female presents with a painful right ankle. She is known to have hypertension and this is well controlled on a thiazide diuretic. She does not smoke or drink and there has been no previous joint problems. On examination she is febrile, the right ankle is swollen, erythematous, warm, tender and range of movement is markedly diminished. In this patient which one of the following tests would you perform next? a) Joint aspiration

295

Mastering MRCP b) Blood cultures c) Serum uric acid level d) MRI of the ankle e) X-ray of the ankle Answer: a) The clinical features suggest the patient has septic arthritis and the next step would be to aspirate to confirm the diagnosis and obtain samples for culture and sensitivity

BOF: 3.126 A 74 year old male presents with a history of breathlessness. He has mild diet controlled diabetes mellitus but apart from this he had been reasonably well. He smoked 10 cigarettes a day and consumed 12 units of alcohol per week. He was a retired lawyer. On examination he did not appear to have lost weight, he looked pale and there was generalised lymphadenopathy and splenomegaly. Investigations Hb 6.3 g/dL WBC 120 x109 /L Platelets 45 x109/L Blood film showed mainly mature lymphocytes In this patient which one of the following would you commence? a) Prednisolone b) Vincristine, prednisolone, doxorubicin, cyclophosphamide c) Melphalan d) Mustine, oncovin, procarbazine , prednisolone e) Hydroxyurea Answer: a) The patient has chronic lymphocytic leukemia. The anaemia may be due to marrow failure. Prednisolone is used for treatment of marrow failure and haemolysis.

BOF:1 A 38 year old female presents with a history of feeling weak, pain in her muscles and bones .No significant past medical history. She is a vegetarian . She is married and has 4 children . on examination you note that she has a waddling gait and she finds it difficult to get up from the seated position . You ask her to squat and rise up and she finds this difficult without using her arms for support. Investigations Corrected calcium 1.8 mmol/L Phosphate 0.6 mmol/L Alkaline phosphatase 330 U/L

296

Mastering MRCP In this patient which one of the following would you prescribe ? a) Magnesium glycerophosphate b) NSAIDs c) Calcium and vitamin D d) Bisphophonate e) Multivitamins Answer :c) The patient has osteomalacia and treatment would be with calcium and vitamin D supplementation

BOF:2 A 42 year old male presents with central chest pain . There is no significant past medical history and he is not on any drugs. He smokes 10 cigarettes a day but does not drink . On examination he is febrile , pulse 100 beats per minute regular , BP 130/80 JVP not raised , heart sounds were normal and there were no murmurs . Lungs were clear. ECG shows widespread ST segment elevation and T inversion. In this patient your next line of management would be : a) Thrombolysis b) Low molecular weight heparin c) Aspirin d) NSAIDs e) Angioplasty Answer: d) The patient has fever tachycardia but no other physical signs. The ECG shows widespread ST elevation and T inversion. This suggests that the patient has pericarditis and treatment should be with NSAIDs

BOF: 3 A 32 year old female presents with a history of pain and swelling of her joints. She has not had any serious illness in the past and is not on any medications. On examination you note that she has swelling, of the distal interphalangeal joints of the index and middle fingers of her right hand and swelling of the distal interphalangeal joints of the 4th and 5th fingers of her left hand. Investigations reveal elevated inflammatory markers. You do a careful examination to look for: a) Malar rash b) Nodules over her elbows c) Iritis d) Pitting of her nails e) Heberden’s nodes Answer: d) The asymmetrical arthropathy affecting the distal interphalangeal joints would suggest that the patient has psoriatic arthropathy. This is associated with pitting of the nails.

297

Mastering MRCP

BOF: 4 A 68 year old male complains of breathlessness. He has had an aortic valve replacement 5 years prior to presentation and is on long term warfarin. Investigations: Hb 10.2 g/dL MCV 68 fl Platelets 375x 109/l WBC 6.4 x109/l INR 3.0 Gastroscopy was reported as normal. Which of the following would you request next? a) Transthoracic echocardiogram b) Blood cultures c) Transoesophageal echocardiogram d) Colonoscopy e) Bone marrow examination The patient has iron deficiency anaemia and in a patient of this age colon cancer becomes a strong possibility and hence colonoscopy is essential. Angiodysplasia of the caecum may occur in association with aortic stenosis but this usually resolves when the valve is replaced.

BOF: 5 In response to hypoxia, which one of the following blood vessels would constrict? a) Coronary arteries b) Pulmonary arteries c) Skin arteries d) Renal arteries e) Hepatic arteries Answer: b) Hypoxic pulmonary vasoconstriction is a physiological phenomenon where pulmonary arteries constrict in response to hypoxia. This allows the lung to redirect blood flow from poorly aerated regions to better aerated regions

BOF: 6 A 45 year old female presents with a history of left flank pain and haematuria. On examination an abdominal mass is palpable in the left flank. The patient undergoes an MRI and the radiologist reports the presence of an angiomyolipoma in the left kidney Which of the following underlying conditions is the patient likely to have? a) Tuberose sclerosis b) Neurofibromatosis c) Sturge Weber syndrome

298

Mastering MRCP d) Von Hippel Lindau syndrome e) Osler Rendu Weber syndrome Answer: a) Angiomyolipoma ü Benign renal neoplasm ü Composed of fat, vascular, and smooth muscle elements May be: ü Isolated angiomyolipoma ü Angiomyolipoma associated with tuberous sclerosis Clinical features ü Majority asymptomatic ü Main presenting symptoms are related to intratumoral or retroperitoneal haemorrhage Other symptoms (patients with associated tuberous sclerosis) ü Flank or abdominal pain ü Weight loss ü Hypertension ü Fever, ü Nausea ü Almost half of these cases tumours are bilateral ( follow up important )

BOF: 7 Which one of the following is a recognised effect of BNP (brain natriuretic peptide, B type natriuretic peptide)? a) Peripheral vasoconstriction b) Constriction of pulmonary arteries c) Increased sympathetic tone d) Inhibition of renin angiotensin aldosterone system e) Decreased natriuresis Answer :d) BNP release ü Left ventricular wall stretched by volume overload ü Cardiac muscle cells secretes BNP precursor ü Precursor converted to pro BNP ü Pro BNP cleaved into · C-terminal BNP (biologically active) · N-terminal BNP (biologically inactive) Action of natriuretic peptides ü Suppresses sympathetic nervous system ü Suppresses renin angiotensin aldosterone system ü Stimulates natriuresis ü Decreases peripheral vascular resistance ü Increases smooth muscle relaxation

299

Mastering MRCP

BOF: 8 A young female who is known to have Turner’s syndrome is seen in the clinic. You have a medical student in with you and he asks you what cardiac abnormalities may occur in this condition. You reply that the main abnormalities are coarctation of the aorta, bicuspid aortic valve and: a) Aortic stenosis b) Ventricular septal defect c) Atrial septal defect d) Pulmonary stenosis e) Hypertrophic obstructive cardiomyopathy Answer: a) The main cardiac abnormalities that may occur in Turner’s syndrome are coarctation of the aorta, bicuspid aortic valve and aortic stenosis

BOF: 9 A 56 year old male is reviewed in the hypertension clinic. His hypertension is well controlled on bendroflumethiazide. Routine biochemistry shows his potassium level to be 3.2 mmol /L Which of the following best describes the mechanism by which bendroflumethiazide causes hypokalaemia? a) Increased filtration of potassium through the glomerulus b) Increased amounts of sodium reaching the distal tubules c) Direct action on potassium co-transporter d) Inhibition of the renin angiotensin aldosterone system e) Inhibition of anti diuretic hormone Answer: b) Bendroflumethiazide ü Inhibits active chloride reabsorption at the early distal tubule via the Na-Cl co-transporter ü Increases excretion of sodium, chloride, and water ü Also inhibits sodium ion transport across the renal tubular epithelium through binding to the thiazide sensitive sodium-chloride transporter ü Increases potassium excretion via the sodium-potassium exchange mechanism

BOF: 10 A34 year old female presents with rapid onset loss of vision in her right eye and retro-orbital pain. On examination the fundus looked normal. In this patient the most likely diagnosis is: a) Optic neuritis b) Retinal vein thrombosis c) Central retinal artery occlusion d) Optic atrophy secondary to high intracranial tension e) Leber’s optic atrophy Answer :a) Optic neuritis ü Inflammation of the optic nerve

300

Mastering MRCP Papillitis ü Optic nerve head affected Retrobulbar neuritis ü Nerve affected more posteriorly Clinical features ü Visual impairment ü Eye pain ü Dyschromatopsia (impairment of colour vision)

BOF: 11 A 46 year old homeless male is brought in to casualty. No formal history can be taken as the patient is drowsy. On examination he is drowsy but responds to pain, pulse 110 beats per minute BP 140 /80 Heart sounds were normal no murmurs, lungs were clear Investigations reveal normal arterial oxygen saturation, blood glucose within normal range and a metabolic acidosis with and increased anion gap Which one of the following conditions would account for this: a) Renal tubular acidosis b) Severe diarrhoea c) Pancreatic fistula d) Methanol ingestion e) Pyelonephritis Answer: d) Elevated anion-gap metabolic acidosis CVS ü Hypoperfusion Lactic acidosis (L-lactate) RS ü Carbon monoxide/cyanide poisoning L-lactate GIT ü Short bowel syndrome D-Lactataemia E&M ü Diabetic ketoacidosis Acetoacetate/beta-hydroxybutyrate ü Maple syrup urine disease KUS ü Renal failure Urate, hippurate, sulphate and phosphate LMS ü Rhabdomyolysis Direct proton release from lysed muscle cells Toxins

301

Mastering MRCP ü Alcoholic ketoacidosis Acetoacetate/beta-hydroxybutyrate ü Methanol or formaldehyde poisoning Formate ü Ethylene glycol poisoning Glycolate and oxalate ü Paraldehyde poisoning Variety of organic acids ü Sulphur poisoning ü Sulphate. ü Pyroglutamic acidaemia 5-oxoprolate Drugs ü Biguanide toxicity (lactic acid ) ü Salicylate poisoning Normal anion-gap metabolic acidosis (hyperchloraemic acidosis) GIT ü Gastrointestinal bicarbonate loss Diarrhoea, pancreatic or intestinal fistulae, ureterosigmoidostomy ü Parenteral or hyperalimentation feeding KUS ü Renal tubular acidosis ü Pyelonephritis Drugs and Toxins ü Ammonium chloride ü Carbonic anhydrase inhibitors

BOF: 12 A 24 year old male presents with a history of painful blistering on his penis. He gives a history of unprotected sexual intercourse several weeks prior to the onset of symptoms. On examination you note blisters on an erythematous base. Some blisters had ruptured leaving shallow ulcers and crusts. In this patient, which of the following would you use as treatment? a) Oral acyclovir b) Local acyclovir c) Topical lignocaine d) Topical podophylline e) Simple analgesics Answer: a) The clinical features suggest the patient has primary genital herpes. This should be treated with a systemic antiviral agent.

302

Mastering MRCP BOF: 13 A 63 year old male who is known to have chronic obstructive pulmonary disease presents with worsening breathlessness and swelling of his ankles. In this patient which of the following physical signs would make you suspect that the patient has left ventricular failure? a) Bilateral basal crepitations b) Pitting ankle oedema c) Pulsus alternans d) Elevated JVP e) Third heart sound Answer: c) Pulsus alternans refers to alternating high and low volume pulses and signifies left ventricular failure The other signs may occur in COPD and cor pulmonale

BOF: 14 A 56 year old male who is known to have COPD has been prescribed an inhaled bronchodilator and demonstrates symptomatic relief with this therapy In this patient use of an inhaled corticosteroid inhaler would: a) Improve life expectancy b) Reduce frequency of exacerbations c) Reduce inhaled bronchodilator requirement d) Improve muscle mass e) Improve exercise tolerance Answer :c) Treatment with inhaled steroids in COPD ü Improves spirometry data ü Decreases inhaled β2-agonist consumption in about 25% of patients with stable COPD ü Rate increases to about 75% in patients who respond to β2 -agonist inhalation

BOF: 15 A 72 year old male presents with a history of sudden onset weakness of the right side of his body of 6 hours duration. He has not seen his general practitioner for years and is not on any medication. He smokes 10 cigarettes a day and consumes 20 units of alcohol per week. On examination you note a right sided weakness involving his face arm and legs. The limbs are flaccid on that side and reflexes are absent. BP 190/120, examination of the optic fundus reveals thickening of the arteries and arteriovenous nipping. A CT scan of his head does not show a haemorrhage. In this patient your next line of management would be: a) Alteplase b) Amlodipine c) Aspirin d) Clopidrogel e) Warfarin

303

Mastering MRCP Answer: c) The patient has an ischemic stroke for 6 hours. It is too late for thrombolysis and treatment would be with aspirin and dipyridamole. Clopidrogel would be used in those who are aspirin intolerant. Antihypertensives are used after 72 hours unless there is hypertensive end organ damage. Grade II retinopathy on examination of the fundus would be against this.

BOF: 16 A 23 year old female has been referred to the hypertension clinic by her general practitioner. As she walks in to the clinic you note that she is short and has a webbed neck. What is the likely cause of her hypertension? a) Coarctation of the aorta b) Renal artery stenosis c) Cushing’s syndrome d) Adult polycystic kidney disease e) Conn’s syndrome Answer: a) Short stature and webbed neck in a female would make you suspect Turner’s syndrome. This may be associated with coarctation of the aorta which can cause hypertension

BOF: 17 Which of the following drugs may cause hypertrichosis? a) Valproate b) Carbamazepine c) Cyclosporine d) Azathioprine e) Methotrexate Answer: c) The following drugs may cause hypertrichosis: ü Acetazolamide ü Phenytoin ü Diazoxide ü Minoxidil ü Cyclosporine ü Penicillamine ü Corticosteroids ü Streptomycin ü Psoralens

BOF: 18 A 47 year old male who is known to have hypertension and epilepsy is seen in the clinic. You note that he has gum hypertrophy. Which one of the following medications is the likely cause of this?

304

Mastering MRCP a) Amlodipine b) Lisinopril c) Bendroflumethiazide d) Carbamazepine e) Atenolol Answer: a) Gum hypertrophy may occur with both major classes of calcium channel blockers and resolve following their cessation Drugs causing gum hypertrophy ü Phenytoin ü Cyclosporine ü Calcium antagonists Valproate: case report of valproate induced gum hypertrophy

BOF: 19 Which of the following conditions is the most common cause of recurrent spontaneous abortion in the first trimester of pregnancy? a) Antiphospholipid syndrome b) Factor V Leiden c) Protein S deficiency d) Antithrombin III deficiency e) Protein C deficiency Answer: a) Antiphospholipid syndrome ü Most important treatable cause of recurrent miscarriage ü Prevalence of antiphospholipid syndrome in women with recurrent abortion is 15%

BOF: 20 A 50 year old female who is known to have rheumatoid arthritis is found to have proteinuria. Which one of the drugs is the likely cause of proteinuria? a) Methotrexate b) Azathioprine c) Sodium auriothiomalate d) Leflunomide e) Cyclophosphamide Answer: c) Drugs causing proteinuria ü Penicillamine ü Gold (and other heavy metals) ü NSAIDs ü Captopril ü Mercurials ü Probenecid

305

Mastering MRCP ü Intravenous drug abuse (heroin)

BOF: 21 Which of the following describes the action of endothelin? a) Potent vasoconstrictor b) Natriuretic c) Inhibits secretion of renin and angiotensin d) Relaxes vascular smooth muscle e) Increases extracellular fluid volume Answer :a) Endothelin ü Peptide produced by a variety of tissues ü Potent vasoconstrictor ü Mitogenic activation of myocardial and vascular smooth muscle cells

BOF: 22 Through which of the following does the maxillary nerve leave the skull? a) Foramen ovale b) Foramen rotundum c) Foramen lacerum d) Superior orbital fissure e) Jugular foramen Answer: b) The maxillary division of the trigeminal nerve leaves the skull through the foramen rotundum

BOF: 23 A 62 year old male has been started on finasteride for management of benign prostatic hyperplasia Which one of the following adverse effects would you warn him about? a) Gynaecomastia b) Postural hypotension c) Oedema d) Palpitations e) Chest pain Answer: a) Finasteride ü Synthetic antiandrogen ü Inhibits type II 5-alpha reductase ü Prevents conversion of testosterone to dihydrotestosterone (DHT) Adverse effects RAG ü Impotence ü Ejaculation disorders

306

Mastering MRCP ü Decreased volume of ejaculate ü Decreased libido ü Testicular pain ü Gynaecomastia IS ü Hypersensitivity ü Urticaria

BOF: 24 A 21 year old male presents with a history of sore throat, pain on swallowing and a muffled voice. Apart from a upper respiratory tract infection a few days prior to the onset of these symptoms, he had been well and was not on any medication. He did not smoke but drank 24 units of alcohol per week. On examination he was febrile, there was drooling of saliva from his mouth, there was no tonsillar enlargement. He had a pulse rate of 110 beats per minute, there was cervical lymphadenopathy and tenderness over the larynx. His respiratory rate was 30 per minute and you could hear stridor In this patient which one of the following organisms is most likely to have caused the condition? a) Haemophilus influenzae type B b) Pneumococci c) Group A beta-haemolytic Streptococcus d) Pseudomonas e) Mycobacterium tuberculosis Answer: a) The clinical features suggest the patient has epiglottitis Epiglottitis ü Acute inflammation in the supraglottic region of the oropharynx ü Inflammation of the epiglottis, vallecula, arytenoids and aryepiglottic folds Clinical Features History ü Sore throat ü Odynophagia / dysphagia ü Muffled voice ü Mild cough ü Usually, no prodromal symptoms in children ü Adults may have preceding upper respiratory infection (URI) symptoms. Examination ü Tripod position (Sitting up on hands with the tongue out and the head forward) ü Fever ü Toxic appearance of patient ü Irritability ü Drooling/inability to handle secretions ü Severe pain on palpation over the larynx ü Cervical lymphadenopathy ü Stridor (late finding indicating advanced airway obstruction ) ü Respiratory distress

307

Mastering MRCP ü Tachycardia Causes ü Haemophilus influenzae type B (most common) ü Pneumococci ü Group A beta-haemolytic Streptococcus ü Pseudomonas ü Mycobacterium tuberculosis ü Viruses ü Local trauma (inhalation of foreign bodies or post intubation)

BOF: 25 A 27 year old female is due to go on an airline flight from London, England to Auckland in New Zealand. There is a history of post operative deep vein thrombosis 2 years ago. You advise her not to take excessive alcohol or sleeping tablets and regularly flex ankles to contract calf muscles In this patient what further action would you recommend? a) No further action b) Aspirin before flight c) Warfarin d) Low molecular weight heparin e) Compression stocking Answer: e) Recommendations to prevent DVT during airline flight lasting > 6hours Low risk ü No history of DVT or PE ü No surgery in the previous 4 weeks ü No other risk factors to indicate moderate/high risk Recommendation ü Do not take excessive alcohol or sleeping tablets ü Regularly flex ankles to contract calf muscles Moderate risk ü Previous history of DVT or PE ü Surgery under general anaesthesia lasting more than 30 minutes in the previous 2 months but not in the last 4 weeks Recommendation ü Do not take excessive alcohol or sleeping tablets ü Regularly flex ankles to contract calf muscles ü Wear compression stocking High risk ü Surgery under general anaesthesia lasting more than 30 minutes in previous 4 weeks Recommendation ü Precautions as for low to moderate plus ü Single injection of low molecular weight heparin before departure Other conditions ü History of DVT (or PE)

308

Mastering MRCP ü Additional major risk factors for ü Recurrence of thrombosis (cancer etc ) Recommendation ü LMWH Recent DVT (or PE) ü On anticoagulant treatment low risk ü Within 2 weeks of a new DVT or PE risk greater Plaster casts ü Consider split cast to reduce the risk of compression Aspirin ü No evidence to support use of aspirin in any category

BOF: 26 A 63 year old male who is on warfarin for atrial fibrillation is due to have a tooth extraction. His anticoagulation is well controlled and he has not had any recent change in medication. What advice would you give prior to the procedure? a) Continue with warfarin b) Switch to aspirin c) Switch to sc heparin d) Switch to LMWH e) Stop warfarin for 72 hours Answer: a) Dental Procedures and Anticoagulation ü Risk of significant bleeding in patients • Oral anticoagulants • Stable INR in the therapeutic range 2-4 ( <4) Very small ü Risk of thrombosis • Increased in patients in whom oral anticoagulants are temporarily discontinued ü Oral anticoagulants should not be discontinued in the majority of patients requiring out-patient dental surgery • Including dental extraction Recommendation ü Patients stably anticoagulated on warfarin (INR 2-4) ü Prescribed a single dose of antibiotics as prophylaxis against endocarditis • No necessity to alter anticoagulant regimen \ Risk of bleeding may be minimised by ü Oxidised cellulose (Surgical) ü Collagen sponges and sutures ü Tranexamic acid mouthwashes used four times a day for 2 days Patients who are stably anticoagulated on warfarin ü Check INR 72 hours prior to dental surgery ü No NSAIDs and COX-2 inhibitors

309

Mastering MRCP BOF: 27 A 72 year old patient is seen in the clinic. You note that he has slow movements, rigidity and a tremor. The patient is on apomorphine Which one of the following describes the action of apomorphine? a) COMT inhibitor b) Dopaminergic c) Anticholinergic d) MAO-B inhibitor e) Barbiturate Answer: b) The clinical features suggest the patient has Parkinsonism Apomorphine is a dopaminergic agent that may be used in the treatment of Parkinsonism

BOF: 28 A middle aged male is brought into casualty by the police. He had been found lying in the snow. On examination he was hypothermic, unconscious and had a bradycardia. An ECG was performed Which one of the following is pathognomonic of hypothermia? a) Atrial fibrillation b) J waves c) Prolongation of the PR interval d) Widening of the QRS complex e) QT prolongation Answer :b) J waves These are rounded (dome like) deflections that occur just after the QRS complex ü Pathognomonic of hypothermia Also occur in E&M ü Hypercalcaemia CNS ü Brain injury ü Subarachnoid haemorrhage ü Cardiopulmonary arrest from over sedation CVS ü Brugada syndrome ü Vasospastic angina ü Idiopathic ventricular fibrillation Other ECG changes in hypothermia ü Atrial fibrillation ü Prolonged PR interval ü QRS widening ü QT elongation

310

Mastering MRCP BOF: 29 A 59 year old male presents with long standing abdominal pain. He has had repeated admissions to hospital with episodes of severe pain. He has no significant past illnesses apart from this pain. He smokes 30 cigarettes a day and consumes 50 units of alcohol per week. On examination of his abdomen you note reticular erythema and pigmentation over the anterior abdominal wall What type of malignancy may occur within this skin lesion in chronic cases? a) Squamous carcinoma b) Basal cell carcinoma c) Melanoma d) Kaposi’s sarcoma e) Cutaneous T cell lymphoma Answer: a) The clinical features would suggest the patient has chronic pancreatitis and the skin lesion is likely to be erythema ab igne Erythema ab igne ü Occurs in skin that is chronically and repeatedly exposed to infrared radiation in the form of heat ü Reticulate, erythematous patches, telangiectasia and hyperpigmentation Complications ü Thermal keratosis ü Squamous cell carcinoma in situ ü Squamous cell carcinoma

BOF: 30 A 67 year old male presents with a history of transient loss of vision in one eye . Which of the following blood vessels is likely to be involved in the pathogenesis of this condition? a) External carotid artery b) Internal carotid artery c) Middle meningeal artery d) Posterior communicating artery e) Vertebral artery Answer: b) The ophthalmic artery is a branch of the internal carotid artery. Within the orbit it supplies the extraocular muscles, the lacrimal gland and the eye through the central retinal artery and the posterior ciliary arteries Amaurosis fugax ü Transient and painless loss of vision in one eye ü Indicates transient retinal ischaemia ü Associated with stenosis of the ipsilateral carotid artery or emboli (cardiac and aortic)

BOF: 31 What is the mechanism of action of Clopidrogel? a) Inhibitor of cyclo-oxygenase b) Adenosine diphosphate receptor antagonist

311

Mastering MRCP c) Inhibition of adenosine uptake into platelets d) Glycoprotein IIb/IIIa antagonist e) Phosphodiesterase inhibitor Answer: b) Clopidogrel ü Adenosine diphosphate (ADP) receptor antagonist ü Competitively inhibits ADP from binding to platelet receptors ü Prevents ADP mediated up-regulation of glycoprotein IIb/IIIa receptor Aspirin ü Non-selective, irreversible inhibitor of cyclo-oxygenase ü Reduces production of thromboxane and prostaglandins ü Antithrombotic action derives from reduction in thromboxane A2 Dipyridamole ü Mechanism not fully understood ü Thought to act by inhibiting adenosine uptake into platelets and reducing ADP-induced aggregation Abciximab Tirofiban ü Glycoprotein IIb/IIIa antagonists

BOF: 32 Which of the following acts on the CD 20 antigen ? a) Infliximab b) Adalilumab c) Tratuzumab d) Rituximab e) Abciximab Answer: d) Rituximab ü Antibody directed against the CD 20 antigen ü Effectively eradicates CD20+ cells several distinct mechanisms, including complement-dependent cytotoxicity (CDC), antibody-dependent cellular cytotoxicity (ADCC) and apoptosis Uses ü Non Hodgkin’s lymphoma ü Chronic lymphocytic leukaemia ü Rheumatoid arthritis ü Renal transplant rejection

BOF: 33 A 27 year old female is brought in to casualty by her friends. They had been at a nightclub and they noticed that she has become very restless and anxious and had felt that people in the nightclub were trying to attack her The friends were not aware of any illicit drug use by the patient and she does not give a coherent history You begin to examine the patient

312

Mastering MRCP Which one of the following signs would suggest the patient has used Ecstasy methylenedioxymethamphetamine (MDMA) a) Hypotension b) Constricted pupils c) Bradycardia d) Cold clammy skin e) Hyperthermia Answer: e) 3, 4-methylenedioxymethamphetamine (MDMA, ecstasy) ü MDMA amphetamine derivative ü Structurally similar to methamphetamine and the hallucinogen mescaline ü Causes catecholamine release from presynaptic vesicles ü Also a selective serotonergic neurotoxin that causes massive release of serotonin , 5-hydroxytryptamine (5-HT) and is postulated to inhibit its uptake MDMA toxicity ü Seizures ü Hyperthermia ü Coagulopathies ü Arrhythmias ü Heart failure ü Stroke, ü Renal and or liver failure ü Most MDMA-related fatalities attributed to symptoms of heat stroke and hyperthermia ü Another major cause of morbidity and mortality is abnormal fluid, electrolyte balance, or both • MDMA stimulates vasopressin release, resulting in SIADH • In conjunction with too much water intake during profuse sweating and salt loss can lead to severe hyponatraemia with subsequent cerebral oedema and seizures. ü Although uncommon arrhythmias heart failure occur ü MDMA-induced myocardial infarction is rarely reported. ü Intracerebral haemorrhage uncommon

BOF: 34 A 72 year old female is admitted with confusion. Her daughter tells you that she had been complaining of backache and had been very thirsty. The patient was known to have cerebrovascular disease and was on aspirin and a statin. On examination you note that she looked unwell ,was pale , her tongue was dry , skin turgor was diminished pulse 110 beats per minute regular, BP 110/80 lying ,80 /40 standing . In this patient which one of the following investigation would you arrange next? a) Plasma protein electrophoresis b) Bone marrow aspiration c) Urine for Bence-Jones protein d) Skeletal survey e) Calcium studies Answer: e)

313

Mastering MRCP The patient has clinical features of salt and water loss and with a preceding history of thirst and backache one should consider multiple myeloma with hypercalcaemia. The first step would be to establish that the patient has hypercalcaemia and treat this before embarking on tests to prove that the patient has myeloma

BOF: 35 A 53 year old female presents with a history of rapid weight gain. There is no past history of note. On examination she is centrally obese, with a prominent suprascapular pad of fat . She is hirsute and has acne. BP 170/110. There are striae over her anterior abdominal wall and she also has evidence of a proximal myopathy. The patient has routine investigations Which one of the following may occur in this condition? a) Hyperchloraemic metabolic acidosis b) Hypokalaemic metabolic alkalosis c) Hyperkalaemic metabolic acidosis d) Hyperkalaemic metabolic alkalosis e) Hypochloraemic metabolic acidosis Answer: b) The clinical features would suggest that the patient has Cushing’s syndrome . In this condition the patient may develop a hypokalaemic metabolic alkalosis due to the mineralocorticoid effects of cortisol

BOF: 36 A 23 year old male who is known to have insulin dependent diabetes mellitus is brought into casualty with a hypoglycemic episode. He is treated with intravenous dextrose and glucagons and makes a satisfactory recovery. He admits to having several such episodes over the preceding months. Which of the following is most likely to enable you to determine the cause of the hypoglycemic episodes? a) Detailed family history b) Careful auscultation of the heart c) Careful examination of the skin d) Detailed neurological examination e) Examination of the thyroid gland Answer: c) Examination of the skin with special emphasis on the insulin injection sites is important in patients with hypoglycemic episodes If the patient has injected insulin into an area of lipodystrophy this could cause either enhanced or delayed absorption of insulin and interfere with glycaemic control To maintain normoglycaemia there must be a balance between food intake, energy expenditure and insulin dosage and absorption In analyzing hypoglycemia the important features are: ü Dietary history ü Exercise history ü Timing of insulin injections

314

Mastering MRCP ü IS Lipodystrophy ü E&M Addison’s disease ü GIT Liver failure ü KUS Renal failure (kidneys break down insulin)

BOF: 37 A 26 year old male is admitted with diarrhoea, loss of weight and weakness. The patient is known to have ulcerative colitis – pancolitis – and has been on medication for the last 3 years. For 5 months prior to admission the patient has been having bad diarrhoea and has been self medicating with varying doses of steroids and asacol. He has also become a vegan as a friend had told him that this would cure his colitis. On examination the patient looked unwell and was underweight. There was alopecia and dermatitis. Pulse 100 beats per minute BP110/70 abdomen soft no lumps no tenderness. In this patient deficiency of which one of the following elements would account for the dermatitis and alopecia? a) Zinc b) Copper c) Selenium d) Chromium e) Magnesium Answer :a) Zinc and manganese deficiency causes dermatitis and hair loss Copper deficiency causes anaemia and neutropaenia Selenium deficiency causes cardiomyopathy Magnesium deficiency causes hypocalcaemia Chromium deficiency causes glucose intolerance

BOF: 38 A 70 year old male who is known to have COPD is admitted with a history of increasing breathlessness and cough productive of green sputum. The patient has no other illnesses and no history of drug allergies. The chest X-ray does not show evidence of consolidation In this patient which one of the following would you use? a) Amoxicillin b) Teicoplanin c) Cefotaxime d) Erythromycin e) Amoxicillin plus macrolide Answer :a) In a patient with COPD cough productive of yellow or green sputum suggests that there is an infection

315

Mastering MRCP Treatment would be with an antibiotic Amoxicillin would be the choice from the drugs listed If there was evidence of consolidation one would add in a macrolide

EMQ: 1 Haematemesis and Melaena a) Mallory-Weiss tear b) Duodenal ulcer c) Oesophageal varices d) Gastric ulcer e) Dieulafoy lesion f) Gastric carcinoma g) Gastritis h) Oesophagitis i) Aortoenteric fistula j) Barrett’s oesophagus Select the most likely diagnosis from the list above for the following clinical scenarios: 1) A middle aged, obese female who has been having pruritus for a long time presents with haematemesis and melaena. On examination she has xanthelasma, no signs of chronic hepatocellular failure, moderate splenomegaly Answer: c) Oesophageal varices In a middle-aged lady with a history of pruritus in relation to a gastrointestinal problem, one should think of primary biliary cirrhosis (PBC). The fact that she has xanthelasma is further evidence in favour of this. The lack of signs of hepatocellular failure with evidence of portal hypertension (splenomegaly) would also be in favour of this. The most likely complication of PBC that would present with haematemesis and melaena would be bleeding oesophageal varices. 2) A 75-year-old male with a history of anorexia and weight loss presents with haematemesis and melaena. On examination he looks cachectic and on palpation of the abdomen a hard, craggy liver is felt. Answer: f) Gastric carcinoma In an elderly patient with anorexia and weight loss, one should consider gastric cancer. The fact that he is cachectic would be further evidence in favour of this diagnosis. The finding of a hard craggy liver would support this, as this finding would suggest liver metastases. 3) A 24-year-old male who has been on an alcoholic binge presents wit haematemesis following a period of prolonged vomiting. On examination he is inebriated but there is no cardiovascular instability and his haemoglobin is within normal limits. Answer: a) Mallory-Weiss tear

316

Mastering MRCP A young man presenting with vomiting after an alcoholic binge should make one infer that there is no underlying medical condition. Prolonged vomiting followed by haematemesis is suggestive f a MalloryWeiss tear and the fact that there is no cardiovascular instability nor a drop in haemoglobin should also point to this diagnosis. 4) A 50-year-old sales representative who has been complaining of abdominal discomfort of several months’ duration .the discomfort is relieved by meals but wakes him up from his sleep. He presents with a 3-day history of melaena. Answer: b) Duodenal ulcer The history of abdominal discomfort relieved by meals and nocturnal discomfort is suggestive of a duodenal ulcer. The age group and the fact that he has a presumably stressful job are also factors in favour of this diagnosis. 5) A 78-year-old male who has had a repair of an abdominal aortic aneurysm presents with melaena, tachycardia a fall in blood pressure and is found to be anaemic. Answer: i) Aortoenteric fistula The history of aortic aneurysm repair should make on suspect that the pathology is in relation to this. Melaena with cardiovascular in stability and anaemia should further this suspicion.

EMQ: 2 Radial Pulse a) Anacrotic b) Collapsing c) Pulsus paradoxus d) Bisferiens e) Bounding f) Radio-femoral delay g) Radio-radial delay h) Small volume, irregular in rhythm and volume i) Jerky upstroke j) Pulsus alternans Select the most likely pulse that will be felt in the clinical scenarios given below: 1) A young man with hypertension. He has visible pulsations in the suprasternal notch, a heaving apex beat and there are prominent periscapular blood vessels. There is a continuous bruit best heard over the back. Answer: f) Radio-femoral delay Hypertension in the young may be due to a number of causes. However, in relation to abnormalities of the pulse there is probably only one that should immediately come to mind and that is coarctation of the aorta, which is associated with radio-femoral delay in the pulse. The presence of visible pulsations in the neck is a sign In favour of this as is the heaving apex beat, the prominent periscapular vessels and the bruit over the back. 2) An elderly female with a tapping apex beat, a loud first heart sound, an opening snap and a rumbling mid-diastolic murmur best heard just medial to the apex beat. There is no pre-systolic accentuation of the murmur. Answer: h) Small volume, irregular in rhythm and volume

317

Mastering MRCP The tapping apex beat is representative of a loud first heart sound and this should suggest mitral stenosis. The opening snap is more evidence in favour of this as is the nature of the murmur described. In mitral stenosis the patient is likely to develop atrial fibrillation and this have a small volume pulse, which would be irregular in both rate and rhythm. The absence of pre-systolic accentuation of the murmur also supports the diagnosis of atrial fibrillation. 3) An elderly man with a heaving apex beat, soft single second heart sound and a harsh ejection systolic murmur radiating to the neck Answer: a) Anacrotic pulse The patient has a heaving apex beat and this would indicate systolic overload (the heart is pumping out against resistance) this would suggest aortic stenosis or severe systemic hypertension (coarctation of the aorta is unlikely at this age). The soft second heart sound would indicate aortic stenosis rather than hypertension. The murmur radiating to the neck would support this diagnosis. In aortic stenosis the character of the pulse is small volume, slow rising (anacrotic) 4) A teenager with severe breathlessness, showing signs of respiratory distress. On auscultation of his chest there are bilateral sibilant rhonchi. Answer: c) Pulsus paradoxus Severe obstructive airway disease results in pulsus paradoxus the decreased venous return to the left atrium that normally occurs is exaggerated. 5) An elderly patient who is breathless and oedematous and has an elevated JVP , displaced apex beat , a 3rd heart sound and bilateral basal crepitations Answer: j) pulsus alternans Breathlessness and oedema in an elderly patient should suggest heart failure. The elevated JVP is in keeping with this. The displaced apex beat suggests left ventricular dilatation in keeping with heart failure and the presence of a 3rd heart sound and bilateral basal crepitations all fit in with a diagnosis of heart failure. Pulsus alternans refers to an alternating high and low volume pulse that occurs in left ventricular failure

EMQ: 3 Headache a) Meningitis b) Subarachnoid haemorrhage c) Subdural haematoma d) Migraine e) Cluster headache f) Temporal arteritis g) Cerebral metastases h) Trigeminal neuralgia i) Benign intracranial hypertension j) Acute glaucoma Select the most likely condition from the list above that would account for the following scenario: 1) A 50-year-old female who has had a mastectomy followed by radiotherapy and chemotherapy for breast cancer presents with a severe headache associated with nausea and vomiting. On examination of the fundus there is bilateral papilloedema.

318

Mastering MRCP Answer: g) cerebral metastases The history of cancer should immediately alert one to the possibility that this is the cause of the problem. Headache associated with nausea and vomiting is suggestive of raised intracranial pressure and this is confirmed by the presence of papilloedema. The most likely diagnosis would be metastatic disease as a consequence of the breast cancer. 2) A 63-year-old female presents with headache. She gives a history of pain in her jaw when she eats. On examination the scalp is tender. Answer: f) temporal arteritis In an elderly patient presenting with headache one should always consider temporal arteritis. The history of jaw claudication is very suggestive of temporal arteritis and the scalp tenderness is also in favour of this diagnosis. 3) A 45-year-old female presents with sudden onset headache. She is drowsy and has neck stiffness Answer: b) subarachnoid haemorrhage Sudden onset headache should make one suspect an intracranial haemorrhage. Associated drowsiness should make one think of raised intracranial pressure. Neck stiffness would suggest meningeal irritation and hence blood in contact with the leptomeninges (i.e.) subarachnoid haemorrhage 4) A 35-year-old female presents with recurrent unilateral throbbing headaches that are accompanied by nausea and vomiting. The headache is preceded by an aura where she experiences the sensation of seeing flashing lights. Answer: d) migraine The history of recurrent unilateral headache should make one suspect migraine. The throbbing nature of the headache is in favour of this. The fact that the headache is preceded by an aura lends weight to this diagnosis. 5) A 75-year-old male presents with headache. He is unsteady on his feet and has periods of drowsiness. His relatives say that he has had several falls in the last few weeks. Answer: c) subdural haematoma In an elderly patient who has had several falls one should think of subdural haematoma. The history of periods of drowsiness would be in favour of this.

EMQ: 4 Anaemia a) Hereditary spherocytosis b) Iron deficiency anaemia c) Folic acid deficiency anaemia d) Vitamin B12 deficiency anaemia e) Sickle cell disease f) Thalassaemia major g) Thalassaemia minor h) Sideroblastic anaemia i) Anaemia of chronic disease j) Aplastic anaemia Select the most likely cause of anaemia from the list above to match the scenarios given below: 1) A 25-year-old male has a macrocytic anaemia. He is on long-term treatment with phenytoin sodium for epilepsy

319

Mastering MRCP Answer: c) folic acid deficiency anaemia A macrocytic anaemia may be due to deficiency of vitamin B12 or folic acid. Other causes such as alcohol abuse, hypothyroidism, drug treatment with drugs such as azathioprine are not relevant to this case. Anticonvulsant drugs (for example phenytoin sodium) interfere with mucosal conjugase. This impairs folate absorption. 2) A 58-year-old northern European female with a macrocytic anaemia. She has blue eyes, white hair and has vitiligo. Answer: d) vitamin B12 deficiency anaemia Macrocytic anaemia may be due to vitamin B12 or folate deficiency. Other causes as outlined above are not relevant to this case. In an elderly female one should consider pernicious anaemia. Pernicious anaemia is more common in people of northern Europe, and is associated with blue eyes, premature greying of the hair and vitiligo. 3) A 23-year-old African patient presents with severe pain in his femur and is found to be anaemic Answer: e) sickle cell disease The fact that the patient is an African should alert one to the possibly of a disease more common in a person of that ethnic origin. Severe bone pain should be the clue to the diagnosis. Vasoocclusive crises occur in patients with sickle cell disease and results in severe pain, which is of ischaemic origin. 4) A 53-year-old postmenopausal female presents with a history of lethargy and is found to have a mass in the right iliac fossa. Answer: b) iron deficiency anaemia In a female presenting with anaemia none should always consider menstrual disturbances. The patient is postmenopausal and hence this does not come into consideration. The lump in the right iliac fossa should make one consider carcinoma of the caecum with consequent iron deficiency. 5) A 58-year-old female with long standing rheumatoid arthritis has a normocytic, normochromic anaemia. White cell count and platelets are within normal limits. She has normal iron studies, her folate and B12 levels are normal and there is no reticulocytosis. Answer: I) anaemia of chronic disease In a patient with rheumatoid arthritis anaemia may be due to multiple causes. Haematinic deficiency, marrow failure and haemolysis have been excluded by the information given. Thus the most likely cause is anaemia of chronic disease.

EMQ: 5 Arthritis a) Rheumatoid arthritis b) Osteoarthritis c) Gout d) Ankylosing spondylitis e) Psoriatic arthropathy f) Reiter’s syndrome g) Septic arthritis h) Rheumatic fever i) Systemic lupus erythematosus j) Pseudogout Select the most likely cause of arthropathy from the list above to match each of the scenarios given below:

320

Mastering MRCP

1) A 24-year-old male presents with a gradual onset of backache and morning stiffness that is improved by exercise. On examination of his spine there is decreased spinal mobility and when he stands with his back to a wall, the occiput does not touch the wall. Answer: d) ankylosing spondylitis When a patient has backache in the context of a question on arthritis consider the spondyloarthropathies (i.e.) a group of inflammatory diseases that predominantly affect the axial skeleton, the peripheral joints and the entheses. No underlying cause for the spondyloarthropathy has been mentioned. In a young male with no underlying cause presenting with an inflammatory spondyloarthropathy one should consider the diagnosis to be ankylosing spondylitis. 2) A 50-year-old female presents with pain and stiffness of her hands and wrists. The stiffness is worse in the morning and gets better as the day progresses. On examination she has swelling of her metacarpophalangeal joints and there is ulnar deviation at the metacarpophalangeal joints. Joint movement is impaired. Answer: a) rheumatoid arthritis In a female presenting with pain in her joints one should consider an inflammatory arthropathy. Stiffness, which is worse in the morning, is in favour of an inflammatory arthropathy rather than osteoarthropathy, which is worse in the latter part of the day. The metacarpophalangeal joints being affected would be in favour of rheumatoid arthritis and the fact that there is ulnar deviation at these joints is also a point in favour of rheumatoid arthritis. 3) A 45-year-old man presents with pain and deformity of the joints of his hands. On examination the nails show pitting and the distal interphalangeal joints are painful and deformed on both sides. Answer: e) psoriatic arthropathy Pitting of the nails should alert one to the fact that this arthropathy is in relation to a skin condition. Pitting of the nails may occur in psoriasis and in alopecia areata. Immediately, one should think of psoriatic arthropathy. Indeed psoriatic arthropathy is one of the conditions that cause arthropathy affecting the distal interphalangeal joints. 4) A 70-year-old male presents with severe pain in the right first metatarsophalangeal joint. The pain woke him up from his sleep. On examination the joint is swollen red and painful and tender. Answer: c) gout There are not many causes of a small joint monoarthropathy (gout and very rarely septic arthritis). In an elderly patient the most likely condition is gout. The fact that there is redness over the joint is also a sign in favour of crystal arthropathy although it may also occur in septic arthritis. 5) A 75-year-old man who has had long-standing problems with his joints presents with pain and swelling of his right knee. On examination he is febrile, the skin overlying the knee looks red, it is hot and there is evidence of fluid within the joint. Answer: g) septic arthritis The fact that he is febrile should alert one to the fact that there is a systemic disturbance probably due to infection. Redness over joint may be due to crystal arthropathy but with the other features given the most likely diagnosis is septic arthritis.

321

Mastering MRCP EMQ: 6 Endocrine conditions a) Hyperthyroidism b) Hypothyroidism c) Cushing’s syndrome d) Addison’s disease e) Acromegaly f) Phaeochromocytoma g) Diabetes mellitus h) Hyperparathyroidism i) SIADH (syndrome of inappropriate ADH secretion) j) Diabetes insipidus 1) A 56-year-old man complaining that he noticed that his shoes were becoming tight. On examination he has prominent supra orbital ridges and his jaw looked prominent. There was malocclusion of the teeth. His hands looked large and the skin over the back of the hands was thickened. There was wasting of the muscles of the thenar eminence. Answer: e) acromegaly A complaint of shoes becoming tight would suggest that there has been increased growth. Increased growth in an adult in the context of endocrine disease should immediately alert one to the possibility of acromegaly. The prominent supraorbital ridges and the prominent jaw are also features in keeping with this diagnosis. Malocclusion of the teeth is further evidence in favour; large hands are a feature of acromegaly and the skin does become thickened in this condition. Wasting of the muscles of the thenar eminence indicates that presence of carpal tunnel syndrome. 2) A patient is referred for investigation of hypertension. On examination she has central obesity and thins arms and legs. She is hirsute .She has an oily skin and acne. There are purple striae on her abdominal wall. She has evidence of proximal muscle weakness. Answer: c) Cushing’s syndrome In the context of endocrine disease, hypertension may be caused by Cushing’s syndrome, Conn’s syndrome, phaeochromocytoma, acromegaly Central obesity with thin limbs should make one think of Cushing’s syndrome Hirsutism, oily skin and acne are all in favour of this. The purple striae over the abdomen are also in favour and proximal muscle weakness is a feature. 3) A 56-year-old female is referred for investigation of a complaint of feeling weak and lethargic. He blood pressure is low and there is a postural drop in blood pressure. On examination there is generalised pigmentation of the skin with more marked pigmentation in the skin creases, in the buccal mucosa and the nipples. Answer: d) Addison’s disease Low blood pressure in the context of endocrine disease should immediately alert one to the possibility of Addison’s disease. The skin pigmentation described is characteristic of Addison’s disease. 4) A 30-year-old female has been referred for investigation of loss of weight. On examination she is thin her skin is sweaty; she has a fine tremor and tachycardia. Answer: a) hyperthyroidism

322

Mastering MRCP Weight loss in the context of endocrine disease should alert one to the possibility of thyrotoxicosis, diabetes mellitus or Addison’s disease. The findings of sweating, fine tremor and tachycardia would suggest hyperthyroidism. 5) A 55-year-old female is referred for investigation of lack of energy. On examination she is overweight, her skin is coarse, dry and cold. There is diffuse loss of hair over her scalp. Her pulse rate is slow. Answer: b) hypothyroidism. Lack of energy in the context of endocrine disease should make one think of thyroid disorders, adrenal disorders, pancreatic disease and electrolyte imbalance The patient being overweight with coarse, dry cold skin should alert one to the possibility of hypothyroidism the diffuse loss of scalp hair and the bradycardia are further features in favour of this diagnosis.

EMQ: 7 Lower limb weakness a) Hereditary motor sensory neuropathy b) Multiple sclerosis c) Cauda equina lesion d) Parasagittal meningioma e) Syringomyelia f) Guillain-Barre syndrome g) Spinal cord compression h) Subacute combined degeneration of the cord i) Motor neurone disease j) Tabes dorsalis 1) A 26-year-old female with spastic paraparesis and optic neuritis Answer: b) multiple sclerosis In a young patient with a neurological deficit one should consider multiple sclerosis. The patient has evidence of scattered lesions anatomically and hence the most likely conditions is multiple sclerosis 2) A 65-year-old female with lower limb weakness, with absent ankle jerks and upgoing plantar response. There is loss of joint position sense and vibration sense in both feet. Answer: h) Subacute combined degeneration of the cord This patient has evidence of motor weakness together with posterior column involvement. This should make you think of subacute combined degeneration of the cord. There are few conditions that cause upgoing plantars with absent ankle jerks. This is further evidence in favour of subacute combined degeneration of the cord. 3) A 75-year-old male with prostate cancer presents with backache and evidence of a spastic paraparesis and incontinence of urine and faeces Answer: g) spinal cord compression In a patient with prostate cancer and backache one should consider metastatic disease. This combined with evidence of spinal cord involvement in the form of a spastic paraparesis and incontinence should make you think of spinal cord compression. 4) A 28-year-old female presents with rapidly progressive weakness in her lower limbs. There is a history of a diarrhoeal illness preceding the onset of the problem. On examination she has a flaccid weakness and tendon reflexes are absent.

323

Mastering MRCP Answer: f) Guillain-Barre syndrome The history of a preceding diarrhoeal illness should alert you to the possibility of an acute inflammatory condition. This coupled with the flaccid weakness and absent reflexes should suggest to you that the most likely diagnosis is Guillain-Barre syndrome (acute inflammatory demyelinating polyradiculoneuropathy) 5) A 50-year-old man with spastic paraparesis, exaggerated reflexes and upgoing plantars. The upper limb muscles are wasted and show fasiculations. No sensory involvement. Answer: I) motor neurone disease Spastic paraparesis, exaggerated reflexes and upgoing plantars suggest upper motor neurone involvement. Wasting and fasiculations show involvement of the lower motor neurone in particular the anterior horn cell. This combination of features suggests motor neurone disease. The absence of sensory signs is further evidence in favour of this.

EMQ: 8 Heart sounds and murmurs a) Mitral stenosis b) Mitral regurgitation c) Aortic stenosis d) Aortic regurgitation e) Mitral valve prolapse f) Hypertrophic obstructive cardiomyopathy g) Pulmonary stenosis h) Atrial septal defect i) Tricuspid regurgitation j) Ventricular septal defect 1) A young female with a mid systolic click and late systolic murmur best heard at the apex. Answer: e) mitral valve prolapse The mid systolic click is caused by the mitral valve prolapsing into the left atrium. The late systolic murmur is due to the prolapsed mitral valve allowing regurgitation of blood from the ventricle to the atrium in late systole. 2) A young boy with fixed splitting of the second heart sound and an ejection systolic murmur in the pulmonary area Answer: h) atrial septal defect Fixed splitting of the second heart sound is caused by an atrial septal defect as the deficit allows the increased flow of blood across the pulmonary valve to balance out between inspiration and expiration. The ejection systolic murmur is caused by increased flow of blood across the pulmonary valve. 3) An elderly lady with a loud first heart sound, opening snap and a rumbling murmur in mid diastole best heard just medial to the apex beat. Answer: a) mitral stenosis The loud first heart sound is caused by thickening of the mitral valve. The opening snap is due to the thickened valve snapping open in diastole and the rumbling mid diastolic murmur is caused by flow of blood across the narrowed mitral valve. 4) A middle-aged man who is known to have ankylosing spondylitis. He has a blowing early diastolic murmur best heard at the left sternal edge. Answer: d) aortic regurgitation

324

Mastering MRCP Aortic regurgitation is a known complication of ankylosing spondylitis. In this condition a blowing early diastolic murmur is heard at the left sternal edge. 5) An elderly lady with a soft first heart sound and a blowing pan systolic murmur best heard at the mitral area and radiating to the axilla. Answer: b) mitral regurgitation The soft first heart sound is suggestive of an incompetent mitral valve. The blowing pan systolic murmur best heard at the apex and radiating to the axilla is most likely to be due to mitral regurgitation.

EMQ: 9 Respiratory Conditions a) Left sided pleural effusion b) Right sided pleural effusion c) Left sided pneumothorax d) Right sided pneumothorax e) Right apical collapse f) Right apical consolidation g) Fibrosing alveolitis h) Bronchiectasis i) Pulmonary embolus j) Chronic bronchitis 1) A 56-year-old male presenting with cough and fever. On examination he is febrile, no clubbing, trachea midline, respiratory movements decreased at right apex, vocal fremitus increased at the right apex, percussion note dull at the right apex. Bronchial breathing at the right apex with fine inspiratory crepitations localised to the right apex. Vocal resonance increased at the right apex. Answer: f) right apical consolidation Cough and fever would suggest an infective disorder of the respiratory tract. The trachea being in the midline means there is no mediastinal shift. Decreased respiratory movements at the right apex suggest that there is an underlying lesion affecting the pleura or parenchyma. Increased vocal fremitus indicates increased conduction of sound from the underlying bronchus due to consolidation of the parenchyma or the bronchus being drawn towards the chest wall. This is the same as increased vocal resonance. Dull percussion note would suggest consolidation, collapse or fibrosis (not pneumonectomy as this has not been mentioned in the history nor has a scar been mentioned) As the trachea is in the midline collapse and fibrosis are not likely This leaves consolidation and with the localised crepitations suggests the patient has an apical pneumonia. 2) A 63-year-old male presents with dyspnoea. On examination he is afebrile, the trachea is deviated to the left, respiratory movements are decreased at the right base, vocal fremitus is decreased at the right base. Percussion note is stony dull at the right base, breath sounds are diminished at the right base, vocal resonance is decreased at the right base and there is aegophony at the upper limit of the area of dullness. Answer: b) right sided pleural effusion Dyspnoea may be due to any lesion of the respiratory system or systemic disease.

325

Mastering MRCP The deviation of the trachea to the left means either pathology in the right side pushing the mediastinum to the left or pathology on the left side pulling the mediastinum to the left. Decreased respiratory movements on the right means the pathology is on the right side. Stony dullness at the right base indicates an effusion and this will explain all the signs that have been discussed so far. Decreased breath sounds and decreased vocal resonance are in keeping with this diagnosis. Aegophony may be heard at the upper border of an effusion. 3) A 62-year-old female with progressive breathlessness. On examination she has clubbing and there are fine late inspiratory crepitations at both lung bases Answer: g) fibrosing alveolitis Progressive breathlessness may be due to any progressive lesion in the reparatory system or systemic disease. Clubbing narrows the causes down considerably (classified as lesions of the bronchi, alveoli, lung parenchyma, pulmonary vasculature pleura Fine inspiratory crepitations at the bases may be due to fibrosing alveolitis or left ventricular failure. In the context of progressive breathlessness and clubbing the diagnosis would be fibrosing alveolitis 4) A 26-year-old male presents with sudden onset chest pain and breathlessness. On examination his trachea is deviated to the right, respiratory movements are decreased on the left hand side. Percussion note is hyper-resonant on the left hand side. Breath sounds are decreased on the left hand side. Answer: c) left sided pneumothorax Breathlessness may be due to any lesion in the respiratory tract. However chest pain in relation to the respiratory system is usually due to lesions in the pleura or chest wall . This rapidly narrows down the possibilities. Tracheal deviation to the right would suggest a lesion on the left pushing to the right or a lesion on the left pulling the mediastinum to the left. Decreased movement on the left indicates that the lesion is in the left hand side. Hyper-resonant percussion note is noted in either a pneumothorax or emphysema. With the foregoing clinical features the most likely lesion is a left sided pneumothorax. Decreased breath sounds on the left are in keeping with this diagnosis. 5) A 40-year-old male with a chronic productive cough and recurrent respiratory infections. He gives a history of whooping cough in childhood. On examination he is clubbed, there are coarse crepitations at both bases of the lings. The crepitations change when the patient is asked to cough. Answer: h) bronchiectasis A productive cough is due to increased production of sputum and this may be due to lesions in the bronchi, alveoli, lung parenchyma or external factors The history of whooping cough in childhood would point to bronchiectasis see ACES for PACES page 239 Clubbing and coarse crepitations at the lung bases especially if the crepitations change with coughing would suggest bronchiectasis see ACES for PACES pages 264-265

EMQ: 10 Skin Lesions a) Papule b) Nodule

326

Mastering MRCP c) Plaque d) Burrow e) Vesicle f) Bulla g) Pustule h) Cyst i) Target lesion j) Weal Match the description below to the lesion listed above 1) A fluid filled lesion greater than 5 mm in diameter Answer: f) bulla 2) A fluid filled lesion less than 5 mm in diameter Answer: e) vesicle 3) Raised flat topped lesions with ill-defined margins Answer: j) weal 4) A raised solid skin lesion less than 1 cm at its greatest diameter Answer: a) papule 5) A raised palpable lesion greater than 1 cm at its greatest diameter Answer: b) nodule

EMQ: 11 Dysphagia a) Pharyngeal pouch b) Carcinoma of the oesophagus c) Benign stricture of the oesophagus d) Left atrial dilatation e) Aneurysm of the aorta f) Achalasia of the cardia g) Systemic sclerosis h) Bulbar palsy i) Impacted foreign body j) Schatzki ring Match the description below to one of the conditions listed above 1) A 70-year-old female with difficulty in swallowing. She has not lost any weight. She has a history of rheumatic fever as a child and on examination she is in atrial fibrillation, has a tapping apex and a loud first heart sound with a rumbling mid-diastolic murmur best heard just medial to the apex beat. Answer: d) left atrial dilatation The clinical features described suggest the patient has mitral stenosis and the patient has atrial fibrillation. This would imply that the left atrium is dilated. Left atrial dilatation could cause external compression of the oesophagus and this could result in dysphagia. 2) A 63-year-old male has a long-standing history of gastro-oesophageal reflux disease. He does not use the medications prescribed but uses antacids as required. He complains of difficulty in swallowing but has not lost any weight. Answer: c) benign stricture of the oesophagus

327

Mastering MRCP The history of longstanding reflux without treatment would increase the risk of benign stricturing. The fact that he has not lost weight would be a factor against cancer of the oesophagus. 3) A 67-year-old male with a history of ischaemic heart disease and stroke presents with a few months difficulty in swallowing. To begin with it affected solids more than liquids but he is now having difficulty with liquids as well. He has lost weight. Answer: b) carcinoma of the oesophagus The history of dysphagia for solids and liquids with loss of weight should make one think of carcinoma of the oesophagus till proved otherwise. 4) A 35-year-old male has long standing difficulty in swallowing. He has difficulty with both solids and liquids. He has not lost weight. Endoscopy shows a dilated oesophagus with food debris in it. Answer: f) achalasia of the cardia A benign cause is more likely in a young patient. The fact that the dysphagia is long standing and he has not lost weight are also features that would go against a diagnosis of malignancy. Solids and liquids being affected equally is a feature suggestive of achalasia of the cardia. A dilated proximal oesophagus would also be a feature in favour of achalasia. 5) A 54-year-old male presents as an emergency with sudden onset of pain in his chest and difficulty in swallowing. This came on whilst he was having a meal in a restaurant. There is complete dysphagia with inability to swallow saliva. Answer: i) impacted foreign body Sudden onset of symptoms whilst having a meal should alert one to the diagnosis of foreign body impaction. Pain and complete dysphagia are additional features in favour of this diagnosis.

EMQ: 12 Skin Lesions a) Papule b) Nodule c) Plaque d) Burrow e) Vesicle f) Bulla g) Pustule h) Cyst i) Target lesion j) Weal Match the description below to the lesion listed above 1) Raised flat topped lesion more than 1 cm in diameter Answer: c) plaque 2) Linear elevation of the epidermis caused by parasitic infestation Answer: d) burrow 3) Pus filled lesion less than 5 mm in diameter Answer: g) pustule 4) Large fluid filled lesion surrounded by a well-defined capsule Answer: h) cyst

328

Mastering MRCP 5) Three concentric zone of skin change. A central dark area or blister surrounded by a pale oedematous zone, which is in turn surrounded by an area of erythema Answer: i) target lesion

EMQ: 13 Causes of cirrhosis of the liver a) Haemochromatosis b) Wilson’s disease c) Primary biliary cirrhosis d) Osler-Rendu-Weber syndrome e) Alcohol f) Methotrexate g) Cardiac cirrhosis h) Budd-Chiari syndrome i) Alpha1 antitrypsin deficiency j) Cystic fibrosis Match the physical signs below to the cause of cirrhosis listed above 1) Parotid enlargement and Dupuytren’s contracture Answer: e) alcohol Parotid enlargement and Dupuytren’s contracture are physical signs that are associated with alcohol abuse 2) Clubbing, coarse crepitations at both lung bases Answer: j) cystic fibrosis Clubbing and coarse crepitations at both lung bases suggest that the patient has bronchiectasis. The association between bronchiectasis and cirrhosis of the liver would be cystic fibrosis. 3) A middle-aged female with xanthelasma Answer: c) primary biliary cirrhosis Primary biliary cirrhosis is associated with raised cholesterol levels and xanthelasma 4) Breathlessness, ankle oedema and a raised jugular venous pressure Answer: g) cardiac cirrhosis A raised JVP would suggest that the liver is congested and if this is long standing it could result in chronic liver disease (known as nutmeg liver in pathology) 5) Multiple telangiectasiae around the lips Answer: d) Osler-Rendu-Weber syndrome Hereditary haemorrhagic telangiectasia or Osler-Rendu-Weber syndrome may cause cirrhosis of the liver

EMQ: 14 Causes of cirrhosis of the liver a) Haemochromatosis b) Wilson’s disease c) Primary biliary cirrhosis d) Osler-Rendu-Weber syndrome e) Alcohol f) Methotrexate

329

Mastering MRCP g) Cardiac cirrhosis h) Budd-Chiari syndrome i) Alpha1 antitrypsin deficiency j) Cystic fibrosis Match the physical signs below to the cause of cirrhosis listed above 1) A young female on the contraceptive pill. On examination there is absent hepatojugular reflux Answer: h) Budd-Chiari syndrome In a young female on the contraceptive pill one should consider thromboembolic disease. Absent hepatojugular reflux occurs when there is an obstruction between the hepatic veins and the superior vena cava. In the context of disease of the liver one should consider Budd-Chiari syndrome. 2) A middle-aged female with ulnar deviation at the metacarpophalangeal joints and swan neck deformity of the fingers Answer: f) methotrexate The physical signs suggest the patient has rheumatoid arthritis. The connection between rheumatoid arthritis and liver disease would be treatment with methotrexate, which is hepatotoxic. 3) A young male with a ring of greenish brown pigment at the junction of the cornea and sclera Answer: b) Wilson’s disease The ring of greenish brown pigment is a Kayser-Fleischer ring, which is seen in Wilson’s disease. Wilson’s disease results in copper deposition in the liver with resultant hepatotoxicity 4) A male with a hyperinflated chest with reduced cardiac and liver dullness on percussion and diminished vesicular breath sounds on auscultation Answer: i) Alpha1 antitrypsin deficiency The signs suggest the patient has emphysema. Alpha1 antitrypsin deficiency causes emphysema and chronic liver disease. 5) A middle-aged male with increased pigmentation of the skin Answer: a) haemochromatosis Iron deposition in haemochromatosis causes pigmentation of the skin and chronic liver disease.

EMQ: 15 Causes of fibrosing alveolitis a) Renal tubular acidosis b) Primary biliary cirrhosis c) Scleroderma d) Rheumatoid arthritis e) Systemic lupus erythematosus f) Dermatomyositis g) Sjogren’s syndrome h) Inflammatory bowel disease i) Autoimmune hepatitis j) Polymyositis Match the cause of fibrosing alveolitis listed above to the clinical features below 1) A middle-aged female with smooth, shiny tight skin over the face and hands, telangiectasia, Raynaud’s phenomenon and calcinosis Answer: c) scleroderma

330

Mastering MRCP 2) A middle-aged female with painful joints, ulnar deviation at the metacarpophalangeal joints and swan neck deformity of the fingers Answer: d) rheumatoid arthritis 3) A middle-aged female who complains of itch and who has xanthelasma Answer: b) primary biliary cirrhosis 4) A middle-aged female with dry eyes and dry mouth Answer: g) Sjogren’s syndrome 5) A young female with a butterfly rash and alopecia Answer e) systemic lupus erythematosus

EMQ: 16 Abdominal pain a) Appendicitis b) Cholecystitis c) Mesenteric angina d) Peptic ulcer e) Biliary colic f) Pancreatitis g) Large bowel infarction h) Crohn’s disease i) Colon cancer j) Irritable bowel syndrome 1) A 67-year-old male who is known to have ischaemic heart diseases and transient ischaemic attacks, presents with abdominal pain related to meals. The pain comes on shortly after he eats and this has made him reduce his intake of food and lose weight. On examination there is a bruit over the upper abdomen. Answer: c) mesenteric angina The history of ischaemic heart disease and transient ischaemic attacks should alert one to the possibility of vascular insufficiency as a cause of the condition. Pain related to meals adds evidence to this possibility. The bruit should give the final clue. Loss of weight is common in patients with mesenteric angina 2) A 56-year-old businessman presents with epigastric pain. The pain is gnawing in nature and is relieved by food but increases when he is hungry and wakes him up from his sleep in the early hours of the morning. He is stressed at work and smokes heavily. Answer: d) peptic ulcer The history of pain relieved by meals and nocturnal pain is suggestive of a duodenal ulcer. 3) A 30-year-old male presents with severe abdominal pain of acute onset. The pain is relieved by leaning forward. He gives a history of binge drinking and admits to a recent binge. On examination he looks unwell, there is generalised tenderness over his abdomen and there is bruising in the flanks. Answer: f) pancreatitis The severe pain precipitated by an alcoholic binge should suggest pancreatitis. The pain relieved by leaning forward is in keeping with a retroperitoneal lesion. Severe tenderness and bruising in the flaks lends weight to the diagnosis. 4) A 72 year old male who has had a previous myocardial infarction and stroke is admitted with abdominal pain, diarrhoea and bleeding per rectum. He awoke from his sleep with severe left sided abdominal pain

331

Mastering MRCP and this was followed by profuse watery diarrhoea and later he began passing fresh blood per rectum. On examination he looked unwell was tachycardic and there was a systolic bruit audible over the central abdomen. Answer: g) large bowel infarction The history of sudden onset abdominal pain followed by diarrhoea and bleeding per rectum is suggestive of a large bowel infarction. The fact that the patient is a known vasculopath lends weight to the diagnosis. The finding of a bruit over the abdomen adds further weight to the diagnosis of a vascular lesion in the abdomen. 5) A 27-year-old female presents with a history of abdominal pain of several months duration. She admits to passing loose motions and she has lost weight. She smokes 20 cigarettes per day. On examination there are painful tender erythematous nodules over her shins and there is a tender mass in the right iliac fossa. Answer: h) Crohn’s disease A young female with abdominal pain, weight loss and diarrhoea should alert one to the possibility of inflammatory bowel disease. Crohn’s disease tends to affect females more than males and the fact that she smokes will be further evidence in favour of this diagnosis (the onset of ulcerative colitis is related to cessation of smoking). The tender red nodules over her shins should make one think of erythema nodosum and the mass in the right iliac fossa would suggest and ileocaecal mass in keeping with Crohn’s disease.

EMQ: 17 Abdominal pain a) Mesenteric angina b) Cholecystitis c) Appendicitis d) Acute intermittent porphyria e) Peptic ulcer f) Biliary colic g) Pancreatitis h) Large bowel infarction i) Crohn’s disease j) Hyperparathyroidism 1) A 16-year-old boy is admitted with abdominal pain. The pain started in the periumbilical region and then shifted to the right iliac fossa. It is a constant pain and he feels nauseated and has vomited several times. On examination there is tenderness and rebound tenderness in the right iliac fossa. Answer: c) appendicitis The patient has the typical features of acute appendicitis. 2) A 27-year-old female is admitted with abdominal pain, vomiting and constipation. She gives a history of depression. She has recently started taking the oral contraceptive pill. There is family history of a similar illness. On examination she is tachycardic and her blood pressure is high. There is evidence of a peripheral motor neuropathy. The nurse on the ward reports to you that she is afraid the patient might have haematuria as a sample of urine taken from the patient had been inadvertently left standing and the urine had turned red. Answer: d) acute intermittent porphyria

332

Mastering MRCP The history of pain, constipation and vomiting in association with features suggestive of nerve damage and autonomic dysfunction should alert one to the possibility of porphyria, which is one of the rare causes of abdominal pain. The positive family history, the possible precipitation of the condition by the oral contraceptive pill and the fact that the urine turned red on standing are all features that lend weight to this diagnosis. 3) A 63-year-old male is admitted with abdominal pain. There is a history of renal colic and the patient also complains of pain in his bones. On further questioning he says he is constipated and passes a lot of urine. The patient has corneal calcification. Answer: j) hyperparathyroidism The patient gives the typical features of hyperparathyroidism; pain in the bones, abdominal pain and renal colic (stones, bones and abdominal groans). Polyuria and constipation would suggest the patient has hypercalcaemia and calcification in the cornea is evidence of hypercalcaemia. 4) A 50-year-old female presents with abdominal pain. The pain is described as a cramping discomfort, which is constant. The pain is severe. It is felt in the epigastrium and radiates to the back. The pain comes on rapidly and lasts from 3- 6 hours. It is associated with nausea and vomiting. The pain is related to meals coming on a few hours after a meal, especially a fatty meal. The patient says she is having recurrent episodes of pain like this. No other significant illnesses, she is married and has 5 children. She is obese. Answer: f) biliary colic The history of epigastric pain radiating to the back and lasting less than 6 hours should make one think of biliary colic. The relation to fatty meals and the fact that the patient is multiparous and obese would also suggest this and pint to her having gallstones. 5) A 53-year-old female presents with abdominal pain. The pain is in the upper abdomen on the right side under the ribs. The pain radiates to the right shoulder. The pain is worse on breathing in deeply. She has nausea and vomiting and fever. On examination she is markedly tender in the right hypochondrium. Answer: b) cholecystitis The pain in the right hypochondrium radiating to the right shoulder should make one think of biliary tract disease. The pain being aggravated by breathing and the fever should suggest inflammation. Tenderness in the right hypochondrium would also suggest inflammation and in this region the most likely condition would be cholecystitis.

EMQ: 18 Endoscopy a) Gastroscopy with banding b) Gastroscopy with injection of adrenaline and heater probe coagulation c) Colonoscopy d) Flexible sigmoidoscopy e) Gastroscopy f) Enteroscopy g) Capsule endoscopy h) Gastroscopy with balloon dilatation i) ERCP j) Gastroscopy with placement of a stent

333

Mastering MRCP 1) A 63-year-old female complains of feeling tired and breathless on exertion. There is no history of postmenopausal bleeding. Investigations show she has iron deficiency anaemia. She has had a gastroscopy, which has been reported as normal with normal distal duodenal biopsies Which of the above would the patient require? Answer; c) colonoscopy In a postmenopausal female with iron deficiency anaemia it is imperative that one excludes colorectal cancer as a cause. Colonoscopy would be though investigation of choice. 2) A 64-year-old male presents with iron deficiency anaemia. He eats a normal diet and eats red meat several times a week. He has been investigated extensively with normal gastroscopy, colonoscopy, barium follow through and enteroscopy. Which investigation might you arrange next to try and determine the cause of his iron deficiency anaemia? Answer: g) capsule endoscopy In a male of this age group who is unlikely to have a dietary deficiency, the chances are that iron deficiency is due to gastrointestinal bleeding. No source has been found despite extensive investigation. Capsule endoscopy may localise a source of blood loss in the small bowel. 3) A 54-year-old alcoholic is admitted with haematemesis. On examination he is jaundiced, has palmar erythema, multiple spider naevi, gynaecomastia, reduced body hair and ascites. Which of the above is the patient likely to require? Answer: a) gastroscopy with banding The physical signs in this patient show he has hepatocellular failure and portal hypertension. This means the patient has cirrhosis of the liver and is thus likely to have oesophageal varices and requires a gastroscopy and probably banding of varices. 4) A 60-year-old female who has been taking ibuprofen for joint pains is admitted with haematemesis and melaena. Her haemoglobin concentration is 6.5 g/dL. Endoscopy shows a chronic duodenal ulcer with adherent clot. Which of the above would the patient require? Answer: b) Gastroscopy with injection of adrenaline and heater probe coagulation The patient has a duodenal ulcer with stigmata of recent haemorrhage. She will require endoscopic intervention with injection of adrenaline and heater probe coagulation 5) A 62-year-old male who has long standing gastro-oesophageal reflux disease presents with dysphagia of several months duration. Endoscopy shows a benign stricture of the lower oesophagus Which of the above would the patient require? Answer: h) gastroscopy with balloon dilatation Benign stricture of the oesophagus would be best dealt with by performing gastroscopy with balloon dilatation

EMQ: 19 Endoscopy a) Gastroscopy with banding b) Gastroscopy with injection of adrenaline and heater probe coagulation c) Colonoscopy d) Flexible sigmoidoscopy e) Gastroscopy

334

Mastering MRCP f) Enteroscopy g) Capsule endoscopy h) Gastroscopy with balloon dilatation i) ERCP j) Gastroscopy with placement of a stent Match the scenario given below to one of the procedures listed above 1) A 67-year-old male presents with jaundice. He has no abdominal pain but he says his urine has become very dark and his stools have become pale and difficult to flush away. The ultrasound scan shows dilated bile ducts but the pancreatic region is not visualized. Answer: i) ERCP The patient has jaundice with pale stools and dark urine. This suggests it is obstructive jaundice. It is painless and this would point to malignancy as the cause of the obstruction rather than stones in the bile duct. The ultrasound has shown dilated bile ducts and this is in keeping with the clinical suspicion of obstructive jaundice. No stones have been seen but the pancreatic region has not been visualized and the suspicion here would be that the cause of the obstruction if cancer of the head of the pancreas. ERCP would be the best investigation to evaluate this, as it would provide imaging, the ability to take specimens for cytology or histology and insert a stent to relieve obstruction. 2) A 72-year-old male presents with dysphagia and loss of weight. Endoscopy has shown an adenocarcinoma of the oesophagus and this has been confirmed by biopsy. CT scan has shown metastatic disease in the liver and lungs Answer: j) gastroscopy with placement of a stent The patient has cancer of the oesophagus with widespread metastases and this would make the disease inoperable. Palliative stenting would be the management option in this patient. 3) A 26-year-old female presents with abdominal pain, loose motions and loss of weight. On examination she has painful tender nodules over her shins and a tender mass in her right iliac fossa. Answer: c) colonoscopy The clinical features suggest the patient has ileocaecal Crohn’s disease. Colonoscopy with terminal ileal intubation would be useful to localize the disease and perform biopsies for histological confirmation. 4) A 24-year-old male presents with diarrhoea of several weeks duration. On further questioning he admits to having loose stools for almost a year. At present he goes to the toilet about 10 times during the day and wakes up from sleep 3 times at night to go to the toilet. He passes very loose stools with blood and mucous. Answer: d) flexible sigmoidoscopy The clinical features of a young male with chronic diarrhoea with blood and mucous would suggest the patient has ulcerative colitis and in this case an unprepared flexible sigmoidoscopy would be the investigation of choice. 5) A 60-year-old male presents with new onset of dyspepsia Answer: e) gastroscopy New onset of dyspepsia and loss of weight would make it mandatory for a patient in this age group to have an endoscopy

EMQ: 20 Abdominal pain a) Acute appendicitis b) Diverticular disease

335

Mastering MRCP c) Abdominal aortic aneurysm d) Perforated peptic ulcer e) Crohn's disease f) Ulcerative colitis g) Acute pancreatitis h) Diverticulitis i) Acute viral hepatitis j) Acute cholecystitis Match one of the options above to the most likely scenario given below: 1) A 55-year-old male who has been taking antacids for long standing dyspepsia, presents with severe upper abdominal pain that has subsequently spread and become generalised. For the week prior to admission he had used ibuprofen to ease pain in his right knee On examination he is afebrile, pulse rate 120 beats per minute, BP 110/60. His abdomen does not move with respiration, it is rigid. Percussion note over the right lower ribs is resonant and bowel sounds are absent Answer: d) perforated peptic ulcer The history of dyspepsia should make one suspect a peptic ulcer. The ingestion of ibuprofen should alert one to the possibility of a complication. The physical signs would suggest air under the diaphragm and peritonitis. 2) A 76-year-old female presents with colicky pain in the left iliac fossa. She also complains of rectal bleeding and diarrhoea. She is febrile. A thickened mass is palpable in the left iliac fossa. It is tender. The white cell count shows a neutrophilia. Answer: h) diverticulitis Pain in the left iliac fossa with diarrhoea and rectal bleeding should make one suspect pathology in the left colon. A palpable mass in this region should reinforce this suspicion. Tenderness would suggest inflammation and the neutrophilia would also favour this. 3) A 27-year-old male presents with anorexia, nausea, vomiting and fever of a few days duration. He has recently returned from a holiday in the Far East. On examination he is febrile, has icteric sclera and has tender hepatomegaly. Answer: i) acute viral hepatitis Fever, anorexia, nausea and vomiting of a few days duration should make one suspect an infective aetiology. Travel to the Far East would add weight to this suspicion. Jaundice would make one suspect that the liver is the focus of the infection and tender hepatomegaly should make one suspect inflammation of the liver. 4) A 35-year old male presents with sudden onset of epigastric pain radiating to the back. The pain is severe and characterized as deep and boring. Eating food worsens the pain and bending forward relieves the pain. On examination the patient is febrile he has a tachycardia, and hypotension. Abdominal examination reveals flank ecchymosis and umbilical ecchymosis. On palpation there is epigastric tenderness with localized guarding and rebound, Bowel sounds are sluggish. Answer: g) acute pancreatitis The history of abdominal pain following an alcoholic binge should alert one to the possibility of pancreatitis. The increase in severity of the pain following food and relief by bending forward is also in favour of this. The systemic signs would favour this diagnosis. The ecchymoses would suggest retroperitoneal haemorrhage in association with pancreatitis. The tenderness in this region would be further evidence in favour of this diagnosis and the sluggish bowel sounds would suggest associated ileus.

336

Mastering MRCP 5) A 75-year-old male presents with central abdominal pain radiating through to his back. On examination he is tachycardic and hypotensive and has a pulsatile mass in the central abdomen with a systolic bruit audible over it Answer: c) abdominal aortic aneurysm In an elderly patient presenting with central abdominal pain radiating to his back one of the possibilities is an aortic aneurysm. The features of a pulsatile mass with a systolic bruit over it should make this possibility more likely.

EMQ: 21 Causes of hypertension a) Phaeochromocytoma b) Polycystic kidneys c) Conn’s syndrome d) Polycythaemia rubra vera e) Renal artery stenosis f) Chronic glomerulonephritis g) Hyperparathyroidism h) Cushing’s syndrome i) Coarctation of the aorta j) Acromegaly Match one of the causes of hypertension listed above to the appropriate clinical scenario given below: 1) A 25-year-old female is found to be hypertensive. On abdominal palpation there are ballotable masses in both flanks Answer: b) polycystic kidneys Ballotable masses in both flanks suggest the patient has bilateral enlargement of the kidneys There are a few causes of bilateral enlargement of the kidneys Out of the causes listed above only polycystic kidneys will present with bilateral enlargement of the kidneys. 2) A 56-year-old female, who is centrally obese, has a moon face and buffalo hump. Abdominal examination reveals multiple striae. Answer: h) Cushing’s syndrome In a centrally obese patient with hypertension, one should consider the endocrine causes of hypertension. The clinical features of a moon face, buffalo hump and abdominal striae should suggest Cushing’s syndrome. 3) An 18-year-old male is found to be hypertensive. On examination of his pulse, there is radio-femoral delay and he has a systolic murmur at the base of the heart. Answer: i) coarctation of the aorta In a young patient presenting with hypertension rare causes should be considered. The finding of radiofemoral delay should make one think of coarctation of the aorta. A systolic murmur may be heard at the base of the heart in coarctation of the aorta due to turbulent blood flow across the aortic valve 4) A 63-year old female presents with hypertension. On abdominal examination a bruit is heard in the upper abdomen. Answer: e) renal artery stenosis

337

Mastering MRCP In a patient with hypertension, the finding of a bruit in the upper abdomen should suggest renal artery stenosis 5) A 63-year-old male is found to be hypertensive. He complains that his wedding ring is tight and he has had to buy bigger shoes. Answer: j) acromegaly The history of his wedding ring becoming tight and his shoe size increasing would suggest an increase in the size of his feet and hands and therefore should suggest acromegaly as the cause of hypertension.

EMQ: 22 a) Sarcoidosis b) Malaria c) Dressler’s syndrome d) Whipple’s disease e) Lymphoma f) Polymyalgia rheumatica g) Infective endocarditis h) Retroperitoneal fibrosis i) Typhoid fever j) Tuberculosis 1) A 45-year-old homeless male, who drinks 50 units of alcohol a week and smokes heavily, is admitted with a history of fever, cough and loss of weight. On examination he looks emaciated, his trachea is deviated to the left, there is flattening of the upper part of his chest on the left hand side, chest movements are decreased on the left hand side, upper zone, breath sounds are vesicular and there are fine crepitations in the left upper zone Answer: j) tuberculosis In a homeless alcoholic presenting with cough, fever and loss of weight; tuberculosis must be one of the first conditions one would consider. The physical signs suggest a chronic illness (emaciation) and left apical fibrosis. Thus the most likely diagnosis is tuberculosis. 2) A 26-year-old male who has been on a holiday to Kenya presents with fever accompanied by chills and rigors. On examination of his abdomen his spleen is just palpable below the left costal margin Answer: b) malaria The history of fever with chills and rigors after returning from Kenya would immediately make one think of malaria. The finding of splenomegaly would also be in favour of this diagnosis. 3) A 32-year-old male presents with a history of fever with night sweats and loss of weight. On examination he has cervical, axillary and inguinal lymphadenopathy and hepatosplenomegaly. Answer: e) lymphoma The history suggests a chronic illness but does not give a significant clue. However the findings on examination would make one think of lymphoma. 4) A 40-year-old businessman presents with fever and loss of weight of a few weeks duration. He gives a history of having had a dental extraction 2 months prior to presentation. On examination he is febrile; there is clubbing and erythematous macules on his palms. His pulse rate is 110 beats per minute; regular, jugular venous pressure is not raised. The apex beat is at the 5th left intercostal space in the mid-clavicular line and there is a pan systolic murmur at the apex and this radiates to his axilla. Answer: g) infective endocarditis

338

Mastering MRCP The history of dental extraction should make one think of infective endocarditis. The clinical features of clubbing and Janeway lesions (erythematous macules on his palms) are in favour of this suspicion. The findings suggestive of mitral regurgitation would make the diagnosis more likely. 5) A 26-year-old male presents with a history of fever. He has been on a backpacking holiday in India. On examination he is febrile, there is an erythematous rash on his trunk, his spleen is palpable. His pulse rate is 60 beats per minute. Answer: i) typhoid fever The history of travel to India with possible exposure to contaminated food or water (backpacking) would make one think of typhoid fever. The rash (rose spots) would be further evidence in favour of this diagnosis. The other supporting evidence is splenomegaly and the relative bradycardia.

EMQ: 23 Lesions seen on ophthalmoscopy a) Papilloedema b) Papillitis c) Optic atrophy d) Glaucoma e) Central retinal artery occlusion f) Angioid streaks g) Central retinal vein occlusion h) Choroidoretinitis i) Retinitis pigmentosa j) Drusen 1) On examination of the fundus you note yellowish-white patchy exudates and haemorrhages Answer: h) choroidoretinitis These are the typical features of acute choroidoretinitis 2) On examination of the fundus you note that the optic cup is filled. The disc is more pink than normal. The margins of the disc are difficult to define. The veins are engorged and non-pulsatile. The blood vessels disappear as the traverse the edge of the disc. Answer: a) papilloedema These are the typical features of papilloedema 3) On examination of the fundus you note multiple discrete yellowish-white spots around the macula and the posterior aspect of the retina Answer: j) drusen These are the typical features of drusen 4) On examination of the fundus you note that the optic cup is deep. The blood vessels appear to climb out of the cup which looks like a well Answer: d) glaucoma This is the typical appearance of glaucoma 5) On examination of the fundus you note that the disc looks pale .the number of blood vessels crossing the discs is reduced Answer: c) optic atrophy This is the typical appearance of optic atrophy

339

Mastering MRCP EMQ: 24 Lesions seen on ophthalmoscopy a) Papilloedema b) Papillitis c) Optic atrophy d) Glaucoma e) Central retinal artery occlusion f) Angioid streaks g) Central retinal vein occlusion h) Choroidoretinitis i) Retinitis pigmentosa j) Drusen 1) On examination of the fundus you note linear dark-red streaks beneath the blood vessels Answer: f) Angioid streaks Angioid streaks present as linear red or grey streaks beneath the retinal vessels 2) On examination of the fundus you note the disc is swollen. It is red and appears cloudy with exudates on the disc and in the overlying vitreous The veins are not distended Answer: b) papillitis This is the typical appearance of papillitis 3) On examination of the fundus you note that the fundus is pale. There are few vessels, which look thin. There is a cherry red spot at the macula Answer: e) central retinal artery occlusion This is the typical appearance of central retinal artery occlusion 4) On examination of the fundus you note black pigment, which has the appearance of bone corpuscles, scattered throughout the retina Answer: I) retinitis pigmentosa This is the typical appearance of retinitis pigmentosa 5) On examination of the fundus you note that the veins are tortuous and engorged. Multiple haemorrhages and soft exudates are seen. There is papilloedema Answer: g) central retinal vein occlusion This is the typical appearance of central retinal vein occlusion. The focus is on the veins

EMQ: 25 Nerve Lesions a) Radial nerve b) Obturator nerve c) Axillary nerve d) Femoral nerve e) Median nerve f) Tibial nerve g) Ulnar nerve h) Common peroneal nerve i) Musculocutaneous nerve

340

Mastering MRCP j) Sciatic nerve 1) A woman presents with weakness of adduction and internal rotation of the hip. She has sensory loss over the medial part of the thigh. She is post-partum and the baby was delivered by use of forceps. Answer: b) obturator nerve The clinical features indicate a lesion of the obturator nerve. This nerve may be damaged by injuries during parturition. 2) A young man who has recently lost a lot of weight presents with a right-sided foot drop. There is weakness of dorsiflexion and eversion at the ankle and there is weakness of toe extension. Sensation is impaired over the lateral aspect of his lower leg and the dorsum of the foot. Answer: h) common peroneal nerve The clinical features suggest the patient has a lesion of the common peroneal nerve. Loss of weight with loss of the protective fat pad over the common peroneal can cause compression injury of this nerve. 3) A patient presents with weakness of all muscles below the knee. The ankle jerk is lost. There is loss of sensation over the foot except the medial border. Answer: j) sciatic nerve The clinical features fit with a lesion of the sciatic nerve 4) A patient presents with weakness of plantar flexion and inversion of the foot. There is weakness of flexion of the toes. There is a claw like deformity of the toes. Sensation is lost over the sole of the foot. Answer: f) tibial nerve The clinical features fit with a lesion of the tibial nerve. 5) A patient presents with weakness of the triceps muscle and a wrist drop. There is sensory loss over the first dorsal interosseus muscle. Answer: a) radial nerve The clinical features fit in with a lesion of the radial nerve in the axilla

EMQ: 26 Nerve Lesions a) Radial nerve b) Obturator nerve c) Axillary nerve d) Femoral nerve e) Median nerve f) Tibial nerve g) Ulnar nerve h) Common peroneal nerve i) Musculocutaneous nerve j) Sciatic nerve 1) A patient presents with weakness of shoulder abduction. There is a small area of anaesthesia over the lower part of the deltoid Answer: c) axillary nerve The clinical features fit in with a lesion of the axillary nerve 2) A patient presents with weakness of knee extension and wasting of the quadriceps muscle. There is loss of the knee jerk. There is sensory loss over the front of the thigh and medial aspect of the lower leg as far as the medial malleolus.

341

Mastering MRCP Answer: d) femoral nerve The clinical features fit in with a lesion of the femoral nerve. 3) A patient presents with weakness of the biceps, coracobrachialis and brachialis. There is sensory loss over the lateral aspect of the forearm Answer: I) musculocutaneous nerve The clinical features fit in with a lesion of the musculocutaneous nerve 4) A patient presents with wasting and weakness of the muscles of the lateral part of the thenar eminence. There is loss of sensation over the lateral three and a half fingers. Tinel’s sign is positive Answer: e) median nerve The clinical features fit in with a lesion of the median nerve 5) A patient presents with wasting and weakness of the small muscles of the hand except the muscles of the lateral part of the thenar eminence. There is sensory loss over the medial one and a half fingers. Answer: g) ulnar nerve The clinical features fit in with a lesion of the ulnar nerve

EMQ: 27 Jaundice a) Viral hepatitis b) Cirrhosis of the liver c) Gilbert’s syndrome d) Carcinoma head of pancreas e) Hepatoma f) Common bile duct stones g) Primary biliary cirrhosis h) Primary sclerosing cholangitis i) Malaria j) Liver metastases 1) A 65-year-old male complains of yellow discolouration of his eyes. He complains of passing dark urine and on direct questioning says that his stools are pale. He has a poor appetite and has lost weight. He denies any abdominal pain. On examination he appears to have lost weight but there are no other abnormalities. Answer: d) Carcinoma head of pancreas Jaundice with pale stools and dark urine suggests obstruction. The lack of pain suggests that it is of gradual onset and extraluminal. Loss of weight would suggest malignancy. 2) A 24-year-old university student who has returned from a backpacking holiday in India presents with fever, abdominal discomfort and passage of dark urine of several days duration. On examination his sclera is icteric, there is 3 fingerbreadths tender hepatomegaly. Answer: a) Viral hepatitis Fever, trip to India suggests infection. Backpacking would suggest inadequate food hygiene. Tender hepatomegaly would suggest hepatitis. 3) A 55-year-old obese female presents with a history of colicky right hypochondrial pain. The pain subsides but she notices yellow discolouration of her eyes. On direct questioning she reveals that her urine is dark in colour and her stools are pale. She is a dinner lady at a school, married and has 5 children.

342

Mastering MRCP On examination she is obese, has fever and is tender over the right hypochondrium. Answer: f) Common bile duct stones Jaundice, dark urine, pale stools suggests obstructive jaundice. Colicky abdominal pain would suggest that the cause of the obstruction is intraluminal. Obese, multi para would suggest gallstones as the cause. 4) A 60-year-old male presents with yellow discolouration of his eyes. He gives a history of altered bowel habit with alternating constipation and loose motions of several months duration. Recently he has noticed that he has passed some blood in his stool. On examination he appears unwell, abdominal examination reveals an enlarged liver that is hard in consistency and the surface of the liver is irregular. A bruit is heard over it. Answer: j) Liver metastases Altered bowel habit with blood in the stools in a 60-year-old male should make one think of bowel cancer. A hard liver with an irregular surface in this context would indicate bowel cancer with liver metastases. 5) A 36-year-old male who is known to have ulcerative colitis, which is well controlled on Asacol 800 mgs t.d.s. presents with fever, abdominal pain, itching and yellow discolouration of his skin . On examination he is febrile, jaundiced and has hepatosplenomegaly. Answer: h) Primary sclerosing cholangitis Fever, jaundice, itching and abdominal pain would suggest cholangitis. In the context of a history of ulcerative colitis one should consider primary sclerosing cholangitis.

EMQ: 28 Loss of Weight a) Achalasia of the cardia b) Carcinoma of the stomach c) Bowel cancer with liver metastases d) Crohn’s disease e) Ulcerative colitis f) Coeliac disease g) Short gut syndrome h) Oesophageal cancer i) Peptic stricture of the oesophagus j) Giardiasis 1) A 20-year-old Irish female presents with loose motions, mouth ulcers and an itchy rash. The stools are bulky and difficult to flush away. On examination she appears to have lost weight and she has a vesicular rash over her lower back. Most of vesicles have been scratched away leaving crusts. Investigations reveal a macrocytic anaemia. Answer: f) Coeliac disease Loose bulky motions with loss of weight suggest malabsorption. With a history of mouth ulcers in an Irish patient one should consider coeliac disease. The history of the itchy vesicular rash would suggest dermatitis herpetiformis, which occurs in coeliac disease 2) A 24-year-old female presents with a rash over her lower legs. She gives a long-standing history of diarrhoea and abdominal pain. She has lost a considerable amount of weight. On examination she appears to have lost weight, there are tender erythematous nodules over her shins and there is a mass in the right iliac fossa.

343

Mastering MRCP Answer: d) Crohn’s disease Diarrhoea and abdominal pain in a young female, one would think of inflammatory bowel disease The description of the rash suggests erythema nodosum A mass in the right iliac fossa would be in keeping with an inflammatory mass associated with Crohn’s disease. 3) A 67-year-old male presents with progressive dysphagia over several months duration. The dysphagia affects solid more than liquids. He smokes 20 cigarettes a day and drinks 35 units of alcohol a week. On examination he looks emaciated and has hepatomegaly. Answer: h) Oesophageal cancer Dysphagia in an older patient with solids affected more than liquids would point to a malignancy The enlarged liver suggesting liver metastases is further evidence in favour of this 4) A 70-year-old female presents with severe anorexia and loss of weight. On examination she looks emaciated and has a palpable lymph node in the right supraclavicular region. Answer: b) Carcinoma of the stomach An elderly patient presenting with anorexia should make one think of cancer of the stomach The fact that she is emaciated is in favour of this and the lymph node in the supraclavicular fossa, Virchow’s node, (Troisier’s sign) is further evidence in favour of this 5) A 35-year-old female presents with loss of weight and diarrhoea. She has had multiple operations for Crohn’s disease of the small and large bowel. On examination she looks emaciated; there are multiple scars on her abdominal wall. There are no peripheral signs of chronic inflammatory disease and investigations reveal that inflammatory markers are not raised Answer: g) Short gut syndrome Diarrhoea and weight loss should suggest malabsorption. In the light of multiple resections of the gut short gut syndrome would be the most likely option.

EMQ: 29 Loss of consciousness a) Subdural haematoma b) Encephalitis c) Subarachnoid haemorrhage d) Status epilepticus e) Meningitis f) Intracerebral haemorrhage g) Hypoglycaemia h) Diabetic ketoacidosis i) Carbon monoxide poisoning j) Heroin overdose 1) An 18 year old male who is an insulin dependent diabetic is brought into casualty by his friends. He has been found unconscious in his room at the university halls of residence in the morning. On examination he does not look dehydrated, he is not rousable even when painful stimuli are used, his respiratory rate is 15 breaths per minute and not deep. Answer g) hypoglycemia

344

Mastering MRCP It is likely that he developed hypoglycaemia as he has been found in the room in the morning presumably before breakfast. It is not diabetic ketoacidosis as he is not dehydrated and not apparently acidotic as his respiration is normal (one would have expected Kussmaul’s respiration if he was in diabetic ketoacidotic coma) 2) An 80-year-old male who has been having frequent falls presents with a fluctuating level of consciousness noted by the carriers at the nursing home he lives in. On clinical examination there are no neurological signs. Answer a) subdural haematoma Falls would predispose an elderly patient to subdural haemorrhage and the fluctuating level of consciousness is one of the features of this condition. 3) A 56-year-old male who has hypertension for which he has not been taking medication presents with a history of headache, nausea and vomiting and left sided weakness of sudden onset. Answer: f) Intracerebral haemorrhage Uncontrolled hypertension would predispose the patient to an intracerebral haemorrhage, which could cause a stroke. The history of headache, nausea and vomiting in association with stroke would make haemorrhage more likely. 4) A 22-year-old university student is admitted with fever and a rash. On examination he is febrile, has a haemorrhagic, rash he is drowsy and there is neck stiffness. Answer: e) meningitis Fever and rash with loss of consciousness and neck stiffness should make one think of meningococcal meningitis 5) A 45-year-old female presents with a history of sudden onset occipital headache followed by a deteriorating level of consciousness. On examination there is neck stiffness. Answer: c) subarachnoid haemorrhage Sudden onset of occipital headache should make one think of subarachnoid haemorrhage and this is confirmed by the fact that the patient has neck stiffness

EMQ: 30 Sexually transmitted infections a) Neisseria gonorrhoeae b) Treponema pallidum c) Haemophilus ducreyi d) Chlamydia trachomatis e) HIV f) Human Papilloma Virus g) Ureaplasma urealyticum (NGU) h) Haemophilus vaginalis (NGU) i) Mycoplasma genitalium (NGU) j) Klebsiella granulomatis (Granuloma inguinale, donovanosis) 1) A 32 year old male presents with complaining of ulcers on his penis. He gives a history of unprotected sexual intercourse 2 seeks prior to presentation. On examination he is noted to have ulcerated nodules on the shaft of his penis. They are painless. Answer: j) Klebsiella granulomatis Granuloma inguinale, donovanosis

345

Mastering MRCP Sexually transmitted infection Causative organism Klebsiella granulomatis 10–40 days following contact Lesions at the region of contact Shaft of the penis Labia Perineum Rarely, the vaginal wall or cervix Small, painless nodules appear Later the nodules ulcerate creating open, fleshy, oozing lesion Infection spreads, mutilating the infected tissue 3 weeks erythromycin, streptomycin, or tetracycline 12 weeks of treatment with ampicillin 2) A 34 year old male presents with a history of a burning sensation on passing urine and increased frequency of micturition. He also has a white discharge per urethra. Gram stain done on the discharge does not show any gram negative cocci. Which organism is the most likely to be responsible? Answer: d) Chlamydia trachomatis The patient has urethritis and no gonococci have been demonstrated. The patient therefore has non gonococcal urethritis of non specific urethritis. The commonest cause of NGU or NSU is Chlamydia NGU Urethritis · Dysuria (pain or a burning sensation upon urination) · White or cloudy discharge · Increased frequency of micturition Clinical Features Males · Discharge from penis · Burning or pain when urinating · Itching · Irritation · Tenderness · Stains on underwear Females · Discharge from vagina · Burning or pain when urinating · Anal or oral infections · Abdominal pain or abnormal vaginal bleeding (Pelvic Inflammatory Disease) Gram’s stain no gonococci Commonest cause of NGU · Chlamydia Other organisms · Ureaplasma urealyticum · Haemophilus vaginalis · Mycoplasma genitalium Treatment

346

Mastering MRCP · Doxycycline 3) A 40 year old male presents with an ulcer on his penis. He gives a history of unprotected sexual intercourse 3 weeks prior to presentation. On examination he has a painless , indurated ulcer on the glans penis and there is regional lymphadenopathy Answer: b) Treponema pallidum Syphilis is caused by Treponema pallidum. Primary syphilis presents with a painless indurated ulcer (chancre) at the site of inoculation and regional lymphadenopathy. Dark ground microscopy will demonstrate spirochetes Antibody tests will be positive Treatment is with penicillin Doxycycline, tetracycline, ceftriaxone, erythromycin for penicillin allergic individuals 4) A 35 year old male presents with ulceration on his penis. He gives a history of unprotected sexual intercourse 6 days prior to presentation. On examination he is noted to have multiple ulcers on his prepuce and frenulum. The ulcers have a necrotic base and ragged edge; they are painful and bleed easily. There is unilateral inguinal lymphadenopathy Answer: c) Haemophilus ducreyi Chancroid Sexually transmitted disease caused by Haemophilus ducreyi Presents with multiple painful ulcers and inguinal lymphadenopathy Swabs for gram stain and PCR Treat with · Azithromycin · Ceftriaxone · Ciprofloxacin · Erythromycin 5) A 24 year old male presents with pain and a burning sensation on passing urine. 2 weeks prior to presentation he has had unprotected sexual intercourse. On examination he is noted to have a purulent discharge per urethra. Gram stain shows gram negative diplococci Answer: a) Neisseria gonorrhoeae Gonorrhoea Sexually transmitted infection caused by Neisseria gonorrhoeae which is a gram negative diplococcus Local infection usually Rarely generalised spread with fever, rash, arthritis, hepatitis Treatment Ciprofloxacin Amoxicillin and probenecid Ceftriaxone Spectinomycin

EMQ: 31 Haematological conditions a) Acute lymphoblastic leukemia b) Chronic lymphocytic leukemia c) Chronic myeloid leukemia

347

Mastering MRCP d) Acute myeloid leukemia e) Polycythaemia rubra vera f) Primary thrombocythaemia g) Myeloma h) Hodgkin’s lymphoma i) Non-Hodgkin’s lymphoma j) Myelofibrosis 1) A 76 year old male presents with pain his chest and back. Chest x-ray shows multiple rib fractures, xray of his spine shows wedge compression of his lumbar vertebrae. His ESR is very high , calcium levels are elevated and there is Rouleaux formation on his blood film Answer: g) Myeloma Myeloma refers to malignant disease of the plasma cells in the bone marrow 2) A 60 year old male presents with a history of fever and tiredness. On examination he is noted to have splenomegaly. Hb 9.0 g/dL WBC 66 x109/L 60% lymphocytes Platelets 120 x109/L Answer: b) Chronic lymphocytic leukemia Chronic lymphocytic leukemia is a chronic haematalogical malignancy of excessive proliferation of lymphocytes B rather than T 3) A 16 year old male presents with a history of fever and bleeding gums. On examination he is found to have lymphadenopathy and hepatosplenomegaly. Hb 8.8g/dL WBC 20 x109/L Platelets 56 x109/L Blood film show multiple lymphoblasts Answer: a) Acute lymphoblastic leukemia An acute haematalogical malignancy of proliferating abnormal lymphoblastic white cells that infiltrate the body; marrow, blood, lungs, central nervous system 4) A 54 year old male presents with a history of feeling tired and weak. On examination there are multiple bruises on his shins. There is hepatosplenomegaly. Hb 9.3 g/dL WBC 45 x109/L Platelets 45 x109/L Blood film shows multiple blast cells Answer: d) Acute myeloid leukemia Acute haematalogical malignancy of proliferating abnormal myeloid white cells the infiltrate the body; marrow, blood , lungs , central nervous system 5) A 60 year old male presents with a history of tiredness and pain in his legs on walking. On examination you note that his face looks plethoric. There is splenomegaly Hb 20 g/dL WBN 8.0 x109/L Platelets 500 x109/L Answer: e) Polycythaemia rubra vera

348

Mastering MRCP

Polycythaemia rubra vera refers to a clonal stem cell disorder where there is proliferation of all progenitor cells due to a genetic failure of apoptosis

The End Dedicated to All Iraqi Doctors When you read this book please ask God to forgive me ‫ﺍﺳﺌﻠﻜﻢ ﺍﻟﺪﻋﺎء ﻻ ﻏﻴﺮ‬

Dr. Ahmed Hakim

Related Documents

Mastering Mrcp Vol2
January 2021 1
Earthship-vol2
January 2021 1
Mastering
January 2021 1
Mastering
February 2021 1
Faraz Pearls For Mrcp
February 2021 1
Mrcp Revision Notes
January 2021 3

More Documents from "Pass MRCP"

Mastering Mrcp Vol2
January 2021 1